Final Study Questions part 2

¡Supera tus tareas y exámenes ahora con Quizwiz!

A client is to have a transsphenoidal hypophysectomy to remove a large, invasive pituitary tumor. The nurse should instruct the client that the surgery will be performed through an incision in the: 1. Back of the mouth. 2. Nose. 3. Sinus channel below the right eye. 4.Upper gingival mucosa in the space between the upper gums and lip.

4 With transsphenoidal hypophysectomy, the sella turcica is entered from below, through the sphenoid sinus. There is no external incision; the incision is made between the upper lip and gums.

When assessing an individual with peripheral vascular disease, which clinical manifestation would indicate complete arterial obstruction in the lower left leg? 1. Aching pain in the left calf. 2. Burning pain in the left calf. 3. Numbness and tingling in the left leg. 4. Coldness of the left foot and ankle

4 Coldness in the left foot and ankle is consistent with complete arterial obstruction. Other expected findings would include paralysis and pallor. Aching pain, a burning sensation, or numbness and tingling are earlier signs of tissue hypoxia and ischemia and are commonly associated with incomplete obstruction.

A 46-year-old female client is admitted for acute renal failure secondary to diabetes and hypertension. Which test is the best indicator of adequate glomerular filtration? A) Serum creatinine. B) Blood Urea Nitrogen (BUN). C) Sedimentation rate. D) Urine specific gravity.

A

The nurse is teaching a female client about the best time to plan sexual intercourse in order to conceive. Which information should the nurse provide? A) Two weeks before menstruation. B) Vaginal mucous discharge is thick. C) Low basal temperature. D) First thing in the morning.

A

Which assessment finding by the nurse during a client's clinical breast examination requires follow-up? A) Newly retracted nipple. B) A thickened area where the skin folds under the breast. C) Whitish nipple discharge. D) Tender lumpiness noted bilaterally throughout the breasts.

A

A client who has type 2 diabetes is prescribed glipizide (Glucotrol). Which precautions does the nurse include in the teaching plan related to this medication? A) "Avoid taking nonsteroidal anti-inflammatory drugs." B) "Change positions slowly when you get up." C) "If you miss a dose of this drug, you can double the next dose." D) "Discontinue the medication if you develop an infection."

A Nonsteroidal anti-inflammatory drugs potentiate the hypoglycemic effects of sulfonylurea agents. Glipizide is a sulfonylurea. The other statements are not applicable to glipizide.

The nurse is caring for an older postoperative client. Which assessment finding causes the nurse to assess further for a wound infection? A) The client is now confused but was not confused previously. B) Moderate serosanguineous drainage is seen on the dressing. C) The white blood cell count is 8000/mm3. D) The white blood cell differential indicates a right shift.

A Older adult clients often do not demonstrate typical signs and symptoms of infection because of the diminished immune function seen with aging. Often, the first sign of infection is mental status changes. Any change in mental status in the older postoperative client should lead the nurse to assess for a wound infection.

The nurse is assessing a client's skin for local signs of infection. Which signs does the nurse assess for? (Select all that apply.) A) Redness B) Fever C) Increased erythrocyte sedimentation rate (ESR) D) Pain E) Swelling F) Warmth

A,D,E,F Localized signs of infection include redness, warmth, pain, swelling, heat, and pus. Fever and increased ESR are systemic signs of infection.

Which of the following instructions are most appropriate in the home management of a patient who has undergone surgery for oral cancer? A. "You should drink plenty of fluids and eat foods you enjoy." B. "It is normal to have some leakage of saliva from the suture line." C. "Lying in a prone position helps decrease swelling at the suture line." D. "You should avoid foods high in protein while your suture line is healing."

A. "You should drink plenty of fluids and eat foods you enjoy."

The nurse on a busy surgical unit is assigning client care to a registered nurse (RN) and a practical nurse (PN). Which client is best to assign to the PN? A. A preoperative client who has developed urinary retention and need a urinary catheter inserted B. A postoperative client who is receiving a unit of packed red blood cells C. A preoperative client who is fearful and anxious about the impending surgery D. A postoperative client who is febrile, has a productive cough, and is complaining of pain

A. A preoperative client who has developed urinary retention and need a urinary catheter inserted

A male client who has never smoked but has had COPD for the past 5 years is now being assessed for cancer of the lung. The nurse knows that he is most likely to develop which type of lung cancer? A. Adenocarcinoma B. Oat-cell carcinoma C. Malignant melanoma D. Squamous-cell carcinoma

A. Adenocarcinoma

When caring for a patient who is 3 hours postoperative laryngectomy, the nurse's highest priority assessment would be: A. Airway patency B. Patient comfort C. Incisional drainage D. Blood pressure and heart rate

A. Airway patency

The patient has an order for each of the following inhalers. Which of the following should the nurse offer to the patient at the onset of an asthma attack? A. Albuterol (Proventil) B. Beclomethasone (Beclovent) C. Ipratropium bromide (Atrovent) D. Salmeterol (Serevent)

A. Albuterol (Proventil)

Following a patient's bone marrow aspiration, which of the following nursing interventions should a nurse anticipate? A. Application of firm pressure to the site B. Positioning the patient in a prone position C. Positioning the patient in a supine position D. Application of a warm, moist compress to the site

A. Application of firm pressure to the site

What nursing intervention is particularly indicated for the second stage of labor? A. Assisting the client to push effectively so that expulsion of the fetus can be achieved B. Providing pain medication to increase the client's tolerance of labor pains C. Assessing the fetal heart rate and pattern for signs of fetal distress D. Monitoring effects of oxytocin administration to help achieve cervical dilation

A. Assisting the client to push effectively so that expulsion of the fetus can be achieved

A client is brought to the ED by ambulance in cardiac arrest with cardiopulmonary resuscitation (CPR) in progress. The client is intubated and is receiving 100% oxygen per self-inflating (ambu) bag. The nurse determines that the client is cyanotic, cold, and diaphoretic. Which assessment is most important for the nurse to obtain? A. Breath sounds over bilateral lung fields. B. Carotid pulsation during compressions C. Deep tendon reflexes D. Core body temperature

A. Breath sounds over bilateral lung fields.

An unconscious client is admitted to the intensive care unit and is placed on a ventilator. The ventilator alarms continuously and the client's oxygen saturation level is 62%. What action should the nurse take first? A. Call respiratory therapy B. Begin manual ventilation immediately C. Monitor oxygen saturation levels q5 minutes D. Silence the alarm and call the technician

A. Call respiratory therapy

An overweight, young adult made who was recently diagnosed with type 2 diabetes mellitus is admitted for a hernia repair. He tells the nurse that he is feeling very weak and jittery. Which actions should the nurse implement? (Select all that apply.) A. Check his fingerstick glucose level B. Assess his skin temperature and moisture C. Measure his pulse and blood pressure D. Document anxiety on the surgical checklist E. Administer a PRN dose of regular insulin

A. Check his fingerstick glucose level B. Assess his skin temperature and moisture C. Measure his pulse and blood pressure

During morning rounds, the nurse finds a client who has no spontaneous respirations and does not respond to shaking. The nurse activates the "Code-Blue" system. While waiting for the code team to arrive, what action should the nurse implement? A. Check the pulse for 10 seconds and begin chest compressions B. Check the airway for a foreign body and remove it if visualized C. Assess blood pressure and pupillary response to light D. Seal the face mask of an Ambu bag over the client's mouth and nose

A. Check the pulse for 10 seconds and begin chest compressions

The nurse is obtaining a client's fingerstick glucose level. After gently milking the client's finger, the nurse observes that the distal tip of the finger appears reddened and engorged. What action should the nurse take? A. Collect the blood sample B. Assess radial pulse volume C. Apply pressure to the site D. Select another finger

A. Collect the blood sample

A client who suffered an electrical injury with the entrance site on the left hand and the exit site on the left foot is admitted to the burn unit. Which intervention is most important for the nurse to include in this client plan of care? A. Continuous cardiac monitoring B. Perform passive range of motion C. Evaluate level of consciousness D. Assess lung sounds q4 hours.

A. Continuous cardiac monitoring

When caring for a client with nephrotic syndrome, which assessment is most important for the nurse to obtain? A. Daily weight B. Vital signs C. Level of consciousness D. Bowel sounds

A. Daily weight

Furosemide is prescribed for a 4-year-old who has ventricular septal defect. Which outcome indicates to the nurse that this pharmacological intervention was effective? A. Daily weight decrease of 2 pounds B. Blood urea nitrogen (BUN) increase from 8 to 12 mg/dl C. urine specific gravity change from 1.021 to 1.031 D. urinary output decrease of 5 ml/hour

A. Daily weight decrease of 2 pounds

A resident in a long-term care facility is diagnosed with hepatitis B. Which intervention should the nurse implement with the staff caring for this client?

A. Determine if all employees have had the hepatitis B vaccine series

The nurse is preparing to administer 1.6 ml of medication IM to a 4-month-old infant. Which intervention should the nurse implement? A. Divide the medication into two injections with volumes under 1 ml B. Inject into the center one-third of the medical aspect of the thigh C. Administer into the ventrogluteal muscle with child prone D. Apply a topical anesthetic ointment prior to the injection

A. Divide the medication into two injections with volumes under 1 ml

In assessing a client 48 hours following a fracture, the nurse observes ecchymosis at the fracture site, and recognizes that hematoma formation at the bone fragment site has occurred. What action should the nurse implement? A. Document the extent of he bruising in the medical record B. Assign a UAP to take vital sign measurements q1h C. Advise the client that anticoagulation therapy may be needed. D. Call the lab to obtain a stat APTT and prothrombin time

A. Document the extent of he bruising in the medical record

The husband of an older woman, diagnosed with pernicious anemia, calls the clinic to report that his wife still has memory loss and some confusion since she received the first dose of nasal cyanocobalamin two days ago. He tells the nurse that he is worried that she may be getting Alzheimer's disease. What action should the nurse take? A. Explain that memory loss and confusion are common with Vitamin B12 deficiency B. Ask if the client is experiencing any change in bowel habits C. Determine if the client is taking iron and folic aid supplements D. Encourage the husband to bring the client to the clinic for a complete blood count

A. Explain that memory loss and confusion are common with Vitamin B12 deficiency

A client is diagnosed with an acute small bowel obstruction. Which assessment finding requires the most immediate intervention by the nurse?

A. Fever of 102° F

A middle-aged woman, diagnosed with Graves' disease, asks the nurse about this condition. Which etiological pathology should the nurse include in the teaching plan about hyperthyroidism? (select all that apply) A. Graves' disease, an autoimmune condition, affects thyroid stimulating hormone receptors B. Large protruding eyeballs are a sign of hyperthyroid function C. Early treatment includes levothyroxine D. T3 and T4 hormone levels are increased E. Weight gain is a common complaint in hyperthyroidism

A. Graves' disease, an autoimmune condition, affects thyroid stimulating hormone receptors B. Large protruding eyeballs are a sign of hyperthyroid function D. T3 and T4 hormone levels are increased

When assessing a patient's sleep-rest pattern related to respiratory health, the nurse would ask if the patient: (Select all that apply.) A. Has trouble falling asleep B. Awakens abruptly during the night C. Sleeps more than 8 hours per night D. Has to sleep with the head elevated

A. Has trouble falling asleep B. Awakens abruptly during the night D. Has to sleep with the head elevated

The nurse is caring for a patient admitted to the hospital with pneumonia. Upon assessment, the nurse notes a temperature of 101.4° F, a productive cough with yellow sputum and a respiratory rate of 20. Which of the following nursing diagnosis is most appropriate based upon this assessment? A. Hyperthermia related to infectious illness B. Ineffective thermoregulation related to chilling C. Ineffective breathing pattern related to pneumonia D. Ineffective airway clearance related to thick secretions

A. Hyperthermia related to infectious illness

When assessing a patient's nutritional-metabolic pattern related to hematologic health, the nurse would: A. Inspect the skin for petechiae. B. Ask the patient about joint pain. C. Assess for vitamin C deficiency. D. Determine if the patient can perform ADLs.

A. Inspect the skin for petechiae.

The nurse calculates the body mass index (BMI) for an obese adult. Which additional assessment finding places the client at high risk for cardiac disease? A. Large waist circumference with central fat B. High serum insulin level C. Hyperpigmentation on neck skin folds D. Poor muscle tone

A. Large waist circumference with central fat

Which activity is best for the nurse to initiate with a depressed client? A. Make cut-out cookies B. Play chess C. Play volleyball D. Watch television

A. Make cut-out cookies

The clinic nurse is reviewing strategies for blood glucose monitoring with a client who is newly diagnosed with diabetes mellitus. When helping the client select a blood glucose meter, which client assessments should the nurse complete? A. Manual dexterity and visual acuity B. Capillary refill time and radial pulse volume C. Deep tendon reflexes and skin color D. Skin elasticity and hand grip strength.

A. Manual dexterity and visual acuity

While obtaining the admission assessment data, which of the following characteristics would a nurse expect a patient with anemia to report? A. Palpitations B. Blurred vision C. Increased appetite D. Feeling of warm flushing sensation

A. Palpitations

The nurse observes an unlicensed assistive personnel (UAP) applying an alcohol-based hand rub while leaving a client's room after taking vital signs. What action should the nurse take? A. Remind the UAP to continue rubbing the hands together until they dry B. Supervise the UAP in the next client's room to evaluate hand hygiene C. Instruct the UAP to return to the clients room to perform hand washing D. Advise the UAP to wear gloves when obtaining vital signs for all clients

A. Remind the UAP to continue rubbing the hands together until they dry

A client who had a below-the-knee amputation is experiencing severe phantom limb pain (PLP) and asks the nurse if mirror therapy will make the pain stop. Which response by the nurse is likely to be most helpful? A. Research indicated that mirror therapy is effective in reducing phantom limb pain B. You can try mirror therapy, but do not expect complete elimination of the pain C. Transcutaneous electrical nerve stimulation (TENS) has been found to be more effective D. Where did you learn about the use of mirror therapy in treating phantom limb pain?

A. Research indicated that mirror therapy is effective in reducing phantom limb pain

The scrub nurse places the fenestrated drapes to expose the operative area for a client who is having a hepatic tumor removed. The scrub nurse should assist with applying the sterile impermeable adhesive drape or surgical skin barrier to which area? A. Right subcostal B. Right lumbar C. Low transverse D. Midline abdominal

A. Right subcostal

A client uses triamcinolone (Kenalog), a corticosteroid ointment, to manage pruritus caused by a chronic skin rash. The client calls the clinic nurse to report increased erythema with purulent exudate at the site. Which action should the nurse implement? A. Schedule an appointment or the client to see the healthcare provider B. Advise the client to apply plastic wrap over the ointment to promote healing C. Instruct the client to continue the ointment until all erythema is relieved D. Explain the client need to complete all prescribed dose of the medication

A. Schedule an appointment for the client to see the healthcare provider.

A nurse is caring for a client with Diabetes Insipidus (DI). Which data warrants the most immediate intervention by the nurse? A. Serum sodium of 185 mEq/L B. Dry skin with inelastic turgor C. Apical rate of 110 beats/minute D. Polyuria and excessive thirst

A. Serum sodium of 185 mEq/L

A nurse stops at the site of a motorcycle accident and finds a young adult male lying face down in the road in a puddle of water. It is raining, no one is available to send for help, and the cell phone is in the car about 50 feet away. What action should the nurse take first? A. Stabilize the victim's neck and roll over to evaluate his status B. Examine the victim's body surfaces for arterial bleeding C. Open the airway and initiate resuscitative measures D. Return to the car to call emergency response 911 for help

A. Stabilize the victim's neck and roll over to evaluate his status

41. Select all that apply. During initial assessment, a nurse should record which of the following manifestations of respiratory distress? A. Tachypnea B. Nasal flaring C. Thready pulse D. Panting or grunting E. Use of intercostal muscles F. An inspiratory-to-expiratory ratio of 1:2

A. Tachypnea D. Panting or grunting

A client with cirrhosis states that his disease was cause by a blood transfusion. What information should the nurse obtain first to provide effective client teaching? A. The year the blood transfusion was received B. The amount of alcohol the client drinks C. How long the client has had cirrhosis D. The client's normal coping mechanisms

A. The year the blood transfusion was received

a males client was transferred yesterday from the Emergency department to the telemetry unit because he had ST depression and resolved chest pain. When his EKG monitor alarms for ventricular tachycardia (VT), what action should the nurse take first? A. determine the clients responsiveness and respirations B. bring the crash cart to the room to defibrillate the client C. immediately initiate chest compressions D. notify the emergency response team

A. determine the clients responsiveness and respirations

To ensure the correct amount of oxygen delivery for a patient receiving 35% oxygen via a Venturi mask, it is most important that the nurse A. keep the air-entrainment ports clean and unobstructed. B. apply an adaptor to increase humidification of the oxygen. C. drain moisture condensation from the oxygen tubing every hour. D. keep the flow rate high enough to keep the bag from collapsing during inspiration.

A. keep the air-entrainment ports clean and unobstructed.

The nurse notices clear nasal drainage in a patient newly admitted with facial trauma, including a nasal fracture. The nurse should: A. test the drainage for the presence of glucose. B. suction the nose to maintain airway clearance. C. document the findings and continue monitoring. D. apply a drip pad and reassure the patient this is normal.

A. test the drainage for the presence of glucose.

During a health fair, a male client with emphysema tells the nurse that he fatigues easily. Assessment reveals marked clubbing of the fingernails and an increased anteroposterior chest diameter. Which instruction is best to provide the client?

A."Pace your activities and schedule rest periods."

A 58-year-old female client tells the nurse that she feels a sense of loss since she has stopped having menstrual periods. She then states, "At least I will no longer have to suffer through those horrible Pap smear tests every year." Which action should the nurse implement?

Advise the client that Pap smear tests should be continued.

When assigning clients on a medical-surgical floor to a registered nurse (RN) and a practical nurse (PN), it is best for the charge nurse to assign which client to the PN?

An older adult client with pneumonia and viral meningitis

The nurse identifies a need for additional teaching when a patient with acute infectious diarrhea states,

"I may use over-the-counter lopermide (Immodium) or Parepectolin as need to control the diarrhea."

The nurse plans to help an 18-year-old developmentally disabled female client ambulate on the first postoperative day. When the nurse tells her it is time to get out of bed, the client becomes angry and yells at the nurse, "Get out of here! I'll get up when I'm ready." Which response should the nurse provide?

"I'll be back in 30 minutes to help you get out of bed and walk around the room."

Following a transsphenoidal hypophysectomy, the nurse should assess the client for: 1. Cerebrospinal fluid (CSF) leak. 2. Fluctuating blood glucose levels. 3. Cushing's syndrome. 4. Cardiac arrhythmias

1 A major focus of nursing care after transsphenoidal hypophysectomy is the prevention of and monitoring for a CSF leak. CSF leakage can occur if the patch or incision is disrupted. The nurse should monitor for signs of infection, including elevated temperature, increased white blood cell count, rhinorrhea, nuchal rigidity, and persistent headache. Hypoglycemia and adrenocortical insufficiency may occur. Monitoring for fluctuating blood glucose levels is not related specifically to transsphenoidal hypophysectomy. The client will be given IV fluids postoperatively to supply carbohydrates. Cushing's disease results from adrenocortical excess, not insufficiency. Monitoring for cardiac arrhythmias is important, but arrhythmias are not anticipated following a transsphenoidal hypophysectomy.

Which of the following is a risk factor for hypovolemic shock? 1. Hemorrhage. 2. Antigen-antibody reaction. 3. Gram-negative bacteria. 4. Vasodilation.

1 Causes of hypovolemic shock include external fluid loss, such as hemorrhage; internal fluid shifting, such as ascites and severe edema; and dehydration. Massive vasodilation is the initial phase of vasogenic or distributive shock, which can be further subdivided into three types of shock: septic, neurogenic, and anaphylactic. A severe antigen-antibody reaction occurs in anaphylactic shock. Gram-negative bacterial infection is the most common cause of septic shock. Loss of sympathetic tone (vasodilation) occurs in neurogenic shock.

The client has been managing angina episodes with nitroglycerin. Which of the following indicate the drug is effective? 1. Decreased chest pain. 2. Increased blood pressure. 3. Decreased blood pressure. 4. Decreased heart rate

1 Nitroglycerin acts to decrease myocardial oxygen consumption. Vasodilation makes it easier for the heart to eject blood, resulting in decreased oxygen needs. Decreased oxygen demand reduces pain caused by heart muscle not receiving sufficient oxygen. While blood pressure may decrease ever so slightly due to the vasodilation effects of nitroglycerine, it is only secondary and not related to the angina the patient is experiencing. Increased blood pressure would mean the heart would work harder, increasing oxygen demand and thus angina. Decreased heart rate is not an effect of nitroglycerine.

Propylthiouracil (PTU) is prescribed for a client with Graves' disease. The nurse should teach the client to immediately report which of the following? 1. Sore throat. 2. Painful, excessive menstruation. 3. Constipation. 4. Increased urine output.

1 The most serious adverse effects of PTU are leukopenia and agranulocytosis, which usually occur within the first 3 months of treatment. The client should be taught to promptly report to the health care provider signs and symptoms of infection, such as a sore throat and fever. Clients having a sore throat and fever should have an immediate white blood cell count and differential performed, and the drug must be withheld until the results are obtained. Painful menstruation, constipation, and increased urine output are not associated with PTU therapy.

Which of the following individuals is most at risk for acquiring acute lymphocytic leukemia (ALL)? The client who is: 1. 4 to 12 years. 2. 20 to 30 years. 3. 40 to 50 years. 4. 60 to 70 years.

1 The peak incidence of ALL is at 4 years of age. ALL is uncommon after 15 years of age. The median age at incidence of CML is 40 to 50 years. The peak incidence of AML occurs at 60 years of age. Two-thirds of cases of chronic lymphocytic leukemia occur in clients older than 60 years of age.

The nurse's best explanation for why the severely neutropenic client is placed in reverse isolation is that reverse isolation helps prevent the spread of organisms: 1. To the client from sources outside the client's environment. 2. From the client to health care personnel, visitors, and other clients. 3. By using special techniques to dispose of contaminated materials. 4. By using special techniques to handle the client's linens and personal items.

1 The primary purpose of reverse isolation is to reduce transmission of organisms to the client from sources outside the client's environment.

A client is scheduled for an arteriogram. The nurse should explain to the client that the arteriogram will confirm the diagnosis of occlusive arterial disease by: 1. Showing the location of the obstruction and the collateral circulation. 2. Scanning the affected extremity and identifying the areas of volume changes. 3. Using ultrasound to estimate the velocity changes in the blood vessels. 4. Determining how long the client can walk.

1 An arteriogram involves injecting a radiopaque contrast agent directly into the vascular system to visualize the vessels. It usually involves computed tomographic scanning. The velocity of the blood flow can be estimated by duplex ultrasound. The client's ankle-brachial index is determined, and then the client is requested to walk. The normal response is little or no drop in ankle systolic pressure after exercise.

A 68-year-old client on day 2 after hip surgery has no cardiac history but reports having chest heaviness. The first nursing action should be to: 1. Inquire about the onset, duration, severity, and precipitating factors of the heaviness. 2. Administer oxygen via nasal cannula. 3. Offer pain medication for the chest heaviness. 4. Inform the physician of the chest heaviness.

1 Further assessment is needed in this situation. It is premature to initiate other actions until further data have been gathered. Inquiring about the onset, duration, location, severity, and precipitating factors of the chest heaviness will provide pertinent information to convey to the physician.

A client is currently receiving an infusion labeled Heparin Sodium 25,00 units in 5% Dextrose Injection 500 ml at 14 ml/hour. A prescription is received to change the rate of the infusion to 900 units of Heparin per hour. The nurse should set the infusion pump to deliver how many ml.hour?

18 ml/hour

A client with thyrotoxicosis says to the nurse, "I am so irritable. I am having problems at work because I lose my temper very easily." Which of the following responses by the nurse would give the client the most accurate explanation of her behavior? 1. "Your behavior is caused by temporary confusion brought on by your illness." 2. "Your behavior is caused by the excess thyroid hormone in your system." 3. "Your behavior is caused by your worrying about the seriousness of your illness."

2 A typical sign of thyrotoxicosis is irritability caused by the high levels of circulating thyroid hormones in the body. This symptom decreases as the client responds to therapy. Thyrotoxicosis does not cause confusion. The client may be worried about her illness, and stress may influence her mood; however, irritability is a common symptom of thyrotoxicosis and the client should be informed of that fact rather than blamed.

The nurse is evaluating the client's learning about combination chemotherapy. Which of the following statements by the client about reasons for using combination chemotherapy indicates the need for further explanation? 1. "Combination chemotherapy is used to interrupt cell growth cycle at different points." 2. "Combination chemotherapy is used to destroy cancer cells and treat side effects simultaneously." 3. "Combination chemotherapy is used to decrease resistance." 4. "Combination chemotherapy is used to minimize the toxicity from using high doses of a single agent."

2 Combination chemotherapy does not mean two groups of drugs, one to kill the cancer cells and one to treat the adverse effects of the chemotherapy. Combination chemotherapy means that multiple drugs are given to interrupt the cell growth cycle at different points, decrease resistance to a chemotherapy agent, and minimize the toxicity associated with use of a high dose of a single agent (ie, by using multiple agents with different toxicities).

Which of the following is an expected outcome when a client is receiving an IV administration of furosemide? 1. Increased blood pressure. 2. Increased urine output. 3. Decreased pain. 4. Decreased premature ventricular contractions.

2 Furosemide is a loop diuretic that acts to increase urine output. Furosemide does not increase blood pressure, decrease pain, or decrease arrhythmias.

The nurse is completing a health assessment of a 42-year-old female with suspected Graves' disease. The nurse should assess this client for: 1. Anorexia. 2. Tachycardia. 3. Weight gain. 4. Cold skin.

2 Graves' disease, the most common type of thyrotoxicosis, is a state of hypermetabolism. The increased metabolic rate generates heat and produces tachycardia and fine muscle tremors. Anorexia is associated with hypothyroidism. Loss of weight, despite a good appetite and adequate caloric intake, is a common feature of hyperthyroidism. Cold skin is associated with hypothyroidism.

Which of the following is the most appropriate diet for a client during the acute phase of myocardial infarction? 1. Liquids as desired. 2. Small, easily digested meals. 3. Three regular meals per day. 4. Nothing by mouth

2 Recommended dietary principles in the acute phase of MI include avoiding large meals because small, easily digested foods are better tolerated. Fluids are given according to the client's needs, and sodium restrictions may be prescribed, especially for clients with manifestations of heart failure. Cholesterol restrictions may be prescribed as well. Clients are not prescribed diets of liquids only or restricted to nothing by mouth unless their condition is very unstable.

Which is a priority assessment for the client in shock who is receiving an IV infusion of packed red blood cells and normal saline solution? 1. Fluid balance. 2. Anaphylactic reaction. 3. Pain. 4. Altered level of consciousness.

2 The client who is receiving a blood product requires astute assessment for signs and symptoms of allergic reaction and anaphylaxis, including pruritus (itching), urticaria (hives), facial or glottal edema, and shortness of breath. If such a reaction occurs, the nurse should stop the transfusion immediately, but leave the IV line intact, and notify the physician. Usually, an antihistamine such as diphenhydramine hydrochloride (Benadryl) is administered. Epinephrine and corticosteroids may be administered in severe reactions. Fluid balance is not an immediate concern during the blood administration. The administration should not cause pain unless it is extravasating out of the vein, in which case the IV administration should be stopped. Administration of a unit of blood should not affect the level of consciousness.

Which nursing intervention is most important in preventing septic shock? 1. Administering IV fluid replacement therapy as prescribed. 2. Obtaining vital signs every 4 hours for all clients. 3. Monitoring red blood cell counts for elevation. 4. Maintaining asepsis of indwelling urinary catheters.

4

Upon entering the room of a patient who has just returned from surgery for total laryngectomy and radical neck dissection, a nurse should recognize a need for intervention when finding A. a gastrostomy tube that is clamped. B. the patient coughing blood-tinged secretions from the tracheostomy. C. the patient positioned in a lateral position with the head of the bed flat. D. 200 ml of serosanguineous drainage in the patient's portable drainage device.

C. the patient positioned in a lateral position with the head of the bed flat.

A client is admitted to the hospital with a diagnosis of severe acute diverticulitis. Which nursing intervention has the highest priority

Place the client on NPO status.

A client has undergone insertion of a permanent pacemaker. When developing a discharge teaching plan, the nurse writes a goal of, "The client will verbalize symptoms of pacemaker failure." Which symptoms are most important to teach the client? A) Facial flushing. B) Fever. C) Pounding headache. D) Feelings of dizziness.

D

A client reports unprotected sexual intercourse one week ago and is worried about HIV exposure. An initial HIV antibody screen (ELISA) is obtained. The nurse teaches the client that seroconversion to HIV positive relies on antibody production by B lymphocytes after exposure to the virus. When should the nurse recommend the client return for repeat blood testing? A) 6 to 18 months. B) 1 to 12 months. C) 1 to 18 weeks. D) 6 to 12 weeks.

D

A client taking furosemide (Lasix), reports difficulty sleeping. What question is important for the nurse to ask the client? A) What dose of medication are you taking? B) Are you eating foods rich in potassium? C) Have you lost weight recently? D) At what time do you take your medication?

D

A client who is HIV positive asks the nurse, "How will I know when I have AIDS?" Which response is best for the nurse to provide? A) Diagnosis of AIDS is made when you have 2 positive ELISA test results. B) Diagnosis is made when both the ELISA and the Western Blot tests are positive. C) I can tell that you are afraid of being diagnosed with AIDS. Would you like for me to call your minister? D) AIDS is diagnosed when a specific opportunistic infection is found in an otherwise healthy individual.

D

A client who is fully awake after a gastroscopy asks the nurse for something to drink. After confirming that liquids are allowed, which assessment action should the nurse consider a priority? A) Listen to bilateral lung and bowel sounds. B) Obtain the client's pulse and blood pressure. C) Assist the client to the bathroom to void. D) Check the client's gag and swallow reflexes.

D

A client with a completed ischemic stroke has a blood pressure of 180/90 mm Hg. Which action should the nurse implement? A) Position the head of the bed (HOB) flat. B) Withhold intravenous fluids. C) Administer a bolus of IV fluids. D) Give an antihypertensive medications.

D

During a health fair, a 72-year-old male client tells the nurse that he is experiencing shortness of breath. Auscultation reveals crackles and wheezing in both lungs. Suspecting that the client might have chronic bronchitis, which classic symptom should the nurse expect this client to have? A) Racing pulse with exertion. B) Clubbing of the fingers. C) An increased chest diameter. D) Productive cough with grayish-white sputum.

D

Dysrhythmias are a concern for any client. However, the presence of a dysrhythmia is more serious in an elderly person because A) elderly persons usually live alone and cannot summon help when symptoms appear. B) elderly persons are more likely to eat high-fat diets which make them susceptible to heart disease. C) cardiac symptoms, such as confusion, are more difficult to recognize in the elderly. D) elderly persons are intolerant of decreased cardiac output which may result in dizziness and falls.

D

When reviewing the purposes of a family assessment, the nurse educator would identify a need for further teaching if the student responded that family assessment is used to gain an understanding of the family. A) development. B) function. C) structure. D) political views.

D An understanding of the political views of family members is not a primary purpose of a family assessment. A family assessment provides the nurse with information and an understanding of family dynamics. This is important to nurses for the provision of quality health care. A family assessment provides an understanding of family development, function, and structure.

A client is being treated for dehydration. Which statement made by the client indicates understanding of this condition? a. I will use a salt substitute when making and eating my meals. b. I must drink a quart of water or other liquid each day. c. I will not drink liquids after 6 PM so I won't have to get up at night. d. I will weigh myself each morning before I eat or drink.

D Because 1 L of water weighs 1 kg, change in body weight is a good measure of excess fluid loss or fluid retention. Weight loss greater than 0.5 lb daily is indicative of excessive fluid loss. The other statements are not indicative of practices that will prevent dehydration.

An older adult client is admitted with an infection. On assessment, the nurse finds the client slightly confused. Vital signs are as follows: temperature 99.2° F (37.3° C), blood pressure 100/60 mm Hg, pulse 100, and respiratory rate 20. Which action by the nurse is most appropriate? A) Document the findings and continue to monitor. B) Assess the client's pain level and treat if needed. C) Perform a Mini-Mental Status Examination. D) Assess the client for other signs of infection.

D Because of an age-related decline in immune function, an older adult's normal temperature may be 1° to 2° lower than normal. A temperature of 99.2° F may be a fever in this population. Often a change in mental status is an early sign of illness for the older adult. The nurse should assess for other indications of infection.

A client with a history of heart failure is being discharged. Which priority instruction will assist the client in the prevention of complications associated with heart failure? A) "Eat six small meals daily instead of three larger meals." B) "When you feel short of breath, take an additional diuretic." C) "Avoid drinking more than 3 quarts of liquids each day." D) "Weigh yourself daily while wearing the same amount of clothing."

D Clients with heart failure are instructed to weigh themselves daily to detect worsening heart failure early, and thus avoid complications. Other signs of worsening heart failure include increasing dyspnea, exercise intolerance, cold symptoms, and nocturia.

The nurse evaluates that teaching for the patient with iron deficiency anemia has been effective when the patient states A. "I will need to take the iron supplements the rest of my life." B. "I will increase my dietary intake of milk and milk products." C. "I should increase my activity to increase my aerobic capacity." D. "I should take the iron for several months after my blood is normal."

D. "I should take the iron for several months after my blood is normal."

The nurse plans to help an 18-year-old developmentally disabled female client ambulate on the first postoperative day. When the nurse tells her it is time to get out of bed, the client becomes angry and yells at the nurse. "Get out of here! I'll get up when I'm ready." Which response should the nurse provide? A. "Your healthcare provider has prescribed ambulation on the first postoperative day." B. "You must ambulate to avoid serious complications that are much more painful." C. "I know how you feel; you're angry about having to do this, but it is required." D. "I'll be back in 30 minutes to help you get out of bed and walk around the room."

D. "I'll be back in 30 minutes to help you get out of bed and walk around the room."

While the school nurse is teaching a group of 14-year-olds, one of the participants remarks, "You are too young to be our teacher! You're not much older than we are!" How should the nurse respond? A. "I think I am qualified to teach this group." B. "How old do you think I am?" C. "Do you think you can teach it any better?" D. "We need to stay focused on the topic."

D. "We need to stay focused on the topic."

A 60-year-old male client with cancer of the liver has been in a hepatic coma for the past 24 hours. On admission, the client signed a release of information to his family. His oldest son arrives from out of town and asks the nurse how his father is doing. Which response is best for the nurse to provide? A. "I know you are concerned about your father. Would you like to talk about your feelings?" B. The healthcare provider will be here this afternoon and can explain your father's condition to you and the rest of the family." C. "Your father's condition is extremely critical. Would you like me to call the hospital chaplain to talk with you?" D. "Your father has given no response for 24 hours. His condition is extremely critical at this time."

D. "Your father has given no response for 24 hours. His condition is extremely critical at this time."

Before starting a transfusion of packed red blood cells for an anemic patient, the nurse would arrange for a peer to monitor his or her other assigned patients for how many minutes when the nurse begins the transfusion? A. 60 B. 5 C. 30 D. 15

D. 15

The patient has an order for albuterol 5 mg via nebulizer. Available is a solution containing 1 mg/ml. How many milliliters should the nurse use to prepare the patient's dose? A. 0.2 B. 2.5 C. 3.75 D. 5.0

D. 5.0

The arterial blood gas (ABG) readings that indicate compensated respiratory acidosis are a PaCO2 of A. 30 mm Hg and bicarbonate level of 24 mEq/L. B. 30 mm Hg and bicarbonate level of 30 mEq/L. C. 50 mm Hg and bicarbonate level of 20 mEq/L. D. 50 mm Hg and bicarbonate level of 30 mEq/L.

D. 50 mm Hg and bicarbonate level of 30 mEq/L.

A patient with COPD is receiving oxygen at 2 L/min. While in the supine position for a bath, the patient complains of shortness of breath. What is the most appropriate first nursing action? A. Increase the flow of oxygen. B. Perform tracheal suctioning. C. Report this to the physician. D. Assist the patient to Fowler's position.

D. Assist the patient to Fowler's position.

The nurse is scheduled to give a dose of ipratropium bromide by metered dose inhaler. The nurse would administer the right drug by selecting the inhaler with which of the following trade names? A. Vanceril B. Pulmicort C. AeroBid D. Atrovent

D. Atrovent

Which of the following physical assessment findings in a patient with pneumonia best supports the nursing diagnosis of ineffective airway clearance? A. Oxygen saturation of 85% B. Respiratory rate of 28 C. Presence of greenish sputum D. Basilar crackles

D. Basilar crackles

The nurse administers an isotonic intravenous solution to a client in septic shock. Which parameter is most important for the nurse to monitor to determine if this treatment is effective? A. White blood cell count (WBC) B. Body temperature C. Hemoglobin and hematocrit D. Blood pressure

D. Blood pressure

Respiratory acidosis is at highest risk in a patient with A. hypokalemia. B. pulmonary fibrosis. C. salicylate overdose. D. COPD.

D. COPD.

What is the goal when planning nursing care for a client with edema and leg discoloration secondary to chronic venous insufficiency? A. Adequate oxygenation will be restored B. Client will manifest normal urine output C. Client will demonstrate improved fluid-balance D. Client's skin integrity will remain intact

D. Client's skin integrity will remain intact

The charge nurse in the Labor and Delivery Unit makes assignments for a nurse and unlicensed assistive personnel (UAP). A client in labor is admitted with contractions occurring every 3 to 5 minutes. Which task should be assigned to the UAP? A. Teach patterned breathing B. Apply external fetal monitor C. Measure the fundal height D. Collet a urine specimen

D. Collet a urine specimen

An adult female with multiple sclerosis (MS) fells while walking to the bathroom. On transfer to the intensive care unit, she is confused and has had projectile vomiting twice. Which intervention should the nurse implement first? A. Determine clients last dose of corticosteroids B. Determine neurological baseline prior to the fall C. Administer a PRN IV antiemetic as prescribed D. Complete head to toe neurological assessment.

D. Complete head to toe neurological assessment.

An older client is admitted with a diagnosis of bacterial pneumonia. Which symptom should the nurse report to the health care provider after assessing the client?

D. Confusion and tachycardia

An older woman who experienced a cerebrovascular accident (CVA) has difficulty with visual perception and she only eats half of the food on her meal tray. Her family expresses concern about her nutritional status. How should the nurse respond to the family's concern? A. Encourage the family to offer to feed the client when she does not eat her entire meal. B. Suggest that the family bring foods from home that the client enjoys C. Explain that weight loss will be reversed after the acute phase of the stroke has ended. D. Demonstrate the use of visual scanning during meals to the client and family.

D. Demonstrate the use of visual scanning during meals to the client and family

The father brings his pre-school-aged son to the rural urgent care clinic because the child fell from a horse earlier today. Which finding indicates to the nurse that further assessment is required for possible abuse or neglect? A. A dislocated shoulder and fractured wrist on X-ray B. Bruises, abrasions, and restricted movement in right shoulder, elbow, and wrist C. Father's presence during the child's assessment and physical examination D. Dry, peeling skin, ridged nails, and significantly underweight

D. Dry, peeling skin, ridged nails, and significantly underweight

The nurse should instruct the parents of an 11-year-old with Type 1 diabetes mellitus to carefully watch their child for symptoms of diabetic ketoacidosis at risk for becoming ketoacidotic? A. While adjusting the amount of the insulin dosage B. When changing to a new brand of insulin C. After skipping two or more meals consecutively D. During the course of an acute illness

D. During the course of an acute illness

The nurse is assigned to care for a patient who has anxiety and an exacerbation of asthma. Which of the following is the primary reason for the nurse to carefully inspect the chest wall of this patient? A. Observe for signs of diaphoresis B. Allow time to calm the patient C. Monitor the patient for bilateral chest expansion D. Evaluate the use of intercostal muscles

D. Evaluate the use of intercostal muscles

A client's telemetry monitor indicates ventricular fibrillation (VF). After delivering one counter shock, the nurse resumes chest compression. After another minute of compressions, the client's rhythm converts to supraventricular tachycardia (SVT) on the monitor. At this point, what is the priority intervention for the nurse? A. Prepare for transcutaneous pacing B. Deliver another defibrillator shock C. Administer IV Epinephrine per ACLS protocol D. Give IV dose of adenosine rapidly over 1-2 seconds.

D. Give IV dose of adenosine rapidly over 1-2 seconds

When planning patient teaching about emphysema, the nurse understands that the symptoms of emphysema are caused by which of the following? A. Hypertrophy and hyperplasia of goblet cells in the bronchi B. Collapse and hypoventilation of the terminal respiratory unit C. An overproduction of the antiprotease alpha1-antitrypsin D. Hyperinflation of alveoli and destruction of alveolar walls

D. Hyperinflation of alveoli and destruction of alveolar walls

When assessing lab values on a patient admitted with septicemia, the nurse would expect to find: A. Increased platelets B. Decreased red blood cells C. Decreased erythrocyte sedimentation rate (ESR) D. Increased bands in the WBC differential (shift to the left)

D. Increased bands in the WBC differential (shift to the left)

Two unlicensed assistive personal (UAP) are arguing loudly in the hallway of the extended care facility about who will shower a male resident who defecated in his bed. What action is best for charge nurse to take? A. Shower the client with the help of a practical nurse B. Reassign the clients care to another staff member C. Document the conflict in the employee personnel files D. Instruct both UAP to shower the client immediately

D. Instruct both UAP to shower the client immediately

A client who took a camping vacation two weeks ago in a county with a tropical climate comes to the clinic describing vague symptoms and diarrhea for the past week. Which finding is most important for the nurse report to the healthcare provider? A. Weakness and fatigue B. Intestinal cramping C. Weight loss D. Jaundiced sclera

D. Jaundiced sclera

The nurse is evaluating whether a patient understands how to safely determine whether a metered dose inhaler is empty. The nurse interprets that the patient understands this important information to prevent medication underdosing when the patient describes which method to check the inhaler? A. Place it in water to see if it floats. B. Shake the canister while holding it next to the ear. C. Check the indicator line on the side of the canister. D. Keep track of the number of inhalations used.

D. Keep track of the number of inhalations used.

A female who was admitted for alcohol detoxification is nauseated and describes feeling like roaches are crawling all over her. She is tremulous, and her blood pressure is 146/92; her pulse rate is 94 beats/minute; and her temperature is 100.8F. Which PRN medication should the nurse administer first? A. Ondansetron (Zofran) B. Acetaminophen (Tylenol) C. Ramiprl (Altace) D. Lorazepam (Ativan)

D. Lorazepam (Ativan)

When planning appropriate nursing interventions for a patient with metastatic lung cancer and a 60-pack-year history of cigarette smoking, the nurse recognizes that the smoking has most likely decreased the patient's underlying respiratory defenses because of impairment of which of the following? A. Reflex bronchoconstriction B. Ability to filter particles from the air C. Cough reflex D. Mucociliary clearance

D. Mucociliary clearance

A client with acquired immunodeficiency syndrome (AIDS) has impaired gas exchange from a respiratory infection. Which assessment finding warrants immediate intervention by the nurse? A. Elevated temperature B. Generalized weakness C. Diminished lung sounds D. Pain when swallowing

D. Pain when swallowing

With the client's eyes closed, the nurse places a common object in the client's hand and asks the client to describe the object. The client accurately names the object. How should the nurse document this assessment finding? A. No paraesthesia present B. Short term memory intact C. Active range of motion D. Positive for stereognosis

D. Positive for stereognosis

An adult client is admitted with diabetic ketoacidosis (DKA) and a urinary tract infection (UTI). Prescriptions for intravenous antibiotics and an insulin infusion are initiated. Which serum laboratory value warrants the most immediate intervention by the nurse? A. Glucose of 350 mg/dl B. White blood cell count of 15, 000 mm3 C. Blood PH of 7.30 D. Potassium of 2.5 mEq/L

D. Potassium of 2.5 mEq/L

The nurse know that a client taking diuretics must be assessed for the development of hypokalemia, and that hypokalemia will create changes in the client's normal ECG tracing. Which ECG change would be an expected finding in the client with hypokalemia? A. Tall, spiked T waves B. A prolonged QT interval C. A widening QRS complex D. Presence of a U wave

D. Presence of a U wave

A female client who had a total thyroidectomy several weeks ago is admitted with myxedema coma. Which finding indicates that the client has been noncompliant with her postoperative treatment plan? A. Systolic blood pressure consistently greater than 160 B. Suppressed levels of thyroid stimulating hormone (TSH) C. Telemetry reveals atrial fibrillation D. Serum T3 and T4 levels below normal

D. Serum T3 and T4 levels below normal

A female client receives a prescription for alendronate sodium (Fosamx) to treat her newly diagnosed osteoporosis. What instruction should the nurse include in the client's teaching plan? A. Eat within 30 minutes of taking the medication B. Ingest an antacid 30 minutes prior to taking the tablet C. Consume a light snack with the medication D. Take on an empty stomach with a full glass of water

D. Take on an empty stomach with a full glass of water

A male client with chronic kidney disease (CKD) is beginning his first hemodialysis 3 times per week. Which short-term goal is most important for the nurse to include in the plan of care for this client as he begins the series? A. Reports subjective symptom's during hemodialysis B. Documents his oral intake during dialysis treatments C. Demonstrates self-care of the arteriovenous (AV) Shunt D. Verbalizes understanding of the reasoning for dialysis

D. Verbalizes understanding of the reasoning for dialysis

After a 92-year-old client fractured a hip trying to get out of bed, a nurse is accused of failing to notify the healthcare provider that the client was disoriented. In determining whether the nurse is guilty, a jury would consider which standard? A. What the nurse was taught in school about similar client care situations B. What an experienced lawyer would advise to be done in a similar situation C. What a well-educated healthcare consumer would expect in the same situation D. What a reasonable and prudent nurse would have done in the same situation

D. What a reasonable and prudent nurse would have done in the same situation

The nurse identifies the nursing diagnosis of activity intolerance for a patient with asthma. The nurse assesses for which of the following etiologic factor for this nursing diagnosis in patients with asthma? A. Anxiety and restlessness B. Effects of medications C. Fear of suffocation D. Work of breathing

D. Work of breathing

A client presents at the clinic with blepharitis. What instruction should the nurse provide for home care? A. use bilateral eye patches while sleeping to prevent injury to eyes B. wear sunglasses when out of doors to prevent photophobia C. apply cool moist compresses for 20 minutes followed by warm moist compresses D. apply warm moist compresses then gently scrub eyelids with dilute baby shampoo

D. apply warm moist compresses then gently scrub eyelids with dilute baby shampoo

What will the nurse teach a patient with pneumonia?

Deep breathing and coughing

The nurse is caring for a client with a fractured right elbow. Which assessment finding has the highest priority and requires immediate intervention?

Deep, unrelenting pain in the right arm

A patient with pneumonia has a nursing diagnosis pf ineffective airway clearance related to pain, fatigue, and thick secretions. An appropriate nursing intervention for the patient is to

Encourage fluid intake of 3L/day

The nurse is assessing clients in an outpatient diabetic clinic. Which entry provides the best medication that the client is adhering to the prescribed diabetic regimen?

Hemoglobin A1C of 6.2%

Rifampin side effects and considerations

Hepatitis, thrombocytopenia, orange discoloration of bodily fluids (Sputum, urine, sweat, tears).

A client in the emergency department is bleeding profusely from a gunshot wound to the abdomen. In what position should the nurse immediately place the client to promote maintenance of the client's blood pressure above a systolic pressure of 90 mm Hg?

Maintain the client in a supine position to reduce diaphragmatic pressure and visualize the wound

An antacid (Maalox) is prescribed for a client with peptic ulcer disease. What is the therapeutic action of this medication which is effective in treating the client's ulcer?

Maintenance of a gastric pH of 3.5 or above

Endoscopic retrograde cholangiopanceatography (ERCP)

Nursing responsibility is to explain procedure to patient, including patients role. Keep patient NPO 8 hour before procedure. Ensure consent is signed. Administer sedation. Administer ABT. Check VS and check for perforation or infection. Check Gag reflex.

A 49-year-old female client arrives at the clinic for an annual exam and asks the nurse why she becomes excessively diaphoretic and feels warm during nighttime. What is the nurse's best response? A) Explain the effect of the follicle-stimulating and luteinizing hormones. B) Discuss perimenopause and related comfort measures. C) Assess lung fields and for a cough productive of blood-tinged mucous. D) Ask if a fever above 101º F has occurred in the last 24 hours.

B

The physician orders Lanoxin(digoxin)0.375 mg po every day. On hand you have 0.25mg/5 mL. How many mL would you give your patient? A) 8 mL B) 7.5 mL C) 7 mL D) 5.5 mL

B

A patient states that his/her legs have pain with walking that decreases with rest. The nurse observes absence of hair on the patient's lower leg and the patient has a thready posterior tibial pulse. How would the nurse position the patient's legs? A) Slightly bent with a pillow under the knees B) Dependent position C) Elevated D) Crossed at the knee

B A patient with arterial insufficiency is taught to position their legs in a dependent position to use gravity to help perfuse the tissues. Crossing legs at the knee may interfere with blood flow. Slightly bent legs do not enhance blood flow.

The nurse is teaching a client with pneumonia ways to clear secretions. Which intervention is the most effective? A) Administering an antiemetic medication B) Increasing fluids to 2 L/day if tolerated C) Administering an antitussive medication D) Having the client cough and deep breathe hourly

B Increasing fluids has been proven to decrease the thickness of secretions, thus allowing them to be expectorated quickly. The other interventions would not be as effective.

The nurse is assigned to care for the following four clients who have the potential for having pain. Which client is most likely not to be treated adequately for this problem? A) Middle-aged woman with a fractured arm B) Client with expressive aphasia C) Younger adult with metastatic cancer D) Client who has undergone an appendectomy

B Populations at highest risk for inadequate pain treatment include older adults, minorities, and those with a history of substance abuse. Nonverbal clients are very difficult to assess for pain because self-report is not possible, and the nurse needs to rely on client behaviors or surrogate reporting.

The nurse auscultates the following lung sound in the client with pneumonia. What is the best intervention? Audio Clip A) Administer IV fluids. B) Have the client use an incentive spirometer. C) Have the client cough and deep breathe. D) Prepare to administer a bronchodilator.

B The sound heard is crackles. Crackles often indicate atelectasis, which can be reversed by using an incentive spirometer. If no spirometer is available, coughing and deep breathing is the next best option. This client does not have wheezing, so bronchodilators are not indicated. IV fluids would not help atelectasis.

The nurse would identify which body systems as directly involved in the process of normal gas exchange? (Select all that apply.) A) Endocrine system B) Neurologic system C) Hepatic system D) Immune system E) Cardiovascular system F) Pulmonary system

B, E, F The neurologic system controls respiratory drive; the respiratory system controls delivery of oxygen to the lung capillaries; and the cardiac system is responsible for the perfusion of vital organs. These systems are primarily responsible for the adequacy of gas exchange in the body. The endocrine and hepatic systems are not directly involved with gas exchange. The immune system primarily protects the body against infection.

The nurse is caring for a client who is experiencing excessive bleeding after receiving unfractionated heparin. What orders does the nurse anticipate from the health care provider? (Select all that apply.) A) Laboratory draw for prothrombin time (PT)/international normalized ratio (INR) B) Laboratory draw for activated partial thromboplastin time (aPTT) C) Administer enoxaparin (Lovenox) D) Administer protamine sulfate E) Administer vitamin K

B,D Protamine sulfate is the antidote for heparin-induced bleeding. Vitamin K is the antidote for warfarin. Warfarin (Coumadin) would increase bleeding. Enoxaparin is another name for heparin.

A 58-year-old client, who has no health problems, asks the nurse about taking the pneumococcal vaccine (Pneumovax). Which statement give by the nurse would offer the client accurate information about this vaccine? A. "The vaccine is given annually before the flue season to those over 50 years of age." B. "The immunization is administered once to older adults or persons with a history of chronic illness." C. "The vaccine is for all ages and is given primarily to those person traveling overseas to infected areas." D. "The vaccine will prevent the occurrence of pneumococcal pneumonia for up to 5 years."

B. "The immunization is administered once to older adults or persons with a history of chronic illness."

If a health care provider is planning to transfuse a patient with a unit of packed red blood cells, which of the following solutions should the health care provider hang with the transfusion? A. 5% dextrose in water B. 0.9% sodium chloride C. 5% dextrose in 0.9% sodium chloride D. 5% dextrose in lactated Ringer's solution

B. 0.9% sodium chloride

Seconal 0.1 gram PRN at bedtime is prescribed for rest. The scored tablets are labeled grain 1.5 per tablet. How many tablets should the nurse plan to administer? A. 1/2 tablet B. 1 tablet C. 1 1/2 tablet D. 2 tablets

B. 1 tablet 15 gr = 1 g, 0.1 x 15 = 1.5 grains

The nurse notes a physician's order written at 10:00 AM for 2 units of packed red blood cells to be administered to a patient who is anemic secondary to chronic blood loss. If the transfusion is picked up at 11:30, the nurse should plan to hang the unit no later than which of the following times? A. 11:45 AM B. 12:00 noon C. 12:30 PM D. 3:30 PM

B. 12:00 noon

The nurse is conducting health assessments. Which assessment finding increases a 56-year-old woman's risk for developing osteoporosis? A. Body mass index (BMI) of 31 B. 20 pack-year history of cigarette smoking C. Birth control pill usage until age 45 D. Diabetes mellitus in family history

B. 20 pack-year history of cigarette smoking

A 7-year-old child admitted to the hospital with acute glomerulonephritis (AGN). When obtaining the nursing history, which finding should the nurse expect to obtain? A. A recent DPT immunization B. A recent strep throat infection C. High blood cholesterol levels on routine screening D. Increased thirst and urination

B. A recent strep throat infection

While the nurse is conducting a daily assessment of an older woman who resides in a long-term facility, the client begins to cry and tell the nurse that her family has stopped calling and visiting. What action should the nurse take first? A. Determine the client's orientation to time and space B. Ask the client when a family member last visited her C. Review the client's record regarding social interactions D. Reassure the client of her family's love for her

B. Ask the client when a family member last visited her

In monitoring a client receiving propylthiouracil (PTU) for hyperthyroidism, an increase in which finding indicates that the medication is producing the desired effect? A. Urinary output B. Body weight C. Pulse rate D. Blood pressure

B. Body weight

A nurse assists a male client with Parkinson's disease (PD) to ambulate in the hallway. The client appears to "freeze" and then carefully lifts one leg and steps forward. He tells the nurse that he is pretending to step over a crack on the floor. How should the nurse respond? A. Re-orient the client to his present location and circumstances B. Confirm that this is an effective technique to help with ambulation C. Assist the client to a carpeted area where he can walk more easily. D. Plan to assess the client's cognition after returning to his room.

B. Confirm that this is an effective technique to help with ambulation

If a patient states, "It's hard for me to breathe and I feel short-winded all the time," what is the most appropriate terminology to be applied in documenting this assessment by a nurse? A. Apnea B. Dyspnea C. Tachypnea D. Respiratory fatigue

B. Dyspnea

A male client with heart failure (HF) calls the clinic and reports that he cannot put his shoes on because they are too tight. Which additional information should the nurse obtain? A. What time did he take his last medications? B. Has his weight changed in the last several days? C. Is he still able to tighten his belt buckle? D. How many hours did he sleep last night?

B. Has his weight changed in the last several days?

When completing a mental health assessment, the nurse wishes to obtain information about a client's insight and judgement. What interview technique is most useful in assessing these mental abilities? A. Ask the client to describe what is meant by the proverb, "A penny saved is a penny earned." B. Have the client describe what should be done if a child was observed alone in a locked car on a hot day C. Show the client a list of 3 words a the beginning of the interview, and ask for word recall later. D. Request that the client complete a mathematical operation, such as counting backward by 9s form 100

B. Have the client describe what should be done if a child was observed alone in a locked car on a hot day

In assessing a client at 34-weeks gestation, the nurse notes that she has a slightly elevated total T4 with a slightly enlarged thyroid, a hematocrit of 28%, a heart rate of 92 beats per minute, and a systolic murmur. Which finding requires follow-up? A.Elevated parathyroid hormone level B. Hematocrit of 28% C. Heart rate of 92 beats per minute D. Systolic murmur

B. Hematocrit of 28%

A client diagnosed with chronic kidney disease (CDK) 2 years ago is regularly treated at a community hemodialysis facility. In assessing the client before his scheduled dialysis treatment, which electrolyte imbalance should the nurse anticipate? A. Hypophosphatemia B. Hypocalcemia C. Hyponatremia D. Hypokalemia

B. Hypocalcemia

As a means of relieving a client's pain associated with osteoarthritis, the nurse plans to provide local rest. To implement this intervention, which action should the nurse take? A. Elevate the affected joint with an icepack on it B. Immobilize the affected joint with a splint C. Administer a prescribed local topical salicylate D. Maintain bedrest with bathroom privileges

B. Immobilize the affected joint with a splint

Which of the following nursing interventions is of the highest priority in helping a patient expectorate thick secretions related to pneumonia? A. Humidify the oxygen as able B. Increase fluid intake to 3L/day if tolerated. C. Administer cough suppressant q4hr. D. Teach patient to splint the affected area.

B. Increase fluid intake to 3L/day if tolerated.

In monitoring a client's respiratory stauts, which symptom is characteristic of early acute (adult) respiratory distress syndrome (ARDS)? A. Coarse breath sounds B. Increased respiratory rate C. Intercostal retractions D. Pleural friction rubs

B. Increased respiratory rate

Which condition should the nurse anticipate as a potential problem in a female client with a neurogenic bladder? A. Stress incontinence. B. Infection. C. Painless, gross hematuria. D. Peritonitis.

B. Infection

In planning strategies to reduce a client's risk for complications following orthopedic surgery, the nurse recognizes which pathology as the underlying cause of osteomyelitis? A. Metastatic process B. Infectious process C. Autoimmune disorder D. Inflammatory disorder

B. Infectious process

Sublingual nitroglycerin is administer to a male client with unstable angina who complains of crushing chest pain. Five minutes later the client becomes nauseated and his blood pressure drops to 60/40. Which intervention should the nurse implement. A. Administer a second dose of nitroglycerin B. Infuse a rapid IV normal saline bolus C. Begin external chest compressions D. give a PRN antiemetic medication

B. Infuse a rapid IV normal saline bolus

When assessing a client who had a supratentorial craniotomy, what action should the nurse implement when determining the client's Glasgow coma scale (GCS) rating? A. Inject cold water into the client's ear. B. Instruct the client to raise an arm C. Determine the intracranial pressure D. Check the patellar and radial reflexes

B. Instruct the client to raise an arm

The nurse is planning discharge care for a male client with metastatic cancer. The client tells the nurse that he plans to return to work despite pain, fatigue, and impending death. Which goal is most important to include in this client's plan of care? A. Implement decisions about future hospice services within the next 3 months B. Maintains pain level below 4 when implementing outpatient pain clinic strategies C. Requests home health care if independence becomes compromised for 5 days D. Arranges for short term counseling if stressors impact work schedule for 2 weeks

B. Maintains pain level below 4 when implementing outpatient pain clinic strategies

A male client with hepatitis A is admitted with elevated hepatic enzymes and jaundice. Which intervention should the nurse implement? A. Provide cloth gowns for the client to wear B. Meticulous hand washing after each client contact C. Place the client in strict airborne precautions D. Use plastic utensils with each meal tray

B. Meticulous hand washing after each client contact

Which abnormal laboratory finding indicates that a client with diabetes needs further evaluation for diabetic nephropathy?

B. Microalbuminuria

A tornado warning alarm has been activated at the local hospital. Which action should the charge nurse working on a surgical unit implement first?

B. Move clients and visitors into the hallways and close all doors to clients' rooms.

The nurse manager is concerned about the number of falls that have occurred on the unit in the last month. Which action is most likely to decrease the number of falls? A. Place all clients on the unit regardless of age at risk for falls B. Obtain the evidence based practice guidelines for fall prevention C. Determine if pain medication is related to those who fell D. Inquire about what other units are doing to prevent falls

B. Obtain the evidence based practice guidelines for fall prevention

A client with metastatic cancer who was taking hydromorphone (Dilaudid) PO at home is now receiving the medication IV while in the hospital. To evaluate if the client is receiving an equianalgesic dose of the Dilaudid, what assessment should the nurse complete? A. Respiratory rate B. Pain scale C. Level of consciousness D. Blood pressure

B. Pain scale

A client on telemetry has a pattern of uncontrolled atrial fibrillation with a rapid ventricular response. Based on this finding, the nurse anticipates assisting the physician with which treatment?

B. Perform synchronized cardioversion.

A clients morning laboratory test results include leukocytes 3,500/mm^3 or 3.5x10^9/L (SI). Based on the laboratory result, which complaint is this client most likely to report to the nurse? A. Inability to walk without shortness of breath B. Persistent cough with yellow-colored sputum C. Superficial cuts do not readily stop bleeding D. A red streak and pain in right calf muscle

B. Persistent cough with yellow-colored sputum

In assessing a pressure ulcer on a client's hip, which action should the nurse include? A. Determine the degree of elasticity surrounding the lesion B. Photograph the lesion with a ruler placed next to the lesion C. Stage the depth of the ulcer using the Braden numeric scale D. Use a gloved finger to palpate for tunneling around the lesion

B. Photograph the lesion with a ruler placed next to the lesion

The nurse reviews the laboratory results of a client during an annual physical examination and identifies a positive guaiac test of stool. Which additional serum laboratory test result should the nurse review? A. Glucose B. Platelet count C. White blood cell count D. Amylase

B. Platelet count

A male client who had colon surgery 3 days ago is anxious and requesting assistance to reposition. While the nurse is turning him, the wound dehiscences and eviscerates. The nurse moistens an available sterile dressing and places it over the wound. What intervention should the nurse implement next? A. Bring additional sterile dressing supplies to the room B. Prepare the client to return to the operating room C. Obtain a sample of the drainage to send to the lab D. Auscultate the abdomen for bowel sound activity

B. Prepare the client to return to the operating room

The nurse is caring for a 73-year-old patient who underwent a left total knee arthroplasty. On the third postoperative day, the patient complains of shortness of breath, slight chest pain, and that "something is wrong." Temperature is 98.4o F, blood pressure 130/88, respirations 36, and oxygen saturation 91% on room air. Which of the following should the nurse first suspect as the etiology of this episode? A. Septic embolus from the knee joint B. Pulmonary embolus from deep vein thrombosis C. New onset of angina pectoris D. Pleural effusion related to positioning in the operating room

B. Pulmonary embolus from deep vein thrombosis

The nurse is interviewing a client who is taking interferon-alfa-2a (Roferon-A) and ribavirin (Virazole) combination therapy for hepatitis C. The client reports experiencing overwhelming feelings of depression. What action should the nurse implement first? A. Recommend mental health counseling. B. Review the medications actions and interactions. C. Assess for the client's daily activity level. D. Provide information regarding a support group.

B. Review the medications actions and interactions.

A male client presents to the clinic with large draining ulcers on his lower legs that are characteristic of Kapok's sarcoma lesions. He is accompanied by two family members. What actions should the nurse take? A. Ask the family members to wear gloves when touching the client B. Send family to the waiting area while the client's history is taken C. Obtain a blood sample to determine of the client is HIV positive D. Complete a head to toe assessment to identify other signs of HIV

B. Send family to the waiting area while the client's history is taken

Before discharge, the nurse discusses activity levels with a 61-year-old patient with COPD and pneumonia. Which of the following exercise goals is most appropriate once the patient is fully recovered from this episode of illness? A. Slightly increase activity over the current level. B. Walk for 20 minutes a day, keeping the pulse rate less than 130 beats per minute. C. Limit exercise to activities of daily living to conserve energy. D. Swim for 10 min/day, gradually increasing to 30 min/day.

B. Walk for 20 minutes a day, keeping the pulse rate less than 130 beats per minute.

A patient with hemophilia is hospitalized with acute knee pain and swelling. An appropriate nursing intervention for the patient includes A. wrapping the knee with an elastic bandage. B. placing the patient on bed rest and applying ice to the joint. C. gently performing range-of-motion (ROM) exercises to the knee to prevent adhesions. D. administering nonsteroidal anti-inflammatory drugs (NSAIDs) as needed for pain.

B. placing the patient on bed rest and applying ice to the joint.

A patient admitted to the emergency department with tension pneumothorax and mediastinal shift following an automobile crash is most likely to exhibit A. bradycardia. B. severe hypotension. C. mediastinal flutter. D. a sucking chest wound.

B. severe hypotension.

A nurse call the nurse to report that at 0900 she administered a PO dose of digoxin to her 4-month-old infant, but at 0920 the baby vomited the medicine. What instruction should the nurse provide to his mother? A. give another dose B. withhold this dose C. administer a half dose now D. mix the next dose with food

B. withhold this dose

A client with early breast cancer receives the results of a breast biopsy and asks the nurse to explain the meaning of staging and the type of receptors found on the cancer cells. Which explanation should the nurse provide? A) Lymph node involvement is not significant. B) Small tumors are aggressive and indicate poor prognosis. C) The tumor's estrogen receptor guides treatment options. D) Stage I indicates metastasis.

C

The nurse is planning care for a client with newly diagnosed diabetes mellitus that requires insulin. Which assessment should the nurse identify before beginning the teaching session? A) Present knowledge related to the skill of injection. B) Intelligence and developmental level of the client. C) Willingness of the client to learn the injection sites. D) Financial resources available for the equipment.

C

The nurse is planning care to prevent complication for a client with multiple myeloma. Which intervention is most important for the nurse to include? A) Safety precautions during activity. B) Assess for changes in size of lymph nodes. C) Maintain a fluid intake of 3 to 4 L per day. D) Administer narcotic analgesic around the clock.

C

What is my favorite football team? A) I hate football!! B) the Fightin' Irish C) GO BUCKS!!!! D) that team up north

C Seriously, if you don't know the instructor's favorite football by this time, you need to listen to your recordings from the first lecture. Just sayin'

The client with a stroke was admitted to a medical-surgical unit. Which tasks does the nurse delegate to the unlicensed assistive personnel? A) Assess level of consciousness. B) Evaluate the pulse oximetry reading. C) Assist the client with meals. D) Complete the nursing care plan.

C The nurse needs to know the five rights of delegation: right task, right circumstances, right person, right communication, and right supervision. Unlicensed assistive personnel can help with feeding, but only the nurse can care plan, assess the level of consciousness, and evaluate the oxygenation of the client.

In assessing a client with an arteriovenous (AV) shunt who is scheduled for dialysis today, the nurse notes the ABSENCE of a thrill or bruit at the shunt site. What action should the nurse take? A.Advise the client that the shunt is intact and ready for dialysis as scheduled. B.Encourage the client to keep the shunt site elevated above the level of the heart. C.Notify the health care provider of the findings immediately. D.Flush the site at least once with a heparinized saline solution.

C Absence of a thrill or bruit indicates that the shunt may be obstructed. The nurse should notify the health care provider (C) so that intervention can be initiated to restore function of the shunt. (A) is incorrect. (B) will not resolve the obstruction. An AV shunt is internal and cannot be flushed (D) without access using special needles.

When conducting a health history assessment, the nurse would want to know what important information about the patient's elimination status? (Select all that apply.) A) Time of day patient defecates B) Patient's preferences for toileting C) List of medications taken by patient D) Recent changes in elimination patterns E) Changes in color, consistency, or odor of stool or urine F) Discomfort or pain with elimination

C,D,E,F Recent changes in elimination patterns, color, consistency, or odor are important for the nurse to know concerning elimination. Discomfort or pain during elimination is important for the nurse to know. A nurse should also know which medications the patient is on as this may affect elimination. Time of day is not important, nor is the patient's preferences for toileting. They are personal preferences and do not affect elimination.

A client with a serum sodium level of 125 mEq/ml or mmol/L (SI)should benefit most from the administration of which intervention solution? A. 10% Dextrose in 0.45% sodium chloride B. 5% Dextrose in 0.2% sodium chloride C. 0.9% sodium chloride solution (normal saline) D. 0.45% sodium chloride solution (half normal saline)

C. 0.9% sodium chloride solution (normal saline)

During a home visit, the nurse assesses the skin of a client with eczema who reports that an exacerbation of symptoms has occurred during the last week. Which information is most useful in determining the possible cause of the symptoms? A. An old friend with eczema came for a visit B. Recently received an influenza immunization C. A grandson and his new dog recently visited D. Corticosteroid cream was applied to eczema

C. A grandson and his new dog recently visited

The nurse is obtaining a blood sample via venipuncture from a preschool-aged child. Which intervention should the nurse implement? A. Explain in very simple terms why the blood is needed B. Encourage the child to talk about this experience C. Apply a large colored band-aid to the puncture site D. Place the labeled specimen in a paper cup for transport

C. Apply a large colored band-aid to the puncture site

An adult, Muslim client with ulcerative colitis was admitted to the post-surgical unit earlier today following a bowel resection with temporary colostomy. Which intervention is most important for the nurse to implement? A. Teach the client how to perform stoma care B. Allow family members to visit whenever they wish C. Assign a care provider of the same gender D. Evaluate the client's current nutritional status

C. Assign a care provider of the same gender

During a clinic visit, a male client with heart failure (HF) reports that he has gained 4 pounds (1.8 kg) in the last 3 days. Which action should the nurse implement? A. Recommend controlled portions at mealtimes B. Assess for bilateral pitting pedal edema C. Auscultate all lung fields for fine crackles D. Encourage a reduced intake of table salts

C. Auscultate all lung fields for fine crackles

A postoperative client reports incisional pain. The client has two prescriptions for PRN analgesia that accompanied the client from the postanasthesia unit. Before selecting which medication to administer, which action should the nurse implement? A. Document the client's report of pain in the electronic medical record B. Determine which prescription will have the quickest onset of action C. Compare the client's pain scale rating with the prescribed dosing D. Ask the client to choose which mediation is needed for pain

C. Compare the client's pain scale rating with the prescribed dosing

Which assessment finding has the highest priority when planning nursing care for a client with peptic ulcer disease (PUD)? A. Epigastric pain after eating B. Dizziness when rising form a sitting position C. Dark tarry liquid stool D. Weight loss of 10 pounds in the past month

C. Dark tarry liquid stool

The nurse is caring for a critically ill client with cirrhosis of the liver who has a nasogastric tube draining bright red blood. The nurse notes that the client's serum hemoglobin and hematocrit are decreased. What additional change in lab data should the nurse expect? A. Increased serum albumin B. Decreased serum creatinine C. Decreased serum ammonia D. Increased liver function tests

C. Decreased serum ammonia

The nurse is triaging victims of a tornado at an emergency shelter. An adult woman who has been wandering and crying comes to the nurse. What action should the nurse take? A. Check the client's temperature, blood sugar, and urine output B. Transport the client for laboratory and electrocardiogram (EKG) C. Delegate care of the crying client to an unlicensed assistant D. Send the client to the shelter's nutrient center to obtain water and food

C. Delegate care of the crying client to an unlicensed assistant

During the admission interview, the nurse learns that a newly admitted adult client has a six month history of reoccurring somatic pain. Which problem is most important for the nurse to further explore with the client? A. Nausea and vomiting B. Episodes of injury related to falls C. Feelings of depression D. Periods of anxiety and restlessness

C. Feelings of depression

A client admitted with an acute myocardial infraction receives a cardiac diet with sodium restriction and complains that his hamburger is flavorless. Which condiment should the nurse offer? A. Pickle relish B. Steak sauce C. Fresh horseradish D. Tomato ketchup

C. Fresh horseradish

What is the correct procedure for performing an ophthalmoscopic examination on a client's right eye? A. Instruct the client to look at the examiner's nose and not move his/her eyes during the exam. B. Set ophthalmoscope on the plus 2 to 3 lens and hold it in front of the examiner's right eye. C. From a distance of 8 to 12 inches and slightly to the side, shine the light into the client's pupil. D. For optimum visualization, keep the ophthalmoscope at least 3 inches for the client's eye

C. From a distance of 8 to 12 inches and slightly to the side, shine the light into the client's pupil.

Which of the following positions is most appropriate for the nurse to place a patient experiencing an asthma exacerbation? A. Supine B. Lithotomy C. High-Fowler's D. Reverse Trendelenburg

C. High-Fowler's

The nurse and a social worker are talking when a male client with psychosis angrily shouts at the nurse, "Stop talking about me." The nurse should document the client is exhibiting which symptom? A. Auditory hallucinations B. Visual hallucinations C. Ideas of reference D. Thought broadcasting

C. Ideas of reference

Which intervention regarding immediate postoperative care should the nurse plan to include in the preoperative teaching of a client scheduled for an incisional rotator cuff repair? A. Ice will be applied to the incision prior to exercising to help decrease pain B. A cast will be used to hold the joint securely in place until it is healed C. It will be necessary to wear a sling to keep the joint still D. A special machine will be used to keep the joint moving

C. It will be necessary to wear a sling to keep the joint still

The nurse is assessing a 4-year-old with eczema. Her skin is dry and scaly, and the mother reports that she frequently scratches her skin to the point of causing bleeding. Which guideline is indicated for care for this child? A. Apply baby lotion to her skin twice daily B. Allow her to wear only 100% cotton clothing C. Keep her nails trimmed short D. Bathe her daily with bath oil

C. Keep her nails trimmed short

A client who has a history of long-standing back pain treated with methadone (Dolophines), is admitted to the surgical unit following urological surgery. Which modifications in the plan of care should the nurse make for this client's pain management during the postoperative period? A. Consult with surgeon about increasing methadone in lieu of parenteral opioids. B. Use minimal parenteral opioids for surgical pain, in addition to oral methadone C. Maintain client's methadone, and medicate surgical pain based on pain rating D. Make no changes in the standard pain management for the surgery and hold methadone.

C. Maintain client's methadone, and medicate surgical pain based on pain rating

An infant has a medical diagnosis of tracheosophageal fistula (TEF). Which nursing intervention is indicated for this infant prior to surgical repair? A. Administer isotonic enemas as prescribed B. Evaluate the infant's tolerance for small volume of formula C. Maintain suction equipment available at all times D. Prepare the child for a barium enema to correct the condition

C. Maintain suction equipment available at all times

A client in the emergency department is bleeding profusely from a gunshot wound to the abdomen. In what position should the nurse immediately place the client to promote maintenance of the client's blood pressure above a systolic pressure of 90 mm Hg?

C. Maintain the client in a supine position to reduce diaphragmatic pressure and visualize the wound.

A 71-year-old patient is admitted with acute respiratory distress related to cor pulmonale. Which of the following nursing interventions is most appropriate during admission of this patient? A. Delay any physical assessment of the patient and review with the family the patient's history of respiratory problems. B. Perform a comprehensive health history with the patient to review prior respiratory problems. C. Perform a physical assessment of the respiratory system and ask specific questions related to this episode of respiratory distress. D. Complete a full physical examination to determine the effect of the respiratory distress on other body functions.

C. Perform a physical assessment of the respiratory system and ask specific questions related to this episode of respiratory distress.

Following a devastating hurricane, a client is admitted for dehydration as the result of vomiting and diarrhea that occurred after ingesting contaminated water. The client expresses feelings of fear and anger about the destruction of homes, the loss of property due to the storm, and the looting that occurred following the storm. According to Maslow's hierarchy of need, what priority need should be addressed first? A. Seld-actualization B. Love and belonging C. Physiological needs D. Safety and security

C. Physiological needs

Before placing a client's dentures in the sink for cleansing, what action should the nurse take? A. Fill the sink with half-strength peroxide solution B. Loosen dried secretions with a toothbrush C. Place a washcloth on the bottom of the sink D. Hold the dentures in a stream of running water

C. Place a washcloth on the bottom of the sink

During discharge teaching for a 65-year-old patient with emphysema and pneumonia, which of the following vaccines should the nurse recommend the patient receive? A. S. aureus B. H. influenzae C. Pneumococcal D. Bacille Calmette-Guérin (BCG)

C. Pneumococcal

After a computer tomography (CT) scan with intravenous contrast medium, a client returns to the room complaining of shortness of breath and itching. Which intervention should the nurse implement? A. Call respiratory therapy to give a breathing treatment B. Send another nurse for an emergency tracheotomy set C. Prepare a dose of epinephrine (Adrenalin) D. Review the client's complete list of allergies

C. Prepare a dose of epinephrine (Adrenalin)

An older adult with heart failure is hospitalized during an acute exacerbation. To reduce cardiac workload, which intervention should the nurse include in the client's plan of care? A. Assist with ambulation in the hallway B. Encourage active range of motion exercises C. Provide a bedside commode for toileting D. Teach to sleep in a slide-laying position

C. Provide a bedside commode for toileting

A client receives a prescription for an intramuscular pain medication. The nurse uses the Z-track method to administer the injection Which rationale supports the nurse's use of this method? A. Prevents injury to the underlying bones, nerves, and blood vessels B. Minimize client's discomfort a the injection site C. Seals needle track to avoid medication leakage through the tissue D. Ensures medication reaches the intramuscular site

C. Seals needle track to avoid medication leakage through the tissue

A client receives a prescription for an intramuscular pain medication. The nurse uses the Z-track method to administer the injection. Which rationale supports the nurse's use of this method? A. Prevents injury to the underlying bones, nerves, and blood vessels B Minimizes client's discomfort at the injection site C. Seals needle track to avoid medication leakage through the tissue D. Ensures medication reaches the intramuscular site

C. Seals needle track to avoid medication leakage through the tissue

The community mental health nurse is planning to visit four clients with schizophrenia. Which client should the nurse see first? A. The young woman who believes she is to blame for her divorce B. The client with a history of substance abuse who is living in a halfway house C. The father who took his children from school because aliens were after them D. The client who needs to be evaluated for medication compliance.

C. The father who took his children from school because aliens were after them

If a nurse is assessing a patient whose recent blood gas determination indicated a pH of 7.32 and respirations are measured at 32 breaths/min, which of the following is the most appropriate nursing assessment? A. The rapid breathing is causing the low pH. B. The nurse should sedate the patient to slow down respirations. C. The rapid breathing is an attempt to compensate for the low pH. D. The nurse should give the patient a paper bag to breathe into to correct the low pH.

C. The rapid breathing is an attempt to compensate for the low pH.

A cardiac catherterization of a client with heart disease indicates the following blockages: 95% proximal left anterior descending (LAD), 99% proximal circumflex, and ? % proximal right coronary artery (RCA). The client later asks the nurse "what does all this mean for me?" What information should the nurse provide? A. Blood supply to the heart is diminished by artherosclerotic lesions, which necessitate lifestyle changes. B. Blood vessels supplying the pumping chamber have blockages indicating a past heart attack. C. Three main arteries have major blockages, with only 1 to 5% of blood flow getting through to the heart muscle. D. The heart is not receiving enough blood, so there is a risk of heart failure and fluid retention.

C. Three main arteries have major blockages, with only 1 to 5% of blood flow getting through to the heart muscle.

While caring for a client with Amyotrophic Lateral Sclerosis (ALS), the nurse performs a neurological assessment every four hours. Which assessment finding warrants immediate intervention by the nurse? A. Inappropriate laughter B. Increasing anxiety C. Weakened cough effort D. Asymmetrical weakness

C. Weakened cough effort

A client with a liver abscess undergoes surgical evacuation and drainage of the abscess. Which laboratory value is most important for the nurse to monitor following the procedure? A. Serum creatinine B. Blood urea nitrogen (BUN) C. White blood cell count D. Serum glucose

C. White blood cell count

A client with cholelithiasis has a gallstone lodged in the common bile duct and is unable to eat or drink without becoming nauseated and vomiting. Which finding should the nurse report to the healthcare provider. A. Belching B. Amber urine C. Yellow sclera D. Flatulence

C. Yellow sclera

A client who was splashed with a chemical has both eyes covered with bandages. When assisting the client with eating, which intervention should the nurse instruct the unlicensed assistive personal (UAP) to implement? A. feed the client the entire meal B. provide with only finer food C. orient the client to location of food on the plate D. ask family member to visit during meal time to assist with feeding

C. orient the client to location of food on the plate

In older adults, infection after exposure to respiratory illness is most likely to A. result in similar rates of infection as in the younger adult. B. be easily prevented with the use of antibiotics after being exposed. C. result in serious lower respiratory infection related to weakened respiratory muscles and fewer cilia. D. be less serious because the older adult has less contact with younger children who are most likely to carry serious infections.

C. result in serious lower respiratory infection related to weakened respiratory muscles and fewer cilia.

In teaching the patient with COPD about the need for physical exercise, the nurse informs the patient that A. all patients with COPD should be able to increase walking gradually up to 20 min/day. B. a bronchodilator inhaler should be used to relieve exercise-induced dyspnea immediately after exercise. C. shortness of breath is expected during exercise but should return to baseline within 5 minutes after the exercise. D. monitoring the heart rate before and after exercise is the best way to determine how much exercise can be tolerated.

C. shortness of breath is expected during exercise but should return to baseline within 5 minutes after the exercise.

The nurse is preparing an older male client with Parkinson's disease for discharge to home. Which instruction should the nurse provide to promote independence and reduce risk for injury? (select all that apply) A. use a long stride when walking B. try to swing arms while walking C. take scheduled medication on time D. place small rugs over tile flooring E. consider use of recliner lift chair

C. take scheduled medication on time E. consider use of recliner lift chair

The nurse is providing discharge instructions to a client who is receiving prednisone (Deltasone) 5 mg PO daily for a rash due to contact with poison ivy. Which symptom should the nurse tell the client to report to the healthcare provider?

Rapid weight gain

When explaining dietary guidelines to a client with acute glomerulonephritis (AGN), which instruction should the nurse include in the dietary teaching?

Restrict sodium intake

A client with hypothyroidism reports difficulty falling asleep because of feelings of depression. Which action should the nurse implement?

Review most recent thyroid function test results

The nurse is counseling a healthy 30-year-old female client regarding osteoporosis prevention. Which activity would be most beneficial in achieving the client's goal of osteoporosis prevention?

cross country skiing

which statement about a patient with a tube feeding indicates best practice for patient safety & quality care? A) if the tube becomes clogged, use 30 mL of water for flushing, while applying gentle pressure with a 50 mL piston syringe B) when administering medications, use cold water to dissolve the drug before administering it C) use cranberry juice to flush the tube if it is clogged D) administer drugs down the feeding tube without flushing first, but flush the feeding tube after the drug is given

A

Alteplase recombinant, or tissue plasminogen activator (t-PA), a thrombolytic enzyme, is administered during the first 6 hours after onset of myocardial infarction (MI) to: 1. Control chest pain. 2. Reduce coronary artery vasospasm. 3. Control the arrhythmias associated with MI. 4. Revascularize the blocked coronary artery.

4 The thrombolytic agent t-PA, administered intravenously, lyses the clot blocking the coronary artery. The drug is most effective when administered within the first 6 hours after onset of MI. The drug does not reduce coronary artery vasospasm; nitrates are used to promote vasodilation. Arrhythmias are managed by antiarrhythmic drugs. Surgical approaches are used to open the coronary artery and re-establish a blood supply to the area.

A client is taking furosemide (Lasix) and becomes confused. Which potassium level does the nurse correlate with this condition? A) 2.9 mEq/L B) 5.0 mEq/L C) 6.0 mEq/L D) 3.8 mEq/L

A Hypokalemia decreases cerebral function and is manifested by lethargy, confusion, inability to perform problem-solving tasks, disorientation, and coma. Normal potassium levels are 3.5 to 5.0 mEq/L. At 2.9 mEq/L, potassium is too low, and this could lead to neurologic manifestations.

The nurse administers 6 units of regular insulin and 10 units NPH insulin at 7 AM. At what time does the nurse assess the client for problems related to the NPH insulin? A) 4 PM B) 11 PM C) 8 AM D) 8 PM

A NPH is an intermediate-acting insulin with an onset of 1.5 hours, peak of 4 to 12 hours, and duration of action of 22 hours. Checking the client at 8:00 AM would be too soon; 8:00 PM and 11:00 PM would be too late.

The nurse is administering a nystatin suspension (Mycostatin) for stomatitis. Which instruction will the nurse provide to the client when administering this medication? A."Hold the medication in your mouth for a few minutes before swallowing it." B."Do not drink or eat milk products for 1 hour prior to taking this medication." C."Dilute the medication with juice to reduce the unpleasant taste and odor." D."Take the medication before meals to promote increased absorption."

A Mycostatin is prescribed for fungal infections of the mouth. The client should swish the medication in the mouth for 2 minutes and then swallow (A). (B) does not affect administration of this medication. The medication should not be diluted because this will reduce its effectiveness (C). (D) is not necessary.

The most appropriate measure for a nurse to use in assessing core body temperature when there are suspected problems with thermoregulation is a(n) A) rectal thermometer. B) tympanic membrane sensor. C) temporal thermometer scan. D) oral thermometer.

A The most reliable means available for assessing core temperature is a rectal temperature, which is considered the standard of practice. An oral temperature is a common measure but not the most reliable. A temporal thermometer scan has some limitations and is not the standard. The tympanic membrane sensor could be used as a second source for temperature assessment.

A client is admitted to the hospital with a diagnosis of severe acute diverticulitis. Which assessment finding should the nurse expect this client to exhibit? A) Lower left quadrant pain and a low-grade fever. B) Severe pain at McBurney's point and nausea. C) Abdominal pain and intermittent tenesmus. D) Exacerbations of severe diarrhea.

A

Which instruction should the nurse teach a female client about the prevention of toxic shock syndrome?

"Change your tampon frequently."

An older female client with dementia is transferred from a long-term care unit to an acute care unit. The client's children express concern that their mother's confusion is worsening. How should the nurse respond?

"Confusion in an older person often follows relocation to new surroundings."

The nurse is completing an admission interview for a client with Parkinson disease. Which question will provide additional information about manifestations the client is likely to experience?

"Have you ever been 'frozen' in one spot, unable to move?

A splint is prescribed for nighttime use by a client with rheumatoid arthritis. Which statement by the nurse provides the most accurate explanation for use of the splints? A) Prevention of deformities. B) Avoidance of joint trauma. C) Relief of joint inflammation. D) Improvement in joint strength.

A

An adolescent receives a prescription for an injection of S-matriptan succinate, 4 mg subcutaneously for a migraine headache. Using a vial labeled, 6 mg/0.5 ml, how many ml should the nurse administer? (nearest hundredth)

0.33 ml

A school-aged child who weighs 42 pounds receives a post-tonsillectomy prescription for promethazine (Phenergan) 0.5 mg/kg IM to prevent postoperative nausea. The medication is available in 25 mg/ml ampules. How many ml should the nurse administer?

0.4 ml

A client who weighs 175 pounds is receiving IV bolus dose of heparin 80 units/kg. The heparin is available in a 2 ml vial, labeled 10,000 units/ml. How many ml should the nurse administer? (Enter numeric value only. If rounding is required, round to the nearest tenth.)

0.6 ml

The healthcare provider prescribes epoetin alfa (Procrit) 8,200 units subcutaneously for a client with chronic kidney disease (CKD). The 2 ml multidose vial is labeled, "Each 1 ml of solution contains 10,000 units of epoetin alfa." How many ml should the nurse administer?

0.8

When assessing the client with Hodgkin's disease, the nurse should observe the client for which of the following findings? 1. Herpes zoster infections. 2. Discolored teeth. 3. Hemorrhage. 4. Hypercellular immunity.

1 Herpes zoster infections are common in clients with Hodgkin's disease. Discoloring of the teeth is not related to Hodgkin's disease but rather to the ingestion of iron supplements or some antibiotics such as tetracycline. Mild anemia is common in Hodgkin's disease, but the platelet count is not affected until the tumor has invaded the bone marrow. A cellular immunity defect occurs in Hodgkin's disease in which there is little or no reaction to skin sensitivity tests. This is called anergy.

A client with diabetes is taking insulin lispro (Humalog) injections. The nurse should advise the client to eat: 1. Within 10 to 15 minutes after the injection. 2. 1 hour after the injection. 3. At any time, because timing of meals with lispro injections is unnecessary. 4. 2 hours before the injection.

1 Insulin lispro (Humalog) begins to act within 10 to 15 minutes and lasts approximately 4 hours. A major advantage of Humalog is that the client can eat almost immediately after the insulin is administered. The client needs to be instructed regarding the onset, peak, and duration of all insulin, as meals need to be timed with these parameters. Waiting 1 hour to eat may precipitate hypoglycemia. Eating 2 hours before the insulin lispro could cause hyperglycemia if the client does not have circulating insulin to metabolize the carbohydrate.

The nurse should teach the diabetic client that which of the following is the most common symptom of hypoglycemia? 1. Nervousness. 2. Anorexia. 3. Kussmaul's respirations. 4. Bradycardia.

1 The four most commonly reported signs and symptoms of hypoglycemia are nervousness, weakness, perspiration, and confusion. Other signs and symptoms include hunger, incoherent speech, tachycardia, and blurred vision. Anorexia and Kussmaul's respirations are clinical manifestations of hyperglycemia or ketoacidosis. Bradycardia is not associated with hypoglycemia; tachycardia is.

The nurse is assessing a client with chronic renal failure (CRF). Which finding is most important for the nurse to respond to first? A) Potassium 6.0 mEq. B) Daily urine output of 400 ml. C) Peripheral neuropathy. D) Uremic fetor.

A

Seconal 0.1 gram PRN at bedtime is prescribed for rest. The scored tablets are labeled grain 1.5 per tablet. How many tablets should the nurse plan to administer?

1 tablet

Sepsis

2 blood draws from 2 different sites before start ABT.

Sputum Test for TB

3 seperate sputum test over 3 days. Airborne precautions until 3 negative sputum samples. HEPA mask required.

A client with a psychotic disorder is receiving haloperidol (Haldol) 3 mg IM q30 minutes x 3 hours for agitation control. The medication is available in 5 mg/ml. How many ml will the client receive over the next 3 hours?

3.6 ml

The healthcare provider prescribes an IV solution of regular insulin (Hummulin-R) 100 units in 250 ml of 0.45% saline to infuse at 12 units/hour. The nurse should program the infusion pump to deliver how many ml/hour?

30

Sodium nitroprusside (Nipride) at 0.8 mcg/kg/minute is prescribed for a client who weighs 65 kg. The available IV solution is labeled Nipride 50 mg in 500 ml D5W. The nurse should program the infusion pump to deliver how many ml/hour?

31 ml/hour

The nurse is instructing the client on insulin administration. The client is performing a return demonstration for preparing the insulin. The client's morning dose of insulin is 10 units of regular and 22 units of NPH. The nurse checks the dose accuracy with the client. The nurse determines that the client has prepared the correct dose when the syringe reads how many units?

32 units

The nurse is preparing a teaching plan for healthy adults. Which individual is most likely to maintain optimum health?

A teacher whose blood glucose levels average 126 daily with oral antidiabetic drugs

The nurse is performing hourly neurologic checks for a client with a head injury. Which new assessment finding warrants the most immediate intervention by the nurse?

A unilateral pupil that is dilated and nonreactive to light

A client is admitted with infection and a high fever. Which assessments by the nurse take priority? (Select all that apply.) A) Skin turgor B) Pulse quality C) Blood pressure D) Bowel sounds E) Respiratory effort F) Mental status

A, B, C, F Dehydration can accompany fever, especially if the client is sweating profusely. Blood pressure, pulse quality, and skin turgor are assessments of fluid status. Mental status changes can accompany fluid losses, especially in older clients.

When caring for a patient with metastatic cancer, the nurse notes a hemoglobin level of 8.7 g/dl and hematocrit of 26%. The nurse would place highest priority on initiating interventions that will reduce which of the following? A. Fatigue B. Thirst C. Headache D. Abdominal pain

A. Fatigue

When admitting a 45-year-old female with a diagnosis of pulmonary embolism, the nurse will assess the patient for which of the following risk factors? (Select all that apply.) A. Obesity B. Pneumonia C. Hypertension D. Cigarette smoking

A. Obesity C. Hypertension D. Cigarette smoking

Which expected outcome statement should the nurse include in a teaching plan designed to assist a client with management of an acute attack of gout? A. The client will avoid use of alcohol in managing stress. B. The client will implement a high-purine daily dietary regime. C. The client will use local heat applications for acute pain. D. The client will stop antigout medication once pain subsides.

A. The client will avoid use of alcohol in managing stress.

When planning care for a client with acute pancreatitis, which nursing intervention has the highest priority? A. Withhold food and fluid intake B. Initiate IV fluid replacement C. Administer antiemetics as needed D. Evaluate intake and output ratio

A. Withhold food and fluid intake

To find the infection site associated with acute lymphangitis, the nurse should look _____ to the inflammation. A. distal B. anterior C. proximal D. contralateral

A. distal

A client with stage IV bone cancer is admitted to the hospital for pain control. The client verbalizes continuous, severe pain of 8 on a 1 to 10 scale. Which intervention should the nurse implement?

Administer opioid and non-opioid medication simultaneously

Bronchoscopy

After procedure keep the patient NPO until gag reflux returns.

A client tells the nurse that her biopsy results indicate that the cancer cells are well-differentiated. How should the nurse respond?

Ask the client if the healthcare provider has given her any information about the classification of her cancer

The nurse is reviewing the routine medications taken by a client with chronic angle closure glaucoma. Which medication prescription should the nurse question? A. An antianginal with a therapeutic effect of vasodilation. B. An anticholinergic with a side effect of pupillary dilation. C. An antihistamine with a side effect of sedation. D. A corticosteroid with a side effect of hyperglycemia.

B. An anticholinergic with a side effect of pupillary dilation.

When assigning clients on a medical-surgical floor to an RN and a PN, it is best for the charge nurse to assign which client to the PN?

B. An older adult client with pneumonia and viral meningitis

An elderly client is experiencing disturbed sleep patterns. Which interventions should the nurse implement to help the client attain maximal sleep function? (select all that apply) A. Discuss need for adult day services B. Assess daytime somnolence C. Recommend evaluation of home safety D. Review medication side effect profiles E. Refer to social worker for elder care

B. Assess daytime somnolence D. Review medication side effect profiles

A female client enters the clinic and insists on being seen. She is weak, nervous, and reports a racing heart beat and recent weight loss of 15 pounds. After ruling out substance withdrawal, the healthcare provider suspects hyperthyroidism and admits her for further testing. Which action should the nurse implement? A. Begin preparing client for thyroidectomy procedure B. Space the client's care to provide periods of rest C. Assess the client for hyperactive bowel sounds D. Provide warm blankets to prevent heat loss

B. Space the client's care to provide periods of rest

The nurse is providing preoperative education for a Jewish client scheduled to receive a xenograft graft to promote burn healing. Which information should the nurse provide this client? A. Grafting increases the risk for bacterial infections B. The xenograft is taken from nonhuman sources C. Grafts are later removed by a debriding procedure D. As the burn heals, the graft permanently attaches

B. The xenograft is taken from nonhuman sources

Based on the analysis of the client's atrial fibrillation, the nurse should prepare the client for which treatment protocol? A) Diuretic therapy. B) Pacemaker implantation. C) Anticoagulation therapy. D) Cardiac catheterization.

C

The nurse should be correct in withholding a dose of digoxin in a client with congestive heart failure without specific instruction from the healthcare provider if the client's A) serum digoxin level is 1.5. B) blood pressure is 104/68. C) serum potassium level is 3. D) apical pulse is 68/min.

C

O-H- A) who cares! B) I cannot pick this so I will lose points (this is for the Michigan Fans!!) C) I-O D) O-No!!

C Sucking up here with C may be dependent on how low your accumulative test scores are...

The nurse is completing an admission interview for a client with Parkinson's disease. Which question will provide additional information about manifestations that the client is likely to experience? A."Have you ever experienced any paralysis of your arms or legs?" B."Do you have frequent blackout spells?" C."Have you ever been frozen in one spot, unable to move?" D."Do you have headaches, especially ones with throbbing pain?"

C Clients with Parkinson's disease frequently experience difficulty in initiating, maintaining, and performing motor activities. They may even experience being rooted to the spot and unable to move (C). Parkinson's disease does not typically cause (A, B, or D).

When preparing to administer an ordered blood transfusion, the nurse selects which of the following intravenous solutions to use when priming the blood tubing? A. 5% dextrose in water B. Lactated Ringer's C. 0.9% sodium chloride D. 0.45% sodium chloride

C. 0.9% sodium chloride

During the change of shift report, the charge nurse reviews the infusions being received by clients on the oncology unit. The client receiving which infusion should be assessed first?

C. Continuous epidural infusion of morphine

During assessment of a 45-year-old patient with asthma, the nurse notes wheezing and dyspnea. The nurse interprets that these symptoms are related to which of the following pathophysiologic changes? A. Laryngospasm B. Overdistention of the alveoli C. Narrowing of the airway D. Pulmonary edema

C. Narrowing of the airway

A patient with an acute pharyngitis is seen at the clinic with fever and severe throat pain that affects swallowing. On inspection the throat is reddened and edematous with patchy yellow exudates. The nurse anticipates that collaborative management will include A. treatment with antibiotics. B. treatment with antifungal agents. C. a throat culture or rapid strep antigen test. D. treatment with medication only if the pharyngitis does not resolve in 3 to 4 days.

C. a throat culture or rapid strep antigen test.

What test do you use to verify pulmonary embolus?

CT scan

The nurse initiates neuro checks for a client who is at risk for neurologic compromise. Which manifestation typically provides the first indication of altered neuro function?

Change in level of consciousness

A 70-year-old male client with type 2 diabetes mellitus (DM) is hospitalized with an infected ulcer on his great right toe. Which instruction should the nurse emphasize during discharge teaching?

Check the insides and linings of all enclosed shoes before putting the shoes on

A client is admitted with a diagnosis of chronic obstructive pulmonary disease (COPD). What is the nurse's rationale for keeping the client's oxygen administration level at 3 L/min or less?

Chronic hypoxemia creates the urge to breathe in COPD.

A male client with pernicious anemia takes supplemental folate and self-administers monthly Vitamin B12 injections. He reports feeling increasingly fatigued. Which laboratory value should the nurse review?

Complete blood count

An older client is admitted with a diagnosis of bacterial pneumonia. The nurse's assessment of the client will most likely reveal which sign/symptom?

Confusion and tachycardia

During the shift report, the charge nurse informs a nurse that she has been assigned to another unit for the day. The nurse begins to sigh deeply and tosses about her belongings as she prepares to leave, making it known that she is very unhappy about being "floated" to the other unit. What is the best immediate action for the charge nurse to take?

Continue with shift report and talk to the nurse about the incident at a later time.

A 57-year-old male client is scheduled to have a stress-thallium test the following morning and is NPO after midnight. At 0130, he is agitated because he cannot eat and is demanding food. Which response is best for the nurse to provide to this client? A) I'm sorry sir, you have a prescription for nothing by mouth from midnight tonight. B) I will let you have one cracker, but that is all you can have for the rest of tonight. C) What did the healthcare provider tell you about the test you are having tomorrow? D) The test you are having tomorrow requires that you have nothing by mouth tonight.

D

An elderly client is admitted with a diagnosis of bacterial pneumonia. The nurse's assessment of the client is most likely to reveal which sign/symptom? A) Leukocytosis and febrile. B) Polycythemia and crackles. C) Pharyngitis and sputum production. D) Confusion and tachycardia.

D

The healthcare provider prescribes aluminum and magnesium hydroxide (Maalox), 1 tablet PO PRN, for a client with chronic renal failure who is complaining of indigestion. What intervention should the nurse implement? A) Administer 30 minutes before eating. B) Evaluate the effectiveness 1 hour after administration. C) Instruct the client to swallow the tablet whole. D) Question the healthcare provider's prescription.

D

The nurse assesses a client with advanced cirrhosis of the liver for signs of hepatic encephalopathy. Which finding should the nurse consider an indication of progressive hepatic encephalopathy? A) An increase in abdominal girth. B) Hypertension and a bounding pulse. C) Decreased bowel sounds. D) Difficulty in handwriting.

D

The nurse is assisting a client out of bed for the first time after surgery. What action should the nurse do first? A) Place a chair at a right angle to the bedside. B) Encourage deep breathing prior to standing. C) Help the client to sit and dangle legs on the side of the bed. D) Allow the client to sit with the bed in a high Fowler's position.

D

The nurse is planning to initiate a socialization group for older residents of a long-term facility. Which information is most useful to the nurse when planning activities for the group? A) The length of time each group member has resided at the nursing home. B) A brief description of each resident's family life. C) The age of each group member. D) The usual activity patterns of each member of the group.

D

The nurse accidentally administers 10 mg of morphine intravenously to a client who had been given another dose of morphine, 5 mg IV, about 30 minutes earlier. What action must the nurse be prepared to take? A) Assist with intubation. B) Monitor pain level. C) Administer oxygen. D) Administer naloxone (Narcan).

D A combined dose of 15 mg of morphine may cause severe respiratory depression in some clients. Naloxone is an opioid antagonist that can be used (intravenously) as the first intervention to reverse respiratory depression due to a morphine overdose. Then administration of oxygen may be needed if the client's oxygen saturation decreases. Intubation may occur if the client does not respond to the Narcan, and respiratory depression becomes a respiratory arrest. Naloxone may be repeated, but the pain level of the client needs to be monitored because Narcan can promote withdrawal symptoms.

A nurse is caring for an older adult client who lives alone. Which economic situation presents the most serious problem for this client? a. costs of creating a living will b. stock market fluctuations c. increased provider benefits d. social security as the basis of income

D Older adults on fixed incomes are unable to adjust their income to meet rising costs associated with meeting basic needs

A client has a deep wound covered with a wet-to-damp dressing. Which intervention does the nurse include on this client's care plan? A) Apply a new dressing when the seal breaks and the dressing leaks. B) Change the dressing when the current dressing is saturated. C) Leave the dressing intact until next week. D) Change the dressing every 6 hours around the clock.

D Wet-to-damp dressings are changed every 4 to 6 hours to provide maximum débridement. Synthetic dressings can be left in place for extended periods of time but need to be changed if the seal breaks and the exudate is leaking. Dry gauze dressings should be changed when the outer layer becomes saturated.

The nurse is preparing a 45-year-old client for discharge from a cancer center following ileostomy surgery for colon cancer. Which discharge goal should the nurse include in this client's discharge plan? A.Reduce the daily intake of animal fat to 10% of the diet within 6 weeks. B.Exhibit regular, soft-formed stool within 1 month. C.Demonstrate the irrigation procedure correctly within 1 week. D.Attend an ostomy support group within 2 weeks.

D Attending a support group (D) will be beneficial to the client and should be encouraged because adaptation to the ostomy can be difficult. This goal is attainable and is measurable. (A) is not specifically related to ileostomy care. The client with an ileostomy will not be able to accomplish (B). (C) is not necessary.

The nurse receives the client's next scheduled bag of TPN labeled with the additive NPH insulin. Which action should the nurse implement? A.Hang the solution at the current rate. B.Refrigerate the solution until needed. C.Prepare the solution with new tubing. D.Return the solution to the pharmacy.

D Only regular insulin is administered by the IV route, so the TPN solution containing NPH insulin should be returned to the pharmacy (D). (A, B, and C) are not indicated because the solution should not be administered.

The nurse is assessing a 75-year-old client for symptoms of hyperglycemia. Which symptom of hyperglycemia is an OLDER adult most likely to exhibit? A.Polyuria B.Polydipsia C.Weight loss D.Infection

D Signs and symptoms of hyperglycemia in older adults may include fatigue, infection (D), and evidence of neuropathy (e.g., sensory changes). The nurse needs to remember that classic signs and symptoms of hyperglycemia, such as (A, B, and C) and polyphagia, may be absent in older adults.

The nurse notes that the client's drainage has decreased from 50 to 5 mL/hr 12 hours after chest tube insertion for hemothorax. What is the best initial action for the nurse to take? A.Document this expected decrease in drainage. B.Clamp the chest tube while assessing for air leaks. C.Milk the tube to remove any excessive blood clot buildup. D.Assess for kinks or dependent loops in the tubing.

D The least invasive nursing action should be performed first to determine why the drainage has diminished (D). (A) is completed after assessing for any problems causing the decrease in drainage. (B) is no longer considered standard protocol because the increase in pressure may be harmful to the client. (C) is an appropriate nursing action after the tube has been assessed for kinks or dependent loops.

Which nursing action is necessary for the client with a flail chest?

D. Encourage coughing and deep breathing.

A nurse is performing assessment for a patient diagnosed with chronic obstructive pulmonary disease (COPD). Which of the following findings should the nurse expect to observe? A. Nonproductive cough B. Prolonged inspiration C. Vesicular breath sounds D. Increased anterior-posterior chest diameter

D. Increased anterior-posterior chest diameter

Which change in lab values indicates to the nurse that a client with rheumatoid arthritis may be experiencing an adverse effect of methotrexate (Mexate) therapy?

Decrease in hemoglobin

A 77-year-old female client is admitted to the hospital. She is confused and has had no appetite for several days. She has been nauseated and vomited several times prior to admission. She is currently complaining of a headache. Her pulse rate is 43 beats/min. The nurse is most concerned about the client's history related to what medication?

Digitalis (Lanoxin)

An adult resident in a long-term care facility is diagnosed with hepatitis B. Which intervention should the nurse implement with the staff caring for this client?

Discuss the importance of all employees starting the hepatitis B vaccine series.

The nurse is assessing a client who has tinea pedis. Which question will allow the nurse to gather further information about this condition?

Do you see any improvement when using tolnaftate?

A healthcare worker with no known exposure to tuberculosis has received a Mantoux tuberculosis skin test. The nurse's assessment of the test after 62 hours indicates 5 mm of erythema without induration. What is the best initial nursing action?

Document negative results in the client's medical record

The home health nurse is assessing a male client being treated for Parkinson disease with levodopa-carbidopa (Sinemet). The nurse observes that he does not demonstrate any apparent emotion when speaking and rarely blinks. Which intervention should the nurse implement?

Document the presence of these assessment findings.

What nursing action is necessary for the client with a flail chest?

Encourage coughing and deep breathing.

The nurse is providing care for a client who has had a cervical cord injury. Following reduction of the cervical fracture, a halo vest is placed to maintain realignment of the spinal canal. What intervention is needed to ensure client safety while the halo vest is in place?

Ensure that a set of wrenches are kept in close proximity

Which assessment finding would confirm the specific location of an enteral feeding tube?

Examination of portable radiograph taken after the tube was inserted

Which assessment finding in a client with an acute small bowel obstruction requires the most immediate intervention by the nurse?

Fever of 102° F

RN VS First assistant

First assistant can open and close an incision site

If a patient is taking Heparin infusion and Coumadin and diagnosed with heparin-induced thrombocytopenia (HIT). What are you as the nurse going to do?

Flush all the IV lines with normal saline

A client is being discharged following radioactive seed implantation for prostate cancer. What is the most important information for the nurse to provide this client's family?

Follow exposure precautions.

Which food is most important for the nurse to encourage a male client with osteomalacia to include in his daily diet?

Fortified milk and cereals

The nurse is administering a nystatin suspension (Mycostatin) for stomatitis. Which instruction will the nurse provide to the client when administering this medication?

Hold the medication in the mouth for a few minutes before swallowing it.

A female client with diabetes mellitus type 2 has a plantar foot ulcer. When developing a teaching plan regarding foot care, what information should the nurse obtain first from this client?

How the client examines her feet

A patient treated for vomiting for several days from an unknown cause is admitted to the hospital. The nurse anticipates collaborative care to include

IV replacement of fluid and electrolytes.

The nurse is receiving report from PACU about a client with a Penrose drain who is to be admitted to the surgical nursing unit. Before choosing a room for this client, which information is most important for the nurse to obtain?

If the client's wound is infected

Day of Surgery and Wedding Ring

If the patient does not want to remove their wedding ring if can be taped securely to the finger to prevent loss.

An emaciated homeless client presents to the emergency department complaining of a productive cough with blood-tinged sputum and night sweats. What action is most important for the emergency department triage nurse to implement for this client?

Initiate airborne infection precautions.

A client with draining skin lesions of the lower extremity is admitted with possible Methicillin-Resistant Staphylococcus Aureus (MRSA). Which nursing interventions should the nurse include in the plan on care? (Select all that apply.)

Institute contact precautions for staff and visitors. Send wound drainage for culture and sensitivity. Monitor the client's white blood cell count.

Metabolic Acidosis Respirations

Kussmaul respirations

Nursing management of the patient with chronic gastritis includes teaching the patient to

Maintain a non irritating diet with six small meals a day

Delegation of Blood transfusions

NAP can obtain blood products from the blood bank as directed by RN. Take VS before the transfusion and after the first 15 minutes.

Jaundice (History)

Need for patient teaching inf they take tylenol.

A female client who received a nephrotoxic drug is admitted with acute renal failure and asks the nurse if she will need dialysis for the rest of her life. The nurse should explain which pathophysiologic consequence that supports the need for temporary dialysis until acute tubular necrosis subsides?

Oliguria

In assessing a client diagnosed with primary aldosteronism, the nurse expects the laboratory test results to indicate a decreased serum level of which substance?

Potassium

A client with chronic kidney disease (CDK) arrives at the clinic reporting shortness of breath on exertion and extreme weakness. Vital signs are temperature 100.4 F (38 C), heart rate 110 beats/minute, respirations 28 breaths/minute, and blood pressure 175/98 mmHg. The client usually receives dialysis three times a week but missed the last treatment. STAT blood specimens are sent to the laboratory for analysis. Which laboratory results should the nurse report to the healthcare provider immediately?

Potassium 6.5 mEq/L (mmol/L)

A client with Guillain-Barre syndrome has paralysis of all extremities and requires mechanical ventilation. The nurse observes that the client is not blinking. Which action should the nurse implement?

Protect cornea with lubricant and eye shields

Which consideration is most important when the nurse is assigning a room for a client being admitted with progressive systemic sclerosis (scleroderma)?

Provide a room that can be kept warm.

A male client in skeletal traction tells the nurse that he is frustrated because he needs help repositioning himself in bed. Which intervention should the nurse implement?

Provide an overhead trapeze to the bed for the client to use

A client with non-insulin-dependent diabetes mellitus (NIDDM) takes metformin (Glucophage) daily. The client is scheduled for major surgery requiring general anesthesia the next day. The nurse anticipates the use of which approach to best manage the client's diabetes while the client is NPO during the perioperative period?

Regular insulin subcutaneously per sliding scale

The nurse determines that a client who arrives in the preoperative holding area before surgery is allergic to bananas. Which action should the nurse implement prior to taking the client into the operative area?

Replace latex-containing devices in the OR with alternate synthetic materials

A postoperative client receives a Schedule II opioid analgesic for pain. Which assessment finding requires the most immediate intervention by the nurse?

Respiratory rate of 12 per minute with O2 saturation of 85%

Which description of symptoms is characteristic of a client diagnosed with trigeminal neuralgia (tic douloureux)?

Sudden, stabbing, severe pain over the lip and chin

After attending a class on reducing cancer risk factors, a client selects bran flakes with 2% milk and orange slices from a breakfast menu. In evaluating the client's learning, the nurse affirms that the client has made good choices, and makes what additional recommendation?

Switch to skim milk.

A client with multiple sclerosis has urinary retention related to sensorimotor deficits. Which action should the nurse include in the client's plan of care?

Teach the client techniques for performing intermittent catheterization

A 55-year-old male client has been admitted to the hospital with a medical diagnosis of chronic obstructive pulmonary disease (COPD). Which risk factor is the most significant in the development of this client's COPD?

The client smokes 1 to 2 packs of cigarettes per day.

A 58-year-old client, who has no health problems, asks the nurse about taking the pneumococcal vaccine (Pneumovax). Which statement given by the nurse would offer the client accurate information about this vaccine?

The immunization is administered once to older adults or persons with a history of chronic illness."

Respiratory Assessment

The patient who smokes should be encouraged to stop at least 6 weeks preoperatively to decrease the risk of intraoperative and postoperative respiratory complications. Draw ABGs prior and after. Assess lung sounds.

A home health nurse knows that a 70-year-old male client who is convalescing at home following a hip replacement is at risk for developing decubitus ulcers. Which physical characteristic of aging contributes to such a risk?

Thinning of the skin with loss of elasticity

The presence of bronchovesicular breath sounds in the peripheral lung fields is described as?

abnormal lung sounds

Abdomen auscultation need further evaluation

absent

The primary advantage of the use of (Versed) as an adjunct to general anesthesia is its

amnestic effect

Because of rapid elimination of volatile liquids used for general anesthesia, the nurse should anticipate that early in the anesthesia recovery period, the patient will need

analgesic medication

Adventitous sounds

are extra breath sounds that are abnormal.

Wheezing

are short high pitched sounds made on expiratory

Patients fear of death

can be extremely detrimental. Attitude and emotional state influence the stress response, and thus the surgical outcome.

The strict vegetarian is at highest risk for the development of ?

cobalamin deficiency anemia

A patient has a platelet count of 50,000/uL and is diagnosed with immune thrombocytopenia purpura. The nurse anticipates that initial treatment will include

corticosteroids

Histamine (H2)-receptor antagonist (Pepcid)

decrease HCL acid secretion, increase pH, and decrease gastric volume. Taken to prevent stress ulcers

Benzodiazepines (Valium)

decrease anxiety, induce sedation, amnesic effects

NG tube removal after surgery

first priority is to ambulate

Palpate the liver

place your left hand behind the patient to support the right eleventh and twelfth ribs. Press the left hand forward and place the right hand on patients right abdomen later to the rectus muscle.

A family member was taught to suction a client's tracheostomy prior to the client's discharge from the hospital. Which observation by the nurse indicates that the family member is capable of correctly performing the suctioning technique? A. Turns on the continuous wall suction to -190 mm Hg B. Inserts the catheter until resistance or coughing occurs C. Withdraws the catheter while maintaining suctioning D. Re-clears the tracheostomy after suctioning the mouth

B. Inserts the catheter until resistance or coughing occurs

A 25-year-old client was admitted yesterday after a motor vehicle collision. Neurodiagnostic studies showed a basal skull fracture in the middle fossa. Assessment on admission revealed both halo and Battle signs. Which new symptom indicates that the client is likely to be experiencing a common life-threatening complication associated with basal skull fracture? A. Bilateral jugular vein distention. B. Oral temperature of 102 degrees F. C. Intermittent focal motor seizures. D. Intractable pain in the cervical region.

B. Oral temperature of 102 degrees F.

While teaching a patient with asthma about the appropriate use of a peak flow meter, the nurse instructs the patient to do which of the following? A. Use the flow meter each morning after taking medications to evaluate their effectiveness. B. Empty the lungs and then inhale quickly through the mouthpiece to measure how fast air can be inhaled. C. Keep a record of the peak flow meter numbers if symptoms of asthma are getting worse. D. Increase the doses of the long-term control medication if the peak flow numbers decrease.

C. Keep a record of the peak flow meter numbers if symptoms of asthma are getting worse.

During report, the nurse learns that a client with tumor lysis syndrome is receiving an IV infusion containing insulin. Which assessment should the nurse complete first? A. Review the client's history for diabetes mellitus. B. Observe the extremity distal to the IV site. C. Monitor the client's serum potassium and blood glucose. D. Evaluate the client's oxygen saturation and breath sounds.

C. Monitor the client's serum potassium and blood glucose.

A client with chronic asthma is admitted to postanesthesia complaining of pain at level 8 of 10, with a BP of 124/78, pulse of 88 beats/min, and respirations of 20 breaths/min. The postanesthesia recovery prescription is, "Morphine 2 to 4 mg IV push while in recovery for pain level over 5." What intervention should the nurse implement? A. Give the medication as prescribed to decrease the client's pain. B. Call the anesthesia provider for a different medication for pain. C. Use nonpharmacologic techniques before giving the medication. D. Reassess pain level in 30 Minutes and medicate if it remains elevated.

B. Call the anesthesia provider for a different medication for pain.

Which of the following is an indication of a complication of septic shock? 1. Anaphylaxis. 2. Acute respiratory distress syndrome (ARDS). 3. Chronic obstructive pulmonary disease (COPD). 4. Mitral valve prolapse.

2

A nurse has two middle-aged clients who have a prescription to receive a blood transfusion of packed red blood cells at the same time. The first client's blood pressure dropped from the preoperative value of 120/80 mm Hg to a postoperative value of 100/50. The second client is hospitalized because he developed dehydration and anemia following pneumonia. After checking the patency of their IV lines and vital signs, what should the nurse do next? 1. Call for both clients' blood transfusions at the same time. 2. Ask another nurse to verify the compatibility of both units at the same time. 3. Call for and hang the first client's blood transfusion. 4. Ask another nurse to call for and hang the blood for the second client.

4

A client with heart disease is on a continuous telemetry monitor and has developed sinus bradycardia. In determining the possible cause of the bradycardia, the nurse assesses the client's medication record. Which medication is most likely the cause of the bradycardia? A) Propanolol (Inderal). B) Captopril (Capoten). C) Furosemide (Lasix). D) Dobutamine (Dobutrex).

A

When teaching diaphragmatic breathing to a client with chronic obstructive pulmonary disease (COPD), which information should the nurse provide? A) Place a small book or magazine on the abdomen and make it rise while inhaling deeply. B) Purse the lips while inhaling as deeply as possible and then exhale through the nose. C) Wrap a towel around the abdomen and push against the towel while forcefully exhaling. D) Place one hand on the chest, one hand the abdomen and make both hands move outward.

A

Which nursing intervention best assists a bedridden client to keep skin intact? A) Use a lift sheet to move the client in bed. B) Turn the client every 2 to 4 hours. C) Use a foam mattress pad. D) Apply talcum powder to the perineal area.

A Friction forces are generated when the client is dragged or pulled across bed linen; this often leads to altered skin integrity. Using a lift sheet will prevent friction. Keeping the skin clean and dry is an important intervention, but powders should not be used in the perineal area. To minimize vasoconstriction and possible pressure ulcer development from dependency, the client should be turned at a minimum of every 2 hours. A foam mattress will not significantly decrease pressure to an area.

A client is diagnosed with an acute small bowel obstruction. Which assessment finding requires the most immediate intervention by the nurse? A.Fever of 102° F B.Blood pressure of 150/90 mm Hg C.Abdominal cramping D.Dry mucous membranes

A A sudden increase in temperature is an indicator of peritonitis. The nurse should notify the health care provider immediately (A). (B, C, and D) are also findings that require intervention by the nurse, but are of less priority than (A). (B) may indicate a hypertensive condition but is not as acute a condition as peritonitis. (C) is an expected finding in clients with small bowel obstruction and may require medication. (D) indicates probable fluid volume deficit, which requires fluid volume replacement.

A nurse working in a community health setting is performing primary health screenings. Which individual is at highest risk for contracting an HIV infection?

A 17-year-old who is sexually active with numerous partners

During change of shift report, the charge nurse reviews the infusions being received by clients on the oncology unit. The client receiving which infusion should be assessed first?

A continuous epidural infusion of morphine

After initial placement of NG tubes is confirmed, how often must placement be checked? SELECT ALL THAT APPLY? A) before medication administration B) it is not necessary to recheck placement C) every 4-8 hours during feeding D) before intermittent feeding E) according to facility policy

A,C,E

Which is most indicative of pain in an older client who is confused? (Select all that apply). A) Screaming B) Decreased blood pressure C) Crying D) Decreased respirations E) Facial grimace F) Restlessness

A,C,E,F No one scale has been found to be the best tool to use in pain assessment for adults with cognitive impairment. Facial expression, motor behavior, mood, socialization, and vocalization are common indicators of pain in cognitively impaired adults. In acute pain, nonverbal indicators of pain could include increased blood pressure and respirations.

Which of the following test results identify that a patient with an asthma attack is responding to treatment? A. A decreased exhaled nitric oxide B. An increase in CO2 levels C. A decrease in white blood cell count D. An increase in serum bicarbonate levels

A. A decreased exhaled nitric oxide.

When initially teaching a patient the supraglottic swallow following a radical neck dissection, with which of the following foods should the nurse begin? A. Cola B. Applesauce C. French fries D. White grape juice

A. Cola

The nurse is collecting information from a client with chronic pancreatitis who reports persistent gnawing abdominal pain. To help the client manage the pain, which assessment data is most important for the nurse to obtain? A. Eating patterns and dietary intake B. Level and amount of physical activity C. Color and consistency of feces D. Presence and activity of bowel sounds

A. Eating patterns and dietary intake

A client with Cushing's syndrome is recovering from an elective laparoscopic procedure. Which assessment finding warrants immediate intervention by the nurse? A. Irregular apical pulse B. Purple marks on the skin of the abdomen C. Quarter size blood spot on dressing D. Pitting ankle edema

A. Irregular apical pulse

A 63-year-old client with type 2 diabetes mellitus is admitted for treatment of an ulcer on the heel of the left foot that has not healed with wound care. The nurse observes that the entire left foot is darker in color than the right foot. Which additional symptom should the nurse expect to find?

A. Pedal pulses will be weak or absent in the left foot.

To promote airway clearance in a patient with pneumonia, the nurse instructs the patient to do which of the following? (Select all that apply.) A. Splint the chest when coughing B. Maintain a semi-Fowler's position C. Maintain adequate fluid intake D. Instruct patient to cough at end of exhalation

A. Splint the chest when coughing C. Maintain adequate fluid intake D. Instruct patient to cough at end of exhalation

The nurse is responding to telephone messages at a psychiatric day clinic. Which client situation requires immediate intervention by the nurse? A. The wife of a client with post-traumatic stress syndrome reports that her husband is threatening to kill her. B. A client with depression who is crying and tells the nurse that he has has suicidal thoughts C. A young adult diagnosed with a somatoform disorder reports having a severe headache that has become unbearable D. An adult heroin abuser who reports the onset of withdrawal and requests a refill for a prescription for methadone

A. The wife of a client with post-traumatic stress syndrome reports that her husband is threatening to kill her.

A nurse is providing care to an adult female patient and observes that the Hb laboratory analysis result is 9 g/dl. Based on this finding, the nurse should expect to observe A. dyspnea. B. bradycardia. C. warm, dry skin. D. activity tolerance without complaint of fatigue.

A. dyspnea.

Which intervention should the nurse implement for a female client diagnosed with pelvic relaxation disorder? A) Describe proper administration of vaginal suppositories and cream. B) Encourage the client to perform Kegel exercises 10 times daily. C) Explain the importance of using condoms when having sexual intercourse. D) Discuss the importance of keeping a diary of daily temperature and menstrual cycle events.

B

The nurse admitting a patient to the emergency department on a very hot summer day would suspect hyperthermia when the patient demonstrates: A) slow capillary refill. B) red, sweaty skin. C) low pulse rate. D) decreased respirations.

B With hyperthermia, vasodilatation occurs causing the skin to appear flushed and warm or hot to touch. There is an increased respiration rate with hyperthermia. The heart rate increases with hyperthermia. With hypothermia there is slow capillary refill.

Which abnormal laboratory finding indicates that a client with diabetes needs further evaluation for diabetic nephropathy? A.Hypokalemia B.Microalbuminuria C.Elevated serum lipid levels D.Ketonuria

B Microalbuminuria (B) is the earliest sign of diabetic nephropathy and indicates the need for follow-up evaluation. Hyperkalemia, not (A), is associated with end-stage renal disease caused by diabetic nephropathy. (C) may be elevated in end-stage renal disease. (D) may signal the onset of diabetic ketoacidosis (DKA).

Which information about mammograms is most important to provide a post-menopausal female client? A) Breast self-examinations are not needed if annual mammograms are obtained. B) Radiation exposure is minimized by shielding the abdomen with a lead-lined apron. C) Yearly mammograms should be done regardless of previous normal x-rays. D) Women at high risk should have annual routine and ultrasound mammograms.

C

The nurse notes a venous ulcer on the client's left ankle. What additional assessment finding does the nurse expect in this client? A) Absence of hair on the left lower extremity B) Skin surrounding the ulcer mottled but blanchable C) Brownish discoloration of the lower extremity D) Cold and gray-blue lower extremity

C Venous ulcers are characterized by brown pigmentation of the skin of the lower extremity. Mottled skin, the presence of dependent rubor, and cyanosis are features of arterial ulcers.

A client is ready for discharge following the creation of an ileostomy. Which instruction should the nurse include in discharge teaching? A.Replace the stoma appliance every day. B.Use warm tap water to irrigate the ileostomy. C.Change the bag when the seal is broken. D.Measure and record the ileostomy output.

C A seal must be maintained to prevent leakage of irritating liquid stool onto the skin (C). (A) is excessive and can cause skin irritation and breakdown. Ileostomies produce liquid fecal drainage, so (B) is not necessary. (D) is not needed.

During the change of shift report, the charge nurse reviews the infusions being received by the clients on the oncology unit. The client receiving which infusion should be seen first?

C. A continuous epidural infusion of morphine

A client taking tamoxifen citrate following a lumpectomy reports several problems to the nurse. It is most important for the nurse to follow-up on which reported problem? A. Erratic menstrual periods B. Bone pain C. Calf tenderness D. Anorexia and nausea

C. Calf tenderness

When a nurse assesses a client receiving total parenteral nutrition (TPN), which laboratory value is most important for the nurse to monitor regularly?

C. Glucose

What type of anemia is associated with folate deficiency? A. Microcytic B. Pernicious C. Megaloblastic D. Iron deficiency

C. Megaloblastic

A client is ready for discharge following creation of an ileostomy. Which instruction should the nurse include in discharge teaching?

Change the bag when the seal is broken.

After a transurethral resection of the prostate (TURP), a client has bloody urine output with large clots. The nurse implements the postoperative prescription to irrigate the indwelling catheter PRN to maintain the catheter's patency. Which action should the nurse implement?

Clamp the catheter for 30 minutes prior to irrigating with saline

A client experiencing uncontrolled atrial fibrillation is admitted to the telemetry unit. What initial medication should the nurse anticipate administering to the client? A) Xylocaine (Lidocaine). B) Procainamide (Pronestyl). C) Phenytoin (Dilantin). D) Digoxin (Lanoxin).

D

What instruction should the nurse give a client who is diagnosed with fibrocystic changes of the breast? A) Observe cyst size fluctuations as a sign of malignancy. B) Use estrogen supplements to reduce breast discomfort. C) Notify the healthcare provider if whitish nipple discharge occurs. D) Perform a breast self-exam (BSE) procedure monthly.

D

A client with type 2 diabetes takes metformin (Glucophage) daily. The client is scheduled for major surgery requiring general anesthesia the next day. The nurse anticipates which approach to manage the client's diabetes best while the client is NPO during the perioperative period? A.NPO except for metformin and regular snacks B.NPO except for oral antidiabetic agent C.Novolin N insulin subcutaneously twice daily D.Regular insulin subcutaneously per sliding scale

D Regular insulin dosing based on the client's blood glucose levels (sliding scale) is the best method to achieve control of the client's blood glucose while the client is NPO and coping with the major stress of surgery (D). (A) increases the risk of vomiting and aspiration. (B and C) provide less precise control of the blood glucose level.

An older client is admitted with a diagnosis of bacterial pneumonia. Which symptom should the nurse report to the health care provider after assessing the client? A.Leukocytosis and febrile B.Polycythemia and crackles C.Pharyngitis and sputum production D.Confusion and tachycardia

D The onset of pneumonia in the older client may be signaled by general deterioration, confusion, increased heart rate, and/or increased respiratory rate (D). (A, B, and C) are often absent in the older client with bacterial pneumonia.

In the case of pulmonary embolus from deep vein thrombosis, which of the following actions should the nurse take first? A. Notify the physician. B. Administer a nitroglycerin tablet sublingually. C. Conduct a thorough assessment of the chest pain. D. Sit the patient up in bed as tolerated and apply oxygen.

D. Sit the patient up in bed as tolerated and apply oxygen.

A patient's ABGs include a PaO2 of 88 mm Hg and a PaCO2 of 38 mm Hg and mixed venous blood gases include a PvO2 of 40 mm Hg and PvCO2 of 46 mm Hg. These findings indicate that the patient has A. impaired cardiac output. B. unstable hemodynamics. C. inadequate delivery of oxygen to the tissues. D. normal capillary oxygen-carbon dioxide exchange.

D. normal capillary oxygen-carbon dioxide exchange.

A female client who received partial-thickness and full-thickness burns over 40% of her body in a house fire is admitted to the inpatient burn unit. What fluid should the nurse prepare to administer during the acute phase of the client's burn recovery?

Ringer's Lactate

Which statement reflects the highest priority nursing diagnosis for an older client recently admitted to the hospital for a new-onset cardiac dysrhythmia?

Risk for injury related to syncope and confusion

Hypothermia

SPO2 96%. What is the nurse going to do? Turn oxygen up.

Scrub nurse sterile activities

Scrubs, gowns, and gloves self and other members of surgical team. Prepares instruments. Counts sponges, needles, and instruments. Monitors practices of aseptic technique in self and others.

The nurse teaches the patient with a hiatal hernia or GERD to control symptoms by

Sleeping with the head of the bed elevated on 4 to 6 inch blocks

The nurse observes ventricular fibrillation on telemetry and upon entering the client's bathroom finds the client unconscious on the floor. What intervention should the nurse implement first?

Start cardiopulmonary resuscitation

While receiving a unit of packed RBCs, the patient develops chills and a temperature of 102.2 F. The priority action for the nurse to take is

Stop the transfusion and removes the IV catheter

When preparing a male client who has had a total laryngectomy for discharge, what instruction would be most important for the nurse to include in the discharge teaching?

Tell the client to carry a medic alert card that explains his condition.

A male client complains of pain in his right calf, and the nurse determines that his calf is edematous and deep red. What intervention has the highest priority?

Tell the client to remain in bed

A hospitalized male client is receiving nasogastric tube feedings via a small-bore tube and a continuous pump infusion. He begins to cough and produces a moderate amount of white sputum. Which action should the nurse take first?

Turn off the continuous feeding pump.

In evaluating the effects of lactulose (Cephulac), which outcome would indicate that the drug is performing as intended?

Two or three soft stools per day

A nurse is assisting an 82-year-old client with ambulation and is concerned that the client may fall. What area contains the older person's center of gravity?

Upper torso

To prepare the patient for a thoracentesis, the nurse positions the patient

sitting upright with the elbows on an over-the-bed table.

Non Drug Allergies

specifically food and environmental allergies such as latex. You want to make sure they do not use latex prior to surgery and schedule them first for surgery so no latex dust in the air.

A male client has undergone insertion of a permanent pacemaker. When developing a discharge teaching plan, the nurse writes a goal of, "The client will verbalize symptoms of pacemaker failure." Which behavior indicates that the goal has been met? The client

states that changes in the pulse and feelings of dizziness are significant changes.

While receiving a unit of packed RBCs. the patient develops chills and a temperature of 102.2F. The priority action for the nurse to take is?

stop the transfusion and removes the IV catheter.

The nurse is giving preoperative instructions to a 14-year-old female client scheduled for surgery to correct a spinal curvature. Which statement by the client best demonstrates learning has taken place?

"Let me show you the method of turning I will use after surgery."

A client with peripheral vascular disease returns to the surgical care unit after having femoral-popliteal bypass grafting. Indicate in which order the nurse should conduct assessment of this client. 1. Postoperative pain. 2. Peripheral pulses. 3. Urine output. 4. Incision site.

(2,4,3,1) Because assessment of the presence and quality of the pedal pulses in the affected extremity is essential after surgery to make sure that the bypass graft is functioning, this step should be done first. The nurse should next ensure that the dressing is intact, and then that the client has adequate urine output. Lastly, the nurse should determine the client's level of pain.

A client with type 1 diabetes mellitus has influenza. The nurse should instruct the client to: 1. Increase the frequency of self-monitoring (blood glucose testing). 2. Reduce food intake to diminish nausea. 3. Discontinue that dose of insulin if unable to eat. 4. Take half of the normal dose of insulin

1 Colds and influenza present special challenges to the client with diabetes mellitus because the body's need for insulin increases during illness. Therefore, the client must take the prescribed insulin dose, increase the frequency of blood glucose testing, and maintain an adequate fluid intake to counteract the dehydrating effect of hyperglycemia. Clear fluids, juices, and Gatorade are encouraged. Not taking insulin when sick, or taking half the normal dose, may cause the client to develop ketoacidosis.

While administering and infusion of packed RBCs. the RN may delegate which of the following actions to nursing assistive personnel

1. Obtain blood products from the bank and 2. Obtain VS before and after the first 15 minutes.

Which of the following indicates a potential complication of diabetes mellitus? 1. Inflamed, painful joints. 2. Blood pressure of 160/100 mm Hg. 3. Stooped appearance. 4. Hemoglobin of 9 g/dL (90 g/L).

2 The client with diabetes mellitus is especially prone to hypertension due to atherosclerotic changes, which leads to problems of the microvascular and macrovascular systems. This can result in complications in the heart, brain, and kidneys. Heart disease and stroke are twice as common among people with diabetes mellitus as among people without the disease. Painful, inflamed joints accompany rheumatoid arthritis. A stooped appearance accompanies osteoporosis with narrowing of the vertebral column. A low hemoglobin concentration accompanies anemia, especially iron deficiency anemia and anemia of chronic disease.

A client with peripheral vascular disease has undergone a right femoral-popliteal bypass graft. The blood pressure has decreased from 124/80 to 94/62. What should the nurse assess first? 1. IV fluid solution. 2. Pedal pulses. 3. Nasal cannula flow rate. 4. Capillary refill

2 With each set of vital signs, the nurse should assess the dorsalis pedis and posterior tibial pulses. The nurse needs to ensure adequate perfusion to the lower extremity with the drop in blood pressure. IV fluids, nasal cannula setting, and capillary refill are important to assess; however, priority is to determine the cause of drop in blood pressure and that adequate perfusion through the new graft is maintained.

A client is brought to the Emergency Center after a snow-skiing accident. Which intervention is most important for the nurse to implement? A) Review the electrocardiogram tracing. B) Obtain blood for coagulation studies. C) Apply a warming blanket. D) Provide heated PO fluids.

A

After diagnosis and initial treatment of a 3-year-old child with cystic fibrosis, the nurse provides home care instruction to the mother. Which statement by the child's mother indicates that she understands home care treatment to promote pulmonary functions? A. "Chest physiotherapy should be performed twice a day before a meal." B. "Administer a cough suppressant every 8 hours.' C. "Maintain supplemental oxygen at 4 to 6 L/minute." D. "Energy should be conserved by scheduling minimally strenuous activities."

A. "Chest physiotherapy should be performed twice a day before a meal."

An older male client comes to the geriatric screening clinic complaining of pain in his left calf. The nurse notices a reddened area on the calf of his right leg that is warm to the touch and the nurse suspects that the client may have thrombophlebitis. Which additional assessment is most important for the nurse to perform?

Auscultate the client's breath sounds.

Client census is often used to determine staffing needs. Which method of obtaining census determination for a particular unit provides the best formula for determining long-range staffing patterns?

Average daily census

A 43-year-old homeless, malnourished female client with a history of alcoholism is transferred to the ICU. She is placed on telemetry, and the rhythm strip shown is obtained. The nurse palpates a heart rate of 160 beats/min, and the client's blood pressure is 90/54. Based on these finding, which IV medication should the nurse administer? A. Amiodarone (Cordarone) B. Magnesium sulfate C. Lidocaine (Xylocaine) D. Procainamide (Pronestyl)

B. Magnesium sulfate

Which abnormal lab finding indicates that a client with diabetes needs further evaluation for diabetic nephropathy? A. Hypokalemia B. Microalbuminauria C. Elevated serum lipids D. Ketonuria

B. Microalbuminuria

A nurse is reviewing the hematologic test results for a patient in whom the hematocrit (Hct) is reported at a reading of 30%. Based on this result, the nurse should interpret that the patient A. is susceptible to bleeding disorders. B. has fewer red blood cells than normal. C. is experiencing an inflammatory response. D. is experiencing an acute hemolytic crisis.

B. has fewer red blood cells than normal.

After Procedure colonoscopy

Be aware patient may experience abdominal cramping. Observe for rectal bleeding and signs of perforation (e.g., malaise, abdominal distension, tenesmus). Check vital signs.

A client with pheochromocytoma reports the onset of a severe headache. The nurse observes that the client is very diaphoretic. Which assessment data should the nurse obtain next?

Blood pressure

Before procedure colonoscopy

Bowel preparation is done. May be on clear liquid diet for 1-2 days. Enema given the night before. 1 gallon of GoLYTELY evening before. Explain the that the flexible scope will be inserted in side lying position. Sedation will be given.

A 77-year-old female client is admitted to the hospital. She is confused, has no appetite, is nauseated and vomiting, and is complaining of a headache. Her pulse rate is 43 beats per minute. Which question is a priority for the nurse to ask this client or her family on admission? "Does the client A) have her own teeth or dentures?" B) take aspirin and if so, how much?" C) take nitroglycerin?" D) take digitalis?"

D

Twelve hours after chest tube insertion for hemothorax, the nurse notes that the client's drainage has decreased from 50 ml/hr to 5 ml/hr. What is the best inital action for the nurse to take? A. Document this expected decrease in drainage. B. Clamp the chest tube while assessing for air leaks. C. Milk the tube to remove any excessive blood clot build up. D. Assess for kinks or dependent loops in the tubing.

D. Assess for kinks or dependent loops in the tubing.

The nurse who works in labor and delivery is reassigned to the cardiac care unit for the because of a low census in labor and delivery. Which assignment is best for the charge nurse to give this nurse? A. Transfer a client to another hit B. Monitor the central telemetry C. Perform the admission of a new client D. Assis cardiac nurse with their assignments

D. Assis cardiac nurse with their assignments

A client with cirrhosis develops increasing pedal edema and ascites. Which dietary modification is most important for the nurse to teach this client?

D. Restrict salt and fluid intake.

To prevent atelectasis in an 82-year-old patient with a hip fracture, a nurse should A. supply oxygen. B. suction the upper airway. C. ambulate the patient frequently. D. assist the patient with aggressive coughing and deep breathing.

D. assist the patient with aggressive coughing and deep breathing.

A client with type 2 diabetes mellitus (DM) is admitted to the hospital for uncontrolled DM. Insulin therapy is initiated with an initial dose of Humulin N insulin at 0800. At 1600, the client complains of diaphoresis, rapid heartbeat, and feeling shaky. What should the nurse do first?

Determine the client's current glucose level

What is the correct location for placement of the hands for manual chest compressions during cardiopulmonary resuscitation (CPR) on the adult client?

Just above the xiphoid process on the lower third of the sternum

Which abnormal lab finding indicates that a client with diabetes needs further evaluation for diabetic nephropathy?

Microalbuminuria

A 62-year-old woman who lives alone tripped on a rug in her home and fractured her hip. Which predisposing factor most likely contributed to the fracture in the proximal end of her femur?

Osteoporosis resulting from declining hormone levels

When somebody is taking Heparin what test are you going to use to monitor it?

PTT

Hypoxemia

PaO2 is at 55% and SpO2 is 88

An emaciated homeless client presents to the emergency department complaining of a productive cough, with blood-tinged sputum and night sweats. Which action is most important for the emergency department triage nurse to implement for this client? A.Initiate airborne infection precautions. B.Place a surgical mask on the client. C.Don an isolation gown and latex gloves. D.Start protective (reverse) isolation precautions.

This client is exhibiting classic symptoms of tuberculosis (TB), and the client is from a high-risk population for TB. Therefore, airborne infection precautions (A), which are indicated for TB, should be used with this client. (B) is used with DROPLET precautions. There is no evidence that (C or D) would be warranted at this time.

A client with unstable asthma had an emergent cardiac catheterization. Which complication should the nurse monitor for in the initial 24 hours after the procedure?

Thrombus formation

A patient who has had a transfusion-related acute lung injury (TRALI) is to receive a transfusion of packed red blood cells (PRBCs). How do you reduce the reaction?

Transfuse only leukocyte-reduced PRBCs.

Thoracentesis What Position you going to place them in?

Upright leaning over table to increase lung expansion

A patients tracheostomy tube becomes dislodged with vigorous coughing. The first action by the nurse is to?

attempt to replace the tube

Hearing aides and glasses to surgical suite

bring to the nurses desk so when the patient awakes he will not feel as disorientated without having his glasses and hearing aides.

Anticholinergics (atropine)

decrease oral and respiratory secretions

Symptom of primary pulmonary hypertension

dyspnea with exertion.

Sedative medication prior to surgery

given by the anesthesiologist

In caring for a client with acute diverticulitis, which assessment data warrant immediate nursing intervention? The client

has a rigid hard abdomen and elevated white blood cell count (WBC).

To determine when the patient with a tracheostomy tube can be effectively swallow, the nurse deflates the cuff and?

has the patient drink a small amount of blue-colored water, observing for coughing and colored secretions.

Peritonsillar abscess

is a complication of acute pharyngitis or acute tonsillitis when bacterial infection invades one or both tonsils. The tonsil may enlarge sufficiently to threaten airway patency. Also known as the hot potato voice. The patient experience a high fever, leukocytosis, and chills.

A male client has just undergone a laryngectomy and has a cuffed tracheostomy tube in place. When initiating bolus tube feedings postoperatively, when should the nurse inflate the cuff?

just prior to tube feeding

A client with Parkinson disease is taking carbidopa-levodopa (Sinemet). Which observation by the nurse would indicate that the desired outcome of the medication is being achieved?

lessing of tremors

Coarse Crackles

long duration, discontinuous, low pitched sounds caused by air passing through airway intermittently occluded by mucus, unstable bronchial wall, or fold of mucosa; evident with inspiration. COPD, heart failure, and pneumonia.

Following a patient's esophagogastrostomy for cancer of the esophagus, it is important for the nurse to

maintain the patient in sem-fowler's or fowler's position

Saw Palmetto

may have additive effects with other hormone therapies.

Garlic

may increase bleeding, especially in patients taking anticoagulants

Absent breath sounds

no sound evident over entire lung or area of lung. Pleural effusion or lobectomy

T-tube gallbladder

normal color is green bile

A 25-year-old client was admitted yesterday after a motor vehicle collision. Neurodiagnostic studies showed a basal skull fracture in the middle fossa. Assessment on admission revealed both halo and Battle signs. Which new symptom indicates that the client is likely to be experiencing a common life-threatening complication associated with a basal skull fracture?

oral temp 102 F

Normal Arterial Blood Gases

pH 7.35-7.45, PaO2 80-100mm Hg, SaO2 >95%, PaCO2 32-48 mm Hg, HCO3- 22-26 mEq/L

How anethetist puts somebody to sleep

picture a relaxing place and push medications through the IV.

To promote effective coughing, deep breathing, and ambulation in the postoperative patient, it is most important for the nurse to

provide adequate and regular pain medication.

A female client with a nasogastric tube attached to low suction states that she is nauseated. The nurse assesses that there has been no drainage through the nasogastric tube in the last 2 hours. What action should the nurse take first?

reposition the patient on her side

A client with cirrhosis develops increasing pedal edema and ascites. What dietary modification is most important for the nurse to teach this client?

restrict salts and fluid intakes

The nurse receives the client's next scheduled bag of total parental nutrition (TPN) labeled with the additive NPH insulin. What action should the nurse implement?

return solution to pharmacy

Before administering a bolus of intermittent tube feeding to a patient with percutaneous endoscopic gastrostomy (PEG), the nurse aspirates 220mL of gastric contents. The nurse should

return the aspirate to the stomach and continue with the tube feeding as planned.

he nurse includes frequent oral care in the plan of care for a client scheduled for an esophagogastrostomy for esophageal cancer. This intervention is included in the client's plan of care to address which nursing diagnosis?

risk for infection

A patient with an SaO2 of 85% has a PaO2 of 50mm Hg. This indicates?

shift to the left in the oxygen-hemoglobin dissociation curve that could be caused by hypothermia.

Fine Crackles

short-duration, discontinuous, high pitched sounds heard just before the end of inspiration. In pneumonia or heart failure.

A break in sterile technique during surgery would occur then the scrub nurse touches

the mask with gloved hands

RN delegate to a surgical technologist

the surgical technologist can pass instruments to the doctor

Blood Glucose prior to surgery

to check their metabolic status, blood sugar, and their diabetes mellitus. Determine if insulin needs to be given prior to surgery.

The nurse evaluates that patient teaching about a high-calorie, high protein diet has been effective when the patient selects for breakfast from the hospital menu

two poached eggs, hash brown potatoes, and whole milk

Abnormal Breathe sounds

used to describe bronchial or bronchovesicular sounds heard in the peripheral lung fields.

Diagnose Pulmonary Emboli

with at CAT scan or MRI

A client with hypertension has been receiving ramipril (Altace) 5 mg PO daily for 2 weeks and is scheduled to receive a dose at 0900. At 0830 the client's blood pressure is 120/70. Which action should the nurse take? A. Administer the dose as prescribed. B. Hold the dose and contact the healthcare provider. C. Hold the dose and recheck the blood pressure in 1 hour. D. Check the healthcare provider's prescription to clarify the dose.

A. Administer the dose as prescribed.

In assessing an older client with dementia for sundowning syndrome, what assessment technique is best for the nurse to use? A. Observe for tiredness at the end of the day. B. Perform a neurologic exam and mental status exam. C. Monitor for medication side effects. D. Assess for decreased gross motor movement.

A. Observe for tiredness at the end of the day.

The nurse reviews pursed lip breathing with a patient newly diagnosed with emphysema. The nurse reinforces that this technique will assist respiration by which of the following mechanisms? A. Preventing bronchial collapse and air trapping in the lungs during exhalation B. Increasing the respiratory rate and giving the patient control of respiratory patterns C. Loosening secretions so that they may be coughed up more easily D. Promoting maximal inhalation for better oxygenation of the lungs

A. Preventing bronchial collapse and air trapping in the lungs during exhalation

The nurse is caring for a patient with an acute exacerbation of asthma. Following initial treatment, which of the following findings indicates to the nurse that the patient's respiratory status is improving? A. Wheezing becomes louder B. Vesicular breath sounds decrease C. Aerosol bronchodilators stimulate coughing D. The cough remains nonproductive

A. Wheezing becomes louder

The nurse is teaching a patient how to self-administer ipratropium (Atrovent) via a metered dose inhaler. Which of the following instructions given by the nurse is most appropriate to help the patient learn proper inhalation technique? A. "Avoid shaking the inhaler before use." B. "Breathe out slowly before positioning the inhaler." C. "After taking a puff, hold the breath for 30 seconds before exhaling." D. "Using a spacer should be avoided for this type of medication."

B. "Breathe out slowly before positioning the inhaler."

An older female client with dementia is transferred from a long term care unit to an acute care unit. The client's children express concern that their mother's confusion is worsening. How should the nurse respond? A. "It is to be expected that older people will experience progressive confusion." B. "Confusion in an older person often follows relocation to new surroundings." C. "The dementia is progressing rapidly, but we will do everything we can to keep your mother safe." D. "The acute care staff is not as experienced as the long-term care staff at dealing with dementia."

B. "Confusion in an older person often follows relocation to new surroundings."

A patient has been receiving oxygen per nasal cannula while hospitalized for COPD. The patient asks the nurse whether oxygen use will be needed at home. Which of the following would be the most appropriate response by the nurse? A. "Long-term home oxygen therapy should be used to prevent respiratory failure." B. "Oxygen will be needed when your oxygen saturation drops to 88% and you have symptoms of hypoxia." C. "Long-term home oxygen therapy should be used to prevent heart problems related to emphysema." D. "Oxygen will not be needed until or unless you are in the terminal stages of this disease."

B. "Oxygen will be needed when your oxygen saturation drops to 88% and you have symptoms of hypoxia."

Debilitating anginal pain can be decreased in some clients by the administration of beta-blocking agents such as nadolol (Corgard). Which client requires the nurse to use extreme caution when administering Corgard? A. A 56-year-old air traffic controller who had bypass surgery 2 years ago. B. A 47-year-old kindergarten teacher diagnosed with asthma 40 years ago C. A 52-year-old unemployed stock broker who refuses treatment for alcoholism D. A 60-year-old retired librarian who takes a diuretic daily for hypertension.

B. A 47-year-old kindergarten teacher diagnosed with asthma 40 years ago

When assigning clients on a medical-surgical floor to a RN and a LPN, it is best for the charge nurse to assign which client to the LPN? A. A child with bacterial meningitis with recent seizures. B. An older adult client with pneumonia and viral meningitis. C. A female client in isolation wiht meningococcal meningitis. D. A male client 1 day post-op after drainage of a brain abscess.

B. An older adult client with pneumonia and viral meningitis.

An older male client comes to the geriatric screening clinic complaining of pain in his left calf. The nurse notices a reddened area on the calf of his right leg that is warm to touch and the nurse suspects that the client may have thrombophlebitis. Which addition assessment is most important for the nurse to perform? A. Measure calf circumference. B. Auscultate the client's breath sounds. C. Observe for ecchymosis and petechiae. D. Obtain the client's blood pressure.

B. Auscultate the client's breath sounds.

The nurse is planning the care for a client who is admitted with the syndrome of inappropriate antidiuretic hormone secretion (SIADH). Which interventions should the nurse include in this client's plan of care? (Select all that apply.) A. Salt-free diet B. Quiet environment C. Deep tendon reflex assessments D. Neurologic checks E. Daily weights F. Unrestricted intake of free water

B. Quiet environment C. Deep tendon reflex assessments D. Neurologic checks E. Daily weights

A female client with a nasogastric tube attached to low suction states that she is nauseated. The nurse assesses that there has been no drainage through the nasogastric tube in the last 2 hours. Which action should the nurse take first? A. Irrigate the nasogastric tube with sterile normal saline. B. Reposition the client on her side. C. Advance the nasogastric tube 5 cm. D. Administer an intravenous antiemetic as prescribed.

B. Reposition the client on her side

The nurse observes ventricular fibrillation on telemetry and upon entering the clients bathroom finds the client unconscious on the floor. What intervention should the nurse implement first? A. Administer an antidysrhythmic medication. B. Start cardiopulmonary resuscitation. C. Defibrillate the client at 200 joules. D. Assess the client's pulse oximetry.

B. Start cardiopulmonary resuscitation.

Which description of symptoms is characteristic of a client with diagnosed with trigeminal neuralgia (tic douloureux)? A. Tinnitus, vertigo, and hearing difficulties. B. Sudden, stabbing, severe pain over the lip and chin. C. Unilateral facial weakness and paralysis. D. Difficulty in talking, chewing, and swallowing.

B. Sudden, stabbing, severe pain over the lip and chin.

The nurse is assigned to care for a patient in the emergency department admitted with an exacerbation of asthma. The patient has received a β-adrenergic bronchodilator and supplemental oxygen. If the patient's condition does not improve, the nurse should anticipate which of the following is likely to be the next step in treatment? A. Pulmonary function testing B. Systemic corticosteroids C. Biofeedback therapy D. Intravenous fluids

B. Systemic corticosteroids

The nurse evaluates that discharge teaching for a patient hospitalized with pneumonia has been most effective when the patient states which of the following measures to prevent a relapse? A. "I will increase my food intake to 2400 calories a day to keep my immune system well." B. "I must use home oxygen therapy for 3 months and then will have a chest x-ray to reevaluate." C. "I will seek immediate medical treatment for any upper respiratory infections." D. "I should continue to do deep-breathing and coughing exercises for at least 6 weeks."

D. "I should continue to do deep-breathing and coughing exercises for at least 6 weeks."

The nurse determines that the patient understood medication instructions about the use of a spacer device when taking inhaled medications after hearing the patient state which of the following as the primary benefit? A. "Now I will not need to breathe in as deeply when taking the inhaler medications." B. "This device will make it so much easier and faster to take my inhaled medications." C. "I will pay less for medication because it will last longer." D. "More of the medication will get down into my lungs to help my breathing."

D. "More of the medication will get down into my lungs to help my breathing."

The nurse witnesses a baseball player receive a blunt trauma to the back of the head with a softball. What assessment data should the nurse collect immediately? A. Reactivity of deep tendon reflexes, comparing upper to lower extremities. B. Vital signs readings, excluding blood pressure if need equipment is unavailable. C. Memory of events that occurred before and after the blow to the head. D. Ability to spontaneously open the eyes before any tactile stimuli are given.

D. Ability to spontaneously open the eyes before any tactile stimuli are given.

A client who is receiving an ACE inhibitor for hypertension calls the clinic and reports the recent onset of a cough to the nurse. What action should the nurse implement? A. Advise the client to come to the clinic immediately for further assessment. B. Instruct the client to discontinue use of the drug, and make an appointment at the clinic. C. Suggest that the client lear to accept the cough as a side effect to a necessary prescription. D. Encourage the client to keep taking the drug until seen by the HCP.

D. Encourage the client to keep taking the drug until seen by the HCP.

When caring for a patient with COPD, the nurse identifies a nursing diagnosis of imbalanced nutrition less than body requirements after noting a weight loss of 30 lb. Which of the following would be an appropriate intervention to add to the plan of care for this patient? A. Teach the patient to use frozen meals at home that can be microwaved. B. Provide a high-calorie, high-carbohydrate, nonirritating, frequent feeding diet. C. Order fruits and fruit juices to be offered between meals. D. Order a high-calorie, high-protein diet with six small meals a day.

D. Order a high-calorie, high-protein diet with six small meals a day.

The nurse is assisting a patient to learn self-administration of beclomethasone two puffs inhalation q6hr. The nurse explains that the best way to prevent oral infection while taking this medication is to do which of the following as part of the self-administration techniques? A. Chew a hard candy before the first puff of medication. B. Ask for a breath mint following the second puff of medication. C. Rinse the mouth with water before each puff of medication. D. Rinse the mouth with water following the second puff of medication.

D. Rinse the mouth with water following the second puff of medication.

The nurse is assessing a client who presents with jaundice. Which assessment finding is the most significant indication that further follow up is needed? A. Urine specific gravity of 1.03 with a urine output of 500 ml in 8 hours B. Frothy, tea-colored urine C. Clay-colored stools and complaints of pruritus D. Serum amylase and lipase levels that are twice their normal levels

D. Serum amylase and lipase levels that are twice their normal levels

While ambulating a patient with metastatic lung cancer, the nurse observes a drop in oxygen saturation from 93% to 86%. Which of the following nursing interventions is most appropriate based upon these findings? A. Continue with ambulation as this is a normal response to activity. B. Move the oximetry probe from the finger to the earlobe for more accurate monitoring during activity. C. Obtain a physician's order for supplemental oxygen to be used during ambulation and other activity. D. Obtain a physician's order for arterial blood gas determinations to verify the oxygen saturation.

C. Obtain a physician's order for supplemental oxygen to be used during ambulation and other activity.

The nurse know that normal lab values expected for an adult may vary in an older client. Which data would the nurse expect to find when reviewing laboratory values of an 80-year-old man who is in good overall health. A. Complet blood count reveals increased WBC and decreased RBC counts. B. Chemistries reveal an increased serum bilirubin with slightly increased liver enzymes. C. Urinalysis reveals slight protein in the urine and bacteriuria with pyuria. D. Serum electrolytes reveal a decreased sodium level with an increased potassium level.

C. Urinalysis reveals slight protein in the urine and bacteriuria with pyuria.

After the administration of t-PA, the nurse should: 1. Observe the client for chest pain. 2. Monitor for fever. 3. Review the 12-lead electrocardiogram (ECG). 4. Auscultate breath sounds

1 Although monitoring the 12-lead ECG and monitoring breath sounds are important, observing the client for chest pain is the nursing assessment priority because closure of the previously obstructed coronary artery may recur. Clients who receive t- PA frequently receive heparin to prevent closure of the artery after administration of t- PA. Careful assessment for signs of bleeding and monitoring of partial thromboplastin time are essential to detect complications. Administration of t-PA should not cause fever.

An overweight client taking warfarin (Coumadin) has dry skin due to decreased arterial blood flow. What should the nurse instruct the client to do? Select all that apply. 1. Apply lanolin or petroleum jelly to intact skin. 2. Follow a reduced-calorie, reduced-fat diet. 3. Inspect the involved areas daily for new ulcerations. 4. Instruct the client to limit activities of daily living (ADLs). 5.Use an electric razor to shave

1,2,3,5 Maintaining skin integrity is important in preventing chronic ulcers and infections. The client should be taught to inspect the skin on a daily basis. The client should reduce weight to promote circulation; a diet lower in calories and fat is appropriate. Because the client is receiving Coumadin, the client is at risk for bleeding from cuts. To decrease the risk of cuts, the nurse should suggest that the client use an electric razor. The client with decreased arterial blood flow should be encouraged to participate in ADLs. In fact, the client should be encouraged to consult an exercise physiologist for an exercise program that enhances the aerobic capacity of the body.

The healthcare provider prescribes Morphine Sulfate Oral Solution 38 mg PO q4 hours for a client who is opioid-tolerant. The available 30 ml bottle is labeled, 100 mg/5 ml (20 mg/ml), and is packed with a calibrated oral syringe to provide accurate dose measurements. How many ml should the nurse administer?

1.9 ml

The client who does not respond adequately to fluid replacement has a prescription for an IV infusion of dopamine hydrochloride at 5 mcg/kg/min. To determine that the drug is having the desired effect, the nurse should assess the client for: 1. Increased renal and mesenteric blood flow. 2. Increased cardiac output. 3. Vasoconstriction. 4. Reduced preload and afterload.

2 At medium doses (4 to 8 mcg/kg/min), dopamine hydrochloride slightly increases the heart rate and improves contractility to increase cardiac output and improve tissue perfusion. When given at low doses (0.5 to 3.0 mcg/kg/min), dopamine increases renal and mesenteric blood flow. At high doses (8 to 10 mcg/kg/min), dopamine produces vasoconstriction, which is an undesirable effect. Dopamine is not given to affect preload and afterload.

A client receives a prescription for bacitracin 20,000 units every 12 hours IM. The medication is available in a vial that contains 50,000 units and includes reconstitution instructions: "Use 4.8 ml diluent to yield a total volume of 5 ml." How many ml should the nurse administer?

2 ml

A client with advanced Hodgkin's disease is admitted to hospice because death is imminent. The goal of nursing care at this time is to: 1. Reduce the client's fear of pain. 2. Support the client's wish to discontinue further therapy. 3. Prevent feelings of isolation. 4. Help the client overcome feelings of social inadequacy.

3 Terminally ill clients most often describe feelings of isolation because they tend to be ignored, they are often left out of conversations (especially those dealing with the future), and they sense the attitudes of discomfort that many people feel in their presence. Helpful nursing measures include taking the time to be with the client, offering opportunities to talk about feelings, and answering questions honestly.

If a client displays risk factors for coronary artery disease, such as smoking cigarettes, eating a diet high in saturated fat, or leading a sedentary lifestyle, techniques of behavior modification may be used to help the client change the behavior. The nurse can best reinforce new adaptive behaviors by: 1. Explaining how the risk factor behavior leads to poor health. 2. Withholding praise until the new behavior is well established. 3. Rewarding the client whenever the acceptable behavior is performed. 4. Instilling mild fear into the client to extinguish the behavior.

3 A basic principle of behavior modification is that behavior that is learned and continued is behavior that has been rewarded. Other reinforcement techniques have not been found to be as effective as reward.

The nurse should caution the client with diabetes mellitus who is taking a sulfonylurea that alcoholic beverages should be avoided while taking these drugs because they can cause which of the following? 1. Hypokalemia. 2. Hyperkalemia. 3. Hypocalcemia. 4.Disulfiram (Antabuse)-like symptoms

4 A client with diabetes who takes any first- or second-generation sulfonylurea should be advised to avoid alcohol intake. Sulfonylureas in combination with alcohol can cause serious disulfiram (Antabuse)-like reactions, including flushing, angina, palpitations, and vertigo. Serious reactions, such as seizures and possibly death, may also occur. Hypokalemia, hyperkalemia, and hypocalcemia do not result from taking sulfonylureas in combination with alcohol.

A 51-year-old truck driver who smokes two packs of cigarettes a day and is 30 pounds overweight is diagnosed with having a gastric ulcer. What content is most important for the nurse to include in the discharge teaching for this client? A) Information about smoking cessation. B) Diet instructions for a low-residue diet. C) Instructions on a weight-loss program. D) The importance of increasing milk in the diet.

A

A client is admitted for further testing to confirm sarcoidosis. Which diagnostic test provides definitive information that the nurse should report to the healthcare provider? A) Lung tissue biopsy. B) Positive blood cultures. C) Magnetic resonance imaging (MRI). D) Computerized tomography (CT) of the thorax.

A

A client is placed on a respirator following a cerebral hemorrhage, and vecuronium bromide (Norcuron) 0.04 mg/kg q12h IV is prescribed. Which nursing diagnosis is the priority for this client? A) Impaired communication related to paralysis of skeletal muscles. B) High risk for infection related to increased intracranial pressure. C) Potential for injury related to impaired lung expansion. D) Social isolation related to inability to communicate.

A

A client who is sexually active with several partners requests an intrauterine device (IUD) as a contraceptive method. Which information should the nurse provide? A) Using an IUD offers no protection against sexually transmitted diseases (STD), which increase the risk for pelvic inflammatory disease (PID). B) Getting pregnant while using an IUD is common and is not the best contraceptive choice. C) Relying on an IUD may be a safer choice for monogamous partners, but a barrier method provides a better option in preventing STD transmission. D) Selecting a contraceptive device should consider choosing a successful method used in the past.

A

A female client receiving IV vasopressin (Pitressin) for esophageal varice rupture reports to the nurse that she feels substernal tightness and pressure across her chest. Which PRN protocol should the nurse initiate? A) Start an IV nitroglycerin infusion. B) Nasogastric lavage with cool saline. C) Increase the vasopressin infusion. D) Prepare for endotracheal intubation.

A

A middle-aged male client with diabetes continues to eat an abundance of foods that are high in sugar and fat. According to the Health Belief Model, which event is most likely to increase the client's willingness to become compliant with the prescribed diet? A) He visits his diabetic brother who just had surgery to amputate an infected foot. B) He is provided with the most current information about the dangers of untreated diabetes. C) He comments on the community service announcements about preventing complications associated with diabetes. D) His wife expresses a sincere willingness to prepare meals that are within his prescribed diet.

A

A postmenopausal client asks the nurse why she is experiencing discomfort during intercourse. What response is best for the nurse to provide? A) Estrogen deficiency causes the vaginal tissues to become dry and thinner. B) Infrequent intercourse results in the vaginal tissues losing their elasticity. C) Dehydration from inadequate fluid intake causes vulva tissue dryness. D) Lack of adequate stimulation is the most common reason for dyspareunia.

A

The nurse formulates the nursing diagnosis of, Urinary retention related to sensorimotor deficit for a client with multiple sclerosis. Which nursing intervention should the nurse implement? A) Teach the client techniques of intermittent self-catheterization. B) Decrease fluid intake to prevent over distention of the bladder. C) Use incontinence briefs to maintain hygiene with urinary dribbling. D) Explain that anticholinergic drugs will decrease muscle spasticity.

A

The nurse is assessing a client with bacterial meningitis. Which assessment finding indicates the client may have developed septic emboli? A) Cyanosis of the fingertips. B) Bradycardia and bradypnea. C) Presence of S3 and S4 heart sounds. D) 3+ pitting edema of the lower extremities.

A

The nurse is caring for a client with syndrome of inappropriate antidiuretic hormone (SIADH), which is manifested by which symptoms? A) Loss of thirst, weight gain. B) Dependent edema, fever. C) Polydipsia, polyuria. D) Hypernatremia, tachypnea.

A

The nurse is interviewing a male client with hypertension. Which additional medical diagnosis in the client's history presents the greatest risk for developing a cerebral vascular accident (CVA)? A) Diabetes mellitus. B) Hypothyroidism. C) Parkinson's disease. D) Recurring pneumonia.

A

After attending a class on reducing cancer risk factors, a client selects bran flakes with 2% milk and orange slices from a breakfast menu. In evaluating the client's learning, the nurse affirms that the client has made good choices and makes what additional recommendation? A.Switch to skim milk. B.Switch to orange juice. C.Add a source of protein. D.Add herbal tea.

A Dietary recommendations to reduce cancer risk include reduced consumption of fats, with increased consumption of fruits, vegetables, and fiber. (A) promotes reduced fat consumption. Orange slices provide more fiber than orange juice (B, C, and D) are not standard recommendations for reducing cancer risk.

A client with hypertension has been receiving ramipril (Altace), 5 mg PO, daily for 2 weeks and is scheduled to receive a dose at 0900. At 0830, the client's blood pressure is 120/70 mm Hg. Which action should the nurse take? A.Administer the prescribed dose at the scheduled time. B.Hold the dose and contact the health care provider. C.Hold the dose and recheck the blood pressure in 1 hour. D.Check the health care provider's prescription to clarify dose.

A The client's blood pressure is within normal limits, indicating that the ramipril, an antihypertensive, is having the desired effect and should be administered (A). (B and C) would be appropriate if the client's blood pressure was excessively low (<100 mm Hg systolic) or if the client were exhibiting signs of hypotension such as dizziness. This prescribed dose is within the normal dosage range, as defined by the manufacturer; therefore, (D) is not necessary.

The family suspects that AIDS dementia is occurring in their son who is HIV positive. Which symptom confirms their suspicions?

A change has recently occurred in his handwriting.

Individuals of low socioeconomic status are at an increased risk for infection because of which of the following? (Select all that apply.) A) High cost of medications B) Inadequate nutrition C) Easy access to health screenings D) Uninsured or underinsured status

A, B, D Individuals of low socioeconomic status tend to be part of the underinsured or uninsured population. Lack of insurance decreases accessibility to health care in general and health screening services specifically. High costs of medication and nutritious food also make this population at higher risk for infection.

The nurse is assessing a patient for the adequacy of ventilation. What assessment findings would indicate the patient has good ventilation? (Select all that apply.) A) There is presence of quiet, effortless breath sounds at lung base bilaterally. B) Nail beds are pink with good capillary refill. C) Trachea is just to the left of the sternal notch. D) Respiratory rate is 24 breaths/min. E) The right side of the thorax expands slightly more than the left. F) Oxygen saturation level is 98%.

A,B,F Oxygen saturation level should be between 95 and 100%; nail beds should be pink with capillary refill of about 3 seconds; and breath sounds should be present at base of both lungs. Normal respiratory rate is between 12 and 20 breaths/min. The trachea should be in midline with the sternal notch. The thorax should expand equally on both sides.

The physician has prescribed salmeterol (Serevent) for a patient with asthma. In reviewing the use of dry powder inhalers (DPIs) with the patient, the nurse should provide which of the following instructions? A. "Close lips tightly around the mouthpiece and breathe in deeply and quickly." B. "To administer a DPI, you must use a spacer that holds the medicine so that you can inhale it." C. "Hold the inhaler several inches in front of your mouth and breathe in slowly, holding the medicine as long as possible." D. "You will know you have correctly used the DPI when you taste or sense the medicine going into your lungs."

A. "Close lips tightly around the mouthpiece and breathe in deeply and quickly."

A female client is seen in the Emergency Department with a broken arm. She is visibly anxious and tells the nurse "I am afraid my husband is going to kill me. The beatings are getting worse, and everyone says I should leave him, but I am afraid of what he will do to me and the children if I do leave." What question is most important for the nurse to ask the client? A. "Have you thought about what to do when you are in an unsafe situation?" B. "Have you considered attending family violence programs sponsored by the police department?" C. "Do you know a clergy person you can talk to?" D. "Did you experience violence in your family of origin?"

A. "Have you thought about what to do when you are in an unsafe situation?"

During a health fair, a male client with emphysema tells the nurse that he fatigues easily. Assessment reveals marked clubbing of the fingernails and an increased anteroposterior chest diameter. Which instruction is best to provide the client? A."Pace your activities and schedule rest periods." B."Increase the amount of oxygen you use at night." C."Obtain medical evaluation for antibiotic therapy." D."Reduce your intake of fluids containing caffeine."

A. "Pace your activities and schedule rest periods."

The nurse is performing hourly neurological check for a client with a head injury. Which new assessment finding warrants the most immediate intervention by the nurse? A. A unilateral pupil that is dilated and nonreactive to light. B. Client cries out when awakened by a verbal stimulus. C. Client demonstrates a loss of memory to the events leading up to the injury. D. Onset of nausea, headache, and vertigo.

A. A unilateral pupil that is dilated and nonreactive to light.

The nurse is assessing several clients at a homeless shelter. Which client(s) should the nurse refer for involuntary admission to a psychiatric facility? (select all that apply) A. A young adult who has lost his appetite because the communists are asking him to kill all conservative Catholics B. An adult male who has not eaten for three days and says he is Jesus Christ and that God loves all people C. An older woman who used to be a prostitute and resides in the shelter about three nights weekly D. A middle-aged woman who sits for long periods of time and mumbles to herself about wanting to die E. An adolescent male who works on the streets as a beggar to purchase marijuana, food, and shelter

A. A young adult who has lost his appetite because the communists are asking him to kill all conservative Catholics B. An adult male who has not eaten for three days and says he is Jesus Christ and that God loves all people D. A middle-aged woman who sits for long periods of time and mumbles to herself about wanting to die

The nurse makes a supervisory home visit to observe an unlicensed assistive personnel (UAP) who is providing personal care for a client with Alzheimer's Disease. The nurse observes that whatever the client gets upset, the UAP changes the subject. What action should the nurse take in response to this observation? A. Affirm that the UAP is using an effective strategy to reduce the client's anxiety B. Meet with the UAP later to role model more assertive communication techniques C. Assume care of the client to ensure that effective communication is maintained D. Tell the UAP to offer more choices during the personal care to prevent anxiety

A. Affirm that the UAP is using an effective strategy to reduce the client's anxiety

A charge nurse is establishing priorities of client needs at the beginning of the morning shift. Which client situation has the highest priority for immediate care? A. An elderly client who is trying to climb over the side rails B. A diabetic client who requires insulin before breakfast C. A postoperative client with unremitting nausea D. A 5-year-old child who is crying because the parents left

A. An elderly client who is trying to climb over the side rails

A 45-year-old man with asthma is brought to the emergency department by automobile. He is short of breath and appears frightened. During the initial nursing assessment, which of the following clinical manifestations might be present as an early symptom during an exacerbation of asthma? A. Anxiety B. Cyanosis C. Hypercapnia D. Bradycardia

A. Anxiety

During admission of a patient diagnosed with non-small cell carcinoma of the lung, the nurse questions the patient related to a history of which of the following risk factors for this type of cancer? (Select all that apply.) A. Asbestos exposure B. Cigarette smoking C. Exposure to uranium D. Chronic interstitial fibrosis

A. Asbestos exposure B. Cigarette smoking C. Exposure to uranium

The home care nurse provided self-care instructions for a client with chronic venous insufficiency caused y deep vein thrombosis. Which instructions should the nurse include in the client's discharge teaching plan? (select all that apply) A. Avoid prolonged standing or sitting B. Maintain the bed flat while sleeping C. Cross legs at knee but not at ankle D. Continue wearing elastic stockings E. Use recliner for long periods of sitting

A. Avoid prolonged standing or sitting D. Continue wearing elastic stockings E. Use recliner for long periods of sitting

Before beginning a transfusion of RBCs, which of the following actions by the nurse would be of highest priority to avoid an error during this procedure? A. Check the identifying information on the unit of blood against the patient's ID bracelet. B. Select new primary IV tubing primed with lactated Ringer's solution to use for the transfusion. C. Add the blood transfusion as a secondary line to the existing IV and used the IV controller to maintain correct flow. D. Remain with the patient for 60 minutes after beginning the transfusion to watch for signs of a transfusion reaction. The patient's identifying information (name, date of birth, medical record number) on the identification bracelet should exactly match the information on the blood bank tag that has been placed on the unit of blood. If any information does not match, the transfusions should not be hung because of possible error and risk to the patient.

A. Check the identifying information on the unit of blood against the patient's ID bracelet.

The nurse plans to obtain a urine specimen for culture from a client's indwelling catheter. The nurse enters the room with the syringe and notes that there is 100 ml of urine in the drainage bag, but no urine is in the tubing. What action should the nurse take? A. Clamp the tubing until urine is observed in the tubing B. Obtain sterile normal saline to irrigate the catheter C. Separate the tubing from the catheter and withdraw a urine specimen D. Remove the urine specimen from the drainage bag

A. Clamp the tubing until urine is observed in the tubing

One year after diagnosed with Pneumocystis carinii pneumonia, a client is admitted with respiratory failure. Respirations are shallow with periods of apnea. After the healthcare provider delivers a grim prognosis to the client's family, which intervention should the nurse implement first? A. Clarify client's end of life wishes B. Review client's CD4 cell count C. Obtain arterial blood gases D. Assist with insertion of an airway

A. Clarify client's end of life wishes

Before administering a parenteral nutrition through a central vein, the should confirm information from which sources? (select all that apply) A. Client's identification band B. Healthcare providers prescription C. Dietician's progress notes D. Solution label E. Measured residual volume F. Medication administration record

A. Client's identification band B. Healthcare providers prescription D. Solution label F. Medication administration record

A client who is 3 weeks postpartum is brought to the mental health unit by her husband for admission because she has been verbalizing that baby is evil. After an assessment interview, the nurse determines the client thinks that the baby is going to bring harm to the other children. How should the nurse document the client's altered thought process? A. Delusional thoughts B. Visual hallucinations C. Ideas of reference D. Nihilistic ideas

A. Delusional thoughts

A male client with chronic asthma tells the nurse that he is having more episodes of bronchoconstriction and increased mucous production. Which action should the nurse implement? A. Determine if rescue inhaler is being used first during an acute episodes. B. Inquire about effectiveness of corticosteroid inhaler during pollen days C. Auscultate lungs for adventitious sounds consistent with fluid accumulation D. Ask if an environmental trigger is associated with the asthmatic episodes

A. Determine if rescue inhaler is being used first during an acute episodes.

The register nurse (RN) is observing a newly hired practical nurse (PN) give a newborn a vitamin K (AquaMEPHYTON) injection. The PN uses a filter need to draw 0.25 ml of AquaMEPHYTON into the syringe, cleanses the thigh with alcohol in a circular motion, and prepares to inject the needle at a 90 degree angle in the left vastus lateralis. What action should the RN take? A. Direct the PN to change the filter needle to a 1/2 inch needle B. Confirm that the correct injection technique is being used C. Suggest injecting the medication into the dorsogluteal muscle D. Recommend administration of the injection at a 45 degree angle

A. Direct the PN to change the filter needle to a 1/2 inch needle

To prevent infection by autocontamination during the acute phase of recovery from multiple burns, which intervention is most important for the nurse to implement? A. Dress each wound separately B. Avoid sharing equipment between multiple clients C. Use gown, mask, and gloves with dressing change D. Implement protective isolation

A. Dress each wound separately

The nurse is conducting an osteoporosis screening clinic at a health fair. What information should the nurse provide to individuals who are at risk for osteoporosis? (Select all that apply.)

A. Encourage alcohol and smoking cessation. C. Promote regular weight-bearing exercises. D. Implement a home safety plan to prevent falls.

The nurse is conducting an osteoporosis screening clinic at a health fair. What information should the nurse provide to individuals who are at risk for osteoporosis? (Select all that apply.) A.Encourage alcohol and smoking cessation. B.Suggest supplementing diet with vitamin E. C.Promote regular weight-bearing exercises. D.Implement a home safety plan to prevent falls. E.Propose a regular sleep pattern of 8 hours nightly.

A. Encourage alcohol and smoking cessation. C. Promote regular weight-bearing exercises. D. Implement a home safety plan to prevent falls.

Which content about self-care should the nurse include in the teaching plan of a client who has genital herpes? (Select all that apply.) A. Encourage annual physical and Pap smear. B. Take antiviral medication as prescribed. C. Use condoms to avoid transmission to others. D. Warm sitz baths may relieve itching. E. Use Nystatin suppositories to control itching. F. Douche with weak vinegar solutions to decrease itching.

A. Encourage annual physical and Pap smear. B. Take antiviral medication as prescribed. C. Use condoms to avoid transmission to other D. Warm sitz baths may relieve itching.

Which content about self-care should the nurse include in the teaching plan of a female client who has genital herpes? (Select all that apply.)

A. Encourage annual physical and Pap smear. B. Take antiviral medication as prescribed. C. Use condoms to avoid transmission to others. D. Warm sitz baths may relieve itching.Rationale:

When admitting a patient with the diagnosis of asthma exacerbation, the nurse will assess for which of the following potential triggers? (Select all that apply.) A. Exercise B. Allergies C. Emotional stress D. Decreased humidity

A. Exercise B. Allergies C. Emotional stress

A client is being discharged following radioactive seed implantation for prostate cancer. What is the most important information that the nurse should provide to this client's family?

A. Follow exposure precautions.

Which menu selection by a male client indicates to the nurse that he understands the dietary management of Crohn's disease? A. Grilled chicken sandwich and pasta B. Tossed green salad and breadsticks C. Cheese enchiladas and beans D. Hamburger and French fries

A. Grilled chicken sandwich and pasta

An 81-year-old male client has emphysema. He lives at home with his cat and manages self-care with no difficulty. When making a home visit, the nurse notices that this client's tongue is somewhat cracked and his eyeballs appear sunken into his head. Which nursing intervention is indicated?

A. Help the client determine ways to increase his fluid intake.

A male client with arterial peripheral vascular disease (PVD) complains of pain in his feet. Which instruction should the nurse give to the UPA to quickly relieve the client's pain? A. Help the client to dangle his legs. B. Apply compression stockings. C. Assist with passive leg exercises. D. Ambulate three times daily.

A. Help the client to dangle his legs.

A client is placed on a mechanical ventilator following a cerebral hemorrhage, and vecuronium bromide (Norcuron) 0.04 mg/kg every 12 hours IV is prescribed. What is the priority nursing diagnosis for this client? A. Impaired communication related to paralysis of skeletal muscles. B. Hight risk or infection related to increased ICP. C. Potential for injury related to impaired lung expansion. D. Social isolation related to inability to communicate.

A. Impaired communication related to paralysis of skeletal muscles.

A client with cancer is receiving chemotherapy with a known vesicant. The client's IV has been in place for 72 hours. The nurse determines that a new IV site cannot be obtained, and leaves the present IV in place. What is the greatest clinical risk related to this situation? A. Impaired skin integrity B. Fluid volume excess C. Acute pain and anxiety D. Peripheral neurovascular dysfunction

A. Impaired skin integrity

The mother reports to the nurse that the thick honey-colored crusts on her child's legs began as flat red spots. This is highly indicative of what condition? A. Impetigo B. Eczema C. Ringworm D. Psoriasis

A. Impetigo

A client with a history of asthma and bronchitis arrives at the clinic with shortness of breath, productive cough with thickened tenacious mucous, and the inability to walk up a flight of stairs without experiencing breathlessness. Which action is most important for the nurse to instruct the client about self-care? A. Increase the daily intake of oral fluids to liquefy secretions B. Avoid crowded enclosed areas to reduce pathogen exposure C. Call the clinic if undesirable side effects of mediations occur D. Teach anxiety reduction methods for feelings of suffocation

A. Increase the daily intake of oral fluids to liquefy secretions

The nurse evaluates that a patient is experiencing the expected beneficial effects of ipratropium (Atrovent) after noting which of the following assessment findings? A. Increased peak flow readings B. Increased level of consciousness C. Decreased sputum production D. Increased respiratory rate

A. Increased peak flow readings. Ipratropium is a bronchodilator that should lead to increased PEFRs.

A client is admitted voluntarily to the in-patient psychiatric unit for multi-substance dependency. While the client is in a group therapy, a peace officer approaches the nurse's station demanding to arrest the client. How should the nurse respond? A. Inform the officer that client information cannot be released B. Ensure safety for other client's by handcuffing the client during removal C. Refer the peace officer to the client's healthcare provider D. Call the unit security code and have the peace officer escorted off the unit.

A. Inform the officer that client information cannot be released

An emaciated homeless client presents to the emergency department complaining of a productive cough, with blood-tinged sputum and night sweats. Which action is most important for the emergency department triage nurse to implement for this client?

A. Initiate airborne infection

A nurse is admitting a 4-month-old who has respiratory syncytial virus (RIS). What intervention should the nurse implement first? A. Initiate contact precautions B. Place the infant under a mist tent C. Elevate aretial blood gases D. Obtain the infants vital signs

A. Initiate contact precautions

A male client with major depression who is taking fluoxetine calls the psychiatric clinic complaining of be more agitated, irritable, and anxious than usual. Which intervention should the nurse implement? A. Instruct the client to seek medical attention immediately. B. Explain that these are common side effects of Prozac. C. Encourage him to take the fluoxetine at night with a snack. D. Tell the client to have a complete blood count (CBC) drawn.

A. Instruct the client to seek medical attention immediately.

The nurse who has administered a first dose of oral prednisone (Deltasone) to the patient with asthma writes on the care plan to begin monitoring which of the following patient parameters? A. Intake and output B. Bowel sounds C. Apical pulse D. Deep tendon reflexes

A. Intake and output

Which medication should the nurse anticipate administering to a client who is diagnosed with myxedema coma? A. Intravenous administration of thyroid hormones B. Oral administration of hypnotic agents C. Intravenous bolus of hydrocortisone D. Subcutaneous administration of vitamin K

A. Intravenous administration of thyroid hormones

The wife of a client with Parkinson's disease expresses concern because her husband has lost so much weight. Which teaching is best for the nurse to provide? A. Invite friends over regularly to share in meal times B. Encourage the client to drink clear liquids between meals C. Coach the client to make an intentional effort to swallow D. Talk to the healthcare provider about prescribing an appetite stimulant

A. Invite friends over regularly to share in meal times

The school nurse is planning to begin an obesity screening program in a school system. It is best to begin the screening program with which group? A. Kindergarden B. Third grade C. High school D. Onset of puberty

A. Kindergarden

The nurse is interviewing a client with schizophrenia. Which client behavior requires immediate intervention? A. Muscle spasms of the back and neck B. Rocks back and forth in the chair C. Shuffling gait and stooped posture D. Lip smacking and frequent eye blinking

A. Muscle spasms of the back and neck

The nurse is observing an unlicensed assistive personnel (UPA) who is performing morning care for a bedfast client with Huntington disease. Which care measure is most important for the nurse to supervise? A. Oral care B. Bathing C. Foot care D. Catheter care

A. Oral care

A client with an electrical burn is admitted to the emergency department on a backboard with a cervical collar. Which intervention should the nurse implement? A. Place the client on cardiac telemetry B. Obtain STAT arterial blood gases C. Flush the burned area with sterile normal saline D. Elevate the client's head of bed to 45 degrees

A. Place the client on cardiac telemetry

The legs of a client who is receiving hospice care have become mottled in appearance. When the nurse observes the unlicensed assistive personnel (UAP) place a heating pad on the mottled areas, what action should the nurse take? A. Remove the heating pads and pace a soft blanket over the client's legs and feet B. Advise the UAP to observe the client's skin while the heating pads are in place C. Evaluate the client's feet on a pillow and monitor the client's pedal pulses frequently D. Instruct the UAP to reposition the heating pads to the sides of the legs and feet

A. Remove the heating pads and pace a soft blanket over the client's legs and feet

A 74-year-old male client is admitted to the ICU with a diagnosis of respiratory failure secondary to pneumonia. Currently, he is ventilator-dependent with settings of tidal volume (VT) 750 ml and intermittent mandatory ventilation (IMV) rate of 10. ABG results are pH 7.48; PaCO2 30; PaO2 64; HCO3 25; and FiO2 0.80. Which intervention should the nurse implement first?

Add 5 cm positive end-expiratory pressure (PEEP).

A client with hypertension has been receiving ramipril (Altace) 5 mg PO daily for 2 weeks and is scheduled to receive a dose at 0900. At 0830 the client's blood pressure is 120/70. What action should the nurse take?

Administer the dose as prescribed.

Esophagogastroduodenoscopy (EGD).

After procedure you keep the patient NPO until gag reflex returns. Gently tickle back of throat to determine reflex. Use warm saline gargles for relief of throat. Check temp for 15-30min for 1-2hr.

The nurse is reviewing the routine medications taken by a client with chronic angle closure glaucoma. Which medication prescription should the nurse question?

An anticholinergic with a side effect of pupillary dilation

The nurse notes that a client who is scheduled for surgery the next morning has an elevated blood urea nitrogen (BUN) level. Which client condition is most likely to have contributed to this finding?

Anorexia and vomiting for the past 2 days

Following surgery, Mario complains of mild incisional pain while performing deep- breathing and coughing exercises. The nurse's best response would be: A. "Pain will become less each day." B. "This is a normal reaction after surgery." C. "With a pillow, apply pressure against the incision." D. "I will give you the pain medication the physician ordered."

Answer: (C) "With a pillow, apply pressure against the incision." Applying pressure against the incision with a pillow will help lessen the intra-abdominal pressure created by coughing which causes tension on the incision that leads to pain.

Mary received AtropineSO4 as a pre-medication 30 minutes ago and is now complaining of dry mouth and her PR is higher, than before the medication was administered. The nurse's best A. The patient is having an allergic reaction to the drug. B. The patient needs a higher dose of this drug C. This is normal side-effect of AtSO4 D. The patient is anxious about upcoming surgery

Answer: (C) This is normal side-effect of AtSO4 Atropine sulfate is a vagolytic drug that decreases oropharyngeal secretions and increases the heart rate.

The nurse needs to carefully assess the complaint of pain of the elderly because older people A. are expected to experience chronic pain B. have a decreased pain threshold C. experience reduced sensory perception D. have altered mental function

Answer: (C) experience reduced sensory perception Degenerative changes occur in the elderly. The response to pain in the elderly maybe lessened because of reduced acuity of touch, alterations in neural pathways and diminished processing of sensory data.

A 58-year-old client who has been post-menopausal for five years is concerned about the risk for osteoporosis because her mother has the condition. Which information should the nurse offer? A) Osteoporosis is a progressive genetic disease with no effective treatment. B) Calcium loss from bones can be slowed by increasing calcium intake and exercise. C) Estrogen replacement therapy should be started to prevent the progression osteoporosis. D) Low-dose corticosteroid treatment effectively halts the course of osteoporosis.

B

A female client requests information about using the calendar method of contraception. Which assessment is most important for the nurse to obtain? A) Amount of weight gain or weight loss during the previous year. B) An accurate menstrual cycle diary for the past 6 to 12 months. C) Skin pigmentation and hair texture for evidence of hormonal changes. D) Previous birth-control methods and beliefs about the calendar method.

B

During CPR, when attempting to ventilate a client's lungs, the nurse notes that the chest is not moving. What action should the nurse take first? A) Use a laryngoscope to check for a foreign body lodged in the esophagus. B) Reposition the head to validate that the head is in the proper position to open the airway. C) Turn the client to the side and administer three back blows. D) Perform a finger sweep of the mouth to remove any vomitus.

B

Healthcare workers must protect themselves against becoming infected with HIV. The Center for Disease Control has issued guidelines for healthcare workers in relation to protection from HIV. These guidelines include which recommendation? A) Place HIV positive clients in strict isolation and limit visitors. B) Wear gloves when coming in contact with the blood or body fluids of any client. C) Conduct mandatory HIV testing of those who work with AIDS clients. D) Freeze HIV blood specimens at -70° F to kill the virus.

B

In assessing cancer risk, the nurse identifies which woman as being at greatest risk of developing breast cancer? A) A 35-year-old multipara who never breastfed. B) A 50-year-old whose mother had unilateral breast cancer. C) A 55-year-old whose mother-in-law had bilateral breast cancer. D) A 20-year-old whose menarche occurred at age 9.

B

Physical examination of a comatose client reveals decorticate posturing. Which statement is accurate regarding this client's status based upon this finding? A) A cerebral infectious process is causing the posturing. B) Severe dysfunction of the cerebral cortex has occurred. C) There is a probable dysfunction of the midbrain. D) The client is exhibiting signs of a brain tumor.

B

The nurse is planning care for a client who has a right hemispheric stroke. Which nursing diagnosis should the nurse include in the plan of care? A) Impaired physical mobility related to right-sided hemiplegia. B) Risk for injury related to denial of deficits and impulsiveness. C) Impaired verbal communication related to speech-language deficits. D) Ineffective coping related to depression and distress about disability.

B

The nurse is providing dietary instructions to a 68-year-old client who is at high risk for development of coronary heart disease (CHD). Which information should the nurse include? A) Limit dietary selection of cholesterol to 300 mg per day. B) Increase intake of soluble fiber to 10 to 25 grams per day. C) Decrease plant stanols and sterols to less than 2 grams/day. D) Ensure saturated fat is less than 30% of total caloric intake.

B

Which intervention should the nurse plan to implement when caring for a client who has just undergone a right above-the-knee amputation? A) Maintain the residual limb on three pillows at all times. B) Place a large tourniquet at the client's bedside. C) Apply constant, direct pressure to the residual limb. D) Do not allow the client to lie in the prone position.

B

Which milestone indicates to the nurse successful achievement of young adulthood? A) Demonstrates a conceptualization of death and dying. B) Completes education and becomes self-supporting. C) Creates a new definition of self and roles with others. D) Develops a strong need for parental support and approval.

B

A client has recently been placed on prednisone (Deltasone). What is the highest priority instruction the nurse will provide? A) "Take the drug with food or milk." B) "Report any abdominal pain or dark-colored vomit." C) "Expect to experience weight gain." D) "Watch your diet while on this medication."

B All of these directions are appropriate to give the client; however, telling the client to report abdominal pain and dark-colored vomit is most important because these could signal gastric ulceration.

A patient is being treated with an antibiotic. The nurse explains to the patient that this medication is required for the reduction of inflammation at the injury site because this medication: A) will decrease the pain at the site. B) helps to kill the infection causing the inflammation. C) will reduce the patients fever. D) inhibits cyclooxygenase.

B Antimicrobials treat the underlying cause of the infection which leads to inflammation. Analgesics and nonsteroidal antiinflammatory drugs (NSAIDs) help to treat pain. NSAIDs and other antipyretics are cyclooxygenase inhibitors. Antipyretics help to reduce fever.

The nurse is caring for a client who had a stroke. Which nursing intervention does the nurse implement during the first 72 hours to prevent complications? A) Position with the head of the bed flat to enhance cerebral perfusion. B) Monitor neurologic and vital signs closely to identify early changes in status. C) Administer prescribed analgesics to promote pain relief. D) Cluster nursing procedures together to avoid fatiguing the client.

B Early detection of neurologic, blood pressure, and heart rhythm changes offers an opportunity to intervene in a timely fashion. Evidence is not yet sufficient to recommend a specific back rest elevation after stroke. Analgesics are often held during the first 72 hours to ensure that the client's neurologic status is not altered by pain medications. Preventing fatigue is not a priority in the first 72 hours.

Why does the nurse always ask the client his or her pain level after taking routine vital signs? A) To follow McCaffery's guidelines on pain management B) To ensure that pain assessment occurs on a regular basis C) To determine the need for more frequent vital sign measurement D) To determine whether pain is influencing blood pressure and heart rate

B Making pain the fifth vital sign allows more frequent and accurate assessment, which can contribute to better pain management.

The nurse is caring for a client with peripheral arterial disease. What priority nursing intervention does the nurse perform to promote vasodilation? A) Increase the client's exercise regimen daily. B) Educate the client to abstain from smoking. C) Apply a heating pad to the affected limb. D) Administer an aspirin on a daily basis.

B Smoking causes vasoconstriction, and its effects can last up to 1 hour after the cigarette is finished. Increasing activity may lead to collateral circulation but does not cause vasodilation. Use of a heating pad is contraindicated in the client with peripheral artery disease because of the risk of a burn caused by diminished sensation. The use of aspirin my impede platelet clumping and is contraindicated only when the client is on anticoagulants.

An emergency department (ED) nurse gives report on a client who is being transferred to the medical-surgical floor. Because of an identified risk for suicide, the ED nurse suggests that the floor nurse contact a sitter and behavioral health. This statement represents which part of the SBAR hand-off? A) Situation B) Recommendation C) Background D) Assessment

B The ED nurse is giving recommendations to the medical-surgical floor nurse about interventions to start for the client who is being transferred. No communication is provided in the SBAR report about the situation, background, or assessment.

A client who has had a stroke with left-sided hemiparesis has been referred to a rehabilitation center. The client asks, "Why do I need rehabilitation?" How does the nurse respond? A) "Rehabilitation will reverse any physical deficits caused by the stroke." B) "Rehabilitation will help you function at the highest level possible." C) "If you do not have rehabilitation, you may never walk again." D) "Your doctor knows best and has ordered this treatment for you."

B The goal of rehabilitation is to maximize the client's abilities in all aspects of life. The other responses do not answer the client's question appropriately.

The nurse is caring for a client with Crohn's disease and colonic strictures. Which assessment finding requires the nurse to consult the health care provider immediately? A) Traces of blood in the stool B) Distended abdomen C) Temperature of 100.0° F (37.8° C) D) Crampy lower abdominal pain

B The presence of strictures predisposes the client to intestinal obstruction. Abdominal distention may indicate that the client has developed an obstruction of the large bowel, and the client's provider should be notified right away. Low-grade fever, bloody diarrhea, and crampy abdominal pain are common symptoms of Crohn's disease.

Which instruction should the nurse teach a female client about the prevention of toxic shock syndrome?

B. "Change your tampon frequently."

The patient has an order for albuterol 5 mg via nebulizer. Available is a solution containing 2 mg/ml. How many milliliters should the nurse use to prepare the patient's dose? A. 0.2 B. 2.5 C. 3.75 D. 5.0

B. 2.5

The nurse notes that a client who is scheduled for surgery the next morning has an elevated blood urea nitrogen (BUN) level. Which condition is most likely to have contributed to this finding?

B. Anorexia and vomiting for the past 2 days

The nurse is reviewing routine medications taken by a client with chronic angle closure glaucoma. Which medication prescription should the nurse question?

B. Anticholinergic with a side effect of pupillary dilation

The nurse reports that a client is at risk for a brain attack (stroke) based on which assessment finding? A. Nuchal rigidity B. Carotid bruit C. Jugular vein distention D. Palpable cervical lymph node

B. Carotid bruit

The nurse is giving medication to an older client who has a percutaneous esophageal gastrostomy (PEG) tube in place. Which medication drug form should the nurse question? A. Digoxin (Lanoxin) tablet B. Enteric-coated aspirin (Ecotrin) C. Furosidemide (Lasix) tablet D. Megestrol (Megace) suspension

B. Enteric-coated aspirin (Ecotrin)

A client with Addison's disease started taking hydrocortisone in a divided daily dose last week. It is most important for the nurse to monitor which serum laboratory value? A. Osmolarity B. Glucose C. Albumin D. Platelets

B. Glucose

The nurse is scheduled to give a dose of salmeterol by metered dose inhaler (MDI). The nurse would administer the right drug by selecting the inhaler with which of the following trade names? A. Vanceril B. Serevent C. AeroBid D. Atrovent

B. Serevent

When administering medications, when is the last opportunity for the nurse to discover a near-miss medication error? A. During medication administration documentation B. When determining the client'd identity at the bedside C. After the dose is dispensed by the computer-controlled system D. As the prescription is verified with the client's medical record

B. When determining the client'd identity at the bedside

A 67-year-old woman who lives alone is admitted after tripping on a rug in her home and fractures her hip. Which predisposing factor probably led to the fracture in the proximal end of her femur? A) Failing eyesight resulting in an unsafe environment. B) Renal osteodystrophy resulting from chronic renal failure. C) Osteoporosis resulting from hormonal changes. D) Cardiovascular changes resulting in small strokes which impair mental acuity.

C

A client has been taking oral corticosteroids for the past five days because of seasonal allergies. Which assessment finding is of most concern to the nurse? A) White blood count of 10,000 mm3. B) Serum glucose of 115 mg/dl. C) Purulent sputum. D) Excessive hunger.

C

A client who was in a motor vehicle collision was admitted to the hospital and the right knee was placed in skeletal traction. The nurse has documented this nursing diagnosis in the client's medical record: "Potential for impairment of skin integrity related to immobility from traction." Which nursing intervention is indicated based on this diagnosis statement? A) Release the traction q4h to provide skin care. B) Turn the client for back care while suspending traction. C) Provide back and skin care while maintaining the traction. D) Give back care after the client is released from traction.

C

During an interview with a client planning elective surgery, the client asks the nurse, "What is the advantage of having a preferred provider organization insurance plan?" Which response is best for the nurse to provide? A) Long-term relationships with healthcare providers are more likely. B) There are fewer healthcare providers to choose from than in an HMO plan. C) Insurance coverage of employees is less expensive to employers. D) An individual can become a member of a PPO without belonging to a group.

C

During lung assessment, the nurse places a stethoscope on a client's chest and instructs him/her to say "99" each time the chest is touched with the stethoscope. What should be the correct interpretation if the nurse hears the spoken words "99" very clearly through the stethoscope? A) This is a normal auscultatory finding. B) May indicate pneumothorax. C) May indicate pneumonia. D) May indicate severe emphysema.

C

In assessing a client diagnosed with primary hyperaldosteronism, the nurse expects the laboratory test results to indicate a decreased serum level of which substance? A) Sodium. B) Antidiuretic hormone. C) Potassium. D) Glucose.

C

The nurse is assessing a client who smokes cigarettes and has been diagnosed with emphysema. Which finding should the nurse expect this client to exhibit? A) A decreased total lung capacity. B) Normal arterial blood gases. C) Normal skin coloring. D) An absence of sputum.

C

Which finding should the nurse identify as most significant for a client diagnosed with polycystic kidney disease (PKD)? A) Hematuria. B) 2 pounds weight gain. C) 3+ bacteria in urine. D) Steady, dull flank pain.

C

Which healthcare practice is most important for the nurse to teach a postmenopausal client? A) Wear layers of clothes if experiencing hot flashes. B) Use a water-soluble lubricant for vaginal dryness. C) Consume adequate foods rich in calcium. D) Participate in stimulating mental exercises.

C

The nurse is assessing a client admitted to the cardiac unit. What statement made by the client alerts the nurse to the possibility of right-sided heart failure? A) "I sleep with four pillows at night." B) "I wake up coughing every night." C) "My shoes fit really tight lately." D) "I have trouble catching my breath."

C Signs of systemic congestion occur with right-sided heart failure. Fluid is retained, pressure builds in the venous system, and peripheral edema develops. Left-sided heart failure symptoms include respiratory symptoms. Orthopnea, coughing, and difficulty breathing all could be results of left-sided heart failure.

Which action demonstrates that the nurse understands the purpose of the Rapid Response Team? A) Documenting all changes observed in the client and maintaining a postoperative flow sheet B) Monitoring the client for changes in postoperative status such as wound infection C) Notifying the physician of the client's change in blood pressure from 140 to 88 mm Hg systolic D) Notifying the physician of the client's increase in restlessness after medication change

C The Rapid Response Team (RRT) saves lives and decreases the risk for harm by providing care to clients before a respiratory or cardiac arrest occurs. Although the RRT does not replace the Code Team, which responds to client arrests, it intervenes rapidly for those who are beginning to decline clinically. It would be appropriate for the RRT to intervene when the client has experienced a 52-point drop in blood pressure. Monitoring the client's postoperative status, maintaining a postoperative flow sheet, and notifying the physician of a change in the client's status after a medication change would not be considered activities of the Rapid Response Team.

A young woman is being treated with amoxicillin (Amoxil) for a urinary tract infection. Which is the highest priority instruction for the nurse to give this client? A) "You may experience an irregular heartbeat while on the drug." B) "Watch for blood in your urine while taking this drug." C) "Use a second form of birth control while on the drug." D) "You will experience increased menstrual bleeding while on this drug."

C The client should use a second form of birth control because penicillin seems to reduce the effectiveness of estrogen-containing contraceptives. She should not experience increased menstrual bleeding, an irregular heartbeat, or blood in her urine while taking the medication.

The nurse is caring for a client with Crohn's disease who has developed a fistula. Which nursing intervention is the highest priority? A) Position the client to allow gravity drainage of the fistula. B) Check and record blood glucose levels every 6 hours. C) Encourage the client to consume a diet high in protein and calories. D) Monitor the client's hematocrit and hemoglobin.

C The client with Crohn's disease is already at risk for malabsorption and malnutrition. Malnutrition impairs healing of the fistula and immune responses. Therefore, maintaining adequate nutrition is a priority for this client. The client will require 3000 calories per day to promote healing of the fistula. Monitoring the client's blood sugar and hemoglobin levels is important, but less so than encouraging nutritional intake. The client need not be positioned to facilitate gravity drainage of the fistula, because fistulas often are found in the abdominal cavity.

The nurse is caring for a client who is receiving heparin therapy for a venous thromboembolism (VTE). The client's activated partial thromboplastin time (aPTT) before heparin therapy was 30 seconds. Which aPTT result indicates that anticoagulation is adequate at this time? A) 30 seconds B) 150 seconds C) 60 seconds D) 15 seconds

C Therapeutic aPTT values for clients receiving heparin should range from 1.5 to 2.5 times the control value.

The nurse is completing an admission inter for a client with Parkinson disease. Which question will provide addition information about manifestations the client is likely to experience? A. "Have you ever experienced and paralysis of your arms or legs?" B. " Do you have frequent blackout spells?" C. "Have you ever been 'frozen' in one spot, unable to move?" D. "Do you have headaches, especially ones with throbbing pain?"

C. "Have you ever been 'frozen' in one spot, unable to move?"

While completing an admission assessment for a client with unstable angina, which closed ended question should the nurse ask about the client's chest pain? A. Tell me about the activities that cause your pain? B. When did you first notice the pain in your chest? C. Does your pain occur when walking short decrease? D. How do you feel when the pain becomes noticeable?

C. Does your pain occur when walking short decrease?

A client's serum potassium test level is 6 mEq/L. The laboratory indicates, "Specimen is hemolyzed." What action should the nurse take? A. Encourage the client to eat a banana B. Notify the healthcare provider of the laboratory finding C. Draw a new blood specimen D. Obtain a prescription for sodium polystyrene sulfonate (Kayexalate)

C. Draw a new blood specimen

The nurse evaluates that nursing interventions to promote airway clearance in a patient admitted with COPD are successful based on which of the following findings? A. Absence of dyspnea B. Improved mental status C. Effective and productive coughing D. PaO2 within normal range for the patient

C. Effective and productive coughing

A client with carcinoma of the lung is complaining of weakness and has a serum sodium level of 117 mEq/L. Which nursing problem should the nurse include in this client's plan of care? A. Altered urinary elimination B. Impaired gas exchange C. Fluid volume excess D. Decreased cardiac output

C. Fluid volume excess

A client with ulcerative colitis is admitted to the medical unit during an acute exacerbation. The nurse should instruct the unlicensed assistive personnel (UAP) to report which finding related to the client's bowel movements? A. Hard pellets of stool B. Clay-colored stool C. Stool with fatty streaks D. Blood in the stool

C. Stool with fatty streaks

To reduce the risk for pulmonary complication for a client with Amyotrophic Lateral Sclerosis (ALS), what interventions should the nurse implement? (Select all that apply) A. Initiate passive range of motion exercises B. Establish a regular bladder routine C. Teach the client breathing exercises D. Perform chest physiotherapy E. Encourage use of incentive spirometer

C. Teach the client breathing exercises D. Perform chest physiotherapy E. Encourage use of incentive spirometer

The nurse observes a newly-employed unlicensed assistive personnel (UAP) taking an elderly client's blood pressure. The nurse says "You need to start over. The blood pressure reading you obtained was falsely high for this client." What is the most likely explanation for the erroneous reading? A. The UAP was standing above the sphygmomanometer while taking the reading B. The sphygmomanometer was two feet away from the UAP during the procedure C. The size of the cuff used was too small for this adult client's arm D. The client's arm was elevated above the level of the heart

C. The size of the cuff used was too small for this adult client's arm

A client is being discharged following radioactive seed implantation for prostate cancer. What is the most important information that the nurse should provide to this client's family? A.Follow exposure precautions. B.Encourage regular meals. C.Collect all urine. D.Avoid touching the client.

Clients being treated for prostate cancer with radioactive seed implants should be instructed regarding the amount of time and distance needed to prevent excessive exposure (A) that would pose a hazard to others. (B) is a good suggestion to promote adequate nutrition but is not as important as (A). (C) is unnecessary. Contact with the client (D) IS permitted but should be BRIEF to limit radiation exposure.

A client has a staging procedure for cancer of the breast and ask the nurse which type of breast cancer has the poorest prognosis. Which information should the nurse offer the client? A) Stage II. B) Invasive infiltrating ductal carcinoma. C) T1N0M0. D) Inflammatory with peau d'orange.

D

A client with cirrhosis develops increasing pedal edema and ascites. What dietary modification is most important for the nurse to teach this client? A) Avoid high carbohydrate foods. B) Decrease intake of fat soluble vitamins. C) Decrease caloric intake. D) Restrict salt and fluid intake.

D

A client with diabetes mellitus is experiencing polyphagia. Which outcome statement is the priority for this client? A) Fluid and electrolyte balance. B) Prevention of water toxicity. C) Reduced glucose in the urine. D) Adequate cellular nourishment.

D

A client with gastroesophageal reflux disease (GERD) has been experiencing severe reflux during sleep. Which recommendation by the nurse is most effective to assist the client? A) Losing weight. B) Decreasing caffeine intake. C) Avoiding large meals. D) Raising the head of the bed on blocks.

D

A female client is brought to the clinic by her daughter for a flu shot. She has lost significant weight since the last visit. She has poor personal hygiene and inadequate clothing for the weather. The client states that she lives alone and denies problems or concerns. What action should the nurse implement? A) Notify social services immediately of suspected elderly abuse. B) Discuss the need for mental health counseling with the daughter. C) Explain to the client that she needs to take better care of herself. D) Collect further data to determine whether self-neglect is occurring.

D

Despite several eye surgeries, a 78-year-old client who lives alone has persistent vision problems. The visiting nurse is discussing painting the house with the client. The nurse suggests that the edge of the steps should be painted which color? A) Black. B) White. C) Light green. D) Medium yellow.

D

How should the nurse position the electrodes for modified chest lead one (MCL I) telemetry monitoring? A) Positive polarity right shoulder, negative polarity left shoulder, ground left chest nipple line. B) Positive polarity left shoulder, negative polarity right chest nipple line, ground left chest nipple line. C) Positive polarity right chest nipple line, negative polarity left chest nipple line, ground left shoulder. D) Negative polarity left shoulder, positive polarity right chest nipple line, ground left chest nipple line.

D

In preparing a discharge plan for a 22-year-old male client diagnosed with Buerger's disease (thromboangiitis obliterans), which referral is most important? A) Genetic counseling. B) Twelve-step recovery program. C) Clinical nutritionist. D) Smoking cessation program.

D

The nurse is receiving report from surgery about a client with a penrose drain who is to be admitted to the postoperative unit. Before choosing a room for this client, which information is most important for the nurse to obtain? A) If suctioning will be needed for drainage of the wound. B) If the family would prefer a private or semi-private room. C) If the client also has a Hemovac® in place. D) If the client's wound is infected.

D

Two days postoperative, a male client reports aching pain in his left leg. The nurse assesses redness and warmth on the lower left calf. What intervention should be most helpful to this client? A) Apply sequential compression devices (SCDs) bilaterally. B) Assess for a positive Homan's sign in each leg. C) Pad all bony prominences on the affected leg. D) Advise the client to remain in bed with the leg elevated.

D

What types of medications should the nurse expect to administer to a client during an acute respiratory distress episode? A) Vasodilators and hormones. B) Analgesics and sedatives. C) Anticoagulants and expectorants. D) Bronchodilators and steroids.

D

When providing discharge teaching for a client with osteoporosis, the nurse should reinforce which home care activity? A) A diet low in phosphates. B) Skin inspection for bruising. C) Exercise regimen, including swimming. D) Elimination of hazards to home safety.

D

What is a priority nursing intervention to prevent falls for an older adult client with multiple chronic diseases? A) Requesting that a family member remain with the client to assist in ambulation B) Keeping all four siderails up while the client is in bed C) Placing the client in restraints to prevent movement without assistance D) Providing assistance to the client in getting out of the bed or chair

D Advanced age and multiple illnesses, particularly those that result in alterations in sensation, such as diabetes, predispose this client to falls. The nurse should provide assistance to the client with transfer and ambulation to prevent falls. The client should not be restrained or maintained on bedrest without adequate indication. Although family members are encouraged to visit, their presence around the clock is not necessary at this point.

A client who had a stroke is receiving clopidogrel (Plavix). Which adverse effect does the nurse monitor for in this client? A) New-onset confusion B) Repeated syncope C) Abdominal distention D) Spontaneous ecchymosis

D Clopidogrel (Plavix) is an antiplatelet medication that can cause bleeding, bruising, and liver dysfunction. The nurse should be alert for signs of bleeding, such as ecchymosis, bleeding gums, and tarry stools. Plavix does not cause syncope, confusion, or abdominal distention.

The nurse assesses a client with pneumonia and notes decreased lung sounds on the left side and decreased lung expansion. What is the nurse's best action? A) Increase oxygen flow to 10 L/min. B) Perform an arterial blood gas analysis. C) Have the client cough and deep breathe. D) Check oxygen saturation and notify the health care provider.

D Decreased lung sounds and decreased lung expansion could indicate the development of a complication such as empyema or pus in the pleural space. The nurse should check the client's oxygen saturation and notify the provider. Infection can also move into the bloodstream and result in sepsis, so quick treatment is needed.

The nurse is starting a client on digoxin (Lanoxin) therapy. What intervention is essential to teach this client? A) "Increase your intake of foods high in potassium." B) "Avoid taking aspirin or aspirin-containing products." C) "Hold this medication if your pulse rate is below 80 beats/min." D) "Do not take this medication within 1 hour of taking an antacid."

D Gastrointestinal absorption of digoxin is erratic. Many medications, especially antacids, interfere with its absorption. Clients are taught to hold their digoxin for bradycardia; a heart rate of 80 is too high for this cutoff.

During orientation to an emergency department, the nurse educator would be concerned if the new nurse listed which of the following as a risk factor for impaired thermoregulation? A) Temperature extremes B) Occupational exposure C) Impaired cognition D) Physical agility

D Physical agility is not a risk factor for impaired thermoregulation. The nurse educator would use this information to plan additional teaching to include medical conditions and gait disturbance as risk factors for hypothermia, because their bodies have a reduced ability to generate heat. Impaired cognition is a risk factor. Recreational or occupational exposure is a risk factor. Temperature extremes are risk factors for impaired thermoregulation.

The new nurse is caring for a client with a high temperature. Which action should the nurse perform FIRST? A) Obtaining a fan from central supply for the client's room B) Monitoring the client's temperature more often than ordered C) Sponging the client while monitoring for shivering D) Apply cool packs to the client's axillae and groin

D The use of fans is discouraged to promote cooling in a febrile client because the fan can disperse pathogens. The other actions are appropriate.

Which statement made by a nurse represents the need for further education regarding pain management in older adult clients? a. older adults tend to report pain less often than younger adults b. older clients usually have more experience with pain than younger clients c. older adults are at greatest risk for under treated pain d. older clients have a different pain mechanism and do not feel it as much

D There is no evidence to support the idea that older adult clients perceive pain any differently than younger clients. The other statements are accurate regarding older clients and pain.

A central venous catheter has been inserted via a jugular vein, and a radiograph has confirmed placement of the catheter. A prescription has been received for a medication STAT, but IV fluids have not yet been started. Which action should the nurse take prior to administering the prescribed medication? A.Assess for signs of jugular venous distention. B.Obtain the needed intravenous solution. C.Flush the line with heparinized solution. D.Flush the line with normal saline.

D Medication can be administered via a central line without additional IV fluids. The line should first be flushed with a normal saline solution (D) to ensure patency. Insufficient evidence exists on the effectiveness of flushing catheters with heparin (C). (A) will not affect the decision to administer the medication and is not a priority. Administration of the medication STAT is of greater priority than (B).

A practical nurse (PN) tells the charge nurse in a long-term facility that she does not want to be assigned to one particular resident. She reports that the male client keeps insisting that she is his daughter and begs her to stay in his room. What is the best managerial decision? A.Notify the family that the resident will have to be discharged if his behavior does not improve. B.Notify administration of the PN's insubordination and need for counseling about her statements. C.Ask the PN what she has done to encourage the resident to believe that she is his daughter. D.Reassign the PN until the resident can be assessed more completely for reality orientation.

D Temporary reassignment (D) is the best option until the resident can be examined and his medications reviewed. He may have worsening cerebral dysfunction from an infection or electrolyte imbalance. (A) is not the best option because the family cannot control the resident's actions. The administration may need to know about the situation, but not as a case of insubordination (B). Implying that the PN is somehow creating the situation is inappropriate until a further evaluation has been conducted (C).

The nurse notes that the client's drainage has decreased from 50 to 5 mL/hr 12 hours after chest tube insertion for hemothorax. What is the best initial action for the nurse to take?

D. Assess for kinks or dependent loops in the tubing.

A client who is admitted to the hospital is suspected of having meningitis. The nurse should plan to prepare the client for which diagnostic test? A. CT scan of brain B. Electroencephalogram (EEG) C. Synovial fluid analysis D. Lumbar puncture

D. Lumbar puncture

The nurse plans to administer 1 teaspoon of a liquid medication to a toddler. What is the most accurate way to administer the medication? A. Give medication using a medication dropper B. Administer from a clean teaspoon C. Use a medicine cup to measure the dose D. Measure the medication in an oral syringe

D. Measure the medication in an oral syringe

The nurse is preparing a client for discharge from the hospital following a liver transplant. Which intervention is most important for the nurse to include in this client's discharge teaching plan? A. Keep a record of daily urinary output B. Report the onset of scleral jaundice C. Measure the abdominal girth daily D. Monitor for an elevated temperature

D. Monitor for an elevated temperature

After receiving lactulose, a client with hepatic encephalopathy has several loose stools. What action should the nurse implement? A. Send a stool specimen to the lab B. Measure abdominal girth C. Encourage increased fiber in diet D. Monitor mental status

D. Monitor mental status

A female client who received a nephrotoxic drug is admitted with acute renal failure and asks the nurse if she will need dialysis for the rest of her life. Which pathophysiologic consequence should the nurse explain that supports the need for temporary dialysis until acute tubular necrosis subsides?

D. Nephron obstruction

When providing care for a client following bronchoscopy, which assessment finding should he nurse immediately report to the healthcare provider? A. Slight blood-tinged sputum B. Dyspnea and dysphagia C. Sore throat and hoarseness D. No gag reflex after thirty minutes

D. No gag reflex after thirty minutes

In evaluating effectiveness of postoperative client's intermittent pneumatic compression device, which assessment is most important for the nurse to complete? A. Palpate all peripheral pulse points for volume and strength B. Monitor the amount of drainage from the client's incision C. Evaluate the client's ability to use an incentive spirometer D. Observe both lower extremities for redness and swelling

D. Observe both lower extremities for redness and swelling

Which of the following nursing interventions is most appropriate to enhance oxygenation in a patient with unilateral malignant lung disease? A. Positioning patient on right side. B. Maintaining adequate fluid intake C. Performing postural drainage every 4 hours D. Positioning patient with "good lung down"

D. Positioning patient with "good lung down"

The nurse assesses a client who has been prescribed furosemide (Lasix) for cardiac disease. Which electrocardiographic change would be a concern for a client taking a diuretic?

D. Presence of a U wave

The nurse determines that the patient is not experiencing adverse effects of albuterol (Proventil) after noting which of the following patient vital signs? A. Oxygen saturation 96% B. Respiratory rate of 18 C. Temperature of 98.4° F D. Pulse rate of 76

D. Pulse rate of 76

Following laser trabeculoplasty surgery for open-angle glaucoma, the client reports acute pain deep within the eye. What action should the nurse take? A. Apply bilateral eye shields to reduce photosensitivity B. Administer an antiemetic to prevent vomiting C. Begin postoperative prophylactic antibiotics D. Report the complaint of eye pain to the surgeon

D. Report the complaint of eye pain to the surgeon

The nurse receives the client's next scheduled bag of TPN labeled with the additive NPH insulin. Which action should the nurse implement?

D. Return the solution to the pharmacy.

When preparing to start change-of-shift report, the charge nurse observes an unlicensed assistive personnel (UAP) walking in the hallway with a urine specimen that is not covered. After telling the UAP to cover the specimen, what intervention should the charge nurse implement? A. Ask the nurse educator to review infection control policies with all UAPs. B. Gather input from other staff about the quality of the UAP's performance. C. Direct the UAP to collect all specimens using universal precautions D. Review infection control guidelines with the UAP at the next opportunity

D. Review infection control guidelines with the UAP at the next opportunity

Beginning with the vena cavae, what is the normal sequence of blood circulation through the heart? A. Mitral valve B. Left atrium C. Pulmonary semilunar valve D. Right atrium E. Tricuspid valve F. Aortic semilunar valve G. Left ventricle H. Right ventricle

D. Right atrium E. Tricuspid valve H. Right ventricle C. Pulmonary semilunar valve B. Left atrium A. Mitral valve G. Left ventricle F. Aortic semilunar valve

The nurse is teaching the importance of an exercise regime that includes walking daily for a group of clients with asthma, chronic bronchitis, and emphysema at a pulmonary rehabilitation clinic. Which rationale should the nurse include when motivating the clients?

Daily exercise and walking enhances cardiovascular fitness

A male client with a history of asthma reports having episodes of bronchoconstriction and increased mucous production while exercising. Which action should the nurse implement?

Determine if the client is using an inhaler before exercising

In assessing a client for complications of total parenteral nutrition, it is most important for the nurse to monitor which lab value regularly?

Glucose

The nurse is caring for a patient receiving 1000mL of PN solution over 24 hours. When it is time to change the solution, 150mL remain in the bottle. The most appropriate action by the nurse is to

Hang a new solution and discard the unused solution.

When obtaining a health history from a patient with possible cancer of the mouth, the nurse would expect the patient to report?

Heavy tobacco and alcohol use.

A male client with arterial peripheral vascular disease (PVD) complains of pain in his feet. Which instruction should the nurse give to the unlicensed assistive personnel (UAP) to quickly relieve the client's pain?

Help the client to dangle his legs.

An 81-year-old male client has emphysema. He lives at home with his cat and manages self-care with no difficulty. When making a home visit, the nurse notices that this client's tongue is somewhat cracked and his eyeballs appear sunken into his head. What nursing intervention is indicated?

Help the client to determine ways to increase his fluid intake.

BCG or Bacille Calmette-Guerin

Immigrants have been vaccinate in parts of the world with high TB. Do not give TB test only chest x-ray.

A client is placed on a mechanical ventilator following a cerebral hemorrhage, and vecuronium bromide (Norcuron) 0.04 mg/kg every 12 hours IV is prescribed. What is the priority nursing diagnosis for this client?

Impaired communication related to paralysis of skeletal muscles

Nursing responsibility for ultrasound

Instruct patient to be NPO 8-12 hour before ultrasound. Air or gas can reduce quality of images. Food intake can cause gallbladder contraction, resulting in suboptimal study.

During a health fair, a male client with emphysema tells the nurse that he fatigues easily. Assessment reveals marked clubbing of the fingernails and an increased anteroposterior chest diameter. Which instruction is best to provide the client?

Pace your activities and schedule rest periods

A client with an acute exacerbation of rheumatoid arthritis (RA) has localized pain and inflammation of the fingers and feet; swelling, redness, and restricted joint motion; and reports feeling fatigued. Which nursing diagnosis has the highest priority for this client?

Pain related to joint inflammation

During the assessment of a patient with cobalamin deficiency, the nurse would expect to find that the patient has?

Paresthesia of the hands and feet

A 63-year-old client with type 2 diabetes mellitus is admitted for treatment of an ulcer on the heel of the left foot that has not healed with conventional wound care. The nurse observes that the entire left foot is darker in color than the right foot. Which additional symptom should the nurse expect to find?

Pedal pulses will be weak or absent in the left foot.

The nurse knows that clients taking diuretics must be assessed for the development of hypokalemia, and that hypokalemia will create changes in the client's normal ECG tracing. Which ECG change would be an expected finding in the client with hypokalemia?

Presence of a U wave

The nurse is planning care for a client with diabetes mellitus who has gangrene of the toes to the mid-foot. Which goal should be included in this client's plan of care?

Prevent infection

A female practical nurse (PN) who is a valued employee tells the charge nurse in a long-term facility that she does not want to be assigned to one particular resident. She reports that the male client keeps insisting that she is his daughter and begs her to stay in his room. What is the best managerial decision?

Reassign the PN until the resident can be assessed more completely for reality orientation.

The nurse knows that normal lab values expected for an adult may vary in an older client. Which data would the nurse expect to find when reviewing laboratory values of an 80-year-old man who is in good health overall?

Urinalysis reveals slight protein in the urine and bacteriuria with pyuria.

An older client is admitted after falling while walking. The left leg is externally rotated and shorter than the right leg, and the client is having severe pain and tingling in the left foot. The nurse is unable to palpate the left pedal pulses. Which action is most important for the nurse to implement?

Use a doppler to assess bilateral pedal pulses

A woman who works as a data entry clerk is concerned as to how recent diagnosis of Raynaud's syndrome is going to affect her job performance. Which instruction should the nurse provide this client?

Use a space heater to keep the workspace warm

The nurse is assessing a male client with acute pancreatitis. Which finding requires the most immediate intervention by the nurse?

While the nurse is taking the client's blood pressure, he has a carpal spasm.

On examining a patient 8 hours after formation of a colostomy, the nurse would expect to find

a brick-red, puffy stoma that oozes blood.

Ambulatory Center day of surgery

explain to the patient events such as patient registration, parking, what to wear, what to bring, and the need to have a responsible adult present for transportation home after the surgery.

INH

latent TB and treat 6 to 9 months.

A patient is admitted to the emergency department with acute abdominal pain. The nursing intervention that should be implemented first is

measurement of vital signs

Following a Billroth II procedure, a patient develops dumping syndrome. The nurse explains that the symptoms associated with this problem are caused by

movement of fluid into the small bowel because concentrated food and fluids move rapidly into the intestine.

Ketamine

post surgery monitor for respiratory depression, hypotension, and hallucinations.

A person who has petechiae on skin what are you going to ask that they use?

salicylates.

The nurse is assessing a client with left-sided heart failure. What conditions does the nurse assess for? (Select all that apply.) A) S3/S4 summation gallop B) Cough worsens at night C) Dependent edema D) Pulmonary crackles E) Confusion, restlessness F) Pulmonary hypertension

A,B,D,E Left-sided failure occurs with a decrease in contractility of the heart or an increase in afterload. Most of the signs will be noted in the respiratory system. Right-sided failure occurs with problems from the pulmonary vasculature onward. Signs will be noted before the right atrium or ventricle.

A patient is having inspiratory stridor (crowing respiration) and the nurse suspects he is experiencing a laryngospasm. Which of the following would be most appropriate to implement for a patient experiencing a laryngospasm? A. Administer 100% oxygen. B. Position the patient in high Fowler's position. C. Insert a 16-gauge (large-bore) IV needle. D. Activate the emergency response team (code blue team) to the patient's room.

A. Administer 100% oxygen

An elderly male client tells the nurse that he often wakes up during the night. What action should the nurse implement first? A. Ask the client to describe what happens when he awakens B. Encourage the client to describe his bedtime routines and habits C. Instruct the client to keep a daily sleep journal for one week D. Reassure the client that sleep needs often decrease with age

A. Ask the client to describe what happens when he awakens

A male client with Herpes zoster (shingles) on his thorax tells the nurse that he is having difficulty sleeping. What is the probable etiology of this problem? A. Pain B. Nocturia C. Dyspnea D. Frequent cough

A. Pain

A 15-year-old client with a spinal cord injury develops spastic leg tremors, sweating, and a headache. Which action should the nurse implement? A. Palpate the bladder for distention B. Obtain an oxygen saturation level C. Administer a prescribed analgesic D. Encourage dorsiflexion of the feet

A. Palpate the bladder for distention

Three days after a female client with multiple sclerosis (MS) is admitted to the hospital with a severe urinary tract infection, she reports experiencing double vision. Which intervention should the nurse implement? A. Patch one eye and then the other every few hours B. Encourage bedrest until the diplopia is resolved C. Instruct the client to limit intake of oral fluids D. Administer artificial tear drops to both eyes

A. Patch one eye and then the other every few hours

The nurse is caring for a one-week infant who has a ventriculoperitoneal (VP) shunt that was placed 2 days after birth. Which findings are an indication of a postoperative complication? (select all that apply) A. Poor feeding and vomiting B. Leakage of CSF form the incisional site C. Hyperactive bowel sounds D. Abdominal distension E. White blood cell count of 10, 000/mm^3

A. Poor feeding and vomiting B. Leakage of CSF form the incisional site D. Abdominal distension

42. Select all that apply. Which of the following nursing actions can help clear tracheobronchial secretions in a patient with cystic fibrosis? A. Postural drainage B. Suppressing the cough C. Ensuring adequate hydration D. Administering mucolytic aerosols E. Encouraging the patient to lie flat F. Administering water-soluble vitamins

A. Postural drainage C. Ensuring adequate hydration D. Administering mucolytic aerosols

During assessment of a client in the intensive care unit, the nurse notes that the client's breath sounds are clear upon auscultation, but jugular vein distention and muffled heart sounds are present. Which intervention should the nurse implement? A. Prepare the client for a pericardial tap. B. Administer intravenous furosemide (Lasix). C. Assist the client to cough and deep breathe. D. Instruct the client to restrict oral fluid intake.

A. Prepare the client for a pericardial tap.

While assessing a client's chest tube (CT), the nurse discovers bubbling in the water seal chamber of the chest tube collections device. The client's vital sign are: blood pressure 80/40 mmHg, heart rate 120 beats/minute, respiratory rate 32 breaths/minute, oxygen saturation 88%. Which interventions should the nurse implement? (select all that apply) A. Provide supplemental oxygen B. Auscultate bilateral lung fields C. Administer a nebulizer treatment D. Reinforce occlusive CT dressing E. Give PRN dose of pain medication

A. Provide supplemental oxygen B. Auscultate bilateral lung fields D. Reinforce occlusive CT dressing

43. Select all that apply. Which of the following is included in a comprehensive respiratory assessment? A. Pulse oximetry B. Chest auscultation C. Apical radial pulse D. Nail-bed assessment E. Evaluation of respiratory effort F. Rate and character of respirations

A. Pulse oximetry B. Chest auscultation D. Nail-bed assessment E. Evaluation of respiratory effort F. Rate and character of respirations

After a hospitalization for Syndrome of Inappropriate Antidiuretic Hormone (SIADH), a client develops pontine myselinolysis. Which intervention should the nurse implement first? A. Reorient client to his room B. Place a patch on one eye C. Evaluate client's ability to swallow D. Perform range of motion exercises

A. Reorient client to his room

A client diagnosed with angina pectoris complains of chest pain while ambulating in the hallway. Which action should the nurse implement first? A. Support the client to a sitting position. B. Ask the client to walk slowly back to the room. C. Administer a sublingual nitroglycerin tablet. D. Provide oxygen via nasal cannula.

A. Support the client to a sitting position.

After attending a class on reducing cancer risk factors, a client selects bran flakes with 2% milk and orange slices from a breakfast menu. In evaluating the client's learning, the nurse affirms that the client has made good choices and makes what additional recommendation?

A. Switch to skim milk.

Which instruction should the nurse include in the discharge teaching for a client who has gastroesophageal reflux? A. Teach the client to elevate the head of the bed on blocks B. Remind the client to avoid high-fiber foods C. Encourage the client to lie down and rest after meals. D. Instruct the client to use antacids only as a last resort

A. Teach the client to elevate the head of the bed on blocks

The nurse is caring for a client with a chest tube to water seal drainage that was inserted 10 days ago because of a ruptured bullae and pneumothorax. Which finding should the nurse report to the healthcare provider before the chest tube is removed? A. Tidal of water in the water seal chamber B. Bilateral muffled breath sounds at bases C. Temperature of 101 degrees F D. Absence of chest tube drainage for 2 days.

A. Tidal in the water seal chamber

A client in septic shock jas a double lumen central venous catheter with one liter of 0.9% Normal Saline solution infusing at 1 ml/hour through one lumen and Total Parenteral Nutrition (TPN) Infusing at 50 ml/hour through one port. The nurse prepares newly prescribed IV antibiotics that should take 45 minutes to infuse. What intervention should the nurse implement? A. Use a secondary port of the Normal Saline solution to administer the antibiotic B. Add the antibiotic to the TPN solution, and continue the normal saline solution C. Stop the TPN infusion for the time needed to administer the prescribed antibiotic D. Add the antibiotic to the Normal Saline solution and continue both infusions

A. Use a secondary port of the Normal Saline solution to administer the antibiotic

A patient with acute exacerbation of COPD needs to receive precise amounts of oxygen. Which of the following types of equipment should the nurse prepare to use? A. Venturi mask B. Partial non-rebreather mask C. Oxygen tent D. Nasal cannula

A. Venturi mask

The nurse is investigating a client injury that occurred when a mechanical lift malfunctioned while moving a client from the bed to a orthopedic-chair. Which question should the nurse ask first? A. Was the equipment used according to policy? B. Which day of the week did the injury occur? C. When was the lift last checked by maintenance? D. What mitigating circumstances led to the injury?

A. Was the equipment used according to policy?

A nurse is working on a respiratory care unit where many of the patients are affected by asthma. Which of the following actions by the nurse would most likely increase respiratory difficulty for the patients? A. Wearing perfume to work B. Encouraging patients to ambulate daily C. Allowing the patients to eat green leafy vegetables D. Withholding antibiotic therapy until cultures are obtained

A. Wearing perfume to work

The nurse is caring for a client with a lower left lobe pulmonary abscess. Which position should the nurse instruct the client to maintain? A. left lateral B. Supine, knees flexed C. Dorsal recumbent D. Knee-chest

A. left lateral

Crackles heard on lung auscultation indicate which of the following? 1. Cyanosis. 2. Bronchospasm. 3. Airway narrowing. 4.Fluid-filled alveoli.

4 Crackles are auscultated over fluid-filled alveoli. Crackles heard on lung auscultation do not have to be associated with cyanosis. Bronchospasm and airway narrowing generally are associated with wheezing sounds.

The nurse is assessing a 48-year-old client with a history of smoking during a routine clinic visit. The client, who exercises regularly, reports having pain in the calf during exercise that disappears at rest. Which of the following findings requires further evaluation? 1. Heart rate 57 bpm. 2. SpO2 of 94% on room air. 3. Blood pressure 134/82. 4. Ankle-brachial index of 0.65.

4 An Ankle-Brachial Index of 0.65 suggests moderate arterial vascular disease in a client who is experiencing intermittent claudication. A Doppler ultrasound is indicated for further evaluation. The bradycardic heart rate is acceptable in an athletic client with a normal blood pressure. The SpO2 is acceptable; the client has a smoking history.

The nurse is assigned a group of patients. Which patient would the nurse identify as being at increased risk for impaired gas exchange? A patient: A) with a hemoglobin of 8.5 g/dL B) with a blood glucose of 350 mg/dL C) who has been on anticoagulants for 10 days D) with a heart rate of 100 beats/min and blood pressure of 100/60

A The hemoglobin is low (anemia), therefore the ability of the blood to carry oxygen is decreased. High blood glucose and/or anticoagulants do not alter the oxygen carrying capacity of the blood. A heart rate of 100 beats/min and blood pressure of 100/60 are not indicative of oxygen carrying capacity of the blood.

What interrelated constructs facilitate a nurse to become culturally competent? A) Cultural desire, self-awareness, cultural knowledge, and cultural skill B) Cultural desire, self-awareness, cultural knowledge, and cultural diversity C) Cultural desire, self-awareness, cultural knowledge, and cultural identity D) Cultural diversity, self-awareness, cultural skill, and cultural knowledge

A The process of cultural competence consists of four interrelated constructs: cultural desire, self-awareness, cultural knowledge, and cultural skill. Cultural diversity in the context of health care refers to achieving the highest level of health care for all people by addressing societal inequalities and historical and contemporary injustices. Cultural identity is the norms, values, beliefs, and behaviors of a culture learned through families and group members.

Twelve hours after chest tube insertion for hemothorax, the nurse notes that the client's drainage has decreased from 50 ml/hr to 5 ml/hr. What is the best initial action for the nurse to take?

Assess for kinks or dependent loops in the tubing.

A male client receives a local anesthetic during surgery. During the post-operative assessment, the nurse notices the client is slurring his speech. Which action should the nurse take? A) Determine the client is anxious and allow him to sleep. B) Evaluate his blood pressure, pulse, and respiratory status. C) Review the client's pre-operative history for alcohol abuse. D) Continue to monitor the client for reactivity to anesthesia.

B

The nurse is caring for an overweight client who gained 10 pounds during the previous 2 weeks. The client states that she is hungry all the time and doesn't understand why. Which assessment finding could explain the client's weight gain and hunger? A) The client's glycosylated hemoglobin level is 6%. B) The client started taking dexamethasone (Decadron) daily. C) The client started taking naproxen sodium (Naprosyn) daily. D) The client's thyroxine (T4) level is 8 mcg/dL.

B Dexamethasone is a corticosteroid. These drugs alter carbohydrate, protein, and lipid metabolism, predisposing the client to obesity when taken on a long-term basis. In addition, corticosteroids increase the client's appetite. Naprosyn is an NSAID, which can lead to gastric upset and ulceration and decreased appetite and weight loss. The client's glycosylated hemoglobin and thyroid levels are within normal limits and would not explain the hunger and weight gain.

The nurse is monitoring a client with hypoglycemia. Glucagon provides which function? A) It enhances the activity of insulin, restoring blood glucose levels to normal more quickly after a high-calorie meal. B) It prevents hypoglycemia by promoting release of glucose from liver storage sites. C) It is a storage form of glucose and can be broken down for energy when blood glucose levels are low. D) It converts excess glucose into glycogen, lowering blood glucose levels in times of excess.

B Glycogen is a counterregulatory hormone secreted by the alpha cells of the pancreas when blood glucose levels are low. The actions of glycogen that raise blood glucose levels include stimulating the liver to break down glycogen (glycogenolysis) and forming new glucose from protein breakdown (gluconeogenesis). The other statements are not accurate descriptions of the actions of glucagon.

A client with chronic obstructive pulmonary disease (COPD) reports social isolation. What does the nurse encourage the client to do? A) Participate in community activities. B) Verbalize his or her thoughts and feelings. C) Ask the client's physician for an antianxiety agent. D) Join a support group for people with COPD.

B Many clients with moderate to severe COPD become socially isolated because they are embarrassed by frequent coughing and mucus production. They also can experience fatigue, which limits their activities. The nurse needs to encourage the client to verbalize thoughts and feelings so that appropriate interventions can be selected. Joining a support group would not decrease feelings of social isolation if the client does not verbalize feelings. Antianxiety agents will not help the client with social isolation. Encouraging a client to participate in activities without verbalizing concerns also would not be an effective strategy for decreasing social isolation.

A nurse is caring for several clients. Which client does the nurse assess most carefully for hyperkalemia? a. client with type 2 diabetes taking an oral anti-diabetic agent b. client with heart failure using a salt substitute c. client taking a thiazide diuretic for hypertension d. client taking non-steroidal anti-inflammatory drugs daily

B Many salt substitutes are composed of potassium chloride. Heavy use cna contribute to the development of hyperkalemia. The client should be taught to read labels and to choose a salt substitute that does not contain potassium. NSAIDs promote the retention of sodium but not potassium.

The priority nursing intervention for a patient suspected to be hypothermic would be to: A) hydrate with intravenous (IV) fluids. B) remove wet clothes. C) assess vital signs. D) provide a warm blanket.

B The first thing to do with a patient suspected to be hypothermic is to remove wet clothes, because heat loss is five times greater when clothing is wet. Assessing vital signs is important, but the wet clothes should be removed first. Hydration is very important with hyperthermia and the associated danger of dehydration, but there is not a similar risk with hypothermia. A warm blanket over wet clothes would not be an effective warming strategy.

What is the most important nursing priority for a client who has been admitted for a possible kidney stone? A.Reducing dairy products in the diet B.Straining all urine C.Measuring intake and output D.Increasing fluid intake

B Straining all urine (B) is the most important nursing action to take in this case. Encouraging fluid intake (D) is important for any client who may have a kidney stone, but is even more important to strain all urine. Straining urine will enable the nurse to determine when the kidney stone has been passed and may prevent the need for surgery. (C) is not the highest priority action. (A) is usually not recommended until the stone is obtained and the content of the stone is determined. Even then, dietary restrictions are controversial.

Which assessment is most important for the nurse to include in the daily plan of care for a client with a burned extremity? A. Range of motion B. Distal pulse intensity C. Extremity sensation D. Presence of exudate

B. Distal pulse intensity

A 55-year-old male client is admitted to the coronary care unit having suffered an acute myocardial infarction (MI). Within 24 hours of the occurrence, the nurse can expect to find which systemic sign? A. Elevated serum amylase level B. Elevated CM-MB level C. Prolonged prothrombin time (PT) D. Elevated serum BUN and creatinine

B. Elevated CM-MB level

Which of the following is a factor significant in the development of anemia in men? A. Condom use B. Large hemorrhoids C. A diet high in cholesterol D. Smoking one pack of cigarettes daily

B. Large hemorrhoids

The nurse is assessing a male client with acute pancreatitis. Which finding requires the most immediate intervention by the nurse?

B. While the nurse is taking the client's blood pressure, he has a carpal spasm.

A 75-year-old obese patient who is snoring loudly and having periods of apnea several times each night is most likely experiencing A. narcolepsy. B. sleep apnea. C. sleep deprivation. D. paroxysmal nocturnal dyspnea.

B. sleep apnea.

Which client is at highest risk of compromised immunity? A) Client with extreme anxiety B) Client who is awaiting surgery C) Client who has just had surgery D) Client who just delivered a baby

C Intact skin is a defense to prevent infection; however, a client who has recently had surgery has a portal for organisms to enter the body and cause infection.

What symptom is characteristic of ureteral colic in the clients diagnosed with renal calculi? A. Symptoms of irritation associated with urinary tract infection B. Intense, deep ache in the costovertebral region C. Acute, excruciating, wave-like pain radiating to the genitalia D. Chills, fever, and dysuria

C. Acute, excruciating, wave-like pain radiating to the genitalia

When assessing a patient's respiratory status, which of the following nonrespiratory data are most important for the nurse to obtain? A. Height and weight B. Neck circumference C. Occupation and hobbies D. Usual daily fluid intake

C. Occupation and hobbies

Absorption of vitamin B12 may be decreased in older adults because of decreased A. intestinal motility. B. production of bile by the liver. C. production of intrinsic factor by the stomach. D. synthesis of cobalamin (vitamin B12) by intestinal bacteria.

C. production of intrinsic factor by the stomach.

The nursery nurse identifies a newborn at significant risk for hypothermic alteration in thermoregulation because the patient is: A) large for gestational age. B) well nourished. C) born at term. D) low birth weight.

D Low birth weight and poorly nourished infants (particularly premature infants) and children are at greatest risk for hypothermia. A large for gestational age infant would not be malnourished. An infant born at term is not considered at significant risk. A well nourished infant is not at significant risk.

The nurse is teaching a client who has recently given birth about immunity that has been passed to the newborn. Which statement by the client indicates that additional teaching is needed? A) "My baby received some antibodies from me before birth, and I will give him more when I breast-feed." B) "I had the measles, so my baby will be protected against it until he is old enough to receive the MMR vaccine." C) "Only certain antibodies were able to cross the placenta to protect my baby." D) "I had chickenpox and am immune to it, so my baby will not need to have the chickenpox vaccine."

D The baby receives passive immunity from antibodies that are passed through the placenta in utero. Maternal passive immunity is temporary and will last for only a short time after birth.

The nurse is teaching a female client who uses a contraceptive diaphragm about reducing the risk for toxic shock syndrome (TSS). Which information should the nurse include? (Select all that apply.) A) Remove the diaphragm immediately after intercourse. B) Wash the diaphragm with an alcohol solution. C) Use the diaphragm to prevent conception during the menstrual cycle. D) Do not leave the diaphragm in place longer than 8 hours after intercourse. E) Contact a healthcare provider a sudden onset of fever grater than 101º F appears. F) Replace the old diaphragm every 3 months.

D, E

An older client is admitted with a diagnosis of bacterial pneumonia. The nurse's assessment of the client will most likely reveal which S/SX? A. Leukocytosis and febrile. B. Polycythemia and crackles. C. Pharyngitis and sputum production. D. Confusion and tachycardia.

D. Confusion and tachycardia.

The nurse-manager of a pediatric units needs to assign a room for a 6-month-old diagnosed with respiratory syncytial virus (RSV). Which room assignment is the best for the nurse-manager to make? A. Private room furtherest form the nurses station B. Double room with a 6-month-old on droplet precautions C. Private room with negative air pressure D. Double room with a 4-month-old who has RSV

D. Double room with a 4-month-old who has RSV

After admitting a patient to the medical unit with a diagnosis of pneumonia, the nurse will verify that which of the following physician orders have been completed before administering a dose of cefotetan (Cefotan) to the patient? A. Serum laboratory studies ordered for AM B. Pulmonary function evaluation C. Orthostatic blood pressures D. Sputum culture and sensitivity

D. Sputum culture and sensitivity

Nurse anethetist role

Performing and documenting a preanesthetic assessment and evaluation. Developing and implementing a plan for delivering anesthesia. Anesthetic technique. Administer the anesthesia, adjuvant drugs, and fluids. Noninvasive and invasive monitoring devices. Manage pt. airway and pulmonary status. Manage emergence and recovery from anesthesia. Pain relief. Responding to emergency situations.

The nurse is developing standards of care for a client with gastroesophageal reflux disease and wants to review current evidence for practice. Which one of the following resources will provide the most helpful information? 1. A review in the Cochrane Library. 2. A literature search in a database, such as the Cumulative Index to Nursing and Allied Health Literature (CINAHL). 3. An online nursing textbook. 4. The policy and procedure manual at the health care agency.

1 . The Cochrane Library provides systematic reviews of health care interventions and will provide the best resource for evidence for nursing care. CINAHL offers key word searches to published articles in nursing and allied health literature, but not reviews. A nursing textbook has information about nursing care, which may include evidence-based practices, but textbooks may not have the most up-to-date information. While the policy and procedure manual may be based on evidence-based practices, the most current practices will be found in evidence-based reviews of literature.

The nurse is teaching the client about home blood glucose monitoring. Which of the following blood glucose measurements indicates hypoglycemia? 1. 59 mg/dL (3.3 mmol/L). 2. 75 mg/dL (4.2 mmol/L). 3. 108 mg/dL (6 mmol/L). 4. 119 mg/dL (6.6 mmol/L).

1 Although some individual variation exists, when the blood glucose level decreases to less than 70 mg/dL (3.9 mmol/L), the client experiences or is at risk for hypoglycemia. Hypoglycemia can occur in both type 1 and type 2 diabetes mellitus, although it is more common when the client is taking insulin. The nurse should instruct the client on the prevention, detection, and treatment of hypoglycemia.

The client probably has not progressed to an advanced stage. 98. The nurse is developing a discharge plan about home care with a client who has lymphoma. The nurse should emphasize which of the following? 1. Use analgesics as needed. 2. Take a shower with perfumed shower gel. 3. Wear a mask when outside of the home. 4. Take an antipyretic every morning.

1 Analgesics are used as needed to relieve painful encroachment of enlarged lymph nodes. Perfumed shower gel will increase pruritus. Wearing a mask does not protect the client from infection if pathogens are not spread by airborne droplets. Antipyretics should be used to treat fever symptomatically after infection is ruled out.

When monitoring a client who is receiving tissue plasminogen activator (t-PA), the nurse should have resuscitation equipment available because reperfusion of the cardiac tissue can result in which of the following? 1. Cardiac arrhythmias. 2. Hypertension. 3. Seizure. 4. Hypothermia.

1 Cardiac arrhythmias are commonly observed with administration of t-PA. Cardiac arrhythmias are associated with reperfusion of the cardiac tissue. Hypotension is commonly observed with administration of t-PA. Seizures and hypothermia are not generally associated with reperfusion of the cardiac tissue.

The client with acute lymphocytic leukemia (ALL) is at risk for infection. The nurse should: 1. Place the client in a private room. 2. Have the client wear a mask. 3. Have staff wear gowns and gloves. 4. Restrict visitors

1 Clients with ALL are at risk for infection due to granulocytopenia. The nurse should place the client in a private room. Strict hand-washing procedures should be enforced and will be the most effective way to prevent infection. It is not necessary to have the client wear a mask. The client is not contagious and the staff does not need to wear gloves. The client can have visitors; however, they should be screened for infection and use hand-washing procedures.

The client admitted with peripheral vascular disease (PVD) asks the nurse why her legs hurt when she walks. The nurse bases a response on the knowledge that the main characteristic of PVD is: 1. Decreased blood flow. 2. Increased blood flow. 3. Slow blood flow. 4. Thrombus formation.

1 Decreased blood flow is a common characteristic of all PVD. When the demand for oxygen to the working muscles becomes greater than the supply, pain is the outcome. Slow blood flow throughout the circulatory system may suggest pump failure. Thrombus formation can result from stasis or damage to the intima of the vessels.

The nurse is assessing an older adult with a pacemaker who leads a sedentary lifestyle. The client reports being unable to perform activities that require physical exertion. The nurse should further assess the client for which of the following? 1. Left ventricular atrophy. 2. Irregular heartbeats. 3. Peripheral vascular occlusion. 4. Pacemaker placement

1 In older adults who are less active and do not exercise the heart muscle, atrophy can result. Disuse or deconditioning can lead to abnormal changes in the myocardium of the older adult. As a result, under sudden emotional or physical stress, the left ventricle is less able to respond to the increased demands on the myocardial muscle. Decreased cardiac output, cardiac hypertrophy, and heart failure are examples of the chronic conditions that may develop in response to inactivity, rather than in response to the aging process. Irregular heartbeats are generally not associated with an older sedentary adult's lifestyle. Peripheral vascular occlusion or pacemaker placement should not affect response to stress.

The client asks the nurse to explain what it means that his Hodgkin's disease is diagnosed at stage 1A. Which of the following describes the involvement of the disease? 1. Involvement of a single lymph node. 2. Involvement of two or more lymph nodes on the same side of the diaphragm. 3. Involvement of lymph node regions on both sides of the diaphragm. 4. Diffuse disease of one or more extralymphatic organs

1 In the staging process, the designations A and B signify that symptoms were or were not present when Hodgkin's disease was found, respectively. The Roman numerals I through IV indicate the extent and location of involvement of the disease. Stage I indicates involvement of a single lymph node; stage II, two or more lymph nodes on the same side of the diaphragm; stage III, lymph node regions on both sides of the diaphragm; and stage IV, diffuse disease of one or more extralymphatic organs.

In providing care to the client with leukemia who has developed thrombocytopenia, the nurse assesses the most common sites for bleeding. Which of the following is not a common site? 1. Biliary system. 2. Gastrointestinal tract. 3. Brain and meninges. 4. Pulmonary system.

1 The biliary system is not especially prone to hemorrhage. Thrombocytopenia (a low platelet count) leaves the client at risk for a potentially life-threatening spontaneous hemorrhage in the gastrointestinal, respiratory, and intracranial cavities.

A male client expresses concern about how a hypophysectomy will affect his sexual function. Which of the following statements provides the most accurate information about the physiologic effects of hypophysectomy? 1. Removing the source of excess hormone should restore the client's libido, erectile function, and fertility. 2. Potency will be restored, but the client will remain infertile. 3. Fertility will be restored, but impotence and decreased libido will persist. 4. Exogenous hormones will be needed to restore erectile function after the adenoma is removed.

1 The client's sexual problems are directly related to the excessive prolactin level. Removing the source of excessive hormone secretion should allow the client to return gradually to a normal physiologic pattern. Fertility will return, and erectile function and sexual desire will return to baseline as hormone levels return to normal.

A client with toxic shock has been receiving ceftriaxone sodium (Rocephin), 1 g every 12 hours. In addition to culture and sensitivity studies, which other laboratory findings should the nurse monitor? 1. Serum creatinine. 2. Spinal fluid analysis. 3. Arterial blood gases. 4. Serum osmolality.

1 The nurse monitors the blood levels of antibiotics, white blood cells, serum creatinine, and blood urea nitrogen because of the decreased perfusion to the kidneys, which are responsible for filtering out the Rocephin. It is possible that the clearance of the antibiotic has been decreased enough to cause toxicity. Increased levels of these laboratory values should be reported to the physician immediately. A spinal fluid analysis is done to examine cerebral spinal fluid, but there is no indication of central nervous system involvement in this case. Arterial blood gases are used to determine actual blood gas levels and assess acid-base balance. Serum osmolality is used to monitor fluid and electrolyte balance.

A client with a suspected diagnosis of Hodgkin's disease is to have a lymph node biopsy. The nurse should make sure that personnel involved with the procedure do which of the following when obtaining the lymph node biopsy specimen for histologic examination for this client? 1. Maintain sterile technique. 2. Use a mask, gloves, and a gown when assisting with the procedure. 3. Send the specimen to the laboratory when someone is available to take it. 4. Ensure that all instruments used are placed in a sealed and labeled container.

1 The nurse must ensure that sterile technique is used when a biopsy is obtained because the client is at high risk for infection. In most cases, a lymph node biopsy is sent immediately to the laboratory once it is placed in a specific solution in a closed container. It is not necessary to wear a gown and mask when obtaining the specimen. It is not necessary to use special handling procedures for the instruments used.

The best indicator that the client has learned how to give an insulin self injection correctly is when the client can: 1. Perform the procedure safely and correctly. 2. Critique the nurse's performance of the procedure. 3. Explain all steps of the procedure correctly. 4. Correctly answer a posttest about the procedure

1 The nurse should judge that learning has occurred from the evidence of a change in the client's behavior. A client who performs a procedure safely and correctly demonstrates that he has acquired a skill. Evaluation of this skill acquisition requires performance of that skill by the client with observation by the nurse. The client must also demonstrate cognitive understanding, as shown by the ability to critique the nurse's performance. Explaining the steps demonstrates acquisition of knowledge at the cognitive level only. A posttest does not indicate the degree to which the client has learned a psychomotor skill.

Which of the following is a priority goal for the diabetic client who is taking insulin and has nausea and vomiting from a viral illness or influenza? 1. Obtaining adequate food intake. 2. Managing own health. 3. Relieving pain. 4. Increasing activity.

1 The priority goal for the client with diabetes mellitus who is experiencing vomiting with influenza is to obtain adequate nutrition. The diabetic client should eat small, frequent meals of 50 g of carbohydrate or food equal to 200 cal every 3 to 4 hours. If the client cannot eat the carbohydrates or take fluids, the health care provider should be called or the client should go to the emergency department. The diabetic client is in danger of complications with dehydration, electrolyte imbalance, and ketoacidosis. Increasing the client's health management skills is important to lifestyle behaviors, but it is not a priority during this acute illness of influenza. Pain relief may be a need for this client, but it is not the priority at this time; neither is increasing activity during the illness.

Which of the following findings is the best indication that fluid replacement for the client in hypovolemic shock is adequate? 1. Urine output greater than 30 mL/h. 2. Systolic blood pressure greater than 110 mm Hg. 3. Diastolic blood pressure greater than 90 mm Hg. 4. Respiratory rate of 20 breaths/min.

1 Urine output provides the most sensitive indication of the client's response to therapy for hypovolemic shock. Urine output should be consistently greater than 35 mL/h. Blood pressure is a more accurate reflection of the adequacy of vasoconstriction than of tissue perfusion. Respiratory rate is not a sensitive indicator of fluid balance in the client recovering from hypovolemic shock.

The nurse is planning care for a client who is diagnosed with peripheral vascular disease (PVD) and has a history of heart failure. The nurse should develop a plan of care that is based on the fact that the client may have a low tolerance for exercise related to: 1. Decreased blood flow. 2. Increased blood flow. 3. Decreased pain. 4. Increased blood viscosity.

1 A client with PVD and heart failure will experience decreased blood flow. In this situation, low exercise tolerance (oxygen demand becomes greater than the oxygen supply) may be related to less blood being ejected from the left ventricle into the systemic circulation. Decreased blood supply to the tissues results in pain. Increased blood viscosity may be a component, but it is of much less importance than the disease processes.

A client with heart failure has bilateral +4 edema of the right ankle that extends up to midcalf. The client is sitting in a chair with the legs in a dependent position. Which of the following goals is the priority? 1. Decrease venous congestion. 2. Maintain normal respirations. 3. Maintain body temperature. 4. Prevent injury to lower extremities.

1 Decreasing venous congestion in the extremities is a desired outcome for clients with heart failure. The nurse should elevate the client's legs above the level of the heart to achieve this goal. The client is not demonstrating difficulty breathing or being cold. The nurse should prevent injury to the swollen extremity; however, this is not the priority.

The nurse is caring for a client who recently experienced a myocardial infarction and has been started on clopidogrel (Plavix). The nurse should develop a teaching plan that includes which of the following points? Select all that apply. 1. The client should report unexpected bleeding or bleeding that lasts a long time. 2. The client should take Plavix with food. 3. The client may bruise more easily and may experience bleeding gums. 4. Plavix works by preventing platelets from sticking together and forming a clot. 5. The client should drink a glass of water after taking Plavix.

1,3,4 Plavix is generally well absorbed and may be taken with or without food; it should be taken at the same time every day and, while food may help prevent potential GI upset, food has no effect on absorption of the drug. Bleeding is the most common adverse effect of Plavix; the client must understand the importance of reporting any unexpected, prolonged, or excessive bleeding including blood in urine or stool. Increased bruising and bleeding gums are possible side effects of Plavix; the client should be aware of this possibility. Plavix is an antiplatelet agent used to prevent clot formation in clients that have experienced or are at risk for myocardial infarction, ischemic stroke, peripheral artery disease, or acute coronary syndrome. It is not necessary to drink a glass of water after taking Plavix.

A client is receiving dopamine hydrochloride for treatment of shock. The nurse should: 1. Administer pain medication concurrently. 2. Monitor blood pressure continuously. 3. Evaluate arterial blood gases at least every 2 hours. 4. Monitor for signs of infection.

2 The client who is receiving dopamine hydrochloride requires continuous blood pressure monitoring with an invasive or noninvasive device. The nurse may titrate the IV infusion to maintain a systolic blood pressure of 90 mm Hg. Administration of a pain medication concurrently with dopamine hydrochloride, which is a potent sympathomimetic with dose-related alpha-adrenergic agonist, beta 1-selective adrenergic agonist, and dopaminergic blocking effects, is not an essential nursing action for a client who is in shock with already low hemodynamic values. Arterial blood gas concentrations should be monitored according to the client's respiratory status and acidbase balance status and are not directly related to the dopamine hydrochloride dosage. Monitoring for signs of infection is not related to the nursing action for the client receiving dopamine hydrochloride.

A client is scheduled to have an arteriogram. During the arteriogram, the client reports having nausea, tingling, and dyspnea. The nurse's immediate action should be to: 1. Administer epinephrine. 2. Inform the physician. 3. Administer oxygen. 4. Inform the client that the procedure is almost over.

2 .Clients may have an immediate or a delayed reaction to the radiopaque dye. The physician should be notified immediately because the symptoms suggest an allergic reaction. Treatment may involve administering oxygen and epinephrine. Explaining that the procedure is over does not address the current symptoms

The nurse explains to the client with Hodgkin's disease that a bone marrow biopsy will be taken after the aspiration. What should the nurse explain about the biopsy? 1. "Your biopsy will be performed before the aspiration because enough tissue may be obtained so that you won't have to go through the aspiration." 2. "You will feel a pressure sensation when the biopsy is taken but should not feel actual pain; if you do, tell the doctor so that you can be given extra numbing medicine." 3. "You may hear a crunch as the needle passes through the bone, but when the biopsy is taken, you will feel a suction-type pain that will last for just a moment." 4. "You will be shaved and cleaned with an antiseptic agent, after which the doctor will inject a needle without making an incision to aspirate out the bone marrow."

2 A biopsy needle is inserted through a separate incision in the anesthetized area. The client will feel a pressure sensation when the biopsy is taken but should not feel actual pain. The client should be instructed to inform the physician if pain is felt so that more anesthetic agent can be administered to keep the client comfortable. The biopsy is performed after the aspiration and from a slightly different site so that the tissue is not disturbed by either test. The client will feel a suction-type pain for a moment when the aspiration is being performed, not the biopsy. A small incision is made for the biopsy to accommodate the larger-bore needle. This may require a stitch

The nurse instructs the unlicensed nursing personnel (UAP) on how to provide oral hygiene for clients who cannot perform this task for themselves. Which of the following techniques should the nurse tell the UAP to incorporate into the client's daily care? 1. Assess the oral cavity each time mouth care is given and record observations. 2. Use a soft toothbrush to brush the client's teeth after each meal. 3. Swab the client's tongue, gums, and lips with a soft foam applicator every 2 hours. 4. Rinse the client's mouth with mouthwash several times a day.

2 A soft toothbrush should be used to brush the client's teeth after every meal and more often as needed. Mechanical cleaning is necessary to maintain oral health, stimulate gingiva, and remove plaque. Assessing the oral cavity and recording observations is the responsibility of the nurse, not the nursing assistant. Swabbing with a safe foam applicator does not provide enough friction to clean the mouth. Mouthwash can be a drying irritant and is not recommended for frequent use.

A client has driven himself to the emergency department. He is 50 years old, has a history of hypertension, and informs the nurse that his father died from a heart attack at age 60. The client has indigestion. The nurse connects him to an electrocardiogram monitor and begins administering oxygen at 2 L/min per nasal cannula. The nurse's next action should be to: 1. Call for the physician. 2. Start an IV infusion. 3. Obtain a portable chest radiograph. 4. Draw blood for laboratory studies

2 Advanced cardiac life support recommends that at least one or two IV lines be inserted in one or both of the antecubital spaces. Calling the physician, obtaining a portable chest radiograph, and drawing blood for the laboratory are important but secondary to starting the IV line.

The client is a survivor of non-Hodgkin's lymphoma. Which of the following statements indicates the client needs additional information? 1. "Regular screening is very important for me." 2. "The survivor rate is directly proportional to the incidence of second malignancy." 3. "The survivor rate is indirectly proportional to the incidence of second malignancy." 4. "It is important for survivors to know the stage of the disease and their current treatment plan."

2 It is incorrect that the survivor rate is directly proportional to the incidence of second malignancy. The survivor rate is indirectly proportional to the incidence of second malignancy, and regular screening is very important to detect a second malignancy, especially acute myeloid leukemia or myelodysplastic syndrome. Survivors should know the stage of the disease and their current treatment plan so that they can remain active participants in their health care.

Which of the following clinical manifestations does the nurse most likely observe in a client with Hodgkin's disease? 1. Difficulty swallowing. 2. Painless, enlarged cervical lymph nodes. 3. Difficulty breathing. 4. A feeling of fullness over the liver.

2 Painless and enlarged cervical lymph nodes, tachycardia, weight loss, weakness and fatigue, and night sweats are signs of Hodgkin's disease. Difficulty swallowing and breathing may occur, but only with mediastinal node involvement. Hepatomegaly is a late-stage manifestation.

A client has peripheral vascular disease (PVD) of the lower extremities. The client tells the nurse, "I've really tried to manage my condition well." Which of the following routines should the nurse evaluate as having been appropriate for this client? 1. Resting with the legs elevated above the level of the heart. 2. Walking slowly but steadily for 30 minutes twice a day. 3. Minimizing activity. 4. Wearing antiembolism stockings at all times when out of bed

2 Slow, steady walking is a recommended activity for clients with peripheral vascular disease because it stimulates the development of collateral circulation. The client with PVD should not remain inactive. Elevating the legs above the heart or wearing antiembolism stockings is a strategy for alleviating venous congestion and may worsen peripheral arterial disease

The nurse is assessing the client's use of medications. Which of the following medications may cause a complication with the treatment plan of a client with diabetes? 1. Aspirin. 2. Steroids. 3. Sulfonylureas. 4. Angiotensin-converting enzyme (ACE) inhibitors

2 Steroids can cause hyperglycemia because of their effects on carbohydrate metabolism, making diabetic control more difficult. Aspirin is not known to affect glucose metabolism. Sulfonylureas are oral hypoglycemic agents used in the treatment of diabetes mellitus. ACE inhibitors are not known to affect glucose metabolism.

When assessing a client for early septic shock, the nurse should assess the client for which of the following? 1. Cool, clammy skin. 2. Warm, flushed skin. 3. Increased blood pressure. 4. Hemorrhage.

2 Warm, flushed skin from a high cardiac output with vasodilation occurs in warm shock or the hyperdynamic phase (first phase) of septic shock. Other signs and symptoms of early septic shock include fever with restlessness and confusion; normal or decreased blood pressure with tachypnea and tachycardia; increased or normal urine output; and nausea and vomiting or diarrhea. Cool, clammy skin occurs in the hypodynamic or cold phase (later phase). Hemorrhage is not a factor in septic shock.

The nurse identifies deficient knowledge when the client undergoing induction therapy for leukemia makes which of the following statements? 1. "I will pace my activities with rest periods." 2. "I can't wait to get home to my cat!" 3. "I will use warm saline gargle instead of brushing my teeth." 4. "I must report a temperature of 100°F (37.7°C)."

2 The nurse identifies that the client does not understand that contact with animals must be avoided because they carry infection and the induction therapy will destroy the client's white blood cells (WBCs). The induction therapy will cause anemia, and the client will experience fatigue and will have to pace activities with rest periods. Platelet production will be decreased, and the client will be at risk for bleeding tendencies; oral hygiene will have to be provided by using a warm saline gargle instead of brushing the teeth and gums. The client will be at risk for infection owing to the decrease in WBC production and should report a temperature of 100°F (37.8°C) or higher.

The nurse is teaching a client about risk factors associated with atherosclerosis and how to reduce the risk. Which of the following is a risk factor that the client is not able to modify? 1. Diabetes. 2. Age. 3. Exercise level. 4. Dietary preferences

2 Age is a nonmodifiable risk factor for atherosclerosis. The nurse instructs the client to manage modifiable risk factors such as comorbid diseases (eg, diabetes), activity level, and diet. Controlling serum blood glucose levels, engaging in regular aerobic activity, and choosing a diet low in saturated fats can reduce the risk of developing atherosclerosis.

The client with Hodgkin's disease develops B symptoms. These manifestations indicate which of the following? 1. The client has a low-grade fever (temperature lower than 100°F [37.8°C]). 2. The client has a weight loss of 5% or less of body weight. 3. The client has night sweats. 4. The client probably has not progressed to an advanced stage.

3 A temperature higher than 100.4°F (38°C), profuse night sweats, and an unintentional weight loss of 10% of body weight represent the cluster of clinical manifestations known as the B symptoms. Forty percent of clients with Hodgkin's disease have B symptoms, and B symptoms are more common in advanced stages of the disease.

Which of the following lipid abnormalities is a risk factor for the development of atherosclerosis and peripheral vascular disease? 1. Low concentration of triglycerides. 2. High levels of high-density lipid (HDL) cholesterol. 3. High levels of low-density lipid (LDL) cholesterol. 4. Low levels of LDL cholesterol.

3 An increased LDL cholesterol concentration has been documented as a risk factor for the development of atherosclerosis. LDL cholesterol is not broken down in the liver but is deposited into the intima of the blood vessels. Low triglyceride levels are desirable. High HDL and low LDL levels are beneficial and are known to be protective for the cardiovascular system.

The client with Hodgkin's disease undergoes an excisional cervical lymph node biopsy under local anesthesia. After the procedure, which does the nurse assess first? 1. Vital signs. 2. The incision. 3. The airway. 4. Neurologic signs.

3 Assessing for an open airway is always first. The procedure involves the neck; the anesthesia may have affected the swallowing reflex, or the inflammation may have closed in on the airway, leading to ineffective air exchange. Once a patent airway is confirmed and an effective breathing pattern established, the circulation is checked. Vital signs and the incision are assessed as soon as possible, but only after it is established that the airway is patent and the client is breathing normally. A neurologic assessment is completed as soon as possible after other important assessments.

. A middle-aged adult with a family history of CAD has the following: total cholesterol 198 (11 mmol/L); LDL cholesterol 120 (6.7 mmol/L); HDL cholesterol 58 (3.2 mmol/L); triglycerides 148 (8.2 mmol/L); blood sugar 102 (5.7 mmol/L); and Creactive protein (CRP) 4.2. The health care provider prescribes a statin medication and aspirin. The client asks the nurse why these medications are needed. Which is the best response by the nurse? 1. "The labs indicate severe hyperlipidemia and the medications will lower your LDL, along with a low-fat diet." 2. "The triglycerides are elevated and will not return to normal without these medications." 3. "The CRP is elevated indicating inflammation seen in cardiovascular disease, which can be lowered by the medications prescribed." 4. "These medications will reduce the risk of type 2 diabetes."

3 CRP is a marker of inflammation and is elevated in the presence of cardiovascular disease. The high sensitivity CRP (hs-CRP) is the blood test for greater accuracy in measuring the CRP to evaluate cardiovascular risk. The family history, postmenopausal age, LDL above optimum levels, and elevated CRP place the client at risk of CAD. Statin medications can decrease LDL, whereas statins and aspirin can reduce CRP and decrease the risk of MI and stroke. The blood sugar is within normal limits.

A client is receiving Cilostazol (Pletal) for peripheral arterial disease causing intermittent claudication. The nurse determines this medication is effective when the client reports which of the following? 1. "I am having fewer aches and pains." 2. "I do not have headaches anymore." 3. "I am able to walk further without leg pain." 4. "My toes are turning grayish black in color."

3 Cilostazol is indicated for management of intermittent claudication. Symptoms usually improve within 2 to 4 weeks of therapy. Intermittent claudication prevents clients from walking for long periods of time. Cilostazol inhibits platelet aggregation induced by various stimuli and improving blood flow to the muscles and allowing the client to walk long distances without pain. Peripheral arterial disease causes pain mainly of the leg muscles. "Aches and pains" does not specify exactly where the pain is occurring. Headaches may occur as a side effect of this drug, and the client should report this information to the health care provider. Peripheral arterial disease causes decreased blood supply to the peripheral tissues and may cause gangrene of the toes; the drug is effective when the toes are warm to the touch and the color of the toes is similar to the color of the body.

To help minimize the risk of postoperative respiratory complications after a hypophysectomy, during preoperative teaching, the nurse should instruct the client how to: 1. Use incentive spirometry. 2. Turn in bed. 3. Take deep breaths. 4. Cough.

3 Deep breathing is the best choice for helping prevent atelectasis. The client should be placed in the semi-Fowler's position (or as prescribed) and taught deep breathing, sighing, mouth breathing, and how to avoid coughing. Blow bottles are not effective in preventing atelectasis because they do not promote sustained alveolar inflation to maximal lung capacity. Frequent position changes help loosen lung secretions, but deep breathing is most important in preventing atelectasis. Coughing is contraindicated because it increases intracranial pressure and can cause cerebrospinal fluid to leak from the point at which the sella turcica was entered.

After a myocardial infarction, the hospitalized client is taught to move the legs while resting in bed. The expected outcome of this exercise is to: 1. Prepare the client for ambulation. 2. Promote urinary and intestinal elimination. 3. Prevent thrombophlebitis and blood clot formation. 4. Decrease the likelihood of pressure ulcer formation.

3 Encouraging the client to move the legs while in bed is a preventive strategy taught to all clients who are hospitalized and on bed rest to promote venous return. The muscular action aids in venous return and prevents venous stasis in the lower extremities. These exercises are not intended to prepare the client for ambulation. These exercises are not associated with promoting urinary and intestinal elimination. These exercises are not performed to decrease the risk of pressure ulcer formation

A client is undergoing a bone marrow aspiration and biopsy. What is the best way for the nurse to help the client and two upset family members handle anxiety during the procedure? 1. Allow the client's family to stay as long as possible. 2. Stay with the client without speaking. 3. Encourage the client to take slow, deep breaths to relax. 4. Allow the client time to express feelings.

3 Encouraging the client to take slow, deep breaths during uncomfortable parts of procedures is the best method of decreasing the stress response of tightening and tensing the muscles. Slow, deep breathing affects the level of carbon dioxide in the brain to increase the client's sense of well-being. Allowing the client's family to stay may be appropriate if the family has a calming effect on the client, but this family is upset and may contribute to the client's stress. Silence can be therapeutic, but when the client is faced with a potentially life-threatening diagnosis and a new, invasive procedure, taking deep breaths will be more effective in reducing the stress response. Expressing feelings is important, but deep breathing will promote relaxation; the nurse can encourage the client to express feelings when the procedure is completed.

A nurse is teaching a client with type 1 diabetes mellitus who jogs daily about the preferred sites for insulin absorption. What is the most appropriate site for a client who jogs? 1. Arms. 2. Legs. 3. Abdomen. 4. Iliac crest.

3 If the client engages in an activity or exercise that focuses on one area of the body, that area may cause inconsistent absorption of insulin. A good regimen for a jogger is to inject the abdomen for 1 week and then rotate to the buttock. A jogger may have inconsistent absorption in the legs or arms with strenuous running. The iliac crest is not an appropriate site due to a lack of loose skin and subcutaneous tissue in that area.

. Following diagnosis of angina pectoris, a client reports being unable to walk up two flights of stairs without pain. Which of the following measures would most likely help the client prevent this problem? 1. Climb the steps early in the day. 2. Rest for at least an hour before climbing the stairs. 3. Take a nitroglycerin tablet before climbing the stairs. 4. Lie down after climbing the stairs.

3 Nitroglycerin may be used prophylactically before stressful physical activities such as stair climbing to help the client remain pain free. Climbing the stairs early in the day would have no impact on decreasing pain episodes. Resting before or after an activity is not as likely to help prevent an activity-related pain episode.

Which of the following is the most important goal of nursing care for a client who is in shock? 1. Manage fluid overload. 2. Manage increased cardiac output. 3. Manage inadequate tissue perfusion. 4. Manage vasoconstriction of vascular beds

3 Nursing interventions and collaborative management are focused on correcting and maintaining adequate tissue perfusion. Inadequate tissue perfusion may be caused by hemorrhage, as in hypovolemic shock; by decreased cardiac output, as in cardiogenic shock; or by massive vasodilation of the vascular bed, as in neurogenic, anaphylactic, and septic shock. Fluid deficit, not fluid overload, occurs in shock

The client who experiences angina has been told to follow a low-cholesterol diet. Which of the following meals would be best? 1. Hamburger, salad, and milkshake. 2. Baked liver, green beans, and coffee. 3. Spaghetti with tomato sauce, salad, and coffee. 4. Fried chicken, green beans, and skim milk

3 Pasta, tomato sauce, salad, and coffee would be the best selection for the client following a low-cholesterol diet. Hamburgers, milkshakes, liver, and fried foods tend to be high in cholesterol.

Angiotensin-converting enzyme (ACE) inhibitors may be prescribed for the client with diabetes mellitus to reduce vascular changes and possibly prevent or delay development of: 1. Chronic obstructive pulmonary disease (COPD). 2. Pancreatic cancer. 3. Renal failure. 4. Cerebrovascular accident.

3 Renal failure frequently results from the vascular changes associated with diabetes mellitus. ACE inhibitors increase renal blood flow and are effective in decreasing diabetic nephropathy. Chronic obstructive pulmonary disease is not a complication of diabetes, nor is it prevented by ACE inhibitors. Pancreatic cancer is neither prevented by ACE inhibitors nor considered a complication of diabetes. Cerebrovascular accident is not directly prevented by ACE inhibitors, although management of hypertension will decrease vascular disease.

The nurse is planning care with a client with acute leukemia who has mucositis. The nurse should advise the client that after every meal and every 4 hours while awake the client should use: 1. Lemon-glycerin swabs. 2. A commercial mouthwash. 3. A saline solution. 4. A commercial toothpaste and brush

3 Simple rinses with saline or a baking soda and water solution are effective and moisten the oral mucosa. Commercial mouthwashes and lemon-glycerin swabs contain glycerin and alcohol, which are drying to the mucosa and should be avoided. Brushing after each meal is recommended, but every 4 hours may be too traumatic. During acute leukemia, the neutrophil and platelet counts are often low and a soft-bristle toothbrush, instead of the client's usual brush, should be used to prevent bleeding gums.

The client with acute leukemia and the health care team establish mutual client outcomes of improved tidal volume and activity tolerance. Which measure would be least likely to promote outcome achievement? 1. Ambulating in the hallway. 2. Sitting up in a chair. 3. Lying in bed and taking deep breaths. 4. Using a stationary bicycle in the room.

3 The client with acute leukemia experiences fatigue and deconditioning. Lying in bed and taking deep breaths will not help achieve the goals. The client must get out of bed to increase activity tolerance and improve tidal volume. Ambulating in the hall (using a HEPA filter mask if neutropenic) is a sensible activity and helps improve conditioning. Sitting up in a chair facilitates lung expansion. Using a stationary bicycle in the room allows the client to increase activity as tolerated.

Which of the following conditions is the most significant risk factor for the development of type 2 diabetes mellitus? 1. Cigarette smoking. 2. High-cholesterol diet. 3. Obesity. 4. Hypertension.

3 The most important factor predisposing to the development of type 2 diabetes mellitus is obesity. Insulin resistance increases with obesity. Cigarette smoking is not a predisposing factor, but it is a risk factor that increases complications of diabetes mellitus. A high-cholesterol diet does not necessarily predispose to diabetes mellitus, but it may contribute to obesity and hyperlipidemia. Hypertension is not a predisposing factor, but it is a risk factor for developing complications of diabetes mellitus.

The nurse in the intensive care unit is giving a report to the nurse in the post surgical unit about a client who had a gastrectomy. The most effective way to assure essential information about the client is reported is to: 1. Give the report face to face with both nurses in a quiet room. 2. Audiotape the report for future reference and documentation. 3. Use a printed checklist with information individualized for the client. 4. Document essential transfer information in the client's electronic health record

3 Using an individualized printed checklist assures that all key information is reported; the checklist can then serve as a record to which nurses can refer later. Giving a verbal report leaves room for error in memory; using an audiotape or an electronic health record requires nurses to spend unnecessary time retrieving information.

The nurse is caring for a client with peripheral artery disease who has recently been prescribed clopidogrel (Plavix). The nurse understands that more teaching is necessary when the client states which of the following: 1. "I should not be surprised if I bruise easier or if my gums bleed a little when brushing my teeth." 2. "It doesn't really matter if I take this medicine with or without food, whatever works best for my stomach." 3. "I should stop taking Plavix if it makes me feel weak and dizzy." 4. "The doctor prescribed this medicine to make my platelets less likely to stick together and help prevent clots from forming."

3 Weakness, dizziness, and headache are common adverse effects of Plavix and the client should report these to the physician if they are problematic; in order to decrease risk of clot formation, Plavix must be taken regularly and should not be stopped or taken intermittently. The main adverse effect of Plavix is bleeding, which often occurs as increased bruising or bleeding when brushing teeth. Plavix is well absorbed, and while food may help decrease potential gastrointestinal upset, Plavix may be taken with or without food. Plavix is an antiplatelet agent used to prevent clot formation in clients who have experienced or are at risk for myocardial infarction, ischemic stroke, peripheral artery disease, or acute coronary syndrome.

The nurse is unable to palpate the client's left pedal pulses. Which of the following actions should the nurse take next? 1. Auscultate the pulses with a stethoscope. 2. Call the physician. 3. Use a Doppler ultrasound device. 4. Inspect the lower left extremity

3 When pedal pulses are not palpable, the nurse should obtain a Doppler ultrasound device. Auscultation is not likely to be helpful if the pulse isn't palpable. Inspection of the lower extremity can be done simultaneously when palpating, but the nurse should first try to locate a pulse by Doppler. Calling the physician may be necessary if there is a change in the client's condition.

A client has had a pulmonary artery catheter inserted. In performing hemodynamic monitoring with the catheter, the nurse will wedge the catheter to gain information about which of the following? 1. Cardiac output. 2. Right atrial blood flow. 3. Left end-diastolic pressure. 4. Cardiac index

3 When wedged, the catheter is "pointing" indirectly at the left end-diastolic pressure. The pulmonary artery wedge pressure is measured when the tip of the catheter is slowing inflated and allowed to wedge into a branch of the pulmonary artery. Once the balloon is wedged, the catheter reads the pressure in front of the balloon. During diastole, the mitral valve is open, reflecting left ventricular end diastolic pressure. Cardiac output is the amount of blood ejected by the heart in 1 minute and is determined through thermodilution and not wedge pressure. Cardiac index is calculated by dividing the client's cardiac output by the client's body surface area, and is considered a more accurate reflection of the individual client's cardiac output. Right atrial blood pressure is not measured with the pulmonary artery catheter.

The nurse is assessing an older Caucasian male who has a history of peripheral vascular disease. The nurse observes that the man's left great toe is black. The discoloration is probably a result of: 1. Atrophy. 2. Contraction. 3. Gangrene. 4. Rubor.

3 The term gangrene refers to blackened, decomposing tissue that is devoid of circulation. Chronic ischemia and death of the tissue can lead to gangrene in the affected extremity. Injury, edema, and decreased circulation lead to infection, gangrene, and tissue death. Atrophy is the shrinking of tissue, and contraction is joint stiffening secondary to disuse. The term rubor denotes a reddish color of the skin

The nurse is assessing a client with chronic myeloid leukemia (CML). The nurse should assess the client for: 1. Lymphadenopathy. 2. Hyperplasia of the gum. 3. Bone pain from expansion of marrow. 4. Shortness of breath

4 Although the clinical manifestations of CML vary, clients usually have confusion and shortness of breath related to decreased capillary perfusion to the brain and lungs. Lymphadenopathy is rare in CML. Hyperplasia of the gum and bone pain are clinical manifestations of AML.

The goal of nursing care for a client with acute myeloid leukemia (AML) is to prevent: 1. Cardiac arrhythmias. 2. Liver failure. 3. Renal failure. 4. Hemorrhage.

4 Bleeding and infection are the major complications and causes of death for clients with AML. Bleeding is related to the degree of thrombocytopenia, and infection is related to the degree of neutropenia. Cardiac arrhythmias rarely occur as a result of AML. Liver or renal failure may occur, but neither is a major cause of death in AML.

In assessing a client in the early stage of chronic lymphocytic leukemia (CLL), the nurse should determine if the client has: 1. Enlarged, painless lymph nodes. 2. Headache. 3. Hyperplasia of the gums. 4. Unintentional weight loss.

4 Clients with CLL develop unintentional weight loss; fever and drenching night sweats; enlarged, painful lymph nodes, spleen, and liver; decreased reaction to skin sensitivity tests (anergy); and susceptibility to viral infections. Enlarged, painless lymph nodes are a clinical manifestation of Hodgkin's lymphoma. A headache would not be one of the early signs and symptoms expected in CLL because CLL does not cross the blood-brain barrier and would not irritate the meninges. Hyperplasia of the gums is a clinical manifestation of AML.

Assessment of the diabetic client for common complications should include examination of the: 1. Abdomen. 2. Lymph glands. 3. Pharynx. 4. Eyes.

4 Diabetic retinopathy, cataracts, and glaucoma are common complications in diabetics, necessitating eye assessment and examination. The feet should also be examined at each client encounter, monitoring for thickening, fissures, or breaks in the skin; ulcers; and thickened nails. Although assessments of the abdomen, pharynx, and lymph glands are included in a thorough examination, they are not pertinent to common diabetic complications.

When assessing the lower extremities of a client with peripheral vascular disease (PVD), the nurse notes bilateral ankle edema. The edema is related to: 1. Competent venous valves. 2. Decreased blood volume. 3. Increase in muscular activity. 4. Increased venous pressure.

4 In PVD, decreased blood flow can result in increased venous pressure. The increase in venous pressure results in an increase in capillary hydrostatic pressure, which causes a net filtration of fluid out of the capillaries into the interstitial space, resulting in edema. Valves often become incompetent with PVD. Blood volume is not decreased in this condition. Decreased muscular action would contribute to the formation of edema in the lower extremities.

A client with diabetes begins to cry and says, "I just cannot stand the thought of having to give myself a shot every day." Which of the following would be the best response by the nurse? 1. "If you do not give yourself your insulin shots, you will die." 2. "We can teach your daughter to give the shots so you will not have to do it." 3. "I can arrange to have a home care nurse give you the shots every day." 4. "What is it about giving yourself the insulin shots that bothers you?"

4 The best response is to allow the client to verbalize her fears about giving herself a shot each day. Tactics that increase fear are not effective in changing behavior. If possible, the client needs to be responsible for her own care, including giving selfinjections. It is unlikely that the client's insurance company will pay for home-care visits if the client is capable of self-administration.

The client with type 1 diabetes mellitus is taught to take isophane insulin suspension NPH (Humulin N) at 5 PM each day. The client should be instructed that the greatest risk of hypoglycemia will occur at about what time? 1. 11 AM, shortly before lunch. 2. 1 PM, shortly after lunch. 3. 6 PM, shortly after dinner. 4.1 AM, while sleeping

4 The client with diabetes mellitus who is taking NPH insulin (Humulin N) in the evening is most likely to become hypoglycemic shortly after midnight because this insulin peaks in 6 to 8 hours. The client should eat a bedtime snack to help prevent hypoglycemia while sleeping.

. A client with acute myeloid leukemia (AML) reports overhearing one of the other clients say that AML had a very poor prognosis. The client has understood that the client's physician informed the client that his physician told him that he has a good prognosis. Which is the nurse's best response? 1. "You must have misunderstood. Who did you hear that from?" 2. "AML does have a very poor prognosis for poorly differentiated cells." 3. "AML is the most common nonlymphocytic leukemia." 4. "Your doctor stated your prognosis based on the differentiation of your cells."

4 The statement "Your doctor stated your prognosis based on the differentiation of your cells" addresses the client's situation on an individual basis. The nurse is clarifying that clients have different prognoses—even though they may have the same type of leukemia—because of the cell differentiation. Stating that the client misunderstood is inappropriate for an advocate of the client and serves no useful purpose. The other statements are true but do not address this client's individual concern.

Which client is at greatest risk for coronary artery disease? 1. A 32-year-old female with mitral valve prolapse who quit smoking 10 years ago. 2. A 43-year-old male with a family history of CAD and cholesterol level of 158 (8.8 mmol/L). 3. A 56-year-old male with an HDL of 60 (3.3 mmol/L) who takes atorvastatin. 4. A 65-year-old female who is obese with an LDL of 188 (10.4 mmol/L).

4 The woman who is 65 years old, overweight, and has an elevated LDL is at greatest risk. Total cholesterol greater than 200 (11.1 mmol/L), LDL greater than 100 (5.5 mmol/L), HDL less than 40 (2.2 mmol/L) in men, HDL less than 50 (2.8 mmol/L) in women, men 45 years and older, women 55 years and older, smoking and obesity increase the risk of CAD. Atorvastatin reduces LDL and decreases risk of CAD. The combination of postmenopausal, obesity, and high LDL places this client at greatest risk.

The nurse is obtaining the pulse of a client who has had a femoral-popliteal bypass surgery 6 hours ago. (See below) Which assessment provides the most accurate information about the client's postoperative status? 1. radial pulse 2. femoral pulse 3. apical pulse 4. dorsalis pedis pulse

4 The presence of a strong dorsalis pedis pulse indicates that there is circulation to the extremity distal to the surgery indicating that the graft between the femoral and popliteal artery is allowing blood to circulate effectively. Answer 1 shows the nurse obtaining the radial pulse; answer 2 shows the femoral pulse, which is proximal to the surgery site and will not indicate circulation distal to the surgery site. Answer 3 shows the nurse obtaining an apical pulse.

A client who has been taking warfarin has been admitted with severe acute rectal bleeding and the following laboratory results: International Normalized Ratio (INR), 8; hemoglobin, 11 g/dL (110 g/L); and hematocrit, 33% (0.33). In which order should the nurse implement the following physician prescriptions? 1. Give 1 unit fresh frozen plasma (FFP). 2. Administer vitamin K 2.5 mg by mouth. 3. Schedule client for sigmoidoscopy. 4. Administer IV dextrose 5% in 0.45% normal saline solution.

4,1,2,3 Analysis of the client's laboratory results indicate that an INR of 8 is increased beyond therapeutic ranges. The client is also experiencing severe acute rectal bleeding and has a hemoglobin level in the low range of normal and a hematocrit reflecting fluid volume loss. The nurse should first establish an IV line and administer the dextrose in saline. Next the nurse should administer the FFP. FFP contains concentrated clotting factors and provides an immediate reversal of the prolonged INR. Vitamin K 2.5 mg PO should be given next because it reverses the warfarin by returning the PT to normal values. However, the reversal process occurs over 1 to 2 hours. Last, the nurse can schedule the client for the sigmoidoscopy.

After checking the urinary drainage system for kinks in the tubing, the nurse determines that a client who has returned from the post-anesthesia care has a dark, concentrated urinary output of 54 ml for the last 2 hours. What priority nursing action should be implemented? A) Report the findings to the surgeon. B) Irrigate the indwelling urinary catheter. C) Apply manual pressure to the bladder. D) Increase the IV flow rate for 15 minutes.

A

An 81-year-old male client has emphysema. He lives at home with his cat and manages self-care with no difficulty. When making a home visit, the nurse notices that his tongue is somewhat cracked and his eyeballs are sunken into his head. What nursing intervention is indicated? A) Help the client to determine ways to increase his fluid intake. B) Obtain an appointment for the client to see an ear, nose, and throat specialist. C) Schedule an appointment with an allergist to determine if the client is allergic to the cat. D) Encourage the client to slightly increase his use of oxygen at night and to always use humidified oxygen.

A

An adult client is admitted to the hospital burn unit with partial-thickness and full-thickness burns over 40% of the body surface area. In assessing the potential for skin regeneration, what should the nurse remember about full-thickness burns? A) Regenerative function of the skin is absent because the dermal layer has been destroyed. B) Tissue regeneration will begin several days following return of normal circulation. C) Debridement of eschar will delay the body's ability to regenerate normal tissue. D) Normal tissue formation will be preceded by scar formation for the first year.

A

During assessment of a client with amyotrophic lateral sclerosis (ALS), which finding should the nurse identify when planning care for this client? A) Muscle weakness. B) Urinary frequency. C) Abnormal involuntary movements. D) A decline in cognitive function.

A

Small bowel obstruction is a condition characterized by which finding? A) Severe fluid and electrolyte imbalances. B) Metabolic acidosis. C) Ribbon-like stools. D) Intermittent lower abdominal cramping.

A

The nurse is preparing a teaching plan for a client who is newly diagnosed with Type 1 diabetes mellitus. Which signs and symptoms should the nurse describe when teaching the client about hypoglycemia? A) Sweating, trembling, tachycardia. B) Polyuria, polydipsia, polyphagia. C) Nausea, vomiting, anorexia. D) Fruity breath, tachypnea, chest pain.

A

The nurse working in a postoperative surgical clinic is assessing a woman who had a left radical mastectomy for breast cancer. Which factor puts this client at greatest risk for developing lymphedema? A) She sustained an insect bite to her left arm yesterday. B) She has lost twenty pounds since the surgery. C) Her healthcare provider now prescribes a calcium channel blocker for hypertension. D) Her hobby is playing classical music on the piano.

A

The earliest and most sensitive assessment finding that would indicate an alteration in intracranial regulation would be: A) change in level of consciousness. B) unequal pupil size. C) loss of primitive reflexes. D) inability to focus visually.

A A change in level of consciousness is the earliest and most sensitive indication of a change in intracranial processing. This is assessed with the Glasgow Coma Scale (GCS), which assesses eye opening and verbal and motor response. The inability to focus may indicate a change, but it is not one of the earliest indicators or a component of the GCS. Primitive reflexes refers to those reflexes found in a normal infant that disappear with maturation. These reflexes may reappear with frontal lobe dysfunction and may be tested for with a suspected brain injury, so it would be the reappearance of primitive reflexes. A change in pupil size or unequal pupils may indicate a change, but they are not one of the earliest indicators or a component of the GCS.

A client with a pressure ulcer has the following laboratory values: white blood count 8000/mm3, prealbumin 15.2 mg/dL, albumin 4.2 mg/dL, and lymphocyte count 2000/mm3. Which action by the nurse is most appropriate? A) Request a dietary consult. B) Assess the client's vital signs. C) Document the findings. D) Place the client in isolation.

A Albumin, prealbumin, and lymphocyte counts all give information related to nutritional status. The albumin and lymphocyte counts given are normal. The white blood cell count is not directly related to nutritional status. The prealbumin count is low and is a more specific indicator of nutritional status than is the albumin count. This puts the client at risk for impaired wound healing, so the nurse should request a dietary consult.

The nurse is assessing clients on a cardiac unit. Which client does the nurse assess most carefully for developing left-sided heart failure? A) Middle-aged woman with aortic stenosis B) Older woman who smokes cigarettes daily C) Older man who has had a myocardial infarction D) Middle-aged man with pulmonary hypertension

A Although most people with heart failure will have failure that progresses from left to right, it is possible to have left-sided failure alone for a short period. It is also possible to have heart failure that progresses from right to left. Causes of left ventricular failure include mitral or aortic valve disease, coronary artery disease (CAD), and hypertension.

A confused client is hospitalized for possible pneumonia and is admitted from the emergency department with an indwelling catheter in place. During interdisciplinary rounds the following day, what question by the nurse takes priority? A) "Can we discontinue the in-dwelling catheter?" B) "Will the client be able to return home?" C) "Should we get another chest x-ray today?" D) "Do you want daily weights on this client?"

A An in-dwelling catheter dramatically increases the risks of urinary tract infection and urosepsis. Nursing staff should ensure that catheters are left in place only as long as they are medically needed. The nurse should inquire about removing the catheter. All other questions might be appropriate, but because of client safety, this question takes priority.

Which client statement indicates a good understanding regarding antibiotic therapy for recurrent urinary tract infections? A) "Even if I feel completely well, I should take the medication until it is gone." B) "When my urine no longer burns, I will no longer need to take the antibiotics." C) "If my urine becomes lighter and clearer, I can stop taking my medicine." D) "If I have a fever higher than 100° F (37.8° C), I should take twice as much medicine."

A Antibiotic therapy is most effective, especially for recurrent urinary tract infections, when the client takes the prescribed medication for the entire course, not just when symptoms are present. The other statements demonstrate that additional teaching is needed for the client.

The nurse is assessing a client who reports claudication after walking a distance of one block. The nurse notes a painful ulcer on the fourth toe of the client's right foot. What condition do these findings correlate with? A) Peripheral arterial disease B) Deep vein thrombosis C) Diabetic foot ulceration D) Peripheral venous disease

A Arterial disease is characterized by claudication after walking short distances. Ulcerations caused by peripheral arterial disease are painful and initially are located at the most distal points on the extremity. Diabetic ulcers and venous ulcers are seldom painful and usually tend to occur where pressure is applied.

What statement indicates that the client understands teaching about neutropenia? A) "I will call my doctor if I have an increase in temperature." B) "My grandchildren may get an infection from me." C) "I need to use a soft toothbrush." D) "I have to wear a mask at all times."

A Bone marrow suppression leads to neutropenia and increases the client's risk for infection. Decreased numbers of neutrophils and other white blood cells can minimize the clinical manifestations of infection. For this reason, the client may not develop a high temperature, even with severe infection, and any elevation of temperature should be reported immediately to the health care provider. The client does not need to wear a mask or use a soft toothbrush (although if the client has low platelets, he or she should use a soft toothbrush to avoid causing trauma). The client is not contagious.

The nurse is working on a plan of care with her patient which includes turning and positioning and adequate nutrition to help the patient maintain intact skin integrity. The nurse helps the patient to realize that this breaks the chain of infection by eliminating a: A) portal of entry. B) host. C) mode of transmission. D) reservoir.

A Broken or impaired skin creates a portal of entry for pathogens. By maintaining intact tissue, the patient and the nurse have broken the chain of infection by eliminating a portal of entry. Host is incorrect because you are not eliminating the person or organism. Intact tissue does not eliminate the mode of transmission. Skin can still be used to transfer pathogens regardless of it being intact or broken. Intact skin does not eliminate the location for pathogens to live and grow.

A diabetic client has numbness and reduced sensation. Which intervention does the nurse teach this client to prevent injury? A) "Use a bath thermometer to test the water temperature." B) "Examine your feet daily using a mirror." C) "Wear white socks instead of colored socks." D) "Rotate your insulin injection sites."

A Clients with diminished sensory perception can easily experience a burn injury when bath water is too hot. Instead of checking the temperature of the water by feeling it, they should use a thermometer. Examining the feet daily does not prevent injury, although daily foot examinations are important to find problems so they can be addressed. Rotating insulin and wearing white socks also will not prevent injury.

The nurse is assessing a client's understanding of his hypertension therapy. What client statement indicates a need for further teaching? A) "When my blood pressure is normal, I will no longer need to take medication." B) "If my blood pressure stays under control, I will reduce my risk for a heart attack." C) "If I lose weight, I might be able to reduce my blood pressure medication." D) "When getting out of bed in the morning, I will sit for a few moments then stand."

A Compliance with antihypertensive therapy is difficult for two reasons. First, often clients have no distressing symptoms associated with hypertension and may not believe that they have a problem. Second, many clients believe that once blood pressure is brought back into the normal range, they are "cured" and no longer need to take medication. Losing weight might allow the client to reduce medications. Lowering blood pressure does lower risk for heart attack. Because blood pressure medications often lead to orthostatic hypotension, clients should be taught to change position slowly, sitting first before standing after lying flat.

The nurse is caring for a client who has experienced a stroke. Which nursing intervention for nutrition does the nurse implement to prevent complications from cranial nerve IX impairment? A) Place the client in high Fowler's position. B) Verbalize the placement of food on the client's plate. C) Order a clear liquid diet for the client. D) Turn the client's plate around halfway through the meal.

A Cranial nerve IX, the glossopharyngeal nerve, controls the gag reflex. Clients with impairment of this nerve are at great risk for aspiration. The client should be in high Fowler's position and should drink thickened liquids if swallowing difficulties are present. The client would not have vision problems. Turning the plate around would not prevent a complication, nor would limiting the client's diet to clear liquids.

An older adult client presents with signs and symptoms related to dig toxicity. Which age related change may have contributed to this problem? a. decreased renal blood flow b. increased gastrointestinal motility c. decreased ratio of adipose tissue to lean body mass d. increased total body water

A Decreased renal blood flow and reduced glomerular filtration can result in slower medication excretion time, potentially leading to toxic drug accumulation. Aging results in decreased total body water and gastrointestinal motility and an increase in the ratio of adipose tissue to lean body mass, but is not related to dig toxicity.

The nurse is assessing a client with an early onset of multiple sclerosis (MS). Which clinical manifestations does the nurse expect to see? A) Nystagmus & Diplopia B) Hyperresponsive reflexes C) Excessive somnolence D) Heat intolerance

A Early signs and symptoms of MS include changes in motor skills, vision, and sensation. The other manifestations are later signs of MS.

Which statement by a patient indicates additional teaching is required about the medication warfarin? A) "I will increase the intake of green, leafy vegetables for a more healthful diet." B) "I will restrict the intake of foods high in vitamin C." C) "I will increase the amount of protein in my diet to protect my kidneys." D) "I will continue my diabetic diet and restrict sugar."

A Foods such as green, leafy vegetables have high levels of vitamin K. Warfarin is an anticoagulant that acts by interfering with vitamin K-dependent clotting factors. If the amount of vitamin K is increased in the diet, the medication dose may need to be adjusted. A diabetic diet would be continued as indicated for a patient receiving warfarin. Vitamin C is not related to warfarin.

The nurse notes that the handgrip of the client with hypokalemia has diminished since the previous assessment one hour ago. Which intervention by the nurse is the priority? a. assess the client's respiratory rate, rhythm, and depth b. document findings and monitor the client c. measure the client's pulse and blood pressure d. call the health care provider

A In a client with hypokkalemia, progressive skeletal muscle weakness is associated with increasing severity of hypokalemia. The most life-threatening complication of hypokalemia is respiratory insufficiency. It is imperative for the nurse to perform a respiratory assessment first to make sure that the client is not in immediate jeopardy. Next, the nurse would call the health care provider to obtain orders for potassium replacement.

A client with diabetes is prescribed insulin glargine once daily and regular insulin four times daily. One dose of regular insulin is scheduled at the same time as the glargine. How does the nurse instruct the client to administer the two doses of insulin? A) "Draw up and inject the insulin glargine first, then draw up and inject the regular insulin." B) "First draw up the dose of regular insulin, then draw up the dose of insulin glargine in the same syringe, mix, and inject the two insulins together." C) "First draw up the dose of insulin glargine, then draw up the dose of regular insulin in the same syringe, mix, and inject the two insulins together." D) "Draw up and inject the insulin glargine first, wait 20 minutes, then draw up and inject the regular insulin."

A Insulin glargine must not be diluted or mixed with any other insulin or solution. Mixing results in an unpredictable alteration in the onset of action and time to peak action. The correct instruction is to draw up and inject first the glargine, then the regular insulin right afterward.

Which is the highest priority goal to set for a client with pneumonia? A) Maintenance of SaO2 of 95% B) Walking 20 feet three times daily C) Absence of cyanosis D) Absence of confusion

A Maintenance of an SaO2 of at least 95% is a clear goal that indicates that the client has adequate oxygenation. Absence of cyanosis and the presence of confusion are assessment factors that contribute to evaluation of oxygen; however, they are not absolute measures. Likewise, walking three times a day does not directly address oxygenation.

Controlling pain is important to promoting wellness. Unrelieved pain has been associated with a. prolonged stress response and a cascade of harmful effects system wide. b. decreased tumor growth and longevity c. large tidal volumes and decreased lung capacity d. decreased carbohydrate, protein, and fat destruction

A Pain triggers a number of physiologic stress responses in the human body. Unrelieved pain can prolong the stress response and produce a cascade of harmful effects in all body systems. The stress response causes the endocrine system to release excessive amounts of hormones, such as cortisol, catecholamines, and glucagon. Insulin and testosterone levels decrease. Increased endocrine activity in turn initiates a number of metabolic processes, in particular, accelerated carbohydrate, protein, and fat destruction, whcih can result in weight loss, tachycardia, increased respiratory rate, shock, and even death. The immune system is also affected by pain as demonstrated by research showing a link between unrelieved pain and a higher incidence of nosocomial infections and increased tumor growth. Large tidal volumes are not associated with pain while decreased lung capacity is associated with unrelieved pain. Decreased tumor growth and longevity are not associated with unrelieved pain. Decreased carbs, protein, and fat are not associated with pain or stress response.

When a client is assessed, which behavior best indicates that he or she is experiencing changes associated with acute pain? a. inability to concentrate b. expressed hopelessness c. psychosocial withdrawal d. anger and hostility

A The characteristics most common to chronic pain are psychosocial withdrawal, anger and hostility, depression, and hopelessness. The inability to concentrate is associated much more with acute pain, before any physiologic or behavioral adaptation has occurred.

A client who first experienced symptoms related to a confirmed thrombotic stroke 2 hours ago is brought to the intensive care unit. Which prescribed medication does the nurse prepare to administer? A) Tissue plasminogen activator B) Heparin sodium C) Warfarin (Coumadin) D) Gabapentin (Neurontin)

A The client who has had a thrombotic stroke has a 3-hour time frame from the onset of symptoms to receive recombinant tissue plasminogen activator (rt-PA) to dissolve the cerebral artery occlusion and re-establish blood flow. Clients must meet eligibility criteria for administration of this therapy. The other medications do not assist in the re-establishment of blood flow for a client with a confirmed thrombotic stroke.

A client has been admitted to the intensive care unit with worsening pulmonary manifestations of heart failure. What is the nurse's best action? A) Administer loop diuretics as prescribed. B) Begin cardiopulmonary resuscitation (CPR). C) Promote rest and minimize activities. D) Place the client in a high Fowler's position.

A The client with worsening heart failure is most at risk for pulmonary edema as a consequence of fluid retention. Administering diuretics will decrease the fluid overload, thereby decreasing the incidence of pulmonary edema. High Fowler's position might help the client breathe easier but will not solve the problem. CPR is not warranted in this situation. Rest is important for clients with heart failure, but this is not the priority.

The nurse is caring for a female client who is 5 feet, 7 inches tall and weighs 115 pounds. The client asks the nurse if she needs to lose weight. Which response by the nurse is best? A) "No. In fact, your body mass index suggests that you are already underweight." B) "Yes. Your body mass index suggests you are slightly overweight." C) "Your weight is just fine. Don't worry about it." D) "Maybe. Let's look at your risks for cardiovascular disease."

A The client's body mass index (BMI) is 18.0, so she is already underweight. It is inaccurate to tell the client she is overweight, and it is unnecessary to consider her weight in light of any cardiovascular risk factors. The nurse should not reassure the client that her weight is just fine because she is underweight.

A female client is admitted with an exacerbation of ulcerative colitis. Which laboratory value does the nurse correlate with this condition? A) Erythrocyte sedimentation rate (ESR), 55 mm/hr B) Potassium, 5.5 mEq/L C) Sodium, 144 mEq/L D) Hemoglobin, 14.2 g/dL

A The erythrocyte sedimentation rate (ESR) is an indicator of inflammation, which is elevated during an exacerbation of ulcerative colitis. The normal range for the ESR is 0 to 33 mm/hr. Diarrhea caused by ulcerative colitis will result in loss of potassium and hypokalemia with levels lower than 3.5 mEq/L. Bloody diarrhea will lead to anemia, with hemoglobin levels lower than 12 g/dL in females. The sodium level is normal.

The nurse observes a small opening that is draining purulent material on the skin over the trochanter area of a bedridden client. Which is the nurse's next best action? A) Probe for a larger pocket of necrotic tissue. B) Apply alginate dressing daily. C) Apply a transparent film dressing. D) Measure the reddened area on the skin surface.

A This "hidden" wound may first be observed as a small opening in the skin through which purulent drainage exudes. Applying a transparent film dressing would not help this type of wound to heal. Measuring the reddened area would not assist in determining the actual size of the wound, because internal damage has occurred. Alginate dressings could not be applied if the area were not opened.

A client is admitted to the hospital with a diagnosis of severe acute diverticulitis. Which nursing intervention has the highest priority? A.Place the client on NPO status. B.Assess the client's temperature. C.Obtain a stool specimen. D.Administer IV fluids.

A A client with acute severe diverticulitis is at risk for peritonitis and intestinal obstruction and should be made NPO (A) to reduce risk of intestinal rupture. (B, C, and D) are important but are less of a priority than (A), which is implemented to prevent a severe complication.

In caring for a client with acute diverticulitis, which assessment data warrants immediate nursing intervention? A.The client has a rigid hard abdomen and elevated WBC. B.The client has left lower quadrant pain and an elevated temperature. C.The client is refusing to eat any of the meal and is complaining of nausea. D.The client has not had a bowel movement in 2 days and has a soft abdomen.

A A hard rigid abdomen and elevated WBC is indicative of peritonitis (A), which is a medical emergency and should be reported to the health care provider immediately. (B and C) are expected clinical manifestations of diverticulitis. (D) does not warrant immediate intervention.

During the shift report, the charge nurse informs a nurse that she has been assigned to another unit for the day. The nurse begins to sigh deeply and tosses about her belongings as she prepares to leave, making it known that she is very unhappy about being floated to the other unit. What is the best immediate action for the charge nurse to take? A.Continue with the shift report and talk to the nurse about the incident at a later time. B.Ask the nurse to call the house supervisor to see if she must be reassigned. C.Stop the shift report and remind the nurse that all staff are floated equally. D.Inform the nurse that her behavior is disruptive to the rest of the staff.

A Continuing with the shift report (A) is the best immediate action because it allows the nurse who was floated some cooling off time. At a later time (after the nurse has cooled off) the charge nurse should discuss the conduct of the nurse in private. (B) encourages the nurse to shirk the float assignment. (C) is disruptive. Reprimanding the nurse in front of the staff would increase the nurse's hostility, so the nurse should be counseled in private (D).

Which intervention in a client with dehydration induced confusion is most likely to relieve the confusion? a. increasing the IV flow rate to 250 mL/hr b. applying oxygen by mask or nasal cannula c. placing the client in a high Fowler's position d. Measuring intake and output every four hours

A Dehydration most frequently leads to poor cerebra perfusion and cerebral hypoxia, causing confusion. Applying oxygen can reduce confusion, even if perfusion is still less than optimum. Increasing the IV flow rate would increase perfusion. However, depending on the degree of dehydration, rehydrating the person too rapidly with IV fluids can lead to cerebral edema.

A resident in a long-term care facility is diagnosed with hepatitis B. Which intervention should the nurse implement with the staff caring for this client? A.Determine if all employees have had the hepatitis B vaccine series. B.Explain that this type of hepatitis can be transmitted when feeding the client. C.Assure the employees that they cannot contract hepatitis B when providing direct care. D.Tell the employees that wearing gloves and a gown are required when providing care.

A Hepatitis B vaccine should be administered to all health care providers (A). Hepatitis A (not hepatitis B) can be transmitted by fecal-oral contamination (B). There is a chance that staff could contract hepatitis B if exposed to the client's blood and/or body fluids; therefore, (C) is incorrect. There is no need to wear gloves and gowns except with blood or body fluid contact (D).

A 63-year-old client with type 2 diabetes mellitus is admitted for treatment of an ulcer on the heel of the left foot that has not healed with wound care. The nurse observes that the entire left foot is darker in color than the right foot. Which additional symptom should the nurse expect to find? A.Pedal pulses will be weak or absent in the left foot. B.The client will state that the left foot is usually warm. C.Flexion and extension of the left foot will be limited. D.Capillary refill of the client's left toes will be brisk.

A Symptoms associated with decreased blood supply are weak or absent pedal and tibial pulses (A). The client with diabetes experiences vascular scarring as a result of atherosclerotic changes in the peripheral vessels. This results in compromised perfusion to the dependent extremities, which further delays wound healing in the affected foot. Although flexion and extension may be limited (C), depending on the degree of damage, this is not always the case. (B and D) are signs of adequate perfusion of the foot, which would not be expected in this client.

The nurse is preparing a teaching plan for a group of healthy adults. Which individual is most likely to maintain optimum health? A.A teacher whose blood glucose levels average 126 mg/dL daily with oral antidiabetic drugs B.An accountant whose blood pressure averages 140/96 mm Hg and who says he does not have time to exercise C.A stock broker whose total serum cholesterol level dropped to 290 mg/dL with diet modifications D.A recovering IV heroin user who contracted hepatitis more than 10 years ago

A The diabetic teacher (A) has assumed responsibility for self-care, so among those listed, is the most likely to maintain optimum health. (B) has expressed a lack of interest in health promotion. (C) continues to demonstrate a high-risk cholesterol level despite a reported attempt at dietary modifications. Previous IV drug use and a history of hepatitis (D) make this individual a health risk despite the fact that the individual is in recovery.

In preparing to administer intravenous albumin to a client following surgery, what is the priority nursing intervention? (Select all that apply.) A) Set the infusion pump to infuse the albumin within four hours. B) Compare the client's blood type with the label on the albumin. C) Assign a UAP to monitor blood pressure q15 minutes. D) Administer through a large gauge catheter. E) Monitor hemoglobin and hematocrit levels. F) Assess for increased bleeding after administration.

A, D, E, F

The nurse assesses the patient and notes all of the following. Select all of the findings that indicate the systemic manifestations of inflammation. A) Oral temperature 38.6 F B) WBC 20 C) Thick, green nasal discharge D) Patient reports, "I'm tired all the time. I haven't felt like myself in days" E) Patient complaint of pain at 6 on a 0 to 10 scale on palpation of frontal and maxillary sinuses

A,B,D Systemic manifestations of inflammatory response include elevated temperature, leukocytosis, and malaise and fatigue. Purulent exudates and pain are both considered local manifestations of inflammation.

When reviewing an older client's medical record, which findings lead the nurse to perform a nutrition assessment? (Select all that apply.) A) Widow/widower status B) Chronic constipation C) Cholecystectomy 4 years ago D) Random blood sugar level of 198 mg/dL E) History of depression F) Inability to afford a new pair of glasses

A,B,E,F Many factors contribute to malnutrition in older clients. Depression and loneliness from the loss of a spouse; constipation; poor eyesight; chronic medical problems, including depression; and taking prescription and/or over-the-counter medications can contribute to malnutrition. Blood glucose levels and a previous cholecystectomy would not necessarily contribute.

During preoperative teaching for a male client schedule for repair of an inguinal hernia, the client tells the nurse that he has had several surgeries and understand the need to perform coughing and deep breathing exercise after surgery. How should the nurse respond? A. Ask for a demonstration of these exercises B. Explain that coughing should be avoided C. Review the client previous surgical history D. Document the clients understanding of teaching

A. Ask for a demonstration of these exercises

An 81-year-old male client has emphysema. He lives at home with his cat and manages self-care with no difficulty. When making a home visit, the nurse notices that this client's tongue is somewhat cracked and his eyeballs appear sunken into his head. Which nursing intervention is indicated? A.Help the client determine ways to increase his fluid intake. B.Obtain an appointment for the client to have an eye examination. C.Instruct the client to use oxygen at night and increase the humidification. D.Schedule the client for tests to determine his sensitivity to cat hair.

A. Help the client determine ways to increase his fluid intake.

An older female who ambulates with a quad-cane prefers to use a wheelchair because she has a halting and unsteady gait at times. Which interventions should the nurse implement? (select all that apply) A. Move personal items within client's reach B. Lower bed to the lowest possible position C. Raise all bed rails when client is resting D. Give directions to call for assistance E. Assist client to the bathroom q 2 hours F. Encourage the use of the wheelchair

A. Move personal items within client's reach B. Lower bed to the lowest possible position D. Give directions to call for assistance E. Assist client to the bathroom q 2 hours

While caring for a client with a new onset of diabetes mellitus, which intervention is most important for the nurse to include in the client's plan of care? A. Observe client's glucose self-monitoring technique B. Provide written diabetic diet instructions C. Teach client how to read food labels D. Check accuracy of glucose monitoring equipment

A. Observe client's glucose self-monitoring technique

In caring for a patient with a PCA infusion of morphine sulfate through the right cephalic vein, the nurse assess that client is lethargic, with a blood pressure of 90/60, pulse rate of 118 beats/minute, and a respiratory rate of 8 breaths/minute. What assessment should the nurse perform next? A. Observe the amount and dose of morphine in the PCA pump syringe B. Note the appearance and patency of the client's peripheral IV site C. Palpate the volume of the client's right radial pulse D. Auscultate the client's breath sounds bilaterally

A. Observe the amount and dose of morphine in the PCA pump syringe

An older male adult resident of an extended care facility receives a prescription for diphenhydramine (Benadryl) 25 mg PO to treat generalized pruritus. Two hours after administration of the drug, he continues to experience itching, is confused, and has an unsteady gait. What action should the nurse implement first? A. Place the client on fall precautions B. Apply soft limb restraints to extremities C. Give a second dose of Benadryl D. Lubricate the skin with an emollient

A. Place the client on fall precautions

An older female client with long term type 2 diabetes mellitus (DM) is seen in the clinic for a routine health assessment. To determine if the client is experiencing any long-term complication of DM, which assessments should the nurse obtain? (select all that apply) A. Serum creatinine and blood urea nitrogen (BUN) B. Sensation in feet and legs C. Skin condition of lower extremities D. Visual acuity E Signs of respiratory tract infection

A. Serum creatinine and blood urea nitrogen (BUN) B. Sensation in feet and legs C. Skin condition of lower extremities D. Visual acuity

The DASH (Dietary Approaches to Stop Hypertension) diet is prescribed for a client with uncontrolled hypertension. Which dietary choices should the nurse instruct the client to eat? A. Shredded wheat B. Avocado salad with olives C. Cheddar cheese D. Canned roasted almonds

A. Shredded wheat

The nurse assesses a client being treated for Herpes Zoster (shingles). Which assessments should the nurse include when evaluating the effectiveness of treatment? (Select all that apply) A. Skin integrity B. Functional ability C. Heart sounds D. Pain scale E. Bowel sounds

A. Skin integrity B. Functional ability D. Pain scale

If a nurse is caring for an 80-year-old patient with a temperature of 100.4° F, crackles at the right lung base, pain with deep inspiration, and dyspnea, which of the following orders is the nurse's priority? A. Sputum specimen for culture and sensitivity B. Codeine 15 mg orally every 6 hours as needed C. Incentive spirometer every 2 hours while awake D. Amoxicillin (Amoxil) 500 mg orally 4 times a day

A. Sputum specimen for culture and sensitivity

A young woman is preparing to leave for a 7-day boat trip. She requests a prescription for motion sickness, so the healthcare provider prescribes meclizine (Antivert). Which instruction should the nurse include in this client's teaching? A. Suck on hard candy for a dry mouth while taking this drug B. Sit upright for at least 30 minutes after taken this drug C. Avoid eating shellfish for 24 hours after taking this drug D. Do not drink caffeinated beverages while taking this drug

A. Suck on hard candy for a dry mouth while taking this drug

A client with Type 1 diabetes mellitus (DM) is admitted for an emergency cholecystectomy. To prevent diabetic ketoacidosis (DKA), which intervention is most important for the nurse to implement? A. Supplement insulin needs using a sliding scale B. Monitor the client for polyuria and dehydration C. Measure urine ketones when blood glucose exceeds 300 mg/dl D. Teach the client to monitor blood glucose levels frequently when ill

A. Supplement insulin needs using a sliding scale

The nurse is caring for a postoperative patient with sudden onset of respiratory distress. The physician orders a STAT ventilation-perfusion scan. Which of the following explanations should the nurse provide to the patient about the procedure? A. This test involves injection of a radioisotope to outline the blood vessels in the lungs, followed by inhalation of a radioisotope gas. B. This test will use special technology to examine cross sections of the chest with use of a contrast dye. C. This test will use magnetic fields to produce images of the lungs and chest. D. This test involves injecting contrast dye into a blood vessel to outline the blood vessels of the lungs.

A. This test involves injection of a radioisotope to outline the blood vessels in the lungs, followed by inhalation of a radioisotope gas.

In making client care assignments, which client is best to assign to the practical nurse (PN) working on the unit with a nurse? A. an immobile client receiving low molecular weight heparin Q12H B. a client who is receiving a continuous infusion of heparin and gets out of bed BID C. A client how is being titrated off a heparin infusion and started on PO warfarin D. An ambulatory client receiving warfarin with an INR of 5 seconds.

A. an immobile client receiving low molecular weight heparin Q12H

A patient is admitted to the hospital with fever, chills, a productive cough with rusty sputum, and pleuritic chest pain. Pneumococcal pneumonia is suspected. An appropriate nursing diagnosis for the patient based on the patient's manifestations is A. hyperthermia related to acute infectious process. B. chronic pain related to ineffective pain management. C. risk for injury related to disorientation and confusion. D. ineffective airway clearance related to retained secretions.

A. hyperthermia related to acute infectious process.

When caring for a 2-day-old infant, the nurse observes that the babies legs are flexed with limited abduction. Based on the finding, what action should the nurse take next? A. notify the healthcare provider B. continue care since this is a normal finding C. document the finding in the record D. perform range of motion to the joint

A. notify the healthcare provider

In reviewing the preoperative laboratory findings of an adult male client who is scheduled for colon resection in the morning, the nurse notes that the client has a hemoglobin of 9 grams/dl. After verifying the accuracy of this finding with the laboratory, what action should the nurse take? A. notify the surgeon of the laboratory findings B. collect another specimen for analysis C. confirm the availability of compatible units of blood D. administer a bolus of normal saline preoperatively

A. notify the surgeon of the laboratory findings

Using light pressure with the index and middle fingers, the nurse cannot palpate any of the patient's superficial lymph nodes. The nurse A. records this finding as normal. B. should reassess the lymph nodes using deeper pressure. C. asks the patient about any history of any radiation therapy. D. notifies the health care provider that x-rays of the nodes will be necessary.

A. records this finding as normal.

While caring for a patient with respiratory disease, a nurse observes that the oxygen saturation drops from 94% to 85% when the patient ambulates. The nurse should determine that A. supplemental oxygen should be used when the patient exercises. B. ABG determinations should be done to verify the oxygen saturation reading. C. this finding is a normal response to activity and that the patient should continue to be monitored. D. the oximetry probe should be moved from the finger to the earlobe for an accurate oxygen saturation measurement during activity.

A. supplemental oxygen should be used when the patient exercises.

Mr. Pablo, diagnosed with Bladder Cancer, is scheduled for a cystectomy with the creation of an ileal conduit in the morning. He is wringing his hands and pacing the floor when the nurse enters his room. What is the best approach? A. "Good evening, Mr. Pablo. Wasn't it a pleasant day, today?" B. "Mr, Pablo, you must be so worried, I'll leave you alone with your thoughts. C. "Mr. Pablo, you'll wear out the hospital floors and yourself at this rate." D. "Mr. Pablo, you appear anxious to me. How are you feeling about tomorrow's surgery?"

Answer: (D) "Mr. Pablo, you appear anxious to me. How are you feeling about tomorrow's surgery?" The client is showing signs of anxiety reaction to a stressful event. Recognizing the client's anxiety conveys acceptance of his behavior and will allow for verbalization of feelings and concerns.

Ana's postoperative vital signs are a blood pressure of 80/50 mm Hg, a pulse of 140, and respirations of 32. Suspecting shock, which of the following orders would the nurse question? A. Put the client in modified Trendelenberg's position. B. Administer oxygen at 100%. C. Monitor urine output every hour. D. Administer Demerol 50mg IM q4h

Answer: (D) Administer Demerol 50mg IM q4h Administering Demerol, which is a narcotic analgesic, can depress respiratory and cardiac function and thus not given to a patient in shock. What is needed is promotion for adequate oxygenation and perfusion. All the other interventions can be expected to be done by the nurse.

One day after a Billroth II surgery, a male client suddenly grabs his right chest and becomes pale and diaphoretic. Vital signs are assessed at blood pressure 100/80, pulse 110 beats/min, and respirations 36 breaths/min. What action is most important for the nurse to take?

Apply oxygen at 2 L per nasal cannula.

A 58-year-old client, who has no health problems, asks the nurse about the Pneumovax vaccine. The nurse's response to the client should be based on which information? A) The vaccine is given annually before the flu season to those over 50 years of age. B) The immunization is administered once to older adults or persons with a history of chronic illness. C) The vaccine is for all ages and is given primarily to those persons traveling overseas to areas of infection. D) The vaccine will prevent the occurrence of pneumococcal pneumonia for up to five years.

B

A client is admitted to the hospital with a medical diagnosis of pneumococcal pneumonia. The nurse knows that the prognosis for gram-negative pneumonias (such as E. coli, Klebsiella, Pseudomonas, and Proteus) is very poor because A) they occur in the lower lobe alveoli which are more sensitive to infection. B) gram-negative organisms are more resistant to antibiotic therapy. C) they occur in healthy young adults who have recently been debilitated by an upper respiratory infection. D) gram-negative pneumonias usually affect infants and small children.

B

A client who has heart failure is admitted with a serum potassium level of 2.9 mEq/L. Which action is most important for the nurse to implement? A) Give 20 mEq of potassium chloride. B) Initiate continuous cardiac monitoring. C) Arrange a consultation with the dietician. D) Teach about the side effects of diuretics.

B

A client with a 16-year history of diabetes mellitus is having renal function tests because of recent fatigue, weakness, elevated blood urea nitrogen, and serum creatinine levels. Which finding should the nurse conclude as an early symptom of renal insufficiency? A) Dyspnea. B) Nocturia. C) Confusion. D) Stomatitis.

B

During suctioning, a client with an uncuffed tracheostomy tube begins to cough violently and dislodges the tracheostomy tube. Which action should the nurse implement first? A) Notify the healthcare provider for reinsertion. B) Attempt to reinsert the tracheostomy tube. C) Position the client in a lateral position with the neck extended. D) Ventilate client's tracheostomy stoma with a manual bag-mask.

B

The nurse is assessing a client who has a history of Parkinson's disease for the past 5 years. What symptoms should this client most likely exhibit? A) Loss of short-term memory, facial tics and grimaces, and constant writhing movements. B) Shuffling gait, masklike facial expression, and tremors of the head. C) Extreme muscular weakness, easy fatigability, and ptosis. D) Numbness of the extremities, loss of balance, and visual disturbances.

B

The nurse is assessing a client's laboratory values following administration of chemotherapy. Which lab value leads the nurse to suspect that the client is experiencing tumor lysis syndrome (TLS)? A) Serum PTT of 10 seconds. B) Serum calcium of 5 mg/dl. C) Oxygen saturation of 90%. D) Hemoglobin of 10 g/dl.

B

The nurse is caring for a client with a continuous feeding through a percutaneous endoscopic gastrostomy (PEG) tube. Which intervention should the nurse include in the plan of care? A) Flush the tube with 50 ml of water q 8 hours. B) Check for tube placement and residual volume q4 hours. C) Obtain a daily x- ray to verify tube placement. D) Position on left side with head of bed elevated 45 degrees.

B

The nurse is caring for a client with a stroke resulting in right-sided paresis and aphasia. The client attempts to use the left hand for feeding and other self-care activities. The spouse becomes frustrated and insists on doing everything for the client. Based on this data, which nursing diagnosis should the nurse document for this client? A) Situational low self-esteem related to functional impairment and change in role function. B) Disabled family coping related to dissonant coping style of significant person. C) Interrupted family processes related to shift in health status of family member. D) Risk for ineffective therapeutic regimen management related to complexity of care.

B

The nurse is taking a history of a newly diagnosed Type 2 diabetic who is beginning treatment. Which subjective information is most important for the nurse to note? A) A history of obesity. B) An allergy to sulfa drugs. C) Cessation of smoking three years ago. D) Numbness in the soles of the feet.

B

The nurse working on a telemetry unit finds a client unconscious and in pulseless ventricular tachycardia (VT). The client has an implanted automatic defibrillator. What action should the nurse implement? A) Prepare the client for transcutaneous pacemaker. B) Shock the client with 200 joules per hospital policy. C) Use a magnet to deactivate the implanted pacemaker. D) Observe the monitor until the onset of ventricular fibrillation.

B

Which client should the nurse recognize as most likely to experience sleep apnea? A) Middle-aged female who takes a diuretic nightly. B) Obese older male client with a short, thick neck. C) Adolescent female with a history of tonsillectomy. D) School-aged male with a history of hyperactivity disorder.

B

Which reaction should the nurse identify in a client who is responding to stimulation of the sympathetic nervous system? A) Pupil constriction. B) Increased heart rate. C) Bronchial constriction. D) Decreased blood pressure.

B

The nurse is admitting an older adult with decompensated congestive heart failure. The nursing assessment reveals adventitious lung sounds, dyspnea, and orthopnea. The nurse should question which doctor's order? A) KCl 20 mEq PO two times per day B) Intravenous (IV) 500 mL of 0.9% NaCl at 125 mL/hr C) Oxygen via face mask at 8 L/min D) Furosemide (Lasix) 20 mg PO now

B A patient with decompensated heart failure has extracellular fluid volume (ECV) excess. The IV of 0.9% NaCl is normal saline, which should be questioned because it would expand ECV and place an additional load on the failing heart. Diuretics such as furosemide are appropriate to decrease the ECV during heart failure. Increasing the potassium intake with KCl is appropriate, because furosemide increases potassium excretion. Oxygen administration is appropriate in this situation of near pulmonary edema from ECV excess.

The nurse determines that a client has a Braden Scale score of 9. Which is the nurse's best intervention related to this assessment? A) Increase the client's fluid intake. B) Consult with the health care provider. C) Reassess the client in 3 days. D) Document the finding per protocol.

B A score of 11 or less on the Braden Scale indicates severe risk for pressure ulcer development in terms of decreased sensory perception, exposure to moisture, decreased independent activity, decreased mobility, poor nutrition, and chronic exposure to friction and shear. The nurse needs to consult with the health care provider to relay this information and to obtain more aggressive skin protection measures than are currently provided.

The nurse is preparing to administer a prescribed IV antibiotic to a client admitted with a serious infection. Which action by the nurse is most important? A) Double check the "five rights." B) Assess the client for allergies. C) Teach the client about the drug. D) Check the IV for patency.

B All actions are appropriate and important before administering any medications. However, client safety is the priority. The nurse should first assess the client for medication allergies by asking the client or checking the chart (or both). Ensuring a patent IV and checking the five rights will not protect the client from an allergic reaction.

Which action does the nurse teach a client to reduce the risk for dehydration? A) Avoiding the use of glycerin suppositories to manage constipation B) Maintaining a daily oral intake approximately equal to daily fluid loss C) Restricting sodium intake to no greater than 4 g/day D) Maintaining an oral intake of at least 1500 mL/day

B Although a fixed oral intake of 1500 mL daily is good, the key to prevention of dehydration is to match all fluid losses with the same volume for fluid intake. This is especially true in warm or dry environments, or when conditions result in greater than usual fluid loss through perspiration or ventilation.

The nurse is discharging home a client at risk for venous thromboembolism (VTE) on low-molecular-weight heparin. What instruction does the nurse provide to this client? A) "You must have your aPTT checked every 2 weeks." B) "Notify your health care provider if your stools appear tarry." C) "An IV catheter will be placed to administer your heparin." D) "Massage the injection site after the heparin is injected."

B As with any anticoagulation, low-molecular-weight heparin incurs risk of bleeding. Clients should be taught to report to their health care provider the presence of tarry stools, bleeding gums, hematuria, ecchymosis, or petechiae. Low-molecular-weight heparin does not affect activated partial thromboplastin time (aPTT), as does intravenous heparin. This type of heparin is administered subcutaneously to deliver a slow sustained response. Massaging the site would hasten absorption and decrease effects.

The nurse is caring for four clients. Which client assessment is the most indicative of having pain? A) Client stating that he is "anxious" B) Heart rate of 105 beats/min and restlessness C) Blood pressure 150/70 mm Hg and sleeping D) Postoperative client with a neck incision

B At times clients are unable to verbalize that they are in pain but there are indicators that the client may have acute pain such as increased heart rate, increased blood pressure, increased respirations, sweating, restlessness, and overall distress. All the other distractors could indicate clients who have the potential for being in pain, but restlessness with tachycardia is the most indicative.

The nurse assesses a cut that is 24 hours old and finds that the site is swollen, red, and tender to the touch. Which cell types are responsible for these assessment findings? A) Natural killer cells B) Basophils and eosinophils C) Erythrocytes and platelets D) Plasma cells and B-lymphocytes

B Basophils and eosinophils release histamine, kinins, and other substances that cause the manifestations of inflammation. Erythrocytes carry oxygen, and platelets help stop bleeding. Plasma cells and B-lymphocytes produce antibodies to help fight infection, and natural killer cells destroy invading bacteria.

Which client does the nurse assess to be at greatest risk for pressure ulcer development? A) Client who requires assistance with ambulation B) Incontinent client with limited mobility C) Client with hypertension on multiple medications D) Client who has pneumonia

B Being immobile and being incontinent are two significant risk factors for the development of pressure ulcers. Clients with pneumonia and hypertension do not have specific risk factors. The client who needs assistance with ambulation might be at moderate risk if he or she does not move about much, but having two risk factors makes the last option the person at highest risk.

A client is receiving a chemotherapeutic agent intravenously through a peripheral line. What is the nurse's first action when the client reports burning at the site? A) Apply a cold compress. B) Discontinue the infusion. C) Slow the rate of infusion. D) Check for a blood return.

B Both irritants and vesicants can cause tissue damage. If the nurse suspects extravasation, he or she should immediately stop the infusion. Even if the IV has a good blood return, some of the chemotherapeutic agent can still be leaking into the tissues. Slowing the rate of infusion is not sufficient to prevent further leakage and damage. Applying a cold compress may or may not be the correct action, depending on the specific agent. However, the compress would be applied only after the infusion has been discontinued.

A nurse is assessing clients for fluid and electrolyte imbalances. Which client is at greatest risk for developing hyponatremia? A) Client taking digoxin (Lanoxin) B) Client who is NPO receiving intravenous D5W C) Client taking ibuprofen (Motrin) D) Client taking a sulfonamide antibiotic

B D5W contains no electrolytes. Because the client is not taking any food or fluids by mouth, normal sodium excretion can lead to hyponatremia. The antibiotic, Motrin, and digoxin will not put a client at risk for hyponatremia.

A client asks the nurse why it is important to be weighed every day if he has right-sided heart failure. What is the nurse's best response? A) "The hospital requires that all inpatients be weighed daily." B) "Weight is the best indication that you are gaining or losing fluid." C) "You need to lose weight to decrease the incidence of heart failure." D) "Daily weights will help us make sure that you're eating properly."

B Daily weights are needed to document fluid retention or fluid loss. One liter of fluid equals 2.2 pounds.

The nurse assesses a client's legs. Which assessment finding indicates arterial insufficiency? A) Pain with activity but not while resting B) Dependent mottling and absence of hair C) Full veins present in dependent extremity D) Ankle discoloration and pitting edema

B Dependent mottling and absence of hair is an indication of arterial insufficiency. Pain may be present with activity and at rest. Edema and ankle discoloration would be indicative of venous insufficiency.

A patient is diagnosed with a sprain to her right ankle after a fall. The patient asks the nurse about using ice on her injured ankle. The nurse should tell the patient that: A) ice is not recommended for use on the sprain because it would inhibit the inflammatory response. B) ice should be applied for 15 to 20 minutes every 2 to 3 hours over the next 1 to 2 days. C) she should use ice only when the ankle hurts. D) she should wrap an ice pack around the injured ankle for the next 24 to 48 hours.

B Ice is used on areas of injury during the first 24 to 48 hours after the injury occurs to prevent damage to surrounding tissues from excessive inflammation. Ice should be used for a maximum of 20 minutes at a time every 2 to 3 hours. Ice must be used according to a schedule for it to be effective and not be overused. Using ice more often or for longer periods of time can cause additional tissue damage. Ice is recommended to inhibit the inflammatory process from damaging surrounding tissue.

Before surgery, the nurse observes the client listening to music on the radio. Based on this observation, the nurse may try which nonpharmacologic intervention for pain relief in the postoperative setting? A) Cutaneous skin stimulation B) Imagery C) Radiofrequency ablation D) Hypnosis

B Imagery is a form of distraction in which the client is encouraged to visualize about some pleasant or desirable feeling, sensation, or event. Behaviors that are helpful in assessing a client's capacity for imagery include being able to listen to music or other auditory stimuli.

The Joint Commission focuses on safety in health care. Which action by the nurse reflects The Joint Commission's main objective? A) Performing range-of-motion exercises on the client three times each day B) Assessing the client's respirations when administering opioids C) Delegating to the nursing assistant to give the client a complete bath daily D) Ensuring that the client is eating 100% of the meals served to him or her

B It is important for the nurse to assess respirations of the client when administering opioids because of the possibility of respiratory depression. The other interventions may or may not be necessary in the care of the client and do not focus on safety.

A client is hospitalized with a urinary tract infection (UTI). Which clinical manifestation alerts the nurse to the possibility of a complication from the UTI? A) Hematuria B) Fever and chills C) Cloudy, dark urine D) Burning on urination

B Lower urinary tract infections are rarely associated with systemic symptoms of fever and chills. A client with a UTI who develops fever and chills should be assessed for the development of pyelonephritis. The other options can be seen with UTI.

A client presents to the emergency department after prolonged exposure to the cold. The client is shivering, has slurred speech, and is slow to respond to questions. Which intervention will the nurse prepare for this client FIRST? A) Continuous arteriovenous rewarming B) Dry clothing and warm blankets C) Peritoneal lavage with warmed normal saline D) Administration of warmed IV fluids

B Mild hypothermia is manifested by shivering, slurred speech, poor muscular coordination, and impaired cognitive abilities. Mild hypothermia may be treated with dry clothing and warm blankets. Rewarming should occur slowly by removing wet clothing and providing dry warm blankets first. Other treatments are secondary and should be used to treat moderate to severe hypothermia.

A client receiving intravenous chemotherapy asks the nurse the reason for wearing a mask, gloves, and gown while administering drugs to the client. What is the nurse's best response? A) "I am preventing the spread of infection from you to me or any other client here." B) "The clothing protects me from accidentally absorbing these drugs." C) "The policy is for any nurse giving these drugs to wear a gown, gloves, and mask." D) "These coverings protect you from getting an infection from me."

B Most chemotherapy drugs are absorbed through the skin and mucous membranes. As a result, health care workers who prepare or give these drugs, especially nurses and pharmacists, are at risk for absorbing them. Even at low doses, chronic exposure to chemotherapy drugs can affect health. The Oncology Nursing Society and the Occupational Safety and Health Administration (OSHA) have specific guidelines for using caution and wearing protective clothing whenever preparing, giving, or disposing of chemotherapy drugs.

The nurse is caring for a client who is immobile from a recent stroke. Which intervention does the nurse implement to prevent complications in this client? A) Teach the client to touch and use both sides of the body. B) Apply sequential compression stockings. C) Instruct the client to turn the head from side to side. D) Position the client with the unaffected side down.

B To avoid complications of immobility, such as deep vein thrombosis, the nurse applies sequential compression stockings or pneumatic compression boots. Efforts are made to mobilize the client as much as possible, and the client should be repositioned frequently. The other interventions will not prevent complications of immobility.

The nurse is assessing a male client with acute pancreatitis. Which finding requires the MOST immediate intervention by the nurse? A.The client's amylase level is three times higher than the normal level. B.While the nurse is taking the client's blood pressure, he has a carpal spasm. C.On a 1 to 10 scale, the client tells the nurse that his epigastric pain is at 7. D.The client states that he will continue to drink alcohol after going home.

B A positive Trousseau sign (B) indicates hypocalcemia and always requires further assessment and intervention, regardless of the cause (40% to 75% of those with acute pancreatitis experience hypocalcemia, which can have serious, systemic effects). A key diagnostic finding of pancreatitis is serum amylase and lipase levels that are two to five times higher than the normal value (A). Severe boring pain is an expected symptom for this diagnosis (C), but dealing with the hypocalcemia is a priority over administering an analgesic. Long-term planning and teaching (D) do not have the same immediate importance as a positive Trousseau sign.

A hospitalized client is receiving nasogastric tube feedings via a small-bore tube and a continuous pump infusion. He begins to cough and produces a moderate amount of white sputum. Which action should the nurse take FIRST? A.Auscultate the client's breath sounds. B.Turn off the continuous feeding pump. C.Check placement of the nasogastric tube. D.Measure the amount of residual feeding.

B A productive cough may indicate that the feeding has been aspirated. The nurse should first stop the feeding (B) to prevent further aspiration. (A, C, and D) should all be performed before restarting the tube feeding if no evidence of aspiration is present and the tube is in place.

Which instruction should the nurse teach a female client about the prevention of toxic shock syndrome? A."Get immunization against human papillomavirus (HPV)." B."Change your tampon frequently." C."Empty your bladder after intercourse." D."Obtain a yearly flu vaccination."

B Certain strains of Staphylococcus aureus produce a toxin that can enter the bloodstream through the vaginal mucosa. Changing the tampon frequently (B) reduces the exposure to these toxins, which are the primary cause of toxic shock syndrome. (A) helps prevent cervical cancer, not toxic shock syndrome. (C) can lessen the incidence of urinary tract infection. (D) can help prevent some individuals from contracting the flu and pneumonia, but no relationship to toxic shock syndrome has been proven.

The nurse is reviewing routine medications taken by a client with chronic angle closure glaucoma. Which medication prescription should the nurse question? A.Antianginal with a therapeutic effect of vasodilation B.Anticholinergic with a side effect of pupillary dilation C.Antihistamine with a side effect of sedation D.Corticosteroid with a side effect of hyperglycemia

B Clients with angle-closure glaucoma should not take medications that dilate the pupil (B) because this can precipitate acute and severely increased intraocular pressure. (A, C, and D) do not cause increased intracranial pressure, which is the primary concern with angle-closure glaucoma.

A tornado warning alarm has been activated at the local hospital. Which action should the charge nurse working on a surgical unit implement first? A.Instruct the nursing staff to close all window blinds and curtains in clients' rooms. B.Move clients and visitors into the hallways and close all doors to clients' rooms. C.Visually confirm the location of the tornado by checking the windows on the unit. D.Assist all visitors with evacuation down the stairs in a calm and orderly manner.

B In the event of a tornado, all persons should be moved into the hallways, away from windows, to prevent flying debris from causing injury (B). Although (A) may help decrease the amount of flying debris, it is not safe to leave clients in rooms with closed blinds; (B) is a higher priority at this time. Hospital staff should stay away from windows to avoid injury and should focus on client evacuation into hallways rather than (C). (D) is not the first action that should be taken.

A 58-year-old client who has no health problems asks the nurse about receiving the pneumococcal vaccine (Pneumovax). Which statement given by the nurse would offer the client accurate information about this vaccine? A.The vaccine is given annually before the flu season to those older than 50 years. B.The immunization is administered once to older adults or those at risk for illness. C.The vaccine is for all ages and is given primarily to those persons traveling overseas to areas of infection. D.The vaccine will prevent the occurrence of pneumococcal pneumonia for up to 5 years.

B It is usually recommended that persons older than 65 years and those with a history of chronic illness should receive the vaccine once in their lifetime (B). Some recommend receiving the vaccine at 50 years of age. The influenza vaccine is given once a year, not Pneumovax (A). Although the vaccine might be given to a person traveling overseas, that is not the main rationale for administering the vaccine (C). The vaccine is usually given once in a lifetime (D), but with immunosuppressed clients or clients with a history of pneumonia, revaccination is sometimes required.

The nurse notes that a client who is scheduled for surgery the next morning has an elevated blood urea nitrogen (BUN) level. Which condition is most likely to have contributed to this finding? A.Myocardial infarction 2 months ago B.Anorexia and vomiting for the past 2 days C.Recently diagnosed type 2 diabetes mellitus D.Skeletal traction for a right hip fracture

B The blood urea nitrogen (BUN) level indicates the effectiveness of the kidneys in filtering waste from the blood. Dehydration, which could be caused by vomiting, would cause an increased the BUN level (B). (A) would affect serum enzyme levels, not the BUN level. (C) would primarily affect the blood glucose level; renal failure that could increase the BUN level would be unlikely in a client newly diagnosed with type 2 diabetes. Effects of (D) might affect the complete blood count (CBC) but would not directly increase the BUN level.

A male client has just undergone a laryngectomy and has a cuffed tracheostomy tube in place. When initiating bolus tube feedings postoperatively, when should the nurse inflate the cuff? A.Immediately after feeding B.Just prior to tube feeding C.Continuous inflation is required D.Inflation is not required

B The cuff should be inflated before the feeding to block the trachea and prevent food from entering (B) if oral feedings are started while a cuffed tracheostomy tube is in place. It should remain inflated throughout the feeding to prevent aspiration of food into the respiratory system. (A and D) place the client at risk for aspiration. (C) places the client at risk for tracheal wall necrosis.

When assigning clients on a medical-surgical floor to an RN and a PN, it is best for the charge nurse to assign which client to the PN? A.A young adult with bacterial meningitis with recent seizures B.An older adult client with pneumonia and viral meningitis C.A female client in isolation with meningococcal meningitis D.A male client 1 day postoperative after drainage of a brain abscess

B The most stable client is (B). (A, C, and D) are all at high risk for increased intracranial pressure and require the expertise of the RN for assessment and management of care.

A client on telemetry has a pattern of uncontrolled atrial fibrillation with a rapid ventricular response. Based on this finding, the nurse anticipates assisting the physician with which treatment? A.Administer lidocaine,75 mg intravenous push. B.Perform synchronized cardioversion. C.Defibrillate the client as soon as possible. D.Administer atropine, 0.4 mg intravenous push.

B With uncontrolled atrial fibrillation, the treatment of choice is synchronized cardioversion (B) to convert the cardiac rhythm back to normal sinus rhythm. (A) is a medication used for ventricular dysrhythmias. (C) is not for a client with atrial fibrillation; it is reserved for clients with life-threatening dysrhythmias, such as ventricular fibrillation and unstable ventricular tachycardia. (D) is the drug of choice in symptomatic sinus bradycardia, not atrial fibrillation.

The nurse teaches a client with type 2 diabetes nutritional strategies to decrease obesity. Which food item(s) chosen by the client INDICATES UNDERSTANDING of the teaching? (Select all that apply.) A.White bread B.Salmon C.Broccoli D.Whole milk E.Banana

B, C, E (B, C, and E) provide fresh fruits, lean meats and fish, vegetables, whole grains, and low-fat dairy products. All are recommended by the American Diabetes Association (ADA) and are a part of the My Plate guidelines recommended by the U.S. Department of Agriculture (USDA). Whole milk (D) is high in fat and is not recommended by ADA. White bread is milled, a process that removes the essential nutrients. It should be avoided for weight loss and is a poor choice for the client with diabetes (A).

The nurse is prioritizing care to prevent pressure sores for a client who is immobilized. Which interventions are appropriate? (Select all that apply.) A) Use a rubber ring to decrease sacral pressure when up in the chair. B) Place a small pillow between bony surfaces. C) Keep the heels off the bed surfaces. D) Use a lift sheet to assist with repositioning. E) Reposition the client who is in a chair every 2 hours. F) Elevate the head of the bed to 45 degrees. G) Limit fluids and proteins in the diet.

B,C,D A small pillow decreases the risk for pressure between bony prominences, a lift sheet decreases friction and shear, and heels have poor circulation and are at high risk for pressure sores, so they should be kept off hard surfaces. Head of the bed elevation greater than 30 degrees increases pressure on pelvic soft tissues. Fluids and proteins are important for maintaining tissue integrity. Clients should be repositioned every hour while sitting in a chair. A rubber ring impairs capillary blood flow, increasing the risk for a pressure sore.

An older client just returned from surgery and is rating pain as "8" on a 0 to 10 scale. Which medications are unsafe choices for treatment of severe pain in this older adult? (Select all that apply.) A) Morphine (Durmorph) B) Meperidine (Demerol) C) Propoxyphene (Darvocet) D) Methadone (Dolophine) E) Codeine

B,C,D,E Meperidine, propoxyphene, and codeine are not recommended for older clients because toxic metabolites may accumulate. Codeine may cause constipation as well. Methadone has an extremely long half-life (24 to 36 hours) and has a high potential for sedation and respiratory depression. Morphine is considered the gold standard and may be used in the older adult while monitoring for sedation and respiratory depression is conducted.

The nurse prepares to teach a patient recovering from a myocardial infarction (MI) about combination durg therapy based on "best practice" for controlling hypertension. Which drugs does the nurse include in the teaching plan? SELECT ALL THAT APPLY!!! A) NSAID's B) Aspirin C) Aldosterone antagonists D) ACE Inhibitors or ARB's E) Central alpha Agonists F) Beta Blockers G) Diuretics

B,C,D,F,G

the healthcare provider is working with a client who was recently diagnosed with asthma. Which statement by the client indicates to the nurse that further teaching is needed? A. "It is normal for my heart rate to increase slightly when I use my albuterol inhaler." B. "I should always use my beclomethasone inhaler first, then follow it with my albuterol inhaler." C. "I should wait several minutes between puffs of my inhalers." D. "My albuterol can be used between regular doses if I experience difficulty breathing."

B. "I should always use my beclomethasone inhaler first, then follow it with my albuterol inhaler."

Which client has the highest risk for developing skin cancer? A. A 16-year old dark-skinned female who tans in tanning beds once a week B. A 65 year-old fair-skinned male who is a construction worker C. A 25 year-old dark-skinned male whose mother had skin cancer D. A 70 year-old fair-skinned female who works as a secretary

B. A 65 year-old fair-skinned male who is a construction worker

Which client situation requires the most immediate intervention by the nurse? A. One day day after surgery, a client complains of unpleasant flatulence. B. A bedfast client experiences an episode of urinary incontinence. C. An ambulatory client reports the onset of stress incontinence. D. A client receiving regular dosage of an opioid develops constipation

B. A bedfast client experiences an episode of urinary incontinence.

A newly graduated and licensed registered nurse (RN) is in the second day of orientation to the hospital unit. The education director tells the charge nurse that the new graduate should be assigned to care for one client. Which client is best for the nurse to assign to this new graduate? A. A client returning from surgery following a bowel resection B. A client who is 4-days post myocardial infraction C. A 6-day postoperative client who is receiving a blood transfusion D. A client with AIDS who is in the final stages of dying

B. A client who is 4-days post myocardial infraction

After reviewing the Braden Scale findings of residents at a long-term facility, the charge nurse should to tell the unlicensed assistive personnel (UAP) to prioritize skin care for which client? A. An older adult who is unable to communicate elimination needs B. A older man who sheets are damp each time he turned C. A woman with osteoporosis who is unable to bear weight D. A poorly nourished client who requires liquid supplements

B. A older man who sheets are damp each time he turned

A female resident of a long-term care facility is being admitted to the medical department. The client has a fractured humerus and methicillin-resistant staphylococcus aureus (MRSA). Which room should the charge nurse assign this client? A. A private room, and institute protective environment measures B. A semi-private room with a another client who also has MRSA C. A private isolation room with a vented negative airflow system D. A semi-private room with a client who has hepatitis B (HBV)

B. A semi-private room with a another client who also has MRSA

The nurse is caring for a patient who is to receive a transfusion of two units of packed red blood cells. After obtaining the first unit from the blood bank, the nurse would ask which of the following health team members in the nurses' station to assist in checking the unit before administration? A. Unit secretary B. Another registered nurse C. A physician's assistant D. A phlebotomist

B. Another registered nurse

A fair-skinned female client who is an avid runner is diagnosed with malignant melanoma, which is located on the lateral surface of the lower leg. After wide margin resection, the nurse provides discharge teaching. I t is most important for the nurse to emphasize the need to observe for changes in which characteristic? A. Elasticity of the skin B. Appearance of any moles C. Muscle aches and pains D. Pigmentation of the skin

B. Appearance of any moles

A patient is being discharged from the emergency department after being treated for epistaxis. In teaching the family first aid measures in the event the epistaxis would recur, which of the following measures would the nurse suggest? (Select all that apply.) A. Tilt patients head backwards B. Apply ice compresses to the nose C. Pinch the entire soft lower portion of the nose D. Partially insert a small gauze pad into the bleeding nostril

B. Apply ice compresses to the nose C. Pinch the entire soft lower portion of the nose D. Partially insert a small gauze pad into the bleeding nostril

The nurse who is performing blood sugar and cholesterol screenings at a community health fair determines that a female client's blood sugar is 59 mg/dl at 10:00 a.m. Which nursing intervention is most important for the nurse to implement? A. Check the client's cholesterol B. Ask the client how she's feeling C. Take the client's blood pressure D. Encourage the client to rest

B. Ask the client how she's feeling

Thirty-six hours after cesarean delivery, a client complains of nausea and bloating. Assessment reveals a distended abdomen and no bowel movement since delivery. What intervention should the nurse implement first? A. Increase fiber in diet B. Auscultate the abdomen C. Insert a rectal tube D. Encourage ambulation

B. Auscultate the abdomen

During the transfer of a client who had major abdominal surgery this morning, the postanesthesia care unit (PACU) nurse reports that the client, who is awake and responsive, continues to report pain and nausea after receiving morphine 2 mg IV and ondansetron 4 mg IV 45 minutes ago. Which elements of SBAR communication are missing from the report given by the PACU nurse? A. Situation B. Background C. Assessment D. Recommendation E. Rationales

B. Background C. Assessment D. Recommendation

An older male client diagnosed with end-stage chronic obstructive pulmonary disease (COPD) is on strict bedrest, and asks the nurse, "Why can't I get out of bed? What response is best for the nurse to provide? A. A high-Fowler's position promotes lung expansion B. Bed rest decreases your body's need for oxygen C. You are so weak that you are at risk for falling D. hospital policy requires that you are assisted to get up

B. Bed rest decreases your body's need for oxygen

A male client who reports feeling chronically fatigued has a hemoglobin of 11.0 grams/dl (110mmol/L), hematocrit of 34%, and microcytic and hypochromic red blood cells (RBCs). Based on these findings, which dinner selection should the nurse suggest to the client? A. Cheese pasta and a lettuce and tomato salad B. Beef steak with steamed broccoli and orange slices C. Broiled white fish with a baked sweet potato D. Grilled shrimp and season rice with asparagus salad

B. Beef steak with steamed broccoli and orange slices

A nurse is preparing to establish oxygen therapy for a patient with COPD, and the physician's prescription reads "oxygen per nasal cannula at 5 L per minute." Which of the following actions should the nurse take? A. Administer the oxygen as prescribed. B. Call the physician and question the correct flow rate of the oxygen. C. Establish the oxygen as prescribed and obtain an ABG. D. Change the delivery device from a nasal cannula to a simple oxygen mask.

B. Call the physician and question the correct flow rate of the oxygen.

An older male client with long-standing lung disease is admitted to the medical unit for treatment of pulmonary infection. In assessing for signs of increasing hypoxia, which action should the nurse include? (select all that apply) A. Monitor dryness of mucous membranes B. Check for changes in mentation C. Observe color of skin and nailbeds D. Note appearance of jugular veins E. Assess breathing patterns

B. Check for changes in mentation. C. Observe color of skin and mucous. E. Assess breathing patterns

At 1000 the healthcare provider prescribes an increase dose in the dosage of a client's loop diuretic from 40 mg to 80 mg a day. The nurse has already administered today's 40 mg dose of the loop diuretic at 0600. Which action should the nurse implement? A. Schedule the second 40 mg of the diuretic for administration at 1800 that evening B. Clarify the start date of the new dose with the healthcare provider C. Administer an additional 40 mg of the diuretic after checking the serum potassium level D. Implement the increase in the prescribed dosage the following morning

B. Clarify the start date of the new dose with the healthcare provider

When planning care for a client newly diagnosed with open angle glaucoma, the nurse identifies a priority nursing diagnosis of, "visual sensory/perceptual alterations." This diagnosis is based on which etiology? A. Limited eye movement B. Decreased peripheral vision C. Blurred distance vision D. Photosensitivity

B. Decreased peripheral vision

The nurse administers an oral antiviral to a client with shingles. Which finding is most important for the nurse to report to the healthcare provider? A. elevated liver function tests B. Decreased white blood cell count C. Vomiting and diarrhea D. Pruritus and muscle aches

B. Decreased white blood cell count

The new mother tells the nurse that she does not want her newborn to receive any immunizations. It is the hospital's policy to routinely administer immunizations to all newborns. What intervention should the nurse implement? A. Advise the mother to sign out of the hospital AMA if wishing to refuse the immunizations B. Document that the mother has refused immunization permission for the newborn C. Administer the immunizations after first explaining the hospital policy to the mother D. Report the immunization status of the infant to the office of child protective services

B. Document that the mother has refused immunization permission for the newborn

The nurse observes that a client with COPD is exhibiting fingernail clubbing. What action should the nurse take? A. Administer a PRN dose of albuterol (Proventil) via inhaler. B. Document the assessment finding in the nurses' notes C. Assist the client to a tripod position in the bed. D. Increase the client's oxygen from 1 to 2 liters per minute.

B. Document the assessment finding in the nurses' notes

Select all that apply. Which of the following are significant risk factors for leukemia? A. Being a longtime smoker B. Employment in an oil refinery C. History of hemophilia in parent D. Having Down syndrome E. Having a twin brother with leukemia F. Treatment with an alkylating agent = 3 years ago

B. Employment in an oil refinery D. Having Down syndrome E. Having a twin brother with leukemia F. Treatment with an alkylating agent = 3 years ago

A client in the third trimester of pregnancy reports that she feels some "lumpy places" in her breasts and that her nipples sometimes leak yellowish fluid. She has an appointment with her healthcare provider in two weeks. What action should the nurse take? A. Reschedule the client's appointment for the following day B. Explain that this normal secretion can be assessed at the next visit C. Tell the client to begin nipple stimulation to prepare for breast feeding D. Recommend that the client start wearing a supportive brassiere

B. Explain that this normal secretion can be assessed at the next visit

Two months after treatment with radioactive iodine (I131) (RAI) for hyperthyroidism, a male client reports the onset of extreme fatigue, depression, and hair loss. What action should the nurse take? A. Confirm that the client should continue to take his anti-thyroid medication B. Explain to the client that thyroid replacement hormones may be needed. C. Advise the client to increase his caloric intake and eat high protein foods D. Assure the client that these are time-limited side effects of the treatment

B. Explain to the client that thyroid replacement hormones may be needed.

The nurse caring for a client with dysphagia is attempting to insert a nasogastric tube (NGT), but the client will not swallow and is not gagging. What action should the nurse implement to facilitate the NGT passage into the esophagus? A. Offer the client sips of water or ice and coax to swallow B. Flex the client's head with chin to the chest and insert C. Elevate the bed 90 degrees and hyperextend the head D. Push the NGT beyond the oropharynx gently yet swiftly

B. Flex the client's head with chin to the chest and insert

A client who had a biliopancreatic diversion procedure (BDP) 3 months ago is admitted with a severe dehydration. Which assessment finding warrants immediate intervention by the nurse. A. Strong foul-smelling flatus B. Gastroccult positive emesis C. Complaint of poor night vision D. Loose bowel movements

B. Gastroccult positive emesis

A male client with chronic alcohol use is admitted with signs of early cirrhosis. Which nursing action should the nurse delegate to the unlicensed assistive personnel (UAP)? A. Assist client with preferred meal selections B. Gather oral hygiene products for client's use C. Evaluate client for asterixis D. Assess the client for ascites

B. Gather oral hygiene products for client's use

The nurse is triaging clients in an urgent car clinic. The client with which symptoms should be referred to the health care provider immediately? A. High fever, skin rash, and a productive cough B. Headache, photophobia, and nuchal rigidity C. Nausea, vomiting, and poor skin turgor D. Malaise, fever, and stiff, swollen joints

B. Headache, photophobia, and nuchal rigidity

During a well-baby, 6-month visit, a mother tells the nurse that her infant has had fewer ear infections than her 10-year-old daughter. The nurse should explain that which vaccine is likely to have made the difference in the siblings incidence of otitis media? A. Pneumococcal vaccine B. Hemophilus Influenza type B (H1B) vaccine C. Palrvizumab vaccine for RSV D. Varicella Virus Vaccine Live

B. Hemophilus Influenza type B (H1B) vaccine

An adult male is admitted to the psychiatric unit from the emergency department because he is in the manic stage of bipolar disorder. He has lost 10 pounds in the last two weeks and has not bathed in a week because he has been "trying to start a new business" and "too busy to eat." He is alert and oriented to time, place, and person, but not situation. Which nursing problem has the greatest priority? A. Hygiene self-care deficit B. Imbalanced nutrition C. Disturbed sleep pattern D. Self neglect

B. Imbalanced nutrition

During a home visit, the nurse determines that a male client is experiencing symptoms that should be controlled by his prescribed medication. The client states that he forgot when he was supposed to take his medication. What is the priority nursing problem when the nurse develops the plan of care for the client? A. Self neglect related to loss of cognitive function B. Ineffective health maintenance related to lack of knowledge C. Situational low self-esteem related to symptoms of illness D. Noncompliance related to lifestyle change

B. Ineffective health maintenance related to lack of knowledge

The nurse receives a physician's order to transfuse fresh frozen plasma to a patient suffering from an acute blood loss. Which of the following procedures is most appropriate for infusing this blood product? A. Hand the fresh frozen plasma as a piggyback to a new bag of primary IV solution without KCl. B. Infuse the fresh frozen plasma as rapidly as the patient will tolerate. C. Hang the fresh frozen plasma as a piggyback to the primary IV solution. D. Infuse the fresh frozen plasma as a piggyback to a primary solution of normal saline.

B. Infuse the fresh frozen plasma as rapidly as the patient will tolerate.

A female college student is admitted to the Emergency Department following indigestion of alcohol and pain medication. A nasogastric tube and subclavian line are placed. The nurse auscultates audible breath sounds on the right side, faint sounds on the left side, and chest involvement that occurs only on the right side of the thorax. Which procedure should the nurse prepare for first? A. removal of the subclavian line and preparation for jugular insertion B. Insertion of 16g needle at the 4th intercostal space midclavicular line C. Placement of an endotracheal tube and mechanical ventilation D. Retraction of the nasogastric tube by a length of 2 cm

B. Insertion of 16g needle at the 4th intercostal space midclavicular line

An adult male who returned from a vacation in Mexico three weeks ago calls the clinic complaining of abdominal pain, weight loss, and diarrhea. What action should the nurse take? A. Encourage the client to go to the emergency room B. Instruct the client to bring in a stool sample C. Ask the client if he is experiencing dyspnea D. Tell the client to eat toast and Gatorade sports drink

B. Instruct the client to bring in a stool sample

A male client with Addison's disease tells the nurse that he is taking hydrocortisone in a divided daily dose. He reports increasing fatigue and weakness. What action should the nurse take? A. Advise the client to skip the next scheduled dose of hydrocortisone B. Interview the client about any sources of increased stress in his life C. Instruct the client to limit his intake of oral fluids, especially at night D. Encourage the client to increase daily exercise and physical activity

B. Interview the client about any sources of increased stress in his life

In assessing a client with ulcers on the lower extremity, which findings indicate that the ulcers are likely to be of venous, rather than arterial, origin? A. Black ulcers and dependent rubor B. Irregular ulcer shapes and sever edema C. Absent pedal pulses and shiny skin D. Hairless lower extremities and cool feet

B. Irregular ulcer shapes and severe edema

An adult male client is admitted for Pneumocystis carinal pneumonia (PCP) secondary to AIDSs. While hospitalized, he receives IV pentamidine isethionate therapy. In preparing this client for discharge, what important aspect regarding his medication therapy should the nurse explain? A. IV pentamidine may offer protection to other AIDS-related conditions, such as Kaposi's sarcoma B. It will be necessary to continue prophylactic doses of IV or aerosol pentamidine every month C. IV pentamidine will be given until oral pentamidine can be tolerated D. AZT (Azidothymidine) therapy must be stopped when IV or aerosol pentamidine is being used.

B. It will be necessary to continue prophylactic doses of IV or aerosol pentamidine every month

A client with Alzheimer's Disease falls in the bathroom. The nurse notifies the charge nurse and completes a fall follow-up assessment. What assessment finding warrants immediate intervention by the nurse? A. Urinary incontinence B. Left forearm hematoma C. Disorientation to surroundings D. Dislodged intravenous site

B. Left forearm hematoma

What intervention should the nurse implement to prevent edema and promote healing of a client's incision resulting from an above-the-knee amputation? A. Keep the residual limb in a dependent position B. Maintain the residual limb in a compression dressing C. Inspect the incision hourly for the first 24 hours postoperatively D. Elevate the affected limb on two pillows at all times

B. Maintain the residual limb in a compression dressing

A client who had an emergency appendectomy is being mechanically ventilated, and soft wrist restraints are in place to prevent self extubation. Which outcome is most important for the nurse to include in the client's plan of care? A. Understands pain management scale B. Maintains effective breathing patterns C. Absence of ventilator associated pneumonia D. No injuries related to soft restraints occur

B. Maintains effective breathing patterns

The nurse makes a supervisory home visit to observes an unlicensed assistive personnel (UAP) who is providing personal care for a client with Alzheimer's Disease. The nurse observes that whenever the client gets upset, the UAP changes the subject. What action should the nurse take in response to this observation? A. Affirm that the UAP is using an effective strategy to reduce the client's anxiety B. Meet with the UAP later to role model more assertive communication techniques C. Assume care of the client to ensure that the effective communication is maintained D. Tell the UAP to offer more choices during the personal care to prevent anxiety

B. Meet with the UAP later to role model more assertive communication techniques

The nurse learns that a client in a semi-private room has a postoperative wound that is colonized with a multi-drug resistant organism. What action should the nurse implement? A. Maintain standard precautions and place the roommate in protective isolation B. Move the client to a private room and implement contact precautions for the client C. Initiate airborne precautions for both clients, leaving the in the semi-private room D. Begin droplet precautions for both clients before moving them to private rooms

B. Move the client to a private room and implement contact precautions for the client

A client with superficial burn to the face, neck. and hands resulting from a house fire is admitted to the burn unit. Which assessment finding indicated to the nurse that the client should be monitored for carbon monoxide poisoning? A. Expiratory stridor and nasal flaring B. Mucus membrane cherry red color C. Carbonaceous particles in sputum D. Pulse oximetry reading of 80 percent

B. Mucus membrane cherry red color

A male client with pneumonia is diaphoretic and confused. The cardiac monitor indicates tachycardia with frequent premature ventricular beats. Atrial blood gas (ABG) results are: pH 7.24, PaCO2 65 mmHg, HCO3 24 mEq/L. Which intervention is most important for the nurse to include in the client's plan of care? A. Obtain a 12 lead electrocardiogram (ECG) daily B. Observe frequently for signs of hypoventilation C. Assess lungs for increasing pulmonary secretions D. Maintain patent IV catheter for antibiotic therapy

B. Observe frequently for signs of hypoventilation

The nurse observes that a postoperative client with a continuous bladder irrigation has a large blood clot in the urinary drainage tubing. What action should the nurse perform first? A. Determine the client's blood pressure and apical pulse rate B. Observe the amount of urine in the client's drainage bag C. Obtain a pulse oximeter to assess the client's oxygen saturation D. Review the medication record for recently administered medications

B. Observe the amount of urine in the client's drainage bag

A client with a peripherally inserted central venous catheter (PICC) line has a fever. What client assessment is most important for the nurse to perform? A. Gently palpate the neck for tenderness B. Observe the antecubital fossa for inflammation C. Check for drainage in the subclavian area D. Measure for any increase in abdominal girth

B. Observe the antecubital fossa for inflammation

A client with a productive cough has obtained a sputum specimen for culture as instructed. What is the best initial nursing action? A. Administer the first dose of antibiotic therapy B. Observe the color, consistency, and amount of sputum C. Encourage the client to consume plenty of warm liquids D. Send the specimen to the lab for analysis

B. Observe the color, consistency, and amount of sputum

The nurse assesses a postoperative client. Oxygen is being administered at 2 L/min and a saline lock is in place. Assessment shows cool, pale, moist skin. The client is very restless and has scant urine in the urinary drainage bag. What intervention should the nurse implement first. A. Measure urine specific gravity. B. Obtain IV fluids for infusion per protocol. C. Prepare for insertion of a central venous catheter. D. Auscultate the client's breath sounds.

B. Obtain IV fluids for infusion per protocol.

A male client tells the clinic nurse that he is experiencing burning on urination, and assessment reveals that he had sexual intercourse four days ago with a woman he casually met. Which action should the nurse implement? A. Observe the perineal area for a chancroid-like lesion B. Obtain a specimen of urethral drainage for culture C. Assess for perineal itching, erythema and excoriation D. Identify all sexual partners in the last four days

B. Obtain a specimen of urethral drainage for culture

The nurse is assessing an infant on admission to the newborn nursery and finds that both brachial pulses are bounding, but bilateral femoral pulses are only slightly palpable. Which assessment should the nurse implement next? A. Assess heart sounds for a murmur B. Obtain blood pressures in all extremities C. Listen to the lung fields for fine crackles D. Elevate the legs for evidence of edema

B. Obtain blood pressures in all extremities

The nurse is using a straight urinary catheter kit to collect a sterile urine specimen firm a female client. After positioning and prepping the client, rank the actions in the sequence they should be implemented. A. Don sterile gloves and prepare the sterile field. B. Open the sterile catheter kit close to the client's perineum C. Cleanse the urinary meatus using the solution, swabs, and forceps provided D. Place distal end of the catheter in sterile specimen cup and insert catheter into meatus

B. Open the sterile catheter kit close to the client's perineum A. Don sterile gloves and prepare the sterile field. C. Cleanse the urinary meatus using the solution, swabs, and forceps provided D. Place distal end of the catheter in sterile specimen cup and insert catheter into meatus

The nurse has received funding to design a health promotion project for African-American women who are at risk for developing breast cancer. Which resource is most important in designing this program? A. A lasting of African-American women who live in the community B. Participation of community leaders in planning the program C. Morbidity data for breast cancer in women of all races D. Techical assistacne to produce a video on breast self-examination

B. Participation of community leaders in planning the program

A client who is newly diagnosed with emphysema is being prepared for discharge. Which instruction is best for the nurse to provide the client to assist them with dyspnea self-management? A. Allow additional time to complete physical activities to reduce oxygen demand B. Practice inhaling through the nose and exhaling slowly through pursed lips C. Use a humidifier to increase home air quality humidity between 30-50% D. Strengthen abdominal muscles by alternating leg raises during exhalation

B. Practice inhaling through the nose and exhaling slowly through pursed lips

The nurse is planning care for a client with diabetes mellitus who has gangrene of the toes to the midfoot. Which goal should be included in this client's plan of care?

B. Prevent infection.

The nurse is planning care for a client with diabetes mellitus who has gangrene of the toes to the midfoot. Which goal should be included in this client's plan of care? A.Restore skin integrity. B.Prevent infection. C.Promote healing. D.Improve nutrition.

B. Prevent infection.

The nurse is caring for six clients on a medical-surgical unit. Which interventions should the nurse delegate to the unlicensed assistive personnel (UAP)? (select all that apply) A. Assess daily weights for trends B. Record vital signs every four hours C. Assist with ambulation as prescribed D. Provide oral care after meals E. Monitor for signs of dehydration

B. Record vital signs every four hours C. Assist with ambulation as prescribed D. Provide oral care after meals

The nurse is managing clients who are mechanically ventilated. The client with which assessment finding requires the most immediate intervention by the nurse? A. Audible voice when client is trying to communicate B. Restrained and restless with a low volume alarm C. High pressure alarm when client is coughing D. Diminished breath sounds in the right posterior base

B. Restrained and restless with a low volume alarm

The nurse is teaching a client with glomerulonephritis about self-care. Which dietary recommendations should the nurse encourage the client to follow? A. Increase intake of high-fiber foods, such as bran cereal B. Restrict protein intake by limiting meats and other high-protein foods C. Limit oral fluid intake to 500 ml per day D. Increase intake of potassium-rich foods such as bananas or cantaloupe

B. Restrict protein intake by limiting meats and other high-protein foods

A newborn is apnea for 20 seconds. What action should the nurse implement? A. Place oxygen cannula near the nares B. Stimulate by gently rubbing infant's trunk C. Begin cardiopulmonary resuscitation D. Suction the infant's nano-oropharynx

B. Stimulate by gently rubbing infant's trunk

A male client with chronic renal failure (CRF) is admitted to the intensive care unit after missing his last three appointments at the dialysis center. His arterial blood gas (ABG) results are: pH 7.32; PaCO2 32 mmHG; HCO3 18 mEq/L. Which assessment finding should the nurse expect this client to exhibit? A. Diaphoresis B. Tachypnea C. Hypotension D. Bradycardia

B. Tachypnea

An unlicensed assistive personnel (UAP) is teamed with a nurse who is caring for four clients. Client A is admitted to the medical unit for heart failure (HF) Client B has just returned from surgery Client C, with chronic obstructive pulmonary disease (COPD), needs new oxygen tubing Client D is waiting to be discharged Which activity should the nurse delegate to the UAP as having the highest priority? A. Obtain the daily weight for the client with HF B. Take vital signs of the client who just returned form surgery C. Gather equipment for oxygen tubing change for the client with COPD D. Assist client into the wheelchair for discharge

B. Take vital signs of the client who just returned form surgery

A male client returns to the mental health clinic for assistance with his anxiety reaction that is manifested by a rapid heartbeat, sweating, shaking, and nausea while driving over the bay bridge. What action in the treatment plan should the nurse implement? A.Tell client to drive over the bridge until fear is manageable B. Teach client to listen to music or audio books while driving C. Encourage client to have spouse drive in stressful places D. Recommend that the client avoid driving over the bridge

B. Teach client to listen to music or audio books while driving

The nurse is completing the preoperative assessment of a client who is scheduled for a laparoscopic cholecystectomy under general anesthesia. Which finding warrants notification of the healthcare provider prior to proceeding with the scheduled procedure? A. Light yellow coloring of the client's skin and eyes B. The client's blood pressure reading is 184/88 mm Hg. C. The client vomits 20 ml of clear yellowish fluid D. The IV insertion site is red, swollen, and leaking IV fluid

B. The client's blood pressure reading is 184/88 mm Hg

A client with a prescription for "do not resuscitate" (DNR) begins to manifest signs of impending death. After notifying the family of the client's status, what priority action should the nurse implement? A. The healthcare provider should be notified of the client's status B. The client's need for pain medication should be determined C. The chaplain should be requested to come to the client's bedside D. The client's signs of impending death should be documented

B. The client's need for pain medication should be determined

A 58-year-old client who has no health problems asks the nurse about receiving the pneumococcal vaccine (Pneumovax). Which statement given by the nurse would offer the client accurate information about this vaccine?

B. The immunization is administered once to older adults or those at risk for illness.

A client slips and falls while getting out of bed and the charge nurse instructs the nurse who is caring for the client to complete an incident report. What is the main purpose in having the nurse complete the incident report? A. To ensure that the nurse caring for the client takes responsibility for the incident. B. To provide computer documentation of the incident as a basis for further investigation C. To protect the nurse and the hospital against charges of malpractice D. To reprimand the nurse for not providing safe care for the client

B. To provide computer documentation of the incident as a basis for further investigation

The nurse is explaining the need to reduce salt intake to a client with primary hypertension. What explanation should the nurse provide? A. High salt can damage the lining of the blood vessels B. Too much salt can cause the kidneys to retain fluid C. Excessive salt can cause blood vessels to contract D. Salt can cause inflammation inside the blood vessels

B. Too much salt can cause the kidneys to retain fluid

A client with dementia who is cared for at home by her husband becomes increasingly confused in the evening. Her husband reports to the home health care nurse that his wife often believes that she is waiting for the oil to be changed in her car and insists on leaving. Which recommendation should the nurse provide to this husband? A. Remind his wife that she is just a little confused B. Try to interest his wife in a different activity C. Give his wife a PRN dose of haloperidol (Haldol) D. Show his wife her car that is still in the garage

B. Try to interest his wife in a different activity

The nurse plans to collect a 24-hour urine specimen for a creatine clearance test. Which instruction should the nurse provide to the adult male client? A. Urinate immediately into a urinal, and the lab will collect the specimen every 6 hours, for the next 24 hours B. Urinate at a specified time, discard this urine, and collect all subsequent urine during the next 24 hours C. For the next 24 hours, notify nurse when the bladder is full, and the nurse will collect catheterized specimens D. Cleanse around the meatus, discard first portion of voiding, and collect the rest in a sterile bottle

B. Urinate at a specified time, discard this urine, and collect all subsequent urine during the next 24 hours

A female client who was involved in a motor vehicle collision is admitted with a fractured left femur which is immobilized using a fracture traction splint in preparation for an open reduction internal fixation (ORIF). The nurse determines that her distal pulses are diminished in the left foot. Which interventions should the nurse implement? (Select all that apply.) A. Offer ice chips and oral clear liquids B. Verify pedal pulses using a doppler pulse device C. Monitor left leg for pain, pallor, paresthesia, paralysis, pressure D. Evaluate the application of the splint to the left leg E. Administer oral antispasmodics and narcotic analgesics

B. Verify pedal pulses using a doppler pulse device C. Monitor left leg for pain, pallor, paresthesia, paralysis, pressure D. Evaluate the application of the splint to the left leg

If a patient with an uncuffed tracheostomy tube coughs violently during suctioning and dislodges the tracheostomy tube, a nurse should first A. call the physician. B. attempt to reinsert the tracheostomy tube. C. position the patient in a lateral position with the neck extended. D. cover the stoma with a sterile dressing and ventilate the patient with a manual bag-mask until the physician arrives.

B. attempt to reinsert the tracheostomy tube.

The nurse prepares to suction a client using nasotracheal suctioning. Which ongoing assessments should the nurse plan to complete while performing the procedure? A. amount of wall suction B. color of the lips C. breathing pattern D. breath sounds E. appearance of secretions

B. color of the lips C. breathing pattern E. appearance of secretions

The resurgence in TB resulting from the emergence of multidrug-resistant strains of Mycobacterium tuberculosis is primarily the result of A. a lack of effective means to diagnose TB. B. poor compliance with drug therapy in patients with TB. C. the increased population of immunosuppressed individuals with AIDS. D. indiscriminate use of antitubercular drugs in treatment of other infections.

B. poor compliance with drug therapy in patients with TB.

A client who recently underwent a tracheostomy is being prepared for discharge to home. Which instructions is most important for the nurse to include in the discharge plan? A. explain how to use communication tools B. teach tracheal suctioning techniques C. encourage self-care and independence D. demonstrate how to clean tracheostomy site

B. teach tracheal suctioning techniques

A client has taken steroids for 12 years to help manage chronic obstructive pulmonary disease (COPD). When making a home visit, which nursing function is of greatest importance to this client? Assess the client's A) pulse rate, both apically and radially. B) blood pressure, both standing and sitting. C) temperature. D) skin color and turgor.

C

A client is admitted to the medical intensive care unit with a diagnosis of myocardial infarction. The client's history indicates the infarction occurred ten hours ago. Which laboratory test result should the nurse expect this client to exhibit? A) Elevated LDH. B) Elevated serum amylase. C) Elevated CK-MB. D) Elevated hematocrit.

C

A client taking a thiazide diuretic for the past six months has a serum potassium level of 3. The nurse anticipates which change in prescription for the client? A) The dosage of the diuretic will be decreased. B) The diuretic will be discontinued. C) A potassium supplement will be prescribed. D) The dosage of the diuretic will be increased.

C

A client with multiple sclerosis has experienced an exacerbation of symptoms, including paresthesias, diplopia, and nystagmus. Which instruction should the nurse provide? A) Stay out of direct sunlight. B) Restrict intake of high protein foods. C) Schedule extra rest periods. D) Go to the emergency room immediately.

C

A female client taking oral contraceptives reports to the nurse that she is experiencing calf pain. What action should the nurse implement? A) Determine if the client has also experienced breast tenderness and weight gain. B) Encourage the client to begin a regular, daily program of walking and exercise. C) Advise the client to notify the healthcare provider for immediate medical attention. D) Tell the client to stop taking the medication for a week to see if symptoms subside.

C

After the fourth dose of gentamicin sulfate (Garamycin) IV, the nurse plans to draw blood samples to determine peak and trough levels. When are the best times to draw these samples? A) 15 minutes before and 15 minutes after the next dose. B) One hour before and one hour after the next dose. C) 5 minutes before and 30 minutes after the next dose. D) 30 minutes before and 30 minutes after the next dose.

C

An elderly male client comes to the geriatric screening clinic complaining of pain in his left calf. The nurse notices a reddened area on the calf of his right leg which is warm to the touch and suspects it might be thrombophlebitis. Which type of pain should further confirm this suspicion? A) Pain in the calf awakening him from a sound sleep. B) Calf pain on exertion which stops when standing in one place. C) Pain in the calf upon exertion which is relieved by rest and elevating the extremity. D) Pain upon arising in the morning which is relieved after some stretching and exercise.

C

The nurse is completing an admission interview and assessment on a client with a history of Parkinson's disease. Which question should provide information relevant to the client's plan of care? A) Have you ever experienced any paralysis of your arms or legs? B) Have you ever sustained a severe head injury? C) Have you ever been 'frozen' in one spot, unable to move? D) Do you have headaches, especially ones with throbbing pain?

C

The nurse is teaching a client with maple syrup urine disease (MSUD), an autosomal recessive disorder, about the inheritance pattern. Which information should the nurse provide? A) This recessive disorder is carried only on the X chromosome. B) Occurrences mainly affect males and heterozygous females. C) Both genes of a pair must be abnormal for the disorder to occur. D) One copy of the abnormal gene is required for this disorder.

C

The nurse is working with a 71-year-old obese client with bilateral osteoarthritis (OA) of the hips. What recommendation should the nurse make that is most beneficial in protecting the client's joints? A) Increase the amount of calcium intake in the diet. B) Apply alternating heat and cold therapies. C) Initiate a weight-reduction diet to achieve a healthy body weight. D) Use a walker for ambulation to lessen weight-bearing on the hips.

C

The nurse knows that lab values sometimes vary for the older client. Which data should the nurse expect to find when reviewing laboratory values of an 80-year-old male? A) Increased WBC, decreased RBC. B) Increased serum bilirubin, slightly increased liver enzymes. C) Increased protein in the urine, slightly increased serum glucose levels. D) Decreased serum sodium, an increased urine specific gravity.

C

What discharge instruction is most important for a client after a kidney transplant? A) Weigh weekly. B) Report symptoms of secondary Candidiasis. C) Use daily reminders to take immunosuppressants. D) Stop cigarette smoking.

C

When preparing a client who has had a total laryngectomy for discharge, which instruction is most important for the nurse to include in the discharge teaching? A) Recommend that the client carry suction equipment at all times. B) Instruct the client to have writing materials with him at all times. C) Tell the client to carry a medic alert card stating that he is a total neck breather. D) Tell the client not to travel alone.

C

Which postmenopausal client's complaint should the nurse refer to the healthcare provider? A) Breasts feel lumpy when palpated. B) History of white nipple discharge. C) Episodes of vaginal bleeding. D) Excessive diaphoresis occurs at night.

C

While working in the emergency room, the nurse is exposed to a client with active tuberculosis. When should the nurse plan to obtain a tuberculin skin test? A) Immediately after the exposure. B) Within one week of the exposure. C) Four to six weeks after the exposure. D) Three months after the exposure.

C

The physician orders Lasix (furosemide) 60 mg po every day for your patient. On hand you have Lasix 40 mg. How many tablets will you give the patient? a. 3 b. 1 c. 1 1/2 d. 2 1/5

C 60/40 (desired/have)

The nurse is assessing a client who has undergone a transurethral resection of the prostate (TURP). Which assessment finding requires immediate action by the nurse? A) Having the urge to void continuously while the catheter is inserted B) Passing small blood clots after catheter removal C) Having bright red drainage with multiple blood clots D) Experiencing urinary frequency after catheter removal

C A client who undergoes a TURP is at risk for bleeding during the first 24 hours after surgery. Passage of small blood clots and tissue debris, urinary frequency and leakage, and the urge to void continuously while the client still has the catheter inserted are all considered to be expected complications of the procedure. They will resolve as the client continues to recover and the catheter is removed. However, the presence of bright red blood with clots indicates arterial bleeding and should be reported to the provider.

A client presents with a pressure ulcer on the ankle. Which is the first intervention that the nurse implements? A) Place the client in bed and instruct him or her to elevate the foot. B) Prepare for and assist with obtaining a wound culture. C) Assess the affected leg for pulses, skin color, and temperature. D) Draw blood for albumin, prealbumin, and total protein.

C A client with an ulcer on the foot should be assessed for interruption in arterial flow to the area. This begins with assessment of pulses and color and temperature of the skin. The nurse can also assess for pulses noninvasively with a Doppler if unable to palpate with his or her fingers. Elevation of the foot would impair the ability of arterial blood to flow to the area. Wound cultures are done after it has been determined drainage, odor, and other risks for infection are present. Tests to determine nutritional status and risk assessment would be completed after the initial assessment is done.

Which finding puts a client at greatest risk for wound infection? A) Presence of a deep wound B) Coexisting medical conditions C) Immune compromised status D) Severely reddened skin

C A compromised immune system puts a client at greatest risk for infection. Although all the other options might increase the client's susceptibility, the one with the greatest potential impact is being immune compromised.

A client has a wound on his left trochanter that is 4 inches in diameter, with black tissue at the perimeter, and bone is exposed. Which is the nurse's best action? A) Document as a stage I pressure ulcer and apply a transparent dressing. B) Document as a stage II pressure ulcer and start wet-to-dry gauze treatments. C) Document as a stage IV pressure ulcer and prepare the client for débridement. D) Document as a stage III pressure ulcer and start antibiotic therapy.

C A stage IV ulcer is one in which skin loss is full thickness, with extensive destruction, tissue necrosis, and/or damage to muscle, bone, or supporting structures. Eschar may be present. When the bone of the trochanter area is visible, tissue loss includes muscle loss. A potential intervention consists of débridement of the necrotic tissue and a possible graft to promote healing.

A patient has been newly diagnosed with hypertension. The nurse assesses the need to develop a collaborative plan of care that includes a goal of adhering to the prescribed regimen. When the nurse is planning teaching for the patient, which is the most important initial learning goal? A) The patient will demonstrate coping skills needed to manage hypertension. B) The patient will verbalize the side effects of treatment. C) The patient will select the type of learning materials they prefer. D) The patient will verbalize an understanding of the importance of following the regimen.

C Adults learn best when given information they can understand that is tailored to their learning styles and needs. Verbalizing an understanding is important; however, the nurse will first need to teach the patient.

Which person is at greatest risk for developing a community-acquired pneumonia? A) Young adult aerobics instructor who is a vegetarian B) Middle-aged teacher who typically eats a diet of Asian foods C) Older adult who smokes and has a substance abuse problem D) Older adult with exercise-induced wheezing

C Although age is a factor in the development of community-acquired pneumonia, other lifestyle and exposure factors increase the risk to a greater extent than age. Two conditions that heavily predispose to the development of pneumonia are cigarette smoking and alcoholism. Dietary choices typically do not predispose to the development of pneumonia. Cigarette smoking interferes with the ciliary function of removal of invasive materials. Alcoholism usually results in unbalanced nutrition, as well as decreased immune function. A middle-aged adult, an older adult with wheezing induced by exercise, and a young adult vegetarian would not be at risk for community-acquired pneumonia because they have no predisposing conditions.

The health care provider has prescribed a client sodium warfarin (Coumadin) while he is still receiving intravenous heparin. Which is the nurse's best action? A) Turn off the heparin before administering the warfarin. B) Clarify the warfarin order with the nursing supervisor. C) Administer both heparin and warfarin as prescribed. D) Hold the warfarin dose until the heparin is discontinued.

C Although both heparin and warfarin are anticoagulants, they have different mechanisms of action and onsets of action. Because warfarin has such a slow onset, it must be started while the client is still receiving heparin. Once the warfarin is therapeutic, as evidenced by the international normalized ratio (INR), the client's heparin can be safely discontinued. Effects of heparin will be cleared from the client's bloodstream within a few hours.

The nurse is caring for a client with ulcerative colitis and severe diarrhea. Which nursing assessment is the highest priority? A) Skin integrity B) Blood pressure C) Heart rate and rhythm D) Abdominal percussion

C Although the client with severe diarrhea may experience skin irritation and hypovolemia, the client is most at risk for cardiac dysrhythmias secondary to potassium and magnesium loss from severe diarrhea. The client should have her or his electrolyte levels monitored, and electrolyte replacement may be necessary. Abdominal percussion is an important part of physical assessment but has lower priority for this client than heart rate and rhythm.

The nurse is assessing a client with a long-term history of arthritic pain. Assessment reveals a heart rate of 115 beats/min and blood pressure of 170/80 mm Hg. Which intervention will the nurse carry out first? A) Administer blood pressure medication. B) Administer a drug to lower the heart rate. C) Continue to assess for possible causes of elevated vital signs. D) Assess whether the client needs anti-arthritis medication.

C Arthritis is categorized as chronic pain. With chronic pain, the body adapts by blocking the sympathetic nervous system; this normally causes tachycardia and increased blood pressure. Therefore, this client's high blood pressure and heart rate are not caused by chronic pain and may be a result of a more acute type of pain. Therefore, the best intervention is for the nurse to establish whether the client is having pain other than arthritic pain, and then to decide which intervention should be carried out.

A nurse is caring for several clients at risk for overhydration. The nurse assesses the older client with which finding first? A) Has had diabetes mellitus for 12 years B) Had abdominal surgery and has a nasogastric tube C) Just received 3 units of packed red blood cells D) Uses sodium-containing antacids frequently

C Blood replacement therapy involves intravenous fluid administration, which inherently increases the risk for overhydration. The fact that the fluid consists of packed red blood cells greatly increases the risk, because this fluid increases the colloidal oncotic pressure of the blood, causing fluid to move from interstitial and intracellular spaces into the plasma volume. An older adult may not have sufficient cardiac or renal reserve to manage this extra fluid.

A client has been taught to restrict dietary sodium. Which food selection by the client indicates to the nurse that teaching has been effective? a. a grilled cheese sandwich with tomato soup b. Chinese take-out, including steamed rice c. a chicken leg, one slice of bread with butter, and steamed carrots d. slices of ham and cheese on whole grain crackers

C Clients on restricted sodium diets generally should avoid processed, smoked, and pickled foods and those with sauces and other condiments. Foods lowest in sodium include fish, poultry, and fresh produce. The chinese food likely would have soy sauce, the tomato soup is processed, and the crackers are a snack food - a category of foods often high in sodium.

A client has a urinary tract infection. Which assessment by the nurse is most helpful? A) Palpating and percussing the kidneys and bladder B) Performing a bladder scan to assess post-void residual C) Assessing medical history and current medical problems D) Inquiring about recent travel to foreign countries

C Clients who are severely immune compromised or who have diabetes mellitus are more prone to fungal urinary tract infection. The nurse should assess for these factors. A physical examination and a post-void residual may be needed, but not until further information is obtained. Travel to foreign countries probably would not be as important, because even if exposed, the client needs some degree of immune compromise to develop a fungal urinary tract infection.

During assessment of a client with a 15-year history of diabetes, the nurse notes that the client has decreased tactile sensation in both feet. Which action does the nurse take first? A) Notify the health care provider. B) Document the finding in the client's chart. C) Examine the client's feet for signs of injury. D) Test sensory perception in the client's hands.

C Diabetic neuropathy is common when the disease is of long duration. The client is at great risk for injury in any area with decreased sensation because he or she is less able to feel injurious events. Feet are common locations for neuropathy and injury, so the nurse should inspect them for any signs of injury. After assessing, the nurse should document findings in the client's chart. Testing sensory perception in the hands may or may not be needed. The health care provider can be notified after assessment and documentation have been completed.

The nurse is caring for a client who is disoriented as the result of a stroke. Which action does the nurse implement to help orient this client? A) Turn on the television to a 24-hour news station. B) Provide auditory and visual stimulation simultaneously. C) Ask the family to bring in pictures familiar to the client. D) Maintain a calm and quite environment by minimizing visitors.

C For the client with disorientation, the nurse can request that the family bring in pictures or objects that are familiar to the client. The nurse explains what the object or picture represents in simple terms. These stimuli can be presented several times daily. Visitors can also be familiar stimuli to reorient the client. Too much stimuli and constant stimuli can lead to further confusion.

The nurse is assessing a patient's functional ability. Which activities most closely match the definition of functional ability? A) Healthy individual, college educated, travels frequently, can balance a checkbook B) Healthy individual, works out, reads well, cooks and cleans house C) Healthy individual, volunteers at church, works part time, takes care of family and house D) Healthy individual, works outside the home, uses a cane, well groomed

C Functional ability refers to the individual's ability to perform the normal daily activities required to meet basic needs; fulfill usual roles in the family, workplace, and community; and maintain health and well-being. The other options are good; however, each option has advanced or independent activities in the context of the option.

The client with type 2 diabetes has recently been changed from the oral antidiabetic agents glyburide (Micronase) and metformin (Glucophage) to glyburide-metformin (Glucovance). The nurse includes which information in the teaching about this medication? A) "Glucovance is more effective than glyburide and metformin." B) "Your diabetes is improving and you now need only one drug." C) "Glucovance contains a combination of glyburide and metformin." D) "Glucovance is a new oral insulin and replaces all other oral antidiabetic agents."

C Glucovance is composed of glyburide and metformin. It is given to enhance the convenience of antidiabetic therapy with glyburide and metformin. The other statements are not accurate.

The patient's laboratory report today indicates severe hypokalemia, and the nurse has notified the physician. Nursing assessment indicates that heart rhythm is regular. What is the most important nursing intervention for this patient now? A) Examine sacral area and patient's heels for skin breakdown due to potential edema. B) Establish seizure precautions due to potential muscle twitching, cramps, and seizures. C) Institute fall precautions due to potential postural hypotension and weak leg muscles. D) Raise bed side rails due to potential decreased level of consciousness and confusion.

C Hypokalemia can cause postural hypotension and bilateral muscle weakness, especially in the lower extremities. Both of these increase the risk of falls. Hypokalemia does not cause edema, decreased level of consciousness, or seizures.

The client was given 15 mg of morphine IM for postsurgical pain. When the nurse checks the client for pain relief 1 hour later, the client is sleeping and has a respiratory rate of 10 breaths/min. What is the nurse's first action? A) Administering oxygen by nasal cannula B) Documenting the findings and continuing to monitor C) Arousing the client by calling his or her name D) Administering naloxone (Narcan) IV push

C Many clients experience some degree of respiratory depression with opioid analgesics. If the client can be aroused with minimally intrusive techniques and the rate of respiration is increased spontaneously, no further intervention is required.

A client presents with an acute exacerbation of multiple sclerosis. Which prescribed medication does the nurse prepare to administer? A) Interferon beta-1b (Betaseron) B) Baclofen (Lioresal) C) Methylprednisolone (Medrol) D) Dantrolene sodium (Dantrium)

C Methylprednisolone is the drug of choice for acute exacerbations of the disease. The other medications are not appropriate.

Which statement indicates that the client needs more teaching about mucositis? A) "I will use a soft-bristled toothbrush to prevent trauma." B) "I will rinse my mouth with water after every meal." C) "I should use an alcohol-based mouth rinse to kill bacteria." D) "I cannot use floss because it may irritate my gums."

C Mouthwashes that contain alcohol are drying and can exacerbate mucosal irritation, leading to painful mouth sores. Rinsing the mouth with water or normal saline is indicated. Interventions aimed at decreasing risk for trauma or irritation are matters of priority because of inflammation associated with mucositis.

Which client is at greatest risk for dehydration? a. younger adult client on bedrest b. older adult client receiving hypotonic IV fluid c. older adult client with cognitive impairment d. younger adult client receiving hypertonic IV fluid

C Older adults, because they have less total body water than younger adults, are at greater risk for development of dehydration. Anyone who is cognitively impaired and cannot obtain fluids independently or cannot make his or her need for fluids known is at high risk for dehydration

The nurse is a assessing a client with hypertension. Which client outcome is indicative of effective hypertension management? A) No complaints of sexual dysfunction occur. B) Pedal edema is not present in the lower legs. C) No indication of renal impairment is present. D) The blood pressure reading is 148/94 mm Hg.

C One expected outcome for a client with hypertension is for the client to have no evidence of target organ damage, such as renal or heart disease, that can occur with poorly managed hypertension. Development of pedal edema is not directly related to the management of hypertension. Side effects of some hypertensive agents may interfere with sexual function, but this does not relate to the effectiveness of treatment for hypertension. The blood pressure reading is too high to demonstrate effective management.

When a diabetic patient asks about maintaining adequate blood glucose levels, which of the following statements by the nurse relates most directly to the necessity of maintaining blood glucose levels no lower than about 74 mg/dl? A) "Without a minimum level of glucose circulating in the blood, erythrocytes cannot produce ATP." B) "The presence of glucose in the blood counteracts the formation of lactic acid and prevents acidosis." C) "The central nervous system cannot store glucose and needs a continuous supply of glucose for fuel." D) "Glucose is the only type of fuel used by body cells to produce the energy needed for physiologic activity."

C The brain cannot synthesize or store significant amounts of glucose; thus a continuous supply from the body's circulation is needed to meet the fuel demands of the central nervous system.

A client has a small-bore nasoenteric feeding tube. The nurse assesses the following vital signs: temperature, 100.2° F (37.8° C); pulse, 112 beats/min; respiratory rate, 22 breaths/min; and blood pressure, 106/62 mm Hg. Which action by the nurse takes priority? A) Auscultate bowel sounds and slow the feeding down. B) Remove the tube immediately and notify the heath care provider. C) Auscultate lung sounds and obtain oxygen saturation. D) Add blue dye to the feeding tube formula.

C The client may have aspirated. The nurse should further assess the client's respiratory and oxygenation status. The client may have another reason for the abnormal vital signs, so the nurse should not pull out the tube before performing other assessments. Adding blue dye to the tube feeding formula is not recommended to check for aspiration. Slowing the feeding down will not be helpful.

A client is admitted with left lower lung pneumonia. Which assessment finding does the nurse correlate with this condition? A) Expiratory wheeze on the right side B) Crackles heard on expiration bilaterally C) Dullness to percussion on the lower left side D) Crepitus of the skin around the left lung

C The client with pneumonia may have dullness to percussion on the affected side. The other options are all inconsistent with pneumonia.

The Institute for Healthcare Improvement (IHI) identified interventions to save client lives. Which actions are within the scope of nursing practice to improve quality of care? A) Prescribe aspirin for a client who presents with an acute myocardial infarction B) Insert a central line to give intravenous fluid to a dehydrated client. C) Use sterile technique when changing dressings on a new surgical site. D) Intubate a client whose oxygen saturation is 92%.

C The only intervention identified within the scope of nursing practice is to use sterile technique. Central line insertion, intubation, and prescription are functions of the physician.

The nurse is instructing the nursing assistant to prevent pressure ulcers in a frail older patient; the nursing assistant understands the instruction when she agrees to: A) bathe and dry the skin vigorously to stimulate circulation. B) limit intake of fluid and offer frequent snacks. C) turn the patient at least every 2 hours. D) keep the head of the bed elevated 30 degrees.

C The patient should be turned at least every 2 hours as permanent damage can occur in 2 hours or less. If skin assessment reveals a stage I ulcer while on a 2-hour turning schedule, the patient must be turned more frequently. Limiting fluids will prevent healing; however, offering snacks is indicated to increase healing particularly if they are protein based, because protein plays a role in healing. Use of doughnuts, elevated heads of beds, and overstimulation of skin may all stimulate, if not actually encourage, dermal decline.

The nurse is caring for an anorexic client who is severely malnourished. A nasogastric feeding tube is inserted, and tube feedings are started. Which laboratory finding is the best indication that the client's nutritional status is improving? A) Creatinine has dropped from 1.9 to 0.5 mg/dL. B) Blood urea nitrogen (BUN) level has dropped from 15 to 11 mg/dL. C) Prealbumin level has risen from 9 to 13 mg/dL. D) Sodium has risen from 130 to 144 mg/dL.

C The prealbumin level is a good measure of nutritional status because its half-life is only 2 days, so it reflects current nutritional status. The client's prealbumin level is rising and almost normal, indicating that the client's nutritional status is improving. The other laboratory values are more reflective of fluid balance and kidney function.

The nurse is working at a first aid booth for a spring training game on a hot day. A spectator comes in, reporting that he is not feeling well. Vital signs are temp 104.1 F, pulse 132 BPM, respirs 26 breaths/min, and blood pressure 106/66 mm Hg. He trips over his feet as the nurse leads him to a cot. What is the priory action of the nurse? a. admin tylenol 650 mg orally b. encourage rest, and reassess in 15 minutes c. sponge the victim with cool water and remove his shirt d. encourage drinking of cool water or sports drink

C The spectator shows signs of heat stroke, which is a medical emergency. The spectator should be transported to the ED ASAP. The nurs should take actions to lower his body temp in teh meantime by removing his shirt and sponging his body with cool water. Lowering body temp by drinking cool fluids or taking acetaminophen is not as effective in an emergency situation. The client needs to be cooled quickly and is a priority for treatment

A postoperative client receives a Schedule II opioid analgesic for pain. Which assessment finding requires the most immediate intervention by the nurse? A.Hypoactive bowel sounds with abdominal distention B.Client reports continued pain of 8 on a 10-point scale C.Respiratory rate of 12 breaths/min, with O2 saturation of 85% D.Client reports nausea after receiving the medication

C Administration of a Schedule II opioid analgesic can result in respiratory depression (C), which requires immediate intervention by the nurse to prevent respiratory arrest. (A, B, and D) require action by the nurse but are of less priority than (C).

A client has newly diagnosed diabetes. To delay the onset of microvascular and macrovascular complications in this client, the nurse stresses that the client take which action? A) Restrict fluid intake. B) Prevent ketosis. C) Control hyperglycemia. D) Prevent hypoglycemia.

C Hyperglycemia is a critical factor in the pathogenesis of long-term diabetic complications. Maintaining tight glycemic control will help delay the onset of complications. Preventing hypoglycemia and ketosis, although important, is not as important as maintaining daily glycemic control. Restricting fluid intake is not part of the treatment plan for clients with diabetes.

When educating a client after a total laryngectomy, which instruction would be most important for the nurse to include in the discharge teaching? A.Recommend that the client carry suction equipment at all times. B.Instruct the client to have writing materials with him at all times. C.Tell the client to carry a medical alert card that explains his condition. D.Caution the client not to travel outside the United States alone.

C Neck breathers carry a medical alert card (C) that notifies health care personnel of the need to use mouth to stoma breathing in the event of a cardiac arrest in this client. Mouth to mouth resuscitation will not establish a patent airway. (A and D) are not necessary. There are many alternative means of communication for clients who have had a laryngectomy; dependence on writing messages (B) is probably the least effective.

A client in the emergency department is bleeding profusely from a gunshot wound to the abdomen. In what position should the nurse immediately place the client to promote maintenance of the client's blood pressure above a systolic pressure of 90 mm Hg? A.Place the client in a 45-degree Trendelenburg position to promote cerebral blood flow. B.Turn the client prone to place pressure on the abdominal wound to help staunch the bleeding. C.Maintain the client in a supine position to reduce diaphragmatic pressure and visualize the wound. D.Put the client on the right side to apply pressure to the liver and spleen to stop hemorrhaging.

C Placing the client in a supine position (C) reduces diaphragmatic pressure, thereby enhancing oxygenation, and allows for visualization of the abdominal wound. (A) compromises diaphragmatic expansion and inhibits pressoreceptor activity. (B) places the client at risk of evisceration of the abdominal wound and increased bleeding. (D) will not stop internal bleeding in the liver and spleen caused by the gunshot wound.

When a nurse assesses a client receiving total parenteral nutrition (TPN), which laboratory value is most important for the nurse to monitor regularly? A.Albumin B.Calcium C.Glucose D.Alkaline phosphatase

C TPN solutions contain high concentrations of glucose, so the blood glucose level is often monitored as often as q6h because of the risk for hyperglycemia (C). (A) is monitored periodically because an increase in the albumin level, a serum protein, is generally a desired effect of TPN. (B) may be added to TPN solutions, but calcium imbalances are not generally a risk during TPN administration. (D) may be decreased in the client with malnutrition who receives TPN, but abnormal values, reflecting liver or bone disorders, are not a common complication of TPN administration.

A 62-year-old woman who lives alone tripped on a rug in her home and fractured her hip. Which predisposing factor most likely contributed to the fracture in the proximal end of her femur? A.Failing eyesight resulting in an unsafe environment B.Renal osteodystrophy resulting from chronic kidney disease (CKD) C.Osteoporosis resulting from declining hormone levels D.Cerebral vessel changes causing transient ischemic attacks

C The most common cause of a fractured hip in older women is osteoporosis, resulting from reduced calcium in the bones as a result of hormonal changes in the perimenopausal years (C). (A) may or may not have contributed to the accident, but eye changes were not involved in promoting the hip fracture. (B) is not a common condition of older people but is associated with CKD. Although (D) may result in transient ischemic attacks (TIAs) or stroke, it will not result in fragility of the bones, as does osteoporosis.

The nurse includes frequent oral care in the plan of care for a client scheduled for an esophagogastrostomy for esophageal cancer. This intervention is included in the client's plan of care to address which nursing diagnosis? A.Fluid volume deficit B.Self-care deficit C.Risk for infection D.Impaired nutrition

C The primary reason for performing frequent mouth care preoperatively is to reduce the risk of postoperative infection (C) because these clients may be regurgitating retained food particles, blood, or pus from the tumor. Meticulous oral care should be provided several times a day before surgery. Although oral care will be of benefit to the client who may also be experiencing (A, B, or D), these problems are not the primary reason for the provision of frequent oral care.

The nurse is assessing a group of clients. Which clients are at greater risk for hypothermia or frostbite? (select all that apply) a. an older woman with hypertension b. a young man with a body mass index of 42 c. a young many who has just consumed six martinis d. an older man who smokes a pack of cigarettes a day e. a young woman who is anorexic f. a young woman who is diabetic

C, D, E, F clients with poor nutrition, fatigue, and multiple chronic illnesses are at greater risk for hypothermia. Clients who smoke, consume alcohol, or have impaired peripheral circulation have a higher incidence of frostbite.

What information about nutrition does the nurse teach a client with chronic obstructive pulmonary disease (COPD)? (Select all that apply.) A) "Eat dry foods rather than wet foods, which are heavier." B) "Increase carbohydrate intake for energy." C) "Have about six small meals a day." D) "Practice diaphragmatic breathing against resistance four times daily." E) "Avoid drinking fluids just before and during meals." F) "Eat high-fiber foods to promote gastric emptying." G) "Rest before meals if you have dyspnea."

C, E, G Fluids can make a client feel bloated and should be avoided with meals. Resting before the meal will help a client with dyspnea. Six small meals a day also will help to decrease bloating. Dry foods can cause coughing. Fibrous foods can produce gas, which can cause abdominal bloating and can increase shortness of breath. Diaphragmatic breathing will not necessarily help nutrition.

Which of the following would be included in the assessment of a patient with diabetes mellitus who is experiencing a hypoglycemic reaction? (Select all that apply.) A) Constricted pupils B) Flushed skin C) Tremors D) Nervousness E) Extreme thirst F) Profuse perspiration

C,D,F When hypoglycemia occurs, blood glucose levels fall, resulting in sympathetic nervous system responses such as tremors, nervousness, and profuse perspiration. Dilated pupils would also occur, not constricted pupils. Extreme thirst, flushed skin, and constricted pupils are consistent with hyperglycemia.

A home health nurse is visiting a client with a history of heart failure (HF). When interviewing the client, which question provides the most useful information for the nurse? A. "Have you been weighing yourself once a month?" B. "Have you had any headaches lately?" C. "How many pillows do you sleep on at night?" D. "How much caffeine are you drinking?"

C. "How many pillows do you sleep on at night?"

The nurse is evaluating the health teaching of a female client with condyloma acuminata. Which statement by the client indicates that teaching has been effective? A. "These warts are caused by a fungus" B. "Early treatment is very effective" C. "I need to have regular pap smears" D. "I will clean my hot tub better"

C. "I need to have regular pap smears"

Which of the following statements made by a nurse would indicate proper teaching principles regarding feeding and tracheostomies? A. "Follow each spoon of food consumed with a drink of fluid." B. "Thin your foods to a liquid consistency whenever possible." C. "Tilt your chin forward toward the chest when swallowing your food." D. "Make sure your cuff is overinflated before eating if you have swallowing problems."

C. "Tilt your chin forward toward the chest when swallowing your food."

A 58-year-old client with chronic kidney disease (CKD) is receiving aluminum hydroxide (Amphojel). He tells the nurse that since he does not have indigestion there is no need for him to take the antacid with his meals. Which response is best for the nurse to provide? A. "CKD stresses your body to over-secrete gastric juices, and antacids help neutralize them." B. "OK, I will let your healthcare provider know that you do not need the antacid." C. "Your serum phosphate levels are up, and aluminum antacids prevent absorption of phosphates in foods." D. "I will hold the antacids for now, and if you get indigestion, I will bring it back."

C. "Your serum phosphate levels are up, and aluminum antacids prevent absorption of phosphates in foods."

A female nurse who took drugs from the unit for personal use was temporarily released from duty. After completion of mandatory counseling, the nurse has asked administration to allow her to return to work. When the nurse administrator approaches the charge nurse with the impaired nurse's request, what action is best for the charge nurse to take? A.Since treatment is completed, assign the nurse to routine RN responsibilities B. Ask to meet with the impaired nurse's therapist before allowing her back on the unit C. All the impaired nurse to return to work and monitor medication administration D. Meet with the staff to assess their feelings about the impaired nurse's return to the unit

C. All the impaired nurse to return to work and monitor medication administration

A male client with bipolar disorder has difficulty concentrating and plans to attend group for the first time. He tells the nurse that he will try to stay for the music relaxation group. After 20 minutes in the group, he becomes restless and begins to leave. What should the nurse do? A. Ask the client to stay until the end B. Encourage the client to go to another group C. Allow the client to leave the group D. Offer the client an antianxiety medication

C. Allow the client to leave the group

A female nurse who took drugs from the unit for personal use was temporary released from duty. After completion of mandatory counseling, the nurse has asked administration to allow her to return to work. When the nurse administrator approaches the charge nurse with the impaired nurse's request, what action is best for the charge nurse to take? A. Since treatment is completed, assign the nurse to routine RN responsibilities B. Ask to meet with the impaired nurse's therapist before allowing her back on the unit C. Allow the impaired nurse to return to work and monitor medication administration D. Meet with staff to assess the feelings about the impaired nurse's return to work.

C. Allow the impaired nurse to return to work and monitor medication administration

About 85 victims of a train derailment are brought to the Emergency Department of a small rural hospital. An older male with extensive crush injuries to his lower extremities and pelvis has a blood pressure of 42/28, a thready pulse of 120 beats/minute, and a respiratory rate of 10 breaths/minute with periods of apnea. Using the disaster triage system, which action should the nurse take? A. Obtain the crash cart and defibrillator B. Transport to radiology department C. Assign a black triage color D. Initiate a large bore IV infusion

C. Assign a black triage color

Two days following abdominal surgery a client begins to report camping abdominal pain, and the nurse's inspection the abdomen indicates slight distention. Which action should the nurse implement first? A. Encourage the client to ambulate B. Offer ice ships or warm liquids C. Auscultate the client's abdomen D. Assess the client's temperature

C. Auscultate the client's abdomen

A male client with multiple myeloma is admitted with pneumonia and pancytopenia. The nurse reviews the complete blood cell count findings and identifies a platelet count of 20,000 cells/mm^3. Which intervention should the nurse include in the client's plan of care? A. Pace activities between planned rest periods. B. Monitor intake and output C. Avoid intramuscular injections D. Limit exposure to visitors with respiratory infections

C. Avoid intramuscular injections

The nurse is assessing the normal development of a 9-year-old male infant. Which information should the nurse obtain from the mother? A. Has the child started to walk? B. Is the baby able to lift his head when prone? C. Can the child sit alone? D. Does the baby roll from abdomen to back?

C. Can the child sit alone?

A patient expresses a strong interest in returning to their work, family, and hobbies after having a stroke. Which theory type would the nurse use to develop a plan of care for the best results of this patient's motivation style? a. field b. biological c. cognitive d. sociologic

C. Cognitive theorists believe that attention, relevance, confidence, and satisfaction (ARCS) are the conditions that, when integrated, motivate someone to learn. Field theorists place significance on how achievement, power, the need for affiliation, and avoidance motives influence individual behavior. Sociologic theories are not involved in motivation.

A 12-lead electrocardiogram (ECG) indicated a ST elevation in leads V1 to V4, for a client who reports having chest pain. The healthcare provider prescribes tissue plasminogen activator (t-PA). Prior to initiating the infusion, which intervention is most important for the nurse to implement? A. Place the ECG findings in the client's record B. Obtain a signed informed consent C. Complete pre-infusion checklist D. Insert two large bore IV sites

C. Complete pre-infusion checklist

An adult female client is admitted to the psychiatric unit because of a complex hand washing ritual she preforms daily that takes two hours or longer to complete. She worries about staying clean and refuses to sit on any of the chairs in the day area. This client's hand washing is an example of which clinical behavior? A. Obsession B. Addiction C. Compulsion D. Phobia

C. Compulsion

During a paracentesis, two liters of fluid are removed from the abdomen of a client with ascites. A drainage bag is placed, and 50 ml of clear, straw-colored fluid drains within the first hour. What action should the nurse implement? A. Palpate for abdominal distention B. Send fluid to the lab for analysis C. Continue to monitor the fluid output D. Clamp the drainage tube for 5 minutes

C. Continue to monitor the fluid output

The preeclamptic client who delivered 24 hours ago remains in the labor and delivery recovery room. She continues to receive magnesium sulfate at 3 grams per hour. Her total input is limited to 125 ml per hour, and her urinary output for the last hour was 850 ml. What intervention should the nurse implement? A. Discontinue the magnesium sulfate immediately B. Decrease the client's IV rate to 50 ml per hour C. Continue with the plan of care for this client D. Change the client's diet to NPO status

C. Continue with the plan of care for this client

During the change of shift report, the charge nurse reviews the infusions being received by clients on the oncology unit. The client receiving which infusion should be assessed first? A.Continuous IV infusion of magnesium B.One-time infusion of albumin C.Continuous epidural infusion of morphine D.Intermittent infusion of IV vancomycin

C. Continuous epidural infusion of morphine

A client exposed to tuberculosis is scheduled to begin prophylactic treatment with isoniazid. Which information is most important for the nurse to note before administering the initial dose? A. Conversion of the client's PPD test from negative to positive B. Length of time of the exposure to tuberculosis C. Current diagnosis of hepatitis B D. History of intravenous drug abuse

C. Current diagnosis of hepatitis B

While assessing a client with degenerative joint disease, the nurse observes Heberden's nodes, large prominences on the client's fingers that are reddened. The client reports that the nodes are painful. Which action should the nurse take? A. Review the client's dietary intake of high-protein foods B. Notify the healthcare provider of the finding immediately C. Discuss approaches to the chronic pain control with the client D. Assess the client's radial pulses and capillary refill time

C. Discuss approaches to the chronic pain control with the client

During assessment of a 2-year-old infant, the nurse notices a bluish-black discoloration over the lumbosacral area. Which action should the nurse take? A. Ask the mother about the discoloration B. Report possible child abuse to protective services C. Document the finding in the report D. Gently rub the area with skin cream to promote healing

C. Document the finding in the report

A male client tells the nurse that he is concerned that he may have a stomach ulcer, because he is experiencing heartburn and a dull gnawing pain that is relieved when he eats. What is the best response by the nurse? A. Advice the client the needs to seek immediate medical evaluation and treatment of these symptoms. B. Assure the client that the symptoms may on reflect reflux, since ulcer pain is not relieved with food. C. Encourage the client to obtain a complete a physical exam since the symptoms are consistent with an ulcer D. Instruct the client that these mild symptoms can generally be resolved with changes in the diet.

C. Encourage the client to obtain a complete a physical exam since the symptoms are consistent with an ulcer

The nurse initiates a one-to-one relationship with a 35-year-old depressed female client who was recently admitted to the psychiatric facility. Which nursing action is most effective in promoting the development of a therapeutic relationship? A. At your first meeting clearly define the unit's rules and policies B. Obtain client data from her family to use at the first meeting with the client C. Ensure that scheduled appointments begin according to the schedule D. Re-direct all conversations to discussions about feelings of low self-esteem

C. Ensure that scheduled appointments begin according to the schedule

A newly hired male unlicensed assistive personnel (UAP) is assigned to a home healthcare team along with two experienced UAP's. Which intervention should the home health nurse implement to ensure adequate care for all clients? A. Assign the newly hired UAP to clients who require the least complex level of care B. Ask the most experienced UAP on the team to partner with the newly hired UAP C. Evaluate the newly hired UAP's level of competency by observing him deliver care D. Review the UAP's skills checklist and experience wit the person who hired him

C. Evaluate the newly hired UAP's level of competency by observing him deliver care

Immediately after an elective cardioversion for rapid supraventricular tachycardia (SVT), a male client who was premedicated with hydromorphone (Dilaudid) and midazolam (Versed) is difficult to arouse. His vital signs are: oxygen saturation 94% while receiving oxygen at 2 L/minute per nasal cannula, heart rate 78 beats/minute, respirations 6 breaths/minute, and blood pressure 102/70. Which intervention should the nurse implement? A. Increase oxygen to 4 L/minute B. Prepare for another cardioversion C. Give IV naloxone (Narcan) D. Infuse normal saline IV bolus

C. Give IV naloxone (Narcan)

The nurse is preparing a client for discharge who recently diagnosed with Addison's disease. Which instruction is most important for the nurse to include in the client's discharge teaching plan? A. Use a walker when weakness occurs B. Avoid extreme environmental temperatures C. Increase daily intake of sodium in diet D. Take prescribed cortisone accurately

C. Increase daily intake of sodium in diet

A community health nurse is concerned about the incidence of asthma among preschool aged children in a metropolitan area. Which interventions reflects a primary prevention strategy the nurse might initiate to combat the chronic illness? A. Partner with a major pharmaceutical company to provide nebulizers at a reduced cost to inner-city children B. Refer parents of preschoolers with asthma to a support group sponsored by the American Lung Association C. Inform the city council of the need to strengthen the city's air pollution ordinances D. Offer free asthma screening to children at a health fair sponsored by a local hospital

C. Inform the city council of the need to strengthen the city's air pollution ordinances

When describing patient education approaches, the nurse educator would explain that informal teaching is an approach that a. follows formalized plans b. has standardized content c. often occurs one-to-one d. addresses group needs

C. Informal teaching is individualized one on one teaching which represents the majority of patient education done by nurses that occurs when an intervention is explained or a question is answered. Group needs are often the focus of formal patient education courses or classes. Informal teaching does not necessarily follow a specific formalized plan. It may be planned with specific content, but it is individualized responses to patient needs. Formal teaching involves the use of a curriculum/course plan with standardized content.

What is the correct location for the placement of the hand for manual chest compressions during CPR on the adult client. A. Just above the xiphoid process on the upper third of the sternum. B. Below the xiphoid process midway between the sternum and the umbilicus. C. Just about the xiphoid process on the lower third of the sternum. D. Below the xiphoid process midway between the sternum and the first rib.

C. Just about the xiphoid process on the lower third of the sternum.

Following surgical repair of the bladder, a female client is being discharged from the hospital to home with an indwelling urinary catheter. Which instruction is most important for the nurse to provide to this client? A. Avoid coiling the tubing and keep if free of kinks B. Cleanse the perineal area with soap and water twice daily C. Keep the drainage bag lower than the level of the bladder D. Drink 1,000 ml of fluids daily to irrigate catheter

C. Keep the drainage bag lower than the level of the bladder

What information should the nurse include in the teaching plan of a client diagnosed with gastroesophageal reflux disease (GERD)? A. Sleep without pillows at night to maintain neck alignment. B. Adjust food intake to three full meals per day and no snacks. C. Minimize symptoms by wearing loose, comfortable clothing D. Avoid participation in any aerobic exercise programs

C. Minimize symptoms by wearing loose, comfortable clothing

Clients with tumor lysis syndrome may experience hyperkalemia, requiring the addition of insulin to the IV solution to reduce the serum potassium level. It is most important for the nurse to monitor the client's serum potassium and blood glucose levels to ensure that they are not at dangerous levels (C). (A, B, and D) provide valuable assessment data but are of less priority than (C).

C. Monitor the client's serum potassium and blood glucose levels.

The nurse identifies which recent event as placing a client at high risk for cardiogenic shock? A. Gunshot wounds to the chest and abdomen B. Traumatic amputation of the leg at the groin C. Myocardial infraction in the right ventricle D. Multiple bee stings around the head and neck

C. Myocardial infraction in the right ventricle

While assisting a female client to the toilet, the client begins to have a seizure and the nurse eases her to the floor. The nurse calls for help and monitors the client until the seizing stops. Which intervention should the nurse implement first? A. Document details of the seizure activity B. Observe for lacerations to the tongue C. Observe for prolonged periods of apnea D. Evaluate for evidence of incontinence

C. Observe for prolonged periods of apnea

To assess for the presence of diaphragmatic breathing, what action should the nurse take? A. Attach an apnea monitor to the chest wall B. Auscultate the lung bases anteriorly C. Observe the movement of the abdomen D. Note any gaps between respirations

C. Observe the movement of the abdomen

A client arrives in the emergency department (ED) with slurred speech and right-sided weakness. Which information is most important? A. Family history of stroke B. Changes in vision C. Onset of symptoms D. Severity of headache

C. Onset of symptoms

The nurse determines that a patient is experiencing common adverse effects from the inhaled corticosteroid beclomethasone (Beclovent) after noting which of the following? A. Adrenocortical dysfunction and hyperglycemia B. Elevation of blood glucose and calcium levels C. Oropharyngeal candidiasis and hoarseness D. Hypertension and pulmonary edema

C. Oropharyngeal candidiasis and hoarseness

A 62-year-old woman who lives alone tripped on a rug in her home and fractured her hip. Which predisposing factor most likely contributed to the fracture in the proximal end of her femur?

C. Osteoporosis resulting from declining hormone levels

An older male client tells the nurse that he is losing sleep because he has to get up several times at night to go to the bathroom, that he has trouble starting his urinary stream, and that he does not feel like his bladder is ever completely empty. Which intervention should the nurse implement? A. Collect a urine specimen for culture analysis B. Review the client's fluid intake prior to bedtime C. Palpate the bladder above the symphysis pubis D. Obtain a fingerstick blood glucose level

C. Palpate the bladder above the symphysis pubis

When reviewing the results of a 83-year-old patient's blood tests, which of the following findings would be of most concern to the nurse? A. Platelets of 150,000/µl B. Serum iron of 50 mcg/dl C. Partial thromboplastin time (PTT) of 60 seconds D. Erythrocyte sedimentation rate (ESR) of 35 mm in 1 hour

C. Partial thromboplastin time (PTT) of 60 seconds

Which of the following conditions or factors in a 64-year-old patient diagnosed with head and neck cancer most likely contributed to this health problem? A. Patient's hobby is oil painting. B. Patient's father also had head and neck cancer. C. Patient uses chewing tobacco and drinks beer daily. D. Patient quit school at age 16 and has worked in a butcher shop for more than 40 years.

C. Patient uses chewing tobacco and drinks beer daily.

39. Select all that apply. Which of the following are clinical manifestations of tension pneumothorax? A. Midline trachea B. Severe hypertension C. Progressive cyanosis D. A loud bruit on affected side E. Asymmetrical chest wall movement F. Subcutaneous emphysema in the neck

C. Progressive cyanosis E. Asymmetrical chest wall movement F. Subcutaneous emphysema in the neck

A man who has a known problem with alcohol is accused of stealing from his employer. When he returns home that evening, he accuses his son of stealing from school, and physically abuses the child for what the father describes as the child's dishonest behavior. Which two defense mechanisms are being used by the father? A. Sublimation and displacement B. Denial and sublimation C. Projection and displacement D. Projection and denial

C. Projection and displacement

An older client arrives at the outpatient eye surgery clinic for a right cataract extraction and lens implant. During the immediate postoperative period, which intervention should the nurse implement? A. Teach a family member to administer eye drops B. Encourage deep breathing and coughing exercises C. Provide an eye shield to be worn while sleeping D. Obtain vital signs every 2 hours during hospitalization

C. Provide an eye shield to be worn while sleeping

Which of the following conditions is manifested by unexplained shortness of breath and a high mortality rate? A. Bleeding ulcer B. Transient ischemia C. Pulmonary embolism D. MI

C. Pulmonary embolism

The nurse is preparing a 50 ml dose of 50% Dextrose IV for a client with insulin shock. How should the nurse administer the medication? A. Dilute Dextrose in one liter of 0.9% Normal Saline solution B. Mix Dextrose in a 50 ml piggyback for a total volume of 100 ml. C. Push the undiluted Dextrose slowly through the currently infusing IV D. Ask the pharmacist to add the Dextrose to a TPN solution

C. Push the undiluted Dextrose slowly through the currently infusing IV

When checking a third-grader's height and weight, the school notes that these measurements have not changed in the last year. The child is currently taking daily vitamins, albuterol, and methylphenidate for attention deficit hyperactivity disorder (ADHD). Which intervention should the nurse implement? A. Report the finding to the parents B. Document findings in the child's school life C. Refer child to the family healthcare provider D. Encourage child to get more sleep

C. Refer child to the family healthcare provider

A female client, newly diagnosed with breast cancer, is scheduled for a mastectomy next week. During the preoperative assessment, she complains that her husband has become withdrawn and complains about her irritability, and frequently crying. How should the nurse respond? A. Explain that a positive attitude helps reduce preoperative complications B. Encourage the spouse to be more supportive at this difficult time C. Refer the couple to a counselor to help them with coping strategies D. Inquire if the couple has met with a minister to discuss their feelings

C. Refer the couple to a counselor to help them with coping strategies

What action should the nurse implement to reduce a client's risk for nosocomial infection? A. Apply a face shield or googles before irrigating an infected wound B. Wear sterile gloves to administer an intravenous medication through a saline loc C. Replace continuous tube feeding bag and tubing at least daily D. Obtain a prescription to irrigate a urinary catheter with sterile saline daily

C. Replace continuous tube feeding bag and tubing at least daily

The nurse administers the opioid antagonist naloxone HCL (Narcan) to a young adult client who overdosed on hydromorphone (Dilaudid). Which assessment data indicates that the naloxone is effective? A. Babinski reflex changes from negative to positive B. Pupil size increases from 4 mm to 6 mm C. Respiration increases from 8/minute to 12/minute D. Blood pressure decrease from 140/94 to 120/78

C. Respiration increases from 8/minute to 12/minute

An adult female client is diagnosed with restless leg syndrome and is referred to the sleep clinic. The healthcare provider prescribes ferrous sulfate (Feosol) 325 mg PO daily. Which laboratory values should the nurse monitor? A. Serum electrolytes B. Neutrophils and eosinophils C. Serum iron and ferritin D. Platelet count and hematocrit

C. Serum iron and ferritin

A client with acute renal injury (AKI) who weighs 50 kg and has potassium level of 6.7 mEq/L (6.7 mmol/l) is admitted to the hospital. Which prescribed medication should the nurse administer first A. Sevelamer (RenaGel) one tablet PO. B. Epoetin alfa, recombinant (Epogen) 2, 500 units SUBQ C. Sodium polystyrene (Kayexalate) 15 grams PO D. Calcium acetate (Phos-Lo) one tablet PO

C. Sodium polystyrene (Kayexalate) 15 grams PO

A male client is recovering from an episode of urinary tract calculi. During discharge teaching, the client asks about the dietary restriction he should follow. In discussing fluid intake, the nurse should include which type of fluid limitation A. Low-sodium soups. B. Over all fluid intake C. Tea and hot chocolate D. Citrus fruit juices

C. Tea and hot chocolate

When planning care for a client with rheumatoid arthritis, which intervention is most important for the nurse to include in the plan of care? A. Provide assistive devices to empower client independence B. Implement measures to manage chronic pain C. Teach coping skills for living with a chronic illness D. Schedule rest periods between activates to minimize fatigue.

C. Teach coping skills for living with a chronic illness

A client is admitted to a medical unit with a diagnosis of gastritis and chronic heavy alcohol abuse. What should the nurse administer to prevent the development of Wernicke's syndrome? A. Lorazepam (Ativan) B. Famotidine (Pepcid) C. Thiamine (Vitamin B1) D. Atenolol (Tenormin)

C. Thiamine (Vitamin B1)

A client who is at 36 weeks gestation is admitted with severe preeclampsia. After a 6 gram loading dose of magnesium sulfate is administered, an intravenous infusion of magnesium sulfate at a rate of 2 grams/hour is initiated. Which assessment finding warrants immediate intervention by the nurse? A. Blood pressure 162/94 B. Complaint of headache C. Urine output 20 ml/hour D. Nausea and vomiting

C. Urine output 20 ml/hour

A young female client with seven children is having frequent morning headaches, dizziness, and blurred vision. Her blood pressure (BP) is 168/104 mmHg. The client reports that her husband recently lost his job and she is not sleeping well. After administering a STAT dose of an antihypertensive IV medication, which intervention is most important for the nurse to implement? A. Measure urine output hourly to assess for rental perfusion B. Request a prescription for pain medication C. Use an automated BP machine to monitor for hypotension D. Provide a quiet environment with low lighting

C. Use an automated BP machine to monitor for hypotension

The nurse is preparing a client for a scheduled cesarean section. In which order should the nurse perform these actions? A. Prepping the site with iodophor (Betadine) B. Inserting an indwelling catheter C. Verifying the consent form is signed D. Performing a time-out procedure

C. Verifying the consent form is signed B. Inserting an indwelling catheter A. Prepping the site with iodophor (Betadine) D. Performing a time-out procedure

A male client who had abdominal surgery 5 days ago, and hospitalized because of a surgical wound infection, tells the nurse that he feels like his insides just spilled out when he coughed. What action should the nurse take first? A. Notify the healthcare provider B. Assure the client that such feelings occur with wound infections C. Visualize the abdominal incision D. Obtain sterile towels soaked in saline

C. Visualize the abdominal incision

A male client with bilateral carpal tunnel syndrome reports to the nurse that the pain and tingling he is experiencing worsens at night. What client teaching should the nurse provide? A. Elevate the hands on two pillows at night B. Notify the healthcare provider as soon as possible C. Wear braces as both writs during the night D. Apply cold compresses for 30 min before bedtime

C. Wear braces as both writs during the night

A client in the intensive care unit is being mechanically ventilated , has an indwelling urinary catheter in place, and is exhibiting signs of restlessness. Which action should the nurse take first? A. administer PRN dose of lorazepam B. auscultate bilateral breath sounds C. check urinary catheter for obstruction D. review the heart rhythm on cardiac monitor

C. check urinary catheter for obstruction

An excess of carbon dioxide in the blood causes an increased respiratory rate and volume because CO2 A. displaces oxygen on hemoglobin, leading to a decreased PaO2. B. causes an increase in the amount of hydrogen ions available in the body. C. combines with water to form carbonic acid, lowering the pH of cerebrospinal fluid. D. directly stimulates chemoreceptors in the medulla to increase respiratory rate and volume.

C. combines with water to form carbonic acid, lowering the pH of cerebrospinal fluid.

Following a supraglottic laryngectomy, the patient is taught how to use the supraglottic swallow to minimize the risk of aspiration. In teaching the patient about this technique, the nurse instructs the patient to A. perform Valsalva maneuver immediately after swallowing. B. breathe between each Valsalva maneuver and cough sequence. C. cough after swallowing to remove food from the top of the vocal cords. D. practice swallowing thin, watery fluids before attempting to swallow solid foods.

C. cough after swallowing to remove food from the top of the vocal cords.

If a patient has pernicious anemia, the nurse should provide information regarding A. frequent bouts of dyspnea. B. risks relative to dehydration. C. deficiency of intrinsic factor. D. lack of any effective treatment for this condition.

C. deficiency of intrinsic factor.

The nurse presenting information about fetal development to a group of parents who are attending a Lamaze birthing class. When discussing cephalocaudal fetal development, which information should the nurse provide? A. A set order in fetal development is expected B. growth normally occurs within one organ at a time C. development progresses from head to rump D. organ formation is directed from brain development

C. development progresses from head to rump

A nurse establishes the presence of a tension pneumothorax when assessment findings reveal a(n) A. absence of lung sounds on the affected side. B. inability to auscultate tracheal breath sounds. C. deviation of the trachea toward the side opposite the pneumothorax. D. shift of the point of maximal impulse (PMI) to the left, with bounding pulses.

C. deviation of the trachea toward the side opposite the pneumothorax.

The chronic inflammation of the bronchi characteristic of chronic obstructive pulmonary disease (COPD) results in A. collapse of small bronchioles on expiration. B. permanent, abnormal dilation of the bronchi. C. hyperplasia of mucus-secreting cells and bronchial edema. D. destruction of the elastic and muscular structures of the bronchial wall.

C. hyperplasia of mucus-secreting cells and bronchial edema.

A person complains of fatigue and malaise and has a slight temperature elevation for 2 days before symptoms of influenza (fever, chest congestion, and productive cough) become noticeable. During the time immediately before the illness is diagnosed, the patient A. could avoid contracting the disease if treatment is begun with antibiotics. B. is unable to spread the disease because it is still in the incubation period. C. is in the prodromal stage and is highly contagious and able to spread the disease. D. has a nosocomial infection, which affects approximately two million individuals a year.

C. is in the prodromal stage and is highly contagious and able to spread the disease.

A client with a new diagnosis of Raynaud's disease lives alone. Which instruction should the nurse include in the client's discharge teaching plan? A. have a caregiver for 8 hours daily. B. develop a walking exercise routine C. keep room temperature at 80F D. Wear TED stockings at night

C. keep room temperature at 80F

During care of a patient with multiple myeloma, an important nursing intervention is A. limiting activity to prevent pathologic fractures. B. assessing for changes in size and characteristics of lymph nodes. C. maintaining a fluid intake of 3 to 4 L/day to dilute calcium load. D. administering narcotic analgesics continuously to control bone pain.

C. maintaining a fluid intake of 3 to 4 L/day to dilute calcium load.

If a patient with blood type O Rh- is given AB Rh- blood, the nurse would expect A. the patient's Rh factor to react with the RBCs of the donor blood. B. no adverse reaction because the patient has no antibodies against the donor blood. C. the anti-A and anti-B antibodies in the patient's blood to hemolyze the donor blood. D. the anti-A and anti-B antibodies in the donor blood to hemolyze the patient's blood.

C. the anti-A and anti-B antibodies in the patient's blood to hemolyze the donor blood.

The nurse notes that the only ECG for a 55-year-old male client scheduled for surgery in 2 hours is dated 2 years ago. The client reports that he has a history of "heart trouble," but has no problems at present. Hospital protocol requires that those over 50 years of age have a recent ECG prior to surgery. What nursing action would be best for the nurse to implement?

Call for an ECG to be performed immediately.

The nurse is assessing a client who had a stroke in the right cerebral hemisphere. Which neurologic deficit does the nurse assess for in this client? A) Agraphia B) Aphasia C) Impaired olfaction D) Impaired proprioception

D A stroke to the right cerebral hemisphere causes impaired visual and spatial awareness. The client may present with impaired proprioception and may be disoriented as to time and place. The right cerebral hemisphere does not control speech, smell, or the client's ability to write.

A client with heart failure is prescribed enalapril (Vasotec). What is the nurse's priority teaching for this client? A) "Avoid using aspirin-containing products." B) "Take your medication with food." C) "Check your pulse daily." D) "Avoid using salt substitutes."

D Angiotensin-converting enzyme (ACE) inhibitors inhibit the excretion of potassium. Hyperkalemia can be a life-threatening side effect, and clients should be taught to limit potassium intake. Salt substitutes are composed of potassium chloride.

A nurse is about to administer the first dose of captopril (Capoten) to a client with hypertension. Which is the priority nursing intervention? A) Place the client in Trendelenburg position to facilitate blood flow to the heart. B) Take the client's apical pulse for 1 full minute before drug administration. C) Instruct the client to drink 3 L of fluid daily when taking this medication. D) Educate the client to sit on the side of the bed for a few minutes before rising.

D Angiotensin-converting enzyme (ACE) inhibitors such as captopril can cause severe hypotension with initial use. The client should be instructed to rise slowly and sit on the side of the bed for a few minutes to prevent hypotension-induced falls. No indication is known for assessment of the apical pulse for 1 full minute before taking captopril. Placing the client in a Trendelenburg position is not indicated. In case of a precipitous drop in blood pressure, a modified Trendelenburg position may be used. Adequate fluid intake is necessary but is not the priority in this situation.

The emphasis on understanding cultural influence on health care is important because of: A) disability entitlements. B) HIPAA requirements. C) litigious society. D) increasing global diversity.

D Culture is an essential aspect of health care because of increasing diversity. Disability entitlements refer to defined benefits for eligible mental or physically disabled beneficiaries in relation to housing, employment, and health care. HIPAA requirements refers to the HIPAA Privacy Rule, which protects the privacy of individually identifiable health information; the HIPAA Security Rule, which sets national standards for the security of electronic protected health information; and the confidentiality provisions of the Patient Safety Rule, which protect identifiable information being used to analyze patient safety events and improve patient safety. Litigious society refers to excessively ready to go to law or initiate a lawsuit.

Understanding classifications of pain helps nurses develop a plan of care. A 62-year-old male has fallen while trimming tree branches sustaining tissue injury. He describes his condition as an aching, throbbing back. This is characteristic of: A) mixed pain syndrome. B) chronic pain. C) neuropathic pain. D) nociceptive pain.

D Nociceptive pain refers to the normal functioning of physiological systems that leads to the perception of noxious stimuli (tissue injury) as being painful. Patients describe this type of pain as aching, cramping, or throbbing. Neuropathic pain is pathologic and results from abnormal processing of sensory input by the nervous system as a result of damage to the brain, spinal cord, or peripheral nerves. Patients describe this type of pain as burning, sharp, and shooting. Chronic pain is constant and unrelenting such as pain associated with cancer. Mixed pain syndrome is not easily recognized, is unique with multiple underlying and poorly understood mechanisms like fibromyalgia and low back pain.

While planning care for a patient experiencing fatigue due to chemotherapy, which of the following is the most appropriate nursing intervention? A) Completing all nursing care in the evening when the patient is more rested B) Completing all nursing care in the morning so the patient can rest the remainder of the day C) Limiting visitors, thus promoting the maximal amount of hours for sleep D) Prioritization and administration of nursing care throughout the day

D Pacing activities throughout the day conserves energy, and nursing care should be paced as well. Fatigue is a common side effect of cancer and treatment; and while adequate sleep is important, an increase in the number of hours slept will not resolve the fatigue. Restriction of visitors does not promote healthy coping and can result in feelings of isolation.

In order to provide the best intervention for a patient, the nurse is often responsible for obtaining a sample of exudate for culture. This test will identify: A) what cells are being utilized by the body to attack an infection. B) whether a patient has an infection. C) where an infection is located. D) what specific type of pathogen is causing an infection.

D People can transmit pathogens even if they don't currently feel ill. Some carriers never experience the full symptoms of a pathogen. A CBC will identify that the patient has an infection. Inspection and radiography will help identify where an infection is located. The CBC with differential will identify the white blood cells being used by the body to fight an infection. The culture will grow the microorganisms in the sample for identification of the specific type of pathogen.

The client is receiving an IV of 60 mEq of potassium chloride ina 1000 mL solution of dextrose 5% in 0.45% saline. The client states that the area around the IV site burns. What intervention does the nurse perform first? a. assess for a blood return b. notify the physician c. document the finding d. stop the IV infusion

D Potassium is a severe tissue irritant. The safest action is to discontinue the solution that contains the potassium and discontinue the IV altogether, in which case the client would need another site started. Assessing for a blood return may or may not be successful. The solution could be diluted (less potassium) and the rate could be slowed once it is determined that the needle is in the vein.

The nurse observes skin tenting on the back of the older adult client's hand. Which action by the nurse is most appropriate? A) Examine dependent body areas. B) Notify the physician. C) Document the finding and continue to monitor. D) Assess turgor on the client's forehead.

D Skin turgor cannot be accurately assessed on an older adult client's hands because of age-related loss of tissue elasticity in this area. Areas that more accurately show skin turgor status on an older client include the skin of the forehead, chest, and abdomen. These should also be assessed, rather than merely examining dependent body areas. Further assessment is needed rather than only documenting, monitoring, and notifying the physician.

An older adult client is in physical restraints. Which intervention by the nurse is the priority? A) Assess the client hourly while keeping the restraints in place. B) Assess the client once each shift, releasing the restraints for feeding. C) Assess the client twice each shift while keeping the restraints in place. D) Assess the client every 30 to 60 minutes, releasing restraints every 2 hours.

D The application of restraints can have serious consequences. Thus, the nurse should check the client every 30 to 60 minutes, releasing the restraints every 2 hours for positioning and toileting. The other answers would not be appropriate because the client would not be assessed frequently enough, and circulation to the limbs could be compromised. Assessing every hour and releasing the restraints every 2 hours is in compliance with federal policy for monitoring clients in restraints.

A nurse is explaining to a student nurse about perfusion. The nurse knows the student understands the concept of perfusion when the student states, "Perfusion A) is a normal function of the body, and I don't have to be concerned about it." B) varies as a person ages, so I would expect changes in the body." C) is monitored by the physician, and I just follow orders." D) is monitored by vital signs and capillary refill."

D The best method to monitor perfusion is to monitor vital signs and capillary refill. This allows the nurse to know if perfusion is adequate to maintain vital organs. The nurse does have to be concerned about perfusion. Perfusion is not only monitored by the physician but the nurse too. Perfusion does not always change as the person ages.

The nurse is assessing a client with lung disease. Which symptom does the nurse intervene for first? A) The client's anterior-posterior chest diameter is 2:2. B) Clubbing of the finger tips is noted. C) The client is pale. D) The client has bilateral dependent leg edema.

D The client with bilateral dependent edema may be developing right-sided heart failure in response to respiratory disease. This symptom should be investigated right away and reported to the health care provider. Further assessment is needed. The client with chronic lung disease may develop increased anterior-posterior diameter and clubbing as responses to chronic hypoxia. These symptoms do not require immediate intervention. The client is often pale or has a dusky appearance; this also would not warrant immediate intervention.

Interrelated concepts to the professional nursing role a nurse manager would consider when addressing concerns about the quality of patient education include: A) adherence. B) developmental level. C) motivation. D) technology.

D The interrelated concepts to the professional role of a nurse include health promotion, leadership, technology/informatics, quality, collaboration, and communication. Adherence, culture, developmental level, family dynamics, and motivation are considered interrelated concepts to patient attributes and preference.

The nurse is preparing to administer tube feedings through a client's new Salem sump nasogastric tube. The nurse is unable to withdraw any fluid from the tube before starting the feeding. Which is the priority action of the nurse? A) Start the tube feeding as ordered and check the residual in 30 minutes. B) Inject air into the nasogastric tube while auscultating the client's epigastric area. C) Lower the head of the client's bed and attempt to aspirate fluid again. D) Obtain orders for a chest x-ray to confirm placement before starting the feeding.

D The nurse must verify tube placement before beginning any tube feeding or administering any medications through a tube. The most accurate way to determine placement is via chest x-ray. The nurse could cause the client to aspirate if she or he started the feeding then checked later for placement. Insufflation does not provide accurate results and should not be used to verify tube placement. The nurse must keep the client's head elevated at least 30 degrees.

The nurse is caring for a patient who is being discharged home after a splenectomy. What information on immune function needs to be included in this patient's discharge planning? A) Limiting contact with the general population B) The importance of wearing a face mask in public C) The mechanisms of the inflammatory response D) Basic infection control techniques

D The spleen is one of the major organs of the immune system. Without the spleen, the patient is at higher risk for infection; so, the nurse must be sure that the patient understands basic principles of infection control. The patient with a splenectomy does not need to understand the mechanisms of inflammatory response. The patient with a splenectomy does not need to wear a face mask in public as long as the patient understands and maintains the basic principles of infection control. The patient who has had a splenectomy does not need to limit contact with the general population as long as the patient understands and maintains the basic principles of infection control.

The nurse assesses a client who has been prescribed furosemide (Lasix) for cardiac disease. Which electrocardiographic change would be a concern for a client taking a diuretic? A.Tall, spiked T waves B.A prolonged QT interval C.A widening QRS complex D.Presence of a U wave

D A U wave (D) is a positive deflection following the T wave and is often present with hypokalemia (low potassium level). (A, B, and C) are all signs of hyperkalemia.

A female client who received a nephrotoxic drug is admitted with acute renal failure and asks the nurse if she will need dialysis for the rest of her life. Which pathophysiologic consequence should the nurse explain that supports the need for temporary dialysis until acute tubular necrosis subsides? A.Azotemia B.Oliguria C.Hyperkalemia D.Nephron obstruction

D CKD is characterized by progressive and irreversible destruction of nephrons, frequently caused by hypertension and diabetes mellitus. Nephrotoxins cause acute tubular necrosis, a reversible acute renal failure, which creates renal tubular obstruction from endothelial cells that are sloughed or become edematous. The obstruction of urine flow will resolve (D) with the return of an adequate glomerular filtration rate and, when it does, dialysis will no longer be needed. (A, B, and C) are manifestations seen in the acute and chronic forms of kidney disease.

A client with cirrhosis develops increasing pedal edema and ascites. Which dietary modification is most important for the nurse to teach this client? A.Avoid high-carbohydrate foods. B.Decrease intake of fat-soluble vitamins. C.Decrease caloric intake. D.Restrict salt and fluid intake.

D Salt and fluid restrictions are the first dietary modifications for a client who is retaining fluid as manifested by edema and ascites (D). (A, B, and C) will not affect fluid retention.

The nurse on a medical surgical unit is receiving a client from the postanesthesia care unit (PACU) with a Penrose drain. Before choosing a room for this client, which information is most important for the nurse to obtain? A.If suctioning will be needed for drainage of the wound B.If the family would prefer a private or semiprivate room C.If the client also has a Hemovac in place D.If the client's wound is infected

D The fact that the client has a Penrose drain should alert the nurse to the possibility that the surgical wound is infected (D). Penrose drains provide a sinus tract or opening and are often used to provide drainage of an abscess. To avoid contamination of another postoperative client, it is most important to place any client with an infected wound in a private room. A Penrose drain does not require (A). Although (B) is helpful information, it does not have the priority of (D). A Hemovac (C) is used to drain fluid from a dead space and is not a determinant for the room assignment.

The nurse is giving preoperative instructions to a 14-year-old client scheduled for surgery to correct a spinal curvature. Which statement by the client best demonstrates that learning has taken place? A."I will read all the teaching booklets you gave me before surgery." B."I have had surgery before, so I know what to expect afterward." C."All the things people have told me will help me take care of my back." D."Let me show you the method of turning I will use after surgery."

D The outcome of learning is best demonstrated when the client not only verbalizes an understanding but can also provide a return demonstration (D). A 14-year-old client may or may not follow through with (A), and there is no measurement of learning. (B) may help the client understand the surgical process, but the type of surgery may have been very different, with differing postoperative care. In (C), the client may be saying what the nurse wants to hear without expressing any real understanding of what to do after surgery.

Which nursing action is necessary for the client with a flail chest? A.Withhold prescribed analgesic medications. B.Percuss the fractured rib area with light taps. C.Avoid implementing pulmonary suctioning. D.Encourage coughing and deep breathing.

D Treatment of flail chest is focused on preventing atelectasis and related complications of compromised ventilation by encouraging coughing and deep breathing (D). This condition is typically diagnosed in clients with three or more rib fractures, resulting in paradoxic movement of a segment of the chest wall. (C) should not be avoided because suctioning is necessary to maintain pulmonary toilet in clients who require mechanical ventilation. (A) should not be withheld. (B) should not be applied because the fractures are clearly visible on the chest radiograph.

The nurse is teaching a patient who is to undergo bone marrow aspiration. Which of the following statements made by the nurse would indicate correct instruction regarding the site for the aspiration procedure? A. "The health care provider will perform the aspiration by needle to the femur." B. "The health care provider will perform the aspiration by needle to the scapula." C. "The health care provider will perform the aspiration by needle to the antecubital fossa." D. "The health care provider will perform the aspiration by needle to the posterior iliac crest."

D. "The health care provider will perform the aspiration by needle to the posterior iliac crest."

An adult client is admitted with flank pain and is diagnosed with acute pyelonephritis. What is the priority nursing action? A. Auscultate for presence of bowl sounds. B. Monitor hemoglobin and hematocrit C. Encourage turning and deep breathing D. Administer IV antibiotics as prescribed.

D. Administer IV antibiotics as prescribed

A male client with muscular dystrophy fell in his home and is admitted with a right hip fracture. His right foot is cool, with palpable pedal pulses. Lungs are coarse with diminished bibasilar breath sounds. Vital signs are temperature 101F, heart rate 128 beats/minute, respirations 28 breaths/minute, and blood pressure 122/82. Which intervention is most important for the nurse to implement first? A. Obtain oxygen saturation level B. Encourage incentive spirometry C. Assess lower extremity circulation D. Administer PRN oral antipyretic

D. Administer PRN oral antipyretic

A hospitalized client with chemotherapy-induced stomatitis complains of mouth pain. What is the best initial nursing action? A. Encourage frequent mouth care B. Cleanse the tongue and mouth with glycerin swabs C. Obtain a soft diet for the client D. Administer a topical analgesic per PRN protocol.

D. Administer a topical analgesic per PRN protocol.

Assessment of the fetal heart rate is an important finding when caring for a laboring client. Deceleration in fetal heart rate that are of most concern occur at what time during the contraction cycle? A. Before a contraction B. During a contraction C, Between contractions D. After a contraction

D. After a contraction

Which client should the charge nurse on the oncology unit assign to an RN, rather than a practical nurse (PN) A. A young adult client who is experiencing fatigue while undergoing a series of external beam radiation treatments for stage 1 cancer. B. A middle-aged male client who has just undergone an excision biopsy and has been told that his tumor appears to be benign C. An adult client in remission after a series of chemotherapy treatments who is receiving intramuscular iron injections for anemia D. An elderly female client with cancer and her children who are trying to decide whether to change to palliative care measures or continue disease control

D. An elderly female client with cancer and her children who are trying to decide whether to change to palliative care measures or continue disease control

The nurse assesses the perineum of a client who is complaining of perineal pain 6 hours after a normal delivery, and finds small perineal (vulvar) hematomas. Based on this assessment finding, which treatment should the nurse implement? A. Cleanse the area with warm water B. Prepare for surgical excision C. Spray topical analgesic to the perineum D. Apply ice packs on the perineum

D. Apply ice packs on the perineum

After administering a proton pump inhibitor (PPI), which action should the nurse take to evaluate the effectiveness of the medication? A. Auscultate for bowel sounds in all quadrants B. Monitor the client's serum electrolyte levels C. Measure the client's fluid intake and output D. Ask the client about gastrointestinal pain

D. Ask the client about gastrointestinal pain

A relative comes into the emergency department asking for information about a female adult client who was admitted in stable condition following a motor vehicle collision. What action should the triage nurse take? A. Determine what is happening with the client, then provide the relative with her current status B. Inform the relative that legally no information can be provided to him or her C. Ask the relative to wait in the waiting area until the healthcare provider can see him or her D. Ask the client if she would like to talk with the relative, then bring the relative to the bedside

D. Ask the client if she would like to talk with the relative, then bring the relative to the bedside

The nurse plans to administer 5,000 units of heparin, an anticoagulant. Which procedure should the nurse implement when administering this drug? A. Massage injection site after administration to ensure that the solution is dissolved. B. Prior to injecting the solution, check for bleeding by aspirating the plunger C. Administer IM injections into the fatty portion of the upper arm. D. Assess all needle insertion sites daily for hematoma and signs of inflammation

D. Assess all needle insertion sites daily for hematoma and signs of inflammation

The nurse plans to administer 5,00o units of heparin, an anticoagulant. Which procedure should the nurse implement when administering this drug? A. Massage injection site after administration to ensure that the solution is dissolved B. Prior to injecting the solution, check for bleeding by aspirating the plunger C. Administer IM injections into the fatty portion of the upper arm D. Assess all needle insertion sites daily for hematoma and signs of inflammation

D. Assess all needle insertion sites daily for hematoma and signs of inflammation

An older adult woman with a long history of chronic obstructive pulmonary disease (COPD) is admitted with progressive shortness of breath and a persistent cough. She is anxious and is complaining of a dry mouth. Which intervention should the nurse implement? A. Administer a prescribed sedative B. Encourage client to drink water C. Apply a high-flow venturi mask D. Assist her to an upright position

D. Assist her to an upright position

A 3-year-old boy is brought to the emergency department after the mother found the child in the back yard holding a piece of a toy in his hand and in respiratory distress. The child is dusky with a loud, inspiratory stridor and weak attempts to cough. Which actions should the nurse implement? A. Obtain a pulse oximetry reading and arterial blood gases B. Request a start chest x-ray and prepare medications for an asthmatic episode C. Determine if the child ingested a toxic substance and if vomiting occurred D. Auscultate all pulmonary lung fields and attempt a Heimlich maneuver

D. Auscultate all pulmonary lung fields and attempt a Heimlich maneuver

A client who has had an open cholecystectomy two weeks ago comes to the emergency department with complaints of nausea, abdominal distention, and pain. Which assessment should the nurse implement? A. Obtain a hemoccult of the client's stool B. Palpate the liver and spleen C. Perform a digital rectal exam D. Auscultate all quadrants of the abdomen

D. Auscultate all quadrants of the abdomen

Two days after a nephrectomy, the client reports abdominal pressure and nausea, which assessment should the nurse implement? A. Palpate the abdomen B. Measure hourly urine output C. Ambulate client in hallway D. Auscultate bowels sounds.

D. Auscultate bowels sounds.

After routine physical examination, the healthcare provider admits a woman with a history of Systemic Lupus Erythematosis (SLE) to the hospital because she has 3+ pitting ankle edema and blood in her urine. Which assessment finding warrants immediate intervention by the nurse? A. Dark, rust-colored urine B. Urine output 300 ml/hour C. Joint and muscle aches D. Blood pressure 170/91

D. Blood pressure 170/91

The home health nurse visits a client with heart failure (HF). Assessment findings include: temperature 97.6F, pulse 116 beats/minute, respiratory rate 36 breaths/minute, blood pressure 140/70, pulse oximeter 86% on 2 L/min of oxygen, and crackles are heard throughout the lung fields. Which intervention has the highest priority? A. Start an IV of normal saline (NS) at 100 ml/hr B. Assess for edema and weigh client C. Obtain a prescription for an increased oxygen rate D. Call 911 and prepare the client for transport

D. Call 911 and prepare the client for transport

The nurse is evaluating a male client understanding of diet teaching about the DASH (Dietary Approaches to Stop Hypertension) eating plan. Which behavior indicates that the client is adhering to the eating plan? A. Uses only lactose-free dairy products. B. Enjoys fat free yogurt as an occasional snack food C. No longer includes grains in his daily diet D. Carefully cleans and peels all fresh fruit and vegetables

D. Carefully cleans and peels all fresh fruit and vegetables

While planning care for a client with carpal tunnel syndrome, the nurse identifies a collaborative problem of pain. what is the etiology of this problem? A. Irritation of nerve endings B. Diminished blood flow C. Ischemic tissue changes D. Compression of a nerve

D. Compression of a nerve

At bedtime, an unlicensed assistive personnel (UAP) is positioning a client with obstructive sleep apnea syndrome (OSAS). The UAP elevates the head of the bed and encourages the client to turn on side. In supervising the UAP, what action should the nurse take? A. After leaving the room, discuss correct positioning with the UAP B. Reposition the client in a supine position with the feet elevated pillows C. Remind the UAP to pad the side rails to reduce risk for injury D. Confirm that the UAP has placed the call bell within reach of the client.

D. Confirm that the UAP has placed the call bell within reach of the client.

An adult woman with primary Raynaud phenomenon develops pallor and then cyanosis of her fingers. After warming her hands, the fingers turn red and the client reports a burning sensation. What action should the nurse take? A. Apply a cool compress to the affected fingers for 20 minutes B. Secure a pulse oximeter to monitor the client's oxygen saturation C. Report the finding to the healthcare provider as soon as possible D. Continue to monitor the fingers until color returns to normal

D. Continue to monitor the fingers until color returns to normal

Dinoprostone (Prostin E-2) is prescribed for primigravida who had a missed spontaneous abortion. An increase in which finding should the nurse expect? A. Maternal temperature B. Rh antibody production C. Hemoglobin (Hgb) levels D. Contractions of the uterus

D. Contractions of the uterus

A female client with severe renal impairment is receiving enoxaparin (Lovenox) 30 mg SUBQ BID. Which laboratory value due to enoxaparin should the nurse report to the healthcare provider? A. Calcium 9 mg/dl (9 mmol/L SI) B. Hemoglobin 12 grams/dl (120 mmol/L SI) C. Partial thromboplastin time (PTT) 30 seconds D. Creatinine clearance 25 ml/minute

D. Creatinine clearance 25 ml/minute

The nurse is assessing the emotional status of a client with Parkinson's disease. Which client finding is most helpful in planning goals to meet the client's emotional needs? A. Stares straight ahead without blinking B. Face does not convey any emotion C. Uses a monotone when speaking D. Cries frequently during the interview

D. Cries frequently during the interview

The healthcare provider prescribes Isosorbibide (Isordil) 40 mg every 8 hours for a male client with acute angina pectoris. Which finding should the nurse report to the healthcare provider prior to administering Isordil? A. Serum cholesterol of 200 mg/dl B. Hemoglobin of 14.8 grams/dl C. Takes enteric coated aspirin daily D. Currently takes sildenafil (Viagra)

D. Currently takes sildenafil (Viagra)

Ten hours following thrombolysis for an ST elevation myocardial infarction (STEMI), a client is receiving a lidocaine infusion for isolated runs of ventricular tachycardia (VT). Which finding should the nurse document in the electronic medical record as a therapeutic response to the lidocaine infusion? A. Stabilization of blood pressure ranges B. Cessation of chest pain C. Reduce heart rate D. Decreased frequency of episodes of VT

D. Decreased frequency of episodes of VT

A terminally ill client on a palliative care unit has an advanced directive stipulating comfort measure only. The client has not taken oral fluids in the last 36 hours and is not receiving intravenous fluids. The clients blood pressure is 64/38 and urinary output is 50 ml for the last 12 hours. What is the priority nursing intervention? A. Assess for the presence of dependent edema B. Prepare to initiate intravenous fluids C. Gently massage the client's bladder D. Determine the client's level of discomfort

D. Determine the client's level of discomfort

A client admitted to a surgical unit is being evaluated for an intestinal obstruction. The healthcare provider prescribes a nasogastric tube (NGT) to be inserted and placed to intermittent low wall suction. Which intervention should the nurse implement to facilitate proper tube placement? A. Soak nasogastric tube in warm water B. Insert tube with client's head tilted back C. Apply suction while inserting tube D. Elevate head of bed 60 to 90 degrees

D. Elevate head of bed 60 to 90 degrees

The parents of a 6-year-old recently diagnosed with Duchenne muscular dystrophy tell the nurse that their child wants to continue attending swimming classes. How should the nurse respond? A. Explain that their child is too young to understand risks associated with swimming B. Provide a list of alternative activities that are less likely to cause the child to experience fatigue C. Suggest that the child can be encouraged to participate in a team sport to encourage socialization D. Encourage the parents to allow the child to continue attending swimming lessons with supervision

D. Encourage the parents to allow the child to continue attending swimming lessons with supervision

The healthcare provider prescribes digoxin (Lanoxin) 0.5 mg PO daily for a client with heart failure. When the nurse scans the medication label, "digoxin 0.25 mg/tablet," using an electronic scanner, a "pop-up" window in the electronic medical record indicates "Error in dose." What action should the nurse take? A. Request the pharmacy to deliver the correct dose B. Rescan the medication label until it registers C. Notify the healthcare provider of the error in the data D. Enter the value, 2 tablets, administered

D. Enter the value, 2 tablets, administered

After receiving a prescribed dose of quinapril (Accupril), losartan (Cozaar), and clonidine (Catapres), a female client tells the nurse that she usually takes Accupril at 0800, Cozaar at 1600, and clonidine at 2200 at home. The nurse informs the charge nurse that an error was made during the morning medication administration. What action should the charge nurse implement first? A. Implement orthostatic safety precautions B. Assess the client's level of consciousness C. Increase the client's fluid intake during the day D. Evaluate the clients blood pressure

D. Evaluate the clients blood pressure

A clinical trial is recommended for a client with metastatic breast cancer, but she refuses to participate and tells her family that she does not wish to have further treatments. The client's son and daughter ask the nurse to try to convince their mother to reconsider this decision. How should the nurse respond? A. ask the client with her children present if she fully understands the decision she has made B. Discuss success of clinical trials and ask the client to consider participating for one month C. Explain to the family that they must accept their mother's decision D. Explore the client's decision to refuse treatment and offer support

D. Explore the client's decision to refuse treatment and offer support

A client who has a history of hypothyroidism was initially admitted with lethargy and confusion. Which additional finding warrants the most immediate action by the nurse? A. Facial puffiness and periorbital edema B. Hematocrit of 30% C. Cold and dry skin D. Further decline in level of consciousness

D. Further decline in level of consciousness

Which of the following foods is high in iron? A. Citrus fruits B. Milk products C. Yellow vegetables D. Green leafy vegetables

D. Green leafy vegetables

A young woman with multiple sclerosis just received several immunizations in preparation for moving into a college dormitory. Two days later, she reports to the nurse that she is experiencing increasing fatigue and visual problems. What teaching should the nurse provide? A. Plans to move into the dormitory need to be postponed for at least a semester B. These early signs of an infection may require medical treatment with antibiotics C. These are common side effects of the vaccines and will resolve in a few days D. Immunizations can trigger a relapse of the disease, so get plenty of extra rest

D. Immunizations can trigger a relapse of the disease, so get plenty of extra rest

What is the primary purpose for initiating nursing interventions that promote good nutrition, rest and exercise, and stress reduction for clients diagnosed with an HIV infection? A. Increase ability to carry out activities of daily living B. Promote a feeling of general well-being C. Prevent spread of infection to others D. Improve function of the immune system

D. Improve function of the immune system

Which is the primary purpose for initiating nursing interventions that promote good nutrition, rest and exercise, and stress reduction for clients diagnosed with an HIV infection? A. Increase ability to carry out activities of daily living B. Promote a feeling of general well-being C. Prevent spread of infection to others D. Improve function of the immune system

D. Improve function of the immune system

A client is admitted with syncopal episodes related to a third degree heart block. After the placement of a transcutaneous pacemaker, the nurse observes several episodes of the pacemaker's failure to sense. What action should the nurse take? A. Turn off the pacemaker B. Adjust the miliamperes (mA) C. Increase the pacemaker rate D. Increase the sensitively

D. Increase the sensitively

The nurse is assessing a 75-year-old male client for symptoms of hyperglycemia. Which symptom of hyperglycemia is an older adult most likely to exhibit? A. Polyuria B. Polydipsia C. Weight loss D. Infection

D. Infection

The blood bank notifies the nurse that the two units of blood ordered for an anemic patient are ready for pick up. The nurse should take which of the following actions to prevent an adverse effect during this procedure? A. Immediately pick up both units of blood from the blood bank. B. Regulate the flow rate so that each unit takes at least 4 hours to transfuse. C. Set up the Y-tubing of the blood set with dextrose in water as the flush solution. D. Infuse the blood slowly for the first 15 minutes of the transfusion.

D. Infuse the blood slowly for the first 15 minutes of the transfusion.

After placement of a left subclavian central venous catheter (CVC), the nurse receives report of the X-ray findings that indicate the CVC tip is in the client's superior vena cava. Which action should the nurse implement? A. Notify the healthcare provider of the need to reposition the catheter B. Remove the catheter and apply direct pressure for 5 minutes C. Secure the catheter using aseptic techniques D. Initiate intravenous fluids as prescribed

D. Initiate intravenous fluids as prescribed

An older adult man recently diagnosed with chronic obstructive pulmonary disease (COPD) is admitted with shortness of breath. The nurse observes the client sitting upright and leaning over the bedside table, using accessory muscles to assist in breathing. What action should the nurse take? A. Assist the lien tot a high Fowler's position in bed B. Observe the client for the presence of a barrel chest C. Prepare to transfer the client to a critical care unit D. Instruct the client to pursed lip breathing techniques

D. Instruct the client in pursed lip breathing techniques

A client who is experiencing panic attacks receives a prescription for the benzodiazepine alprazolam (Xanax). Which instructions should the nurse provide the client? A. Explain that it may take up to two weeks before the anxiety starts to get better B. Instruct the client to notify the healthcare provider if tremors of the tongue occur C. Discuss the importance of obtaining monthly blood work to assess for toxicity D. Instruct the client on safety issues this medication causes drowsiness

D. Instruct the client on safety issues this medication causes drowsiness

The nurse is preparing to conduct discharge teaching for a client who had an anaphylactic reaction following administration of ampicillin (Omnipen-N). What instruction is essential for the nurse to provide this client prior to discharge? A. Inform the client that it is essential to take all of the prescribed ampicillin B. Teach the client how to self-administer epinephrine in case a reaction occurs again C. Tell the client to take medication with food to decrease the possibility of future reactions D. Instruct the client to wear a medic-alert bracelet so penicillin will not be given again

D. Instruct the client to wear a medic-alert bracelet so penicillin will not be given again

A female client who has been taking diclofenac (Zipsor) for the past month is admitted with right upper quadrant tenderness, jaundice, and flu-like symptoms. She is also complaining of fatigue, diarrhea, and pruritus. Which intervention is most important to include in this client's plan of care? A. Review results of serum protein electrolytes B. Determine frequently of indigestion C. Evaluate intake and output ratios D. Monitor serum bilirubin levels

D. Monitor serum bilirubin levels

A client with urolithiasis is preparing for discharge after lithotripsy. Which intervention should the nurse include in the client's postoperative discharge instructions? A. Report when hematuria becomes pink tinged B. Use incentive spirometer C. Restrict physical activities D. Monitor urinary stream for decrease in output

D. Monitor urinary stream for decrease in output

The nurse is assessing a 9-year-old boy who is experiencing an acute asthma attack. When auscultating this child's breaths sounds, which finding is the nurse most likely to obtain? A. Diminished breath sounds heard throughout all areas B. Fine crackles upon inspiration C. Louder breath sounds over the lower lung fields D. Musical sounds upon expiration

D. Musical sounds upon expiration

If a patient with arthritis develops iron-deficiency anemia, a nurse should ask about the patient's use of A. alcoholic beverages. B. stool softeners and laxatives. C. caffeinated foods and beverages. D. NSAIDs.

D. NSAIDs.

A client who was discharged 8 months ago with cirrhosis and ascites is admitted with anorexia and recent hemoptysis. The client is drowsy but responds to verbal stimuli. The nurse programs a blood pressure monitor to take readings every 15 minutes,. Which assessment should the nurse implement first? A. Evaluate distal capillary refill for delayed perfusion B. Check the extremities for bruising and petechiae C. Examine the pretibial regions for pitting edema D. Palpate the abdomen for tenderness and rigidity

D. Palpate the abdomen for tenderness and rigidity

A client with alcohol-related liver disease is admitted to the unit. Which prescription should the nurse question as possibly inappropriate for the client? A. Vitamin K1 (AquaMEPHYTON) 5 mg IM daily B. High-calorie, low-sodium diet C. Fluid restriction to 1500 ml/day D. Pentobarbital (Nembutal sodium) 50 mg at bedtime for rest

D. Pentobarbital (Nembutal sodium) 50 mg at bedtime for rest

A male client with diabetes mellitus (DM) is transferred from the hospital to a rehabilitation facility following treatment for a stroke with resulting right hemiplegia. He tells the nurse that his feet are always uncomfortably cool at night, preventing him from falling asleep. Which action should the nurse implement. A. Provide a warming pad (Aqua-pad or K-pad) to feet B. Medicate the client with a prescribed sedative C. Use a bed cradle to hold the covers off the feet D. Place warm blankets next to the client's feet

D. Place warm blankets next to the client's feet

Fluids are restricted to 1,500 ml daily for a male client with acute kidney injury (AKI). He is frustrated and complaining of constant thirst, and the nurse discovers that the family is providing the client with additional fluids. Which intervention should the nurse implement? A. Remove all sources of liquids from the client's room B. Allow family to give client a measured amount of ice chips C. Restrict family visiting until the client's condition is stable D. Provide the client with oral swabs to moisten his mouth

D. Provide the client with oral swabs to moisten his mouth

An 18-year-old female client is admitted to the unit after ingesting an overdose of Phenobarbital (Luminal). She is unresponsive and ABG results are: pH 7.18, PaCO2 60 mmHg, and HCO3 26 mEq/L. Which interpretation of the client's ABG results by the nurse is accurate? A. Metabolic acidosis, uncompensated B. Respiratory acidosis, compensated C. Metabolic alkalosis, compensated D. Respiratory acidosis, uncompensated

D. Respiratory acidosis, uncompensated

The unit manager of an acute care unit evaluates the time management skills of the nursing staff and determines that one staff nurse is consistently behind in meeting the needs of assigned clients. What action should the unit manager take? A. Plan to reassign some of the clients to another nurse the nest day B. Determine if the nurse is having personal problems that affect work. C. Request that the nursing supervisor meet with the nurse today D. Schedule a private meeting with the staff nurse as soon as possible

D. Schedule a private meeting with the staff nurse as soon as possible

An adolescent male is transferred from the medical unit to the mental health unit because his condition is stable after taking an assortment of prescription drugs. Based on the admission interview, the nurse determines that the client is still having suicidal ideations. What intervention is most important for the nurse to implement? A. Try to determine what life event precipitated the suicide attempt B. Place the client in lock-up until the psychiatrist releases him C. Reassure the client that he is in a safe place D. Search the client's belongings for potential weapons

D. Search the client's belongings for potential weapons

An unlicensed assistive personnel (UAP) informs the nurse who is giving medications that a female client is crying. The client was just informed that she has a malignant tumor. What action should the nurse implement first? A. Provide the client with a PRN antianxiety medication and allow privacy for her to grieve B. instruct the UAP to notify the client's spiritual advisor of her need for counseling C. Ask another nurse to finish giving medications and attend to the client immediately D. Tell the client that the nurse will be back to talk to her after medications are given

D. Tell the client that the nurse will be back to talk to her after medications are given

An emergency department nurse is giving discharge instructions to the wife of a young adult male client who sustained a concussion after a fall. The nurse should provide the wife with what instruction as part of the discharge teaching plan? A. Encourage the wife to bring her husband back to the emergency department if he experiences headaches within the next 24 hours B. Teach the wife how to complete a Glasgow Scale (GCS) to do at home C. Provide written instructions on determining pupil constriction D. Tell the wife to bring her husband back to the emergency department if he has projectile vomiting or an unsteady gait

D. Tell the wife to bring her husband back to the emergency department if he has projectile vomiting or an unsteady gait

A nurse is caring for an elderly client who recently attempted suicide with a overdose of sedatives. Which conclusion regarding this client's achievement of normal development is accurate? A. Suicide attempts that occur in the elderly population are most likely due to declining physical health B. The client was unsuccessful in resolving Trust vs. Mistrust issues, resulting in anger turned inward C. Role confusion often occurs in the elderly due to various levels of dementia, which explains suicidal tenderness D. The adult who is unfulfilled at an advanced age fails to achieve ego integrity, and instead experiences despair

D. The adult who is unfulfilled at an advanced age fails to achieve ego integrity, and instead experiences despair

A client with a large pleural effusion undergoes a thoracentesis. Following the procedure, which observation warrants immediate intervention by the nurse? A. The client complains of pain at the insertion site B. The client's chest x-ray indicates decreased pleural effusion C. The client's arterial blood gases are pH 7.35; PaO2 85; PaCO2 35; HCO3 26 D. The client has asymmetrical chest wall expansion

D. The client has asymmetrical chest wall expansion

The nurse in the outpatient department is caring for a client who had a right femoral cardiac catheterization two hours ago. What assessment finding requires immediate intervention? A. The client's right foot is warm to touch B. The client's blood pressure is 110/70 and pulse 88 C. The client's pulse oximeter reading is 98% D. The client wants assistance walking to the bathroom

D. The client wants assistance walking to the bathroom

A client returns to the unit following a suprapubic prostatectomy. He has a three-way catheter in place with a continuous bladder irrigation infusing. Which assessment finding warrants immediate intervention by the nurse? A. True urinary output of 50ml/hr B. Lower abdominal tenderness C. Blood urine output with clots D. Urine leaking around the meatus

D. Urine leaking around the meatus

A frail, elderly female with rheumatoid arthritis (RA) complains to the nurse that the weight of the sheets on her legs hurts all the time. Which action should the nurse implement? A. Soak her hands in warm water when resting B. Administer an analgesic at the hour of sleep C. Provide a soft blanket for covering the client D. Use a bed cradle to keep linens off her legs

D. Use a bed cradle to keep linens off her legs

The nurse is completing a neurological assessment. What observation indicates an abnormal pupil response? A. When shinning the light into the eye, the pupil contracts briskly B. The optic disc appears edematous and engorged C. As the nurse's finger is brought in closer to the eye, the pupils contract D. When shining the light into the right eye, the left pupil does not constrict

D. When shining the light into the right eye, the left pupil does not constrict

After a posterior nasal pack is inserted by a physician, the patient is very anxious and states, "I don't feel like I'm breathing right." The immediate intervention the nurse should initiate is to A. monitor ABGs. B. reassure the patient that this is normal discomfort. C. cut the pack strings and pull the packing out with a hemostat. D. direct a flashlight into the patient's mouth and inspect the oral cavity.

D. direct a flashlight into the patient's mouth and inspect the oral cavity.

The practical nurse (PN) reports to the charge nurse that the unlicensed assistive personal (UAP) did not adhere to the agency's fall prevention protocols when caring for a client at risk for falls. What action should the charge nurse implement? A. encourage the PN to complete an adverse occurrence report B. instruct the PN to supervise the UAP more close the next day C. plan to assign the UAP to more stable clients the next day D. meet with the UAP to discuss the observations made by the PN

D. meet with the UAP to discuss the observations made by the PN

An adult is brought to the emergency department by ambulance following a motorcycle accident. He was not wearing a helmet and presents with periorbital bruising and bloody drainage from both ears. Which assessment finding warrants immediate intervention by the nurse? A. rebound abdominal tenderness B. diminished bilateral breath sounds C. rub pain with deep inspiration D. nausea with projectile vomiting

D. nausea with projectile vomiting

The most appropriate position to assist a patient with chronic obstructive pulmonary disease (COPD) who is having difficulty breathing would be a A. high Fowler's position without a pillow behind the head. B. semi-Fowler's position with a single pillow behind the head. C. right side-lying position with the head of the bed at 45 degrees' elevation. D. sitting upright and forward position with arms supported on an over-the-bed table.

D. sitting upright and forward position with arms supported on an over-the-bed table.

When administering oxygen to a patient with COPD with the potential for carbon dioxide narcosis, the nurse should A. never administer oxygen at a rate of more than 2 L/min. B. monitor the patient's use of oxygen to detect oxygen dependency. C. monitor the patient for symptoms of oxygen toxicity, such as paresthesias. D. use ABGs as a guide to determine what FIO2 level meets the patient's needs.

D. use ABGs as a guide to determine what FIO2 level meets the patient's needs.

When the nurse prepares to administer preoperative medication to a patient, the patient tells the nurse that she really does not understand what the surgeon plans to do.

Notify the healthcare provider because the patient needs further explanation of the planned surgery.

In assessing a client with an arteriovenous shunt who is scheduled for dialysis today, the nurse notes the absence of either a thrill or a bruit at the shunt site. What action should the nurse take?

Notify the healthcare provider of the findings.

The most normal functioning method of speech restoration in the patient with a total larynectomy is

a voice presthesis

PRBCS develops chills, fever, anxiety 30 minutes after infusion started. What does the nurse do?

stop the transfusion.

What is the most important nursing priority for a client who has been admitted for a possible kidney stone?

straining all urine

Proventil side effects

tachycardia, BP changes, nervousness, palpitations, muscle tremors, nausea, vomiting, vertigo, insomnia, dry mouth, headache, hypokalemia.

Pseudomonis

thick green mucous.

The nurse is preparing to administer enoxaparin (Lovenox) 135 mg subcutaneously. The medication is available in a cartridge labeled 150 mg/ml. How many ml should the nurse administer? (enter numeric value only. If rounding is required, round to the nearest tenth.)

0.9 ml

Which description of symptoms is characteristic of a client diagnosed with trigeminal neuralgia (tic douloureux)? A) Tinnitus, vertigo, and hearing difficulties. B) Sudden, stabbing, severe pain over the lip and chin. C) Facial weakness and paralysis. D) Difficulty in chewing, talking, and swallowing.

B

Prior to administering tissue plasminogen activator (t-PA), the nurse should assess the client for which of the following contradictions to administering the drug? 1. Age greater than 60 years. 2. History of cerebral hemorrhage. 3. History of heart failure. 4. Cigarette smoking.

2 A history of cerebral hemorrhage is a contraindication to administration of t- PA because the risk of hemorrhage may be further increased. Age greater than 60 years, history of heart failure, and cigarette smoking are not contraindications.

A postoperative client receives a Schedule II opioid analgesic for pain. Which assessment finding requires the most immediate intervention by the nurse?

C. Respiratory rate of 12 breaths/min, with O2 saturation of 85%

Blood transfusion reactions the following steps need to be taken

1. stop transfusion. 2. maintain patent IV line with saline solution. 3. notify blood bank and the healthcare provider immediately; 4. recheck identifying tags and numbers; 5. monitor VS and urine output; 6. treat symptoms per physician order; 7. save the blood bag and tubing and send them to the blood bank for examination; 8. complete transfusion report; 9. collect required blood and urine specimens at intervals stipulated by hospital policy to evaluate hemolysis and 10. document on transfusion reaction form and patient chart.

A client is receiving an IV solution labeled Heparin Sodium 20,000 units in 5% Dextrose Injection 500 ml at 25 ml/hour. How many units of heparin is the client receiving each hour?

1000 units/hour

The healthcare provider prescribes diltiazem (Cardizem) for a child with hypertension who weighs 66 pounds. Based on the recommended dose of 3.5 mg/kg/day, how many mg should the child receive per day?

105 mg/day

A 34-year-old female is diagnosed with hypothyroidism. The nurse should assess the client for which of the following? Select all that apply. 1. Rapid pulse. 2. Decreased energy and fatigue. 3. Weight gain of 10 lb (4.5 kg). 4. Fine, thin hair with hair loss. 5. Constipation. 6. Menorrhagia.

2, 3, 5, 6. Clients with hypothyroidism exhibit symptoms indicating a lack of thyroid hormone. Bradycardia, decreased energy and lethargy, memory problems, weight gain, coarse hair, constipation, and menorrhagia are common signs and symptoms of hypothyroidism.

A client reports vomiting every hour for the past 8 to 10 hours. The nurse should assess the client for risk of which of the following? Select all that apply. 1. Metabolic acidosis. 2. Metabolic alkalosis. 3. Hypokalemia. 4. Hyperkalemia. 5. Hyponatremia.

2,3 Gastric acid contains a substantial amount of potassium, hydrogen ions, and chloride ions. Frequent vomiting can induce an excessive loss of these acids leading to alkalosis. Excessive loss of potassium produces hypokalemia. Frequent vomiting does not lead to the condition of too much potassium (hyperkalemia) or too little sodium

The nurse is assessing the lower extremities of the client with peripheral vascular disease (PVD). During the assessment, the nurse should expect to find which of the following clinical manifestations of PVD? Select all that apply. 1. Hairy legs. 2. Mottled skin. 3. Pink skin. 4. Coolness. 5. Moist skin.

2,4 Reduction of blood flow to a specific area results in decreased oxygen and nutrients. As a result, the skin may appear mottled. The skin will also be cool to the touch. Loss of hair and dry skin are other signs that the nurse may observe in a client with PVD of the lower extremities.

The nurse is evaluating a client with hyperthyroidism who is taking Propylthiouracil (PTU) 100 mg/day in three divided doses for maintenance therapy. Which of the following statements from the client indicates the desired outcome of the drug? 1. "I have excess energy throughout the day." 2. "I am able to sleep and rest at night." 3. "I have lost weight since taking this medication." 4. "I do perspire throughout the entire day."

2. PTU is a prototype of thioamide antithyroid drugs. It inhibits production of thyroid hormones and peripheral conversion of T4 to the more active T3. A client taking this antithyroid drug should be able to sleep and rest well at night since the level of thyroid hormones is reduced in the blood. Excess energy throughout the day, loss of weight and perspiring through the day are symptoms of hyperthyroidism indicating the drug has not produced its outcome.

Which symptoms should the nurse expect a client to exhibit who is known to have a pheochromocytoma? A) Numbness, tingling, and cramps in the extremities. B) Headache, diaphoresis, and palpitations. C) Cyanosis, fever, and classic signs of shock. D) Nausea, vomiting, and muscular weakness.

B

While performing a skin inspection for a female adult client, the nurse observes a rash that is well circumscribed, has silvery scales and plaques, and is located on the elbows and knees. These assessment findings are likely to indicate which condition? A. Tinea corporis B. Psoriasis C. Herpes Zoster D. Drug reaction

B. Psoriasis

Nursing interventions for the patient with aplastic anemia are directed toward the prevention of the complications of A. fatigue and dyspnea. B. hemorrhage and infection. C. thromboemboli and gangrene. D. cardiac arrhythmias and heart failure.

B. hemorrhage and infection.

Select all that apply. Atelectasis can be caused by A. long-term smoking. B. inadequate surfactant. C. localized airway obstruction. D. an increase in lung expansion. E. an increase in elastic recoil.

B. inadequate surfactant. C. localized airway obstruction. E. an increase in elastic recoil.

Which interventions are necessary to provide safe, quality care to a patient receiving enteral tube feedings? SELECT ALL THAT APPLY!! A) check the residual volume every 4-6 hours B) use clean technique when changing the feeding system C) keep the head of the beg elevated at least 30 degrees D) change the feeding bag & tubing every 12 hours E) allow closed system containers to hang for 24 hours

A,B,C,E

When conducting a health history with a female client with thyrotoxicosis, the nurse should ask about which of the following changes in the menstrual cycle? 1. Dysmenorrhea. 2. Metrorrhagia. 3. Oligomenorrhea. 4. Menorrhagia.

3. A change in the menstrual interval, diminished menstrual flow (oligomenorrhea), or even the absence of menstruation (amenorrhea) may result from the hormonal imbalances of thyrotoxicosis. Oligomenorrhea in women and decreased libido and impotence in men are common features of thyrotoxicosis. Dysmenorrhea is painful menstruation. Metrorrhagia, blood loss between menstrual periods, is a symptom of hypothyroidism. Menorrhagia, excessive bleeding during menstrual periods, is a symptom of hypothyroidism.

What modification is most important for the nurse to recommend to a client with high cholesterol? A. Replace beed with fish or poultry B. Include six servings of fruits and vegetables daily C. Limit portion sizes using the "Plate Method" D. Use vegetables oils in food preparation

A. Replace beed with fish or poultry

A client with chronic kidney disease is being discharged with continuous ambulatory peritoneal dialysis (CAPD). What is the priority nursing diagnosis the nurse should use when developing a discharge teaching plan for this client? A. Altered nutriton B. Impaired mobility C. Risk for sepsis D. Risk for injury

C. Risk for sepsis

Which of the following indicates hypovolemic shock in a client who has had a 15% blood loss? 1. Pulse rate less than 60 bpm. 2. Respiratory rate of 4 breaths/min. 3. Pupils unequally dilated. 4. Systolic blood pressure less than 90 mm Hg.

4 Typical signs and symptoms of hypovolemic shock include systolic blood pressure less than 90 mm Hg, narrowing pulse pressure, tachycardia, tachypnea, cool and clammy skin, decreased urine output, and mental status changes, such as irritability or anxiety. Unequal dilation of the pupils is related to central nervous system injury or possibly to a previous history of eye injury.

Genitourinary System Review prior to surgery

Consider women of childbearing age, you need to determine if they are pregnant or think they could be pregnant. The Surgeon should be informed immediately if the patient states that she might be pregnant because maternal and subsequent fetal exposure to anesthetic during first trimester should be avoided.

A client who is receiving chemotherapy is vomiting. Which nursing intervention should the nurse implement first? A. Teach the client about the importance of hydration B. Report the volume of emesis t the healthcare provider C. Administer ondansetron hydrochloride (Zofran) D. Encourage the client to limit the amount of movement

C. Administer ondansetron hydrochloride (Zofran)

A client who is receiving chemotherapy asks the nurse, "Why is so much of my hair falling out each day?" Which response by the nurse best explains the reason for alopecia? A) Chemotherapy affects the cells of the body that grow rapidly, both normal and malignant. B) Alopecia is a common side effect you will experience during long-term steroid therapy. C) Your hair will grow back completely after your course of chemotherapy is completed. D) The chemotherapy causes permanent alterations in your hair follicles that lead to hair loss.

A

A client receiving cholestyramine (Questran) for hyperlipidemia should be evaluated for what vitamin deficiency? A) K. B) B12. C) B6. D) C.

A

In taking health histories of the following individuals, which client would have the greatest potential for development of head and neck cancer?

An alcoholic, tobacco-chewing auctioneer

The nurse is preparing a 45-year-old female client for discharge from a cancer center following ileostomy surgery for colon cancer. Which discharge goal should the nurse include in this client's discharge plan?

Attend an ostomy support group within 2 weeks.

A 20-year-old female client calls the nurse to report a lump she found in her breast. Which response is the best for the nurse to provide? A) Check it again in one month, and if it is still there schedule an appointment. B) Most lumps are benign, but it is always best to come in for an examination. C) Try not to worry too much about it, because usually, most lumps are benign. D) If you are in your menstrual period it is not a good time to check for lumps.

B

A 32-year-old female client complains of severe abdominal pain each month before her menstrual period, painful intercourse, and painful defecation. Which additional history should the nurse obtain that is consistent with the client's complaints? A) Frequent urinary tract infections. B) Inability to get pregnant. C) Premenstrual syndrome. D) Chronic use of laxatives.

B

In preparing the preoperative teaching plan for a patient who is to undergo a total laryngectomy, a nurse should give highest priority to the A. tracheostomy being in place for 2 to 3 days. B. patient's not being able to speak normally again. C. insertion of a gastrostomy feeding tube during surgery. D. patient's not being able to perform deep-breathing exercises.

B. patient's not being able to speak normally again.

Anticoagulant therapy is used in the treatment of thromboembolic disease because anticoagulants can A. dissolve the thrombi. B. decrease blood viscosity. C. prevent absorption of vitamin K. D. inhibit the synthesis of clotting factors.

D. inhibit the synthesis of clotting factors.

A client with congestive heart failure and atrial fibrillation develops ventricular ectopy with a pattern of 8 ectopic beats/min. What action should the nurse take based on this observation?

Increase oxygen flow via nasal cannula.

The nurse is caring for a client who is 1 day post acute myocardial infarction. The client is receiving oxygen at 2 L/min via nasal cannula and has a peripheral saline lock. The nurse notes that the client is having eight PVCs per minute. Which intervention should the nurse implement first?

Increase the client's oxygen flow rate.

Which assessment finding indicates that the expected outcome of administering donepezil (Aricept) to a client with Alzheimer disease has been accomplished?

Increased ability to solve simple problems

St. John's Wort

May prolong the effects of anesthetic agents. Increases the waking time after surgery.

Cushing syndrome results from a hypersecretion of glucocorticoids in the adrenal cortex. Based on the clinical manifestations of Cushing syndrome, which nursing intervention would be appropriate for a client who is newly diagnosed with Cushing syndrome?

Monitor blood glucose levels daily.

A female client who was involved in a motor vehicle collision with a fractured left femur which is immobilized using a fracture traction splint in preparation for an open reduction internal fixation (ORIF). The nurse determines that her distal pulses are diminished in the left foot. Which interventions should the nurse implement? Select all that apply

Monitor left leg for pain, pallor, paresthesia, paralysis, pressure. Verify pedal pulses using a doppler pulse device. Evaluate the application of the splint to the left leg

A patient is admitted to the hospital for evaluation and treatment of thrombocytopenia. Which of the following actions is most important for the nurse to implement?

Monitoring the patient for headaches, vertigo, or confusion

A severe thunderstorm has moved into a small community, and the tornado warning alarm has been activated at the local hospital. Which action should the charge nurse in the surgical department implement first?

Move clients and visitors into the hallways and close all doors to client rooms.

A client who is being monitored with telemetry has a pattern of uncontrolled atrial fibrillation with a rapid ventricular response. Based on this finding, the nurse anticipates administering which treatment?

Perform synchronized cardioversion

Which of the following statements made by a patient with COPD indicates a need for further education regarding the use of an ipratropium inhaler? A. "I should rinse my mouth following the two puffs to get rid of the bad taste." B. "I should wait at least 1 to 2 minutes between each puff of the inhaler." C. "If my breathing gets worse, I should keep taking extra puffs of the inhaler until I can breathe more easily." D. "Because this medication is not fast-acting, I cannot use it in an emergency if my breathing gets worse.

C. "If my breathing gets worse, I should keep taking extra puffs of the inhaler until I can breathe more easily."

A central venous catheter has been inserted via a jugular vein and a radiography has confirmed placement of the catheter. A prescription has been received for stat medication but IV fluids have not yet been started. What action should the nurse take prior to administering the prescribed medication? A. Assess for signs of jugular vein distention. B. Obtain the needed intravenous solution. C. Administer a bolus of normal saline solution. D. Flush the line with heparinized saline.

C. Administer a bolus of normal saline solution.

The home health nurse is assessing a male client being treated for Parkinson disease with levodopa-carbidopa (Sinemet). The nurse observes that he does not demonstrate any apparent emotions when speaking and rarely blinks. Which intervention should the nurse implement? A. Perform a complete cranial nerve assessment. B. Instruct the client that he may be experiencing medication toxicity. C. Document the presence of these assessment findings. D. Advise the client to seek immediate medical evaluation.

C. Document the presence of these assessment findings.

Nursing assessment findings of jugular vein distention and pedal edema would be indicative of which of the following complications of emphysema? A. Acute respiratory failure B. Pulmonary edema caused by left-sided heart failure C. Fluid volume excess secondary to cor pulmonale D. Secondary respiratory infection

C. Fluid volume excess secondary to cor pulmonale

Which of the following clinical manifestations would the nurse expect to find during assessment of a patient admitted with pneumococcal pneumonia? A. Hyperresonance on percussion B. Fine crackles in all lobes on auscultation C. Increased vocal fremitus on palpation D. Vesicular breath sounds in all lobes

C. Increased vocal fremitus on palpation.


Conjuntos de estudio relacionados

MEDSURG TEST #2: CH 43 - Liver Cancer/Transplant, Pancreas and Gallbladder Problems

View Set

Mathematical Operators and Code Practice

View Set

Patho Wk 7 Ch (36, 37, 38, 40, 41) Song WCU, WEEK 7 PATHO 370, Week 7 Patho WCU, Patho Week 7 Quiz, WCU Patho Week 7 - Ch. 36, 37, 38, 41

View Set

Chapter 46: Antineoplastic Drugs Part 2

View Set

Alg. 1 Topic 5.5: Standard Form of Linear Equations

View Set